Está en la página 1de 322

BRILLIANT PUBLIC SCHOOL,

SITAMARHI

(Affiliated up to +2 level to C.B.S.E., New Delhi)

Class-XII
IIT-JEE Advanced Mathematics
Study Package
Session: 2014-15
Office: Rajopatti, Dumra Road, Sitamarhi (Bihar), Pin-843301
Ph.06226-252314 , Mobile:9431636758, 9931610902
Website: www.brilliantpublicschool.com; E-mail: brilliantpublic@yahoo.com

STUDY PACKAGE
Target: IIT-JEE (Advanced)
SUBJECT: MATHEMATICS-XII
Chapter

Pages Exercises

1 Inverse Trigonometric
Functions
2 Determinants and Matrices

37

53

10

3 Continuity & Differentiability

30

4 Applications of Derivative

51

15

5 Integration

88

6 Area Under Curves

20

7 Differential Equations

34

STUDY PACKAGE
Target: IIT-JEE (Advanced)
SUBJECT: MATHEMATICS
TOPIC: 18 XII M 1. Inverse
Trigonometric Functions
Index:
1. Key Concepts
2. Exercise I to II
3. Answer Key
4. Assertion and Reasons
5. 34 Yrs. Que. from IIT-JEE
6. 10 Yrs. Que. from AIEEE

1.

Principal Values & Domains of Inverse Trigonometric/Circular Functions:


Function

Domain

(i)

y = sin1 x

where

1x1

Range

y
2
2

(ii)

y = cos1 x

where

1x1

0y

(iii)

y = tan1 x

where

xR

<y<
2
2

(iv)

y = cosec 1 x

where

x 1 or x 1

y ,y0
2
2

(v)

y = sec1 x

where

x 1 or x 1

0 y ; y

(vi)

y = cot1 x

where

xR

0<y<

NOTE:
(a)
1st quadrant is common to the range of all the inverse functions.
(b)
3rd quadrant is not used in inverse functions.
(c)
(d)

y 0.
2
No inverse function is periodic. (See the graphs on page 17)

4th quadrant is used in the clockwise direction i.e.

Solved Example # 1

1
1 1
1
.
Find the value of tan cos + tan
2
3

Solution
Let

1
1 1
1
.
y = tan cos + tan

3
2


= tan 3 + 6

= tan
6
1
y=
3
Self practice problems:

Ans.

Find the value of the followings :

1 1
(1)
sin 3 sin 2

(2)

cosec [sec1 ( 2 ) + cot1 ( 1)]

Ans.

Ans.

Solved Example # 2
Find domain of sin1 (2x2 1)
Solution.
Let
y = sin1 (2x2 1)
For y to be defined
1 (2x 2 1) 1

0 2x 2 2

0 x2 1

x [1, 1]
Self practice problems:
Find the domain of followings :
(3)
y = sec1 (x2 + 3x + 1)
2

(4)

x2

y = cos1
2
1+ x

y = tan1 ( x 2 1)
Answers
(3)
( , 3] [ 2, 1] [0, )
(4)
R
(5)
( , 1] [1, )

(5)

2.

Properties of Inverse Trigonometric Functions:


Property - 2(A)
(i)
sin (sin1 x) = x,

1 x 1

(ii)

cos (cos1 x) = x,

1 x 1

(iii)

tan (tan1 x) = x,

xR

(iv)

cot (cot1 x) = x,

xR

(v)

sec (sec 1 x) = x,

x 1, x 1

(vi)

cosec (cosec1 x) = x, x 1, x 1

These functions are equal to identity function in their whole domain which may or may not be R.(See the
graphs on page 18)
Solved Example # 3

Find the value of cosec cot cot 1


.
4

Solution.
Let

y = cosec cot cot 1

cot (cot1 x) = x, x R

.......(i)

1 3
=
cot cot
4
4

from equation (i), we get

3
y = cosec
4
y= 2
Ans.
Self practice problems:

Find the value of each of the following :


(6)

cos sin sin


6

Answers

(7)

3
2

(6)

(7)

1 3

sin cos cos


4

not defined

Property - 2(B)
(i)

sin1 (sin x) = x,

x
2
2

(ii)

cos1 (cos x) = x; 0 x

(iii)

tan1 (tan x) = x;

<x<
2
2

(iv)

cot 1 (cot x) = x;

(v)

sec1 (sec x) = x; 0 x , x

(vi) cosec1 (cosec x) = x;

These are equal to identity function for a short interval of x only.


(See the graphs on page 19-20)

Solved Example # 4
3

Find the value of tan1 tan


4

Solution.
3

Let y = tan1 tan

Note

tan1 (tan x) = x


if x ,
2 2
3

0<x<
x 0,

x
2
2


3
,
4
2 2

3

tan1 tan
4

3
3

,
4
2 2

graph of y = tan1 (tan x) is as :

from the graph we can see that if

3
< x<
,
2
2
then y = tan1 (tan x) can be written as
y=x

y = tan1 tan
4

solved Example # 5
Find the value of sin1 (sin7)

Solution.
Let y = sin1 (sin 7)

Note : sin1 (sin 7) 7 as 7 ,
2 2

5
2
graph of y = sin1 (sin x) is as :

2 < 7 <

From the graph we can see that if 2 x


y = sin1(sin x ) can be written as :
y = x 2

sin1 (sin 7) = 7 2

5
then
2

Similarly if we have to find sin1 (sin(5)) then


3

2 < 5 <
2

y=

123
4

from the graph of sin1 (sin x), we can say that


sin1 (sin(5)) = 2 + (5)
= 2 5
Self practice problems:
(8)
Find the value of cos1 (cos 13)
(9)

Ans.

Find sin1 (sin ), cos1(cos), tan1 (tan ), cot1(cot) for , 3


2

(8)
13 4
;
cos1 (cos ) = 2 ;
(9)
sin-1 (sin) = 3
tan1 (tan ) = 3
;
cot1 (cot ) = 2

Property - 2(C)
(i)
(iii)

1
1 x 1
( x) = sin1 x,
cos1 (x) = cos1 x, 1 x 1
s i n

tan1 ( x) = tan1 x,
cot 1 ( x) = cot1 x,

(ii)
(iv)

xR
xR

The functions sin1 x, tan1 x and cosec1 x are odd functions and rest are neither even nor odd.

Solved Example # 6
Find the value of cos1 {sin( 5)}
Solution.
Let

y = cos1 {sin(5)}
= cos1 ( sin 5)
= cos1 (sin 5)

cos1 ( x) = cos1x, |x| 1

= cos1 cos 5
2

..........(i)

2 < 5 <
2

graph of cos1 (cos x ) is as :

from the graph we can see that if 2 x


then y = cos1 (cosx) can be written as y = x + 2


5
from the graph cos1 cos 5 = 5 + 2 =

2
2

from equation (i), we get

5
y=
2

y=5

3
2

Ans.

Self practice problems:


Find the value of the following

(10)

cos1 ( cos 4)

(12)

1
tan1 cot
4

Answers.

(10)

(11)

(11)

(12)

tan1 tan
8


4 2

5 1

Property - 2(D)
(i)

(iii)

c o s e c

1
;x 1, x 1
x

x = sin1

sec 1 x = cos1

(ii)

1
;x 1, x 1
x

1 1
tan x ; x > 0
cot x =
1
+ tan 1 ; x < 0
x

Solved Example # 7

1 2

Find the value of tan cot


3

Solution
Let

2
y = tan cot 1

cot1 (x) = cot1x, x R


equation (i) can be written as

........(i)

2
y = tan cot 1
3

y = tan cot 1
3

cot1 x = tan1

1
x

3
y = tan tan 1
2

if

x>0

y=

3
2

Self practice problems:


Find the value of the followings
(13)

2
sec cos 1
3

Answers.

(13)

(14)
3
2

(14)

cosec sin1

Property - 2(E)

, 1 x 1
2

(i)

sin1 x + cos1 x =

(iii)

cosec1 x + sec1 x =

tan1 x + cot 1 x =

(ii)

, x 1
2

Solved Example # 8
Find the value of sin (2cos1x + sin1x) when x =
Solution.
Let

1
5

y = sin [2cos1x + sin1x]

,
2

|x| 1

sin1x + cos1x =

y = sin 2 cos 1 x + cos 1 x


2

1
= sin + cos x
2

= cos (cos1x)

y = cos cos 1
5

cos (cos1x) = x

x=

1
5

........(i)
if

x [1, 1]

, xR
2

1
[1, 1]
5
1
1

cos cos 1 =
5
5

1
y= .
5

from equation (i), we get

(16)

x=

Self practice problems:


Solve the following equations

(15)
(16)

5 tan1x + 3 cot1x = 2
4 sin1x = cos1x

Answers.

(15)

x=1

1
2

Property - 2(F)

(i)

2
sin (cos1 x) = cos (sin1 x) = 1 x , 1 x 1

(ii)

tan (cot1 x) = cot (tan1 x) =

(iii)

cosec (sec1 x) = sec (cosec1 x) =

1
, x R, x 0
x
x
x2 1

, x > 1

Solved Example # 9
3

Find the value of sin tan 1 .


4

Solution.

Let

y = sin tan 1
4

..........(i)

Note : To find y we use sin(sin1 x) = x, 1 x 1

For this we convert tan1 x in sin1 x


Let

= tan1

3
4

tan =


3
and 0,
2
4

sin =

3
5

3
sin1 (sin ) = sin1
5


0,
2

equation (ii) can be written as :

3
= sin1
5

from equation (i), we get

..........(ii)
sin1 (sin ) =

3
= tan1
4

y = sin sin 1
5

3
5
Solved Example # 10

y=

1 5
Find the value of tan cos
3
2
Solution.

Let

1 5
y = tan cos

2
3

..............(i)
7

3
tan1 = sin1
4

3

5

Let

5

0, and cos =
3
2

5
=
3
equation (i) becomes

cos1

y = tan
2

1 cos

tan2 =
=
1 + cos
2

tan

............(ii)

=
2

5
3 5
(3 5 ) 2
3
=
=
5
3+ 5
4
1+
3
1

3 5


0,
2

.........(iii)

0,
4
2

>0
2
from equation (iii), we get

tan

3 5

= 2
2

from equation (ii), we get

tan

3 5

y =

Ans.

Solved Example # 11
Find the value of cos (2cos1x + sin1x) when x =
Solution.
Let

y = cos [2cos1x + sin1x]

,
2

|x| 1

sin1x + cos1x =

y = cos 2 cos 1 x + cos 1 x


2

1
cos + cos x
2

sin (cos1x)

y=

sin (cos1x) =

Aliter : Let

sin cos 1
5

1
5

24
5

cos 1

1
=
5

1
5

........(i)

1 x2 , | x | 1

1 1
= 1
=
sin cos
25
5

from equation (i), we get

y=

x=

24
5

cos =

1
and
5

sin =

24
5

24

sin1 (sin ) = sin1 5


0, 2


0,
2

..........(ii)
sin1 (sin ) =

equation (ii) can be written as


24

= sin1

1
= cos1
5

24
1

cos1 = sin1

5

5
Now equation (i) can be written as

24

y = sin sin 1

24
[1, 1]
5

........(iii)

24
1
=
sin sin

24
5

from equation (iii), we get


y=

24
5

Self practice problems:


Find the value of the followings :
(17)

41
tan cos ec 1

(18)

16

sec cot 1

63

(19)

1
1 3

sin cot
2
4

(20)

1
tan 2 tan 1
5

(19)

2 5
5

Answers :

3.

(17)

4
5

(18)

65
16

(20)

7
17

Identities of Addition and Substraction:


A.
(i)

2
2

sin1 x + sin1 y = sin1 x 1 y + y 1 x , x 0, y 0 & (x 2 + y2) 1


2
2

= sin1 x 1 y + y 1 x , x 0, y 0 & x 2 + y2 > 1

Note that:

x 2 + y2 1

0 sin1 x + sin1 y

x 2 + y2 > 1

< sin1 x + sin1 y <


2

(ii)

cos1 x + cos1 y = cos1 x y 1 x

(iii)

tan1 x + tan1 y = tan1

1 y 2 , x 0, y 0

x+y
, x > 0, y > 0 & xy < 1
1 xy

= + tan1

x+y
, x > 0, y > 0 & xy > 1
1 xy

, x > 0, y > 0 & xy = 1


2

xy < 1 0 < tan1 x + tan1 y < ;xy > 1 < tan1 x + tan1 y <
2
2
=

Note that :
B.
(i)

2
2

sin1 x sin1 y = sin1 x 1 y y 1 x , x 0, y 0

(ii)

cos1 x cos1 y = cos1 x y + 1 x

1 y 2 , x 0, y 0, x y

(iii)

xy
tan1 x tan1y = tan1 1 + xy , x 0, y 0

Note: For x < 0 and y < 0 these identities can be used with the help of properties 2(C)
i.e. change x and y to x and y which are positive .
Solved Example # 12
sin1

Show that

3
15
84
+ sin1
= sin1
5
17
85

Solution.
2

3
8226
3
15
15
> 0,
>1
> 0 and + =
5
17
5
7225
17

sin1

3
15
+ sin1
= sin1
5
17

3
225 15
9

1
+
1
5
289 17
25

3 8 15 4
+
.
= sin1 .
5 17 17 5

84
= sin1
85

Solved Example # 13
Evaluate:
12
4
63
+ sin1
tan1
13
5
16

cos1
Solution.
Let

z = cos1

sin1

z = cos1
z=

12
4
63
+ sin1
tan1
13
5
16

4
4

=
cos1
5
5
2

12
63
1 4
tan1
+ cos
.
2
5
13
16

12

63

1 4
cos 1 tan1
cos
5
13
16

4
12
4
12
> 0,
> 0 and
<
5
13
5
13

cos1

4 12
16
4
12
cos1
= cos1 5 13 + 1 25
5
13

equation (i) can be written as

z=

144
63
1

169 = cos 65

63
63

cos1 tan1
65

16
2

63
tan1
z = sin1
65

.........(i)

63

16

.........(ii)

63
63
= tan1

sin1
65

16
from equation (ii), we get
63
tan1
z = tan1
16

63

16

z=0

Solved Example # 14
10

Ans.

5
4

Evaluate tan1 9 + tan1


Solution.

9 > 0,

5
5
> 0 and 9 > 1
4
4

tan1 9 + tan1

9+
4

1 9.
4

= + tan1 ( 1)

5
= + tan1
4

.
4

=
tan1 9 + tan1

5
3
=
.
4
4

Self practice problems:

(22)

4
5
16
+ sin1
+ sin1
5
13
65
If tan14 + tan1 5 = cot1 then find

(23)

Prove that 2 cos1

(24)

tan1 (2x) + tan1 (3x) =

(21)

Evaluate sin1

+ cot1

13
Solve the following equations

(21)

Answers.

1
16
7
+
cos1
=
63
25
2

(22)

19
9

(25)

sin1x + sin1 2x =

(24)

x=

1
6

C.

(i)

(ii)

(iii)

(iv)

(v)

sin 2 x 1 x 2

cos1 (2 x 2 1)

2x

tan

1 x2

2x
sin1
1 + x2

cos1

1 x 2
1+x2

1
2 sin x

1
= 2 sin x

+ 2 sin 1 x

if | x | 1
if

if

x> 1

2
x< 1
2

2 cos 1 x
if
0 x 1
=
1
2 2 cos x if 1 x < 0
2 tan 1x

1
= + 2 tan x
2 tan 1 x

2 tan 1 x

= 2 tan 1x
+ 2 tan 1x

if | x | < 1

if

x< 1

if

x >1

if | x | 1

if

x >1

if

x< 1

2 tan 1x if x 0
=
1
2 tan x if x < 0

(See the graphs on page 20)


Solved Example # 15
Define y = cos1 (4x3 3x) in terms of cos1 x and also draw its graph.
Solution.
Let
Note

y = cos1 (4x3 3x)


Domain : [1, 1] and range : [0, ]

11

(25)

2
3

x=

1
2

Let cos1 x =
[0, ] and x = cos

y = cos1 (4 cos3 3 cos )


...........(i)
y = cos1 (cos 3)

Fig.: Graph of cos1 (cos x)

(i)

[0, ]
3 [0, 3]
to define y = cos1 (cos 3), we consider the graph of cos1 (cos x)
in the interval [0, 3]. Now, from the above graph we can see that
if 0 3
cos1 (cos 3) = 3
from equation (i), we get
y = 3
if
3

y = 3

y = 3 cos1x

(ii)

if < 3 2
from equation (i), we get
y = 2 3
if

y = 2 3

if

y = 2 3cos1 x

1
1
x<
2
2
cos1 (cos 3) = 2 + 3

if

1
x1
2
1
cos (cos 3) = 2 3

if

if

<3 2

2
<
3
3

(iii)

2 < 3 3

from equation (i), we get


y = 2 + 3
if

y = 2 + 3

if

2
<
3

y = 2 + 3 cos1 x

if

1 x<

from (i), (ii) & (iii), we get

2 < 3 3

1
2

1
;
x 1
3 cos x
2

1
1
y = cos1 (4x3 3x) = 2 3 cos 1 x ; x <
2
2

2 + 3 cos 1 x ; 1 x < 1
2

Graph :
For y = cos1 (4x3 3x)
domain : [1, 1]
range : [0, ]
(i)

if

1
x 1 , y = 3 cos1x.
2

dy
=
dx

dy
<0
dx

3
1 x2

if

= 3(1 x2)1/2
1
x , 1
2

1
decreasing if x , 1
2
again if we differentiate equation (i) w.r.t. x, we get

...........(i)

12

d2 y
dx 2

(ii)

d2 y
dx 2

3x
(1 x 2 )3 / 2
1
if x , 1
2

<0

dy
=
dx

increasing

(a)

d2 y
1
if x , 0 then
<0
dx 2
2

(b)

concavity downwards

1
if x , 1
2

1
1
x < , y = 2 3cos1 x.
2
2

if

3
1 x

dy
>0
dx

1 1
if x , and
2 2

concavity downwards

1 1
if x ,
2
2

d2 y
dx

3x
(1 x 2 )3 / 2

1
if x , 0
2

d2 y
1

if x 0, then
>0
dx 2
2

(iii)

concavity upwards

if x 0, 1
2

dy
d2 y
1
then
< 0 and
> 0.
dx
2
dx 2
the graph of y = cos1 (4x3 3x) is as

Similarly if 1 x <

Self practice problems:


(26)

Define y = sin1 (3x 4x3) in terms of sin1x and also draw its graph.

3x x 3
Define y = tan1
1 3x 2

Answers

(27)

(26)

in terms of tan1 x and also draw its graph.

1
;
3 sin x

1
;
y = sin1 (3x 4x3) = 3 sin x

3 sin1 x ;

1
1
x
2
2
1
< x 1
2
1
1 x <
2

graph of y = sin1 (3x 4x3)

13

(27)

y = tan

1
;
3 tan x

3
3x x

1
;
1 3x 2 = + 3 tan x

1
+ 3 tan x ;

<x<

< x <
1

1
3
1
3

<x<

3x x 3

Fig.: Graph of y = tan


2
1 3x
1

D.
x + y + z xyz
If tan1 x + tan1 y + tan1 z = tan1 1 xy yz zx if, x > 0, y > 0, z > 0 & (xy + yz + zx) < 1

NOTE:
(i)
If tan1 x + tan1 y + tan1 z = then x + y + z = xyz
(ii)

If tan1 x + tan1 y + tan1 z =

(iii)

tan1 1 + tan1 2 + tan1 3 =

then xy + yz + zx = 1
2

(iv)

14

tan1 1 + tan1

1
+ tan1 =
3
2
2

Inverse Trigonometric Functions


Some Useful Graphs
1.
(i)


y = sin 1 x, x 1, y ,
2 2

(ii)

y = cos 1 x, x 1, y [0, ]
y

O
1

y = cot 1 x, x R, y (0, )

(iv)


x


y = tan 1 x, x R, y 2, 2 ,

(iii)

(v)


(vi) y = cosec 1 x, x 1, y 2 , 0 U 0 , 2


y = sec 1 x, x 1, y 0, U ,
2 2

15

Part - 2(A)
(i)
y = sin (sin 1 x) = cos (cos 1 x) = x, x [ 1, 1], y [ 1, 1]; y is aperiodic
y
1
y

)45

+1

(ii)

y = tan (tan -1 x) = cot (cot -1 x) = x, x R, y R; y is aperiodic

y
=
x

) 45

y
=x

y = cosec (cosec 1 x) = sec (sec 1 x) = x, x 1, y 1 ; y is aperiodic

y
=
x

(iii)

1
1

16

)45
O

3
2

3
2

x)

(ii)

Part -2(B)

y = sin 1 (sin x), x R, y , , is periodic with period 2
(i)
2 2
y


x

y = cos 1 (cos x), x R, y [0, ], is periodic with period 2


y

x
y=

x+

y=

y=

(iii)


y = tan 1 (tan x), x R ( 2 n 1) , n , y , is periodic with period
2
2 2

y


3
2


x

y=

y=

y=
3

x+

x+
y=

y = sec 1 (sec x ), y is periodic with period 2; x R (2n 1) , n , y 0, 2 U 2 ,


2

y=

y=

y=

(iv)

3
2

17

3
2



,
2
2

(v)

y = cot1 (cot x), y is periodic with period ; x R {n, n }, y 0,

(vi)


y = cosec1 (cosec x), y is periodic with period 2; x R {n, n }, y , {0}
2 2

18

Part - 3(C)

(i)

(ii)

g r a p h

o f

s i n

2 x 1 x2

graph of y = cos1 (2 x 2 1)

Note : In this graph it is advisable not to check its derivability just by the inspection of the graph
because it is difficult to judge from the graph that at x = 0 there is a shapr corner or not.

(iii)

graph of y = tan1

(iv)

graph of y = sin1

(v)

2x
1 x2

2x
1 + x2

graph of y = cos

1 x 2
1+x2

19

KEY CONCEPTS
(INVERSE TRIGONOMETRY FUNCTION)
GENERAL DEFINITION(S):
1.

sin1 x , cos1 x , tan1 x etc. denote angles or real numbers whose sine is x , whose cosine is x
and whose tangent is x, provided that the answers given are numerically smallest available . These
are also written as arc sinx , arc cosx etc .
If there are two angles one positive & the other negative having same numerical value, then
positive angle should be taken .

2.

PRINCIPAL VALUES AND DOMAINS OF INVERSE CIRCULAR FUNCTIONS :


(i)

y = sin1 x where 1 x 1 ; y and sin y = x .


cos1 x

where 1 x 1 ; 0 y and cos y = x .

(ii)

y=

(iii)

y = tan1 x where x R ; < x < and tan y = x .

(iv)

y = cosec1 x where x 1 or x 1 ; y , y 0 and cosec y = x .

(v)

y = sec1 x where x 1 or x 1 ; 0 y ; y

(vi)

y = cot1 x where x R , 0 < y < and cot y = x .

NOTE THAT : (a)


(b)
(c)

and sec y = x .

1st quadrant is common to all the inverse functions .


3rd quadrant is not used in inverse functions .
4th quadrant is used in the

CLOCKWISE DIRECTION

i.e. y 0 .
2

3.

PROPERTIES OF INVERSE CIRCULAR FUNCTIONS :

P
1

(i) sin (sin1 x) = x , 1 x 1

(ii) cos (cos1 x) = x , 1 x 1

(iii) tan (tan1 x) = x , x R

(iv) sin1 (sin x) = x , x

(v) cos1 (cos x) = x ; 0 x

(vi) tan1 (tan x) = x ; < x <

P
2

(i) cosec1 x = sin1

1
x

(ii) sec1 x = cos1

1
x

(iii) cot1 x = tan1

1
x

; x>0
1
x

x 1 , x 1

= + tan1

x 1 , x 1

; x<0

sin1 (x) = sin1 x , 1 x 1


tan1 (x) = tan1 x , x R
cos1 (x) = cos1 x , 1 x 1
cot1 (x) = cot1 x , x R

P
3

(i)
(ii)
(iii)
(iv)

P
4

(i) sin1 x + cos1 x =

(iii) cosec1 x + sec1 x =


P
5

tan1 x + tan1 y = tan1

1 x 1

x+y
1 xy

(ii) tan1 x + cot1 x =

x 1
where x > 0 , y > 0 & xy < 1
20

xR

= + tan1
tan1 x tan1y = tan1
P
6

(i)

x+y
1 xy

xy
1 + xy

where x > 0 , y > 0

2
2
sin1 x + sin1 y = sin1 x 1 y + y 1 x

Note that : x2 + y2 1
(ii)

0 sin1 x + sin1 y

where x 0 , y 0 & x2 + y2 > 1

< sin1 x + sin1 y <


2

(iii)

sin1x sin1y = sin 1 x 1 y 2 y 1 x 2

(iv)

cos1 x + cos1 y = cos1 x y

1 x 2 1 y 2

where x > 0 , y > 0

where x 0 , y 0

x + y + z xyz

If tan1 x + tan1 y + tan1 z = tan1 1 x y y z z x if, x > 0, y > 0, z > 0 & xy + yz + zx < 1

Note : (i)
(ii)

P
8

where x 0 , y 0 & (x2 + y2) 1

2
2
sin1 x + sin1 y = sin1 x 1 y + y 1 x

Note that : x2 + y2 >1

P
7

where x > 0 , y > 0 & xy > 1

If tan1 x + tan1 y + tan1 z = then x + y + z = xyz

then xy + yz + zx = 1
2

If tan1 x + tan1 y + tan1 z =

2 tan1 x = sin1

2x
1 + x2

= cos1

1 x2
1 + x2

= tan1

2x
1 x2

Note very carefully that :


2 tan 1 x

2x
= 2 tan 1 x
sin1
1 + x2
+ 2 tan 1 x

2x
tan1
1 x2

2tan 1 x

= + 2tan 1 x
2tan 1 x

x 1
x>1
x < 1

if
if
if

if
if
if

2 tan 1 x
if x 0
=
1
1 + x2

2
tan
x
if
x<0

1 x
cos1

x <1
x < 1
x >1

REMEMBER THAT :
3
2

(i)

sin1 x + sin1 y + sin1 z =

(ii)

cos1 x + cos1 y + cos1 z = 3

(iii)

tan1 1 + tan1 2 + tan1 3 =

and

x=y= z=1
x = y = z = 1
tan1 1 + tan1 21 + tan1 13 = 2

INVERSE TRIGONOMETRIC FUNCTIONS


SOME USEFUL GRAPHS
1.

y = sin 1 x , x 1 , y 2

2.

21

y = cos 1 x , x 1 , y [0 , ]

3.


y = tan 1 x , x R , y 2 , 2

5.

y = sec 1 x , x 1 , y 0 , 2 2

7. (a) y = sin 1 (sin x) , x R ,

4.

y = cot 1 x , x R , y (0 , )

6. y = cosec 1 x , x 1 ,


y , ,
2 2

7.(b)

Periodic with period 2

y , 0 0 ,

2
2

y = sin (sin 1 x) ,

= x
x [ 1 , 1] , y [ 1 , 1] , y is aperiodic

22

8. (a) y = cos 1(cos x), x R, y [0, ], periodic with period 2

8. (b) y = cos (cos 1 x) ,

= x

= x
x [ 1 , 1] , y [ 1 , 1], y is aperiodic

9. (a) y = tan (tan 1 x) , x R , y R , y is aperiodic


=x

9. (b) y = tan 1 (tan x) ,


= x
xR


n I , y ,
(2 n 1)
2 2
2

periodic with period

10. (a) y = cot 1 (cot x) ,

10. (b) y = cot (cot 1 x) ,

= x
x R {n } , y (0 , ) , periodic with

= x
x R , y R , y is aperiodic

11. (a) y = cosec 1 (cosec x),

11. (b)

= x

x R { n , n I }, y 2 , 0


0 ,
2

y = cosec (cosec 1 x) ,
= x
x 1 , y 1, y is aperiodic

y is periodic with period 2

12. (a) y = sec 1 (sec x) ,

12. (b) y = sec (sec 1 x) ,

= x
y is periodic with period 2 ;
x R

n I
(2 n 1)
2

y 0 ,

2
2

= x
x 1 ; y 1], y is aperiodic
,

23

EXERCISE1
Q.1

Find the following

1
2

(iv) tan1 tan

Q.2

3
3

(vi) tan sin 1 + cot 1


5
2

Find the following :

3

2

1 3
(ii) cos cos 2 + 6

3
4

(iii) tan1 tan (iv) cos1 cos


4
3

3 sin 2
tan

(vi) tan1
+ tan1
where < <
2
2
5
+
3
cos
2

(v) sin cos 1


5

Prove that:
(a) 2 cos1

(c) arc cos


Q.4

(iii) cos1 cos


6

3
(v) cos tan 1

(i) sin sin 1

Q.3

1
(ii) sin 3 sin 2

(i) tan cos 1 + tan 1

3
13

36
5
7
(b) cos 1 + cos 1 + sin 1
=
13
25
325

+ cot1 16 + 1 cos1 7 =
63

2
arc cos
3

6 +1
2 3

25

(d) Solve the inequality: (arc sec x)2 6(arc sec x) + 8 > 0
Find the domain of definition the following functions.
( Read the symbols [*] and {*} as greatest integers and fractional part functions respectively.)
(i) f(x) = arc cos 2 x

1+ x

(iii) f (x) =
(iv) f(x) =

cos (sin x) + sin 1

(ii)

1 + x2
2x

x 3
sin 1
log10 ( 4 x)
2

1 sin x
log 5 (1 4x 2 )

+ cos 1 (1 {x})

, where {x} is the fractional part of x .

(v) f (x) = 3 x + cos 1 3 2 x + log6 (2 x 3) + sin 1 (log2 x)

(vi) f (x) = log10 (1 log7 (x2 5 x + 13)) + cos1

2 + sin

(vii) f(x) = e

sin 1 ( x2 )

x
+ tan 1 1 + n
2

x [x ]

9x
2

2 sin x + 1

(viii) f(x) = sin(cos x) + ln ( 2 cos2 x + 3 cos x + 1) + e cos

2 2 sin x

Q.5

Find the domain and range of the following functions .


(Read the symbols [*] and {*} as greatest integers and fractional part functions respectively.)
(i) f (x) = cot 1(2x x)

1 (log tan x + log


(ii) f (x) = sec
24
3
tan x 3)

(iii)

f(x) = cos1

2 x2 + 1

x2 + 1

(iv) f (x) = tan 1 log 4 5x 2 8x + 4

Q.6

Find the solution set of the equation, 3 cos1 x = sin1 1 x 2

Q.7

Prove that:

(4 x 2

1) .

, |x| 1
2
(b) 2 tan1 (cosec tan1x tan cot1x) = tan1x (x 0)
(a) sin1 cos (sin1 x) + cos1 sin (cos1 x) =

2mn
2pq
2M N
+ tan1 2
where M = mp nq, N = np + mq,
= tan1 2
2
2
2
p q
m n
M N2

(c) tan1

n
q
N
<1 ; <1 and
<1
m
p
M
(d) tan (tan1 x + tan1 y + tan1 z) = cot (cot1 x + cot1 y + cot1 z)
Q.8

Find the simplest value of, arc cos x + arc cos +

Q.9

If cos1

x
2

1
2

x
2.xy
y
x
y
cos + 2 = sin 2 .
+ cos1 = then prove that 2
a
b
ab
a
b

Q.10 If arc sinx + arc siny + arc sinz = then prove that :
(a)

1
3 3 x 2 , x , 1

(x, y, z > 0)

x 1 x + y 1 y + z 1 z = 2 xyz
2

(b) x4 + y4 + z4 + 4 x2y2z2 = 2 (x2 y2 + y2 z2 + z2x2)

Q.11

ab + 1
bc + 1
ca + 1
+ cot1
+ cot1
.
If a > b > c > 0 then find the value of : cot1
ab
bc
ca

Q.12 Solve the following equations / system of equations:


(a) sin1x + sin1 2x =

(b) tan1

(c) tan1(x1) + tan1(x) + tan1(x+1) = tan1(3x)


(e) cos1

x2 1
x2 + 1

+ tan1

2x
x2 1

1 a

= 2
1 b

+ tan1

+ cos1x =

1
1 + 4x

= tan1

2
x2

(f) sin1x + sin1y = 2 & cos1x cos1y =

(d) sin1

1
1 + 2x

(g) 2 tan1x = cos1 1 + a 2 cos1 1 + b 2 (a > 0, b > 0).


Q.13 Let l1 be the line 4x + 3y = 3 and l2 be the line y = 8x. L1 is the line formed by reflecting l1 across the
line y = x and L2 is the line formed by reflecting l2 across the x-axis. If is the acute angle between
a
L1 and L2 such that tan = , where a and b are coprime then find (a + b).
b
Q.14 Let y = sin1(sin 8) tan1(tan 10) + cos1(cos 12) sec1(sec 9) + cot1(cot 6) cosec1(cosec 7).
If y simplifies to a + b then find (a b).

Q.15 Show that : sin 1 sin

33
1
+ cos
7

46

1
cos
+ tan

7
25

13

19 13
1
tan
+ cot cot
=
7

8
8

36
4
8
Q.16 Let = sin1 , = cos1 and = tan1 , find ( + + ) and hence prove that
85
5
15

(i)

cot = cot ,

(ii)

tan tan = 1
where x (0,1]

Q.17 Prove that : sin cot1 tan cos1 x = sin cosec1 cot tan1x = x


Q.18 If sin2x + sin2y < 1 for all x, y R then prove that sin1 (tanx . tany) , .
2 2

Q.19 Find all the positive integral solutions of, tan1x + cos1

1+ y

= sin1

3
.
10


Q.20 Let f (x) = cot1 (x2 + 4x + 2 ) be a function defined R 0, then find the complete set of real
2
values of for which f (x) is onto.

EXERCISE2
Q.1

Prove that: (a) tan + cos 1 + tan cos 1 =


b
b
a
4 2
4 2

(b) cos1

x
y

cos x + cos y
= 2 tan1 tan . tan
2
2
1 + cos x cos y

1 + x2 1 x2

1 + x2 + 1 x2

2b

ab

b + a cos x
(c) 2 tan1 a + b . tan 2 = cos1

a + b cos x

Q.2

If y = tan1

Q.3

If u = cot1 cos2 tan1 cos2 then prove that sin u = tan2 .

Q.4

If = 2 arc tan

prove that x = sin 2y..

1 x2
1 + x
for 0 < x < 1 , then prove that + = , what the
& = arc sin
1 + x2
1 x

value of + will be if x > 1.

Q.5

Q.6

If x 1, then express the function f (x) = sin1 (3x 4x3) + cos1 (4x3 3x) in the form of
2

a cos1 x + b , where a and b are rational numbers.

Find the sum of the series:


n1

tan1 + tan1 + ..... + tan1 1 + 2 2 n 1 + .....

(c)

cot17 + cot113 + cot121 + cot131 + ...... to n terms.

(d)

tan1

2
9

1
3

+ ..... + sin1

(b)

+ sin1

2 1

sin1

(e)
Q.7

(a)

n (n + 1)

+ ......

2 n 1

1
1
1
1
+ tan1 2
+ tan1 2
+ tan1 2
to n terms.
x +x+1
x + 3x + 3
x + 7 x + 13
x + 5x + 7
1
1
1
1
tan1 + tan1 + tan1 + tan1
+ .....
2
8
18
32
2

Solve the following


(a) cot1x + cot1 (n x + 1) = cot1 (n 1)

26

x
x
sec1 = sec1b sec1a
a
b
2x 1
x

1
23
tan1 x + 1 + tan1 2 x + 1 = tan1
36

(b) sec1
(c)

a 1; b 1, a b.

1 1
1 1 3
3
cosec2 tan
+
sec2 tan
as an integral polynomial in & .

2
2
2
2

Q.8

Express

Q.9

Find the integral values of K for which the system of equations ;

K 2
2
=
arc cos x + (arc sin y)
4

4
(arc sin y)2 . (arc cos x) =
16

Q.10 If the value of Lim

Q.11

possesses solutions & find those solutions.

1 + ( k 1) k (k + 1)( k + 2)
120
is equal to
, find the value of k.

k
k ( k + 1)

cos 1

k =2

If X = cosec . tan1 . cos . cot1 . sec . sin1 a & Y = sec cot1 sin tan1 cosec cos1 a ;
where 0 a 1 . Find the relation between X & Y . Express them in terms of a.

1
1

Q.12 Find all values of k for which there is a triangle whose angles have measure tan1 , tan1 + k ,
2
2

and tan1 + 2k .
2

Q.13 Prove that the equation ,(sin1x)3 + (cos1x)3 = 3 has no roots for <
Q.14 Solve the following inequalities :
(a) arc cot2 x 5 arc cot x + 6 > 0

(b) arc sin x > arc cos x

Q.15 Solve the following system of inequations


4 arc tan2x 8arc tanx + 3 < 0 &

(c) tan2 (arc sin x) > 1

4 arc cotx arc cot2 x 3 > 0


cos 1 x
=1
y

(i) sin

Q.16 Consider the two equations in x ;

7
1
and >
32
8

sin 1 x
=0
y

(ii) cos

X2 [1, 1] ; Y1, Y2 I {0} are such that


X1 : the solution set of equation (i)
X2 : the solution set of equation (ii)
Y1 : the set of all integral values of y for which equation (i) possess a solution
Y2 : the set of all integral values of y for which equation (ii) possess a solution
Let : C1 be the correspondence : X1 Y1 such that x C1 y for x X1 , y Y1 & (x , y) satisfy (i).
C2 be the correspondence : X2 Y2 such that x C2 y for x X2 , y Y2 & (x , y) satisfy (ii).
State with reasons if C1 & C2 are functions ? If yes, state whether they are bijjective or into?
T

h e

s e t s

1,

Q.17 Given the functions f(x) = e

( (

cos 1 sin x +
3

))

4 2 cosx
& the function h(x) = f(x)

, g(x) = cosec1

defined only for those values of x, which are common to the domains of the functions f(x) & g(x).
Calculate the range of the function h(x).
Q.18 (a)

(b)

If the functions f(x) = sin1

2
2x
1 1 x are identical functions, then compute
&
g(x)
=
cos
1 + x2
1 + x2

their domain & range .


If the functions f(x) = sin1 (3x 4x3) & g(x) = 3 sin1 x are equal functions, then compute the
maximum range of x.
27

Q.19 Show that the roots r, s, and t of the cubic x(x 2)(3x 7) = 2, are real and positive. Also compute
the value of tan1(r) + tan1(s) + tan1(t).
2x 2 + 4

Q.20 Solve for x : sin1 sin


2 < 3.
1
+
x

EXERCISE3
Q.1

Q.2

The number of real solutions of tan1 x (x + 1) + sin1 x 2 + x + 1 =

is :
2

(A) zero

[JEE '99, 2 (out of 200)]

(B) one

(C) two

Using the principal values, express the following as a single angle :


1

1
5

3 tan1 + 2 tan1 + sin1


2
Solve, sin1

Q.4

Solve the equation:


cos1

( 6x) + cos (3
1

If

sin1 x

3x 2 =

[ REE '99, 6 ]

[REE 2000(Mains), 3 out of 100]

[ REE 2001 (Mains), 3 out of 100]

(B) 1

(C) 1/2

Q.6

Prove that cos tan1 sin cot 1 x =

Q.7

Domain of f (x) =
1 1
(A) ,
2 2

65 5

6
4

x2 x3

+
........ + cos1 x 2 x + x ........ = for 0 < | x | <
2
4

2
4

(A) 1/2

Q.8

142

ax
bx
+ sin1
= sin1x, where a2 + b2 = c2, c 0.
c
c

Q.3

Q.5

(D) infinite

sin 1 (2 x ) +

2 then x equals to

[JEE 2001(screening)]
(D) 1

x2 + 1
x2 + 2

[JEE 2002 (mains) 5]

is

1 3
(B) ,
4 4

1 1
(C) ,
4 4

1 1
(D) ,
4 2
[JEE 2003 (Screening) 3]

If sin cot 1 ( x + 1) = cos(tan 1 x ) , then x =


(A)

1
2

(B)

1
2

(C) 0

28

9
4
[JEE 2004 (Screening)]

(D)

INVERSE TRIGONOMETRY

EXERCISE1
Q 1. (i)
Q.3
Q 4.

Q5.

1
3

, (ii) 1, (iii)

17
4
, (iv) , (v) , (vi)
6
3
5
6

(d)
(i)
(iv)

(, sec 2) [1, )
1/3 x 1 (ii) {1, 1}
x (1/2 , 1/2), x 0

(vi)

{7/3, 25/9}

(i)

D : x R R : [/4 , )

(ii)

D: x n, n + x x = n + n I ;
4
2

(iii)

D: xR

3
, 1
2

Q 6.

Q.12 (a) x =

R : 0 ,

Q 8.
1
2

3
7

(b) x = 3

(e) x = 2 3 or 3 (f) x =
Q.13 57

Q.14 53

, (ii) 1, (iii) , (iv)

(iii) 1 < x < 4


(v)
(3/2 , 2]

(2, 2) {1, 0, 1}

(vii)

1
2

Q 2. (i)

(viii)

{xx = 2n +
2

, (v)

4
, (vi)
5

, n I}
6

R: ,
3 3
2

(iv)

D: xR

Q.11

R : ,
2 4

1
1
,
2
2
ab
(g) x =
1+ ab

(c) x = 0 ,

1
, y=1
2

2
3

Q 19. x = 1 ; y = 2 & x = 2 ; y = 7

(d) x =

3
10

Q.20

1 17
2

EXERCISE2
Q 4.

Q5. 6 cos2x

(b)

2n + 5

(d) arc tan (x + n) arc tan x (e)

Q 6.

(a)

Q 7.

(a) x = n n + 1 or x = n (b) x = ab (c) x =

Q 9. K = 2 ; cos

Q 12. k =

(c) arc cot

2
2
,1 & cos , 1
4
4

11
4

9
9
, so a = 6, b =
2
2

4
3

Q 10. 720

Q 8. (2 + 2) ( + )
Q.11

Q 14. (a) (cot 2 , ) ( , cot 3) (b)

X = Y= 3 a 2

F 2 , 1OP 2 , 1 1 , 2
GH 2 Q (c) 2 2

Q15. tan , cot 1


2

Q16. C1 is a bijective function, C2 is many to many correspondence, hence it is not a function

Q17. [e/6 , e]

Q 18.(a) D : [0, 1] , R : [0, /2] (b) x

Q.19

1
2

3
4

1
(c) D : [ 1, 1] , R : [0, 2]
2

Q.20 x (1, 1)

EXERCISE3
Q.1 C

Q.2

Q.3 x { 1, 0, 1}

Q.4 x =
29

1
3

Q.5 B

Q.7 D

Q.8 A

EXERCISE4 (Inv. Trigono.)


Part : (A) Only one correct option
1.
If cos 1 + cos1 + cos 1 v = 3 then + v + v is equal to
(A) 3
(B) 0
(C) 3
2.
Range of f(x) = sin1 x + tan1 x + sec1 x is
3

(A) ,
4 4

3.

3
(B) ,

4 4

(D) 1

(C) ,
4 4

(D) none of these

= 0 is
The solution of the equation sin1 tan sin1
x 6
4

(A) x = 2
(B) x = 4
(C) x = 4

(D) none of these

4.


The value of sin1 [cos{cos1 (cosx) + sin1 (sin x)}], where x , is
2

5.

(B)
(C)
(D)
2
4
4
2
The set of values of k for which x 2 kx + sin1 (sin 4) > 0 for all real x is
(A) { 0 }
(B) (2, 2)
(C) R
(D) none of these
sin1 (cos(sin1x)) + cos1 (sin (cos1x)) is equal to

(A)

6.

(B)

7.

2
2
cos1 x + 1 x . 1

x2
4

8.

(A) | x | 1
(B) x R
(C) 0 x 1
tan1 a + tan1 b, where a > 0, b > 0, ab > 1, is equal to

(A) 0

9.

1
2

(D)

3
4

x
= cos1 cos1x holds for
2

(D) 1 x 0

a+b
(B) tan1 1 ab

a+b
(C) + tan1

1 ab

a+b
(D) tan1

1 ab

The set of values of x for which the formula 2 sin1x = sin1 (2x

1 x 2 ) is true, is

1
3
3
1
,
,
(C)
(D)

2
2
2 2

The set of values of a for which x 2 + ax + sin1 (x 2 4x + 5) + cos1 (x 2 4x + 5) = 0 has at least one solution is

(A) ( ,
(C) R
11.

a+b

(A) tan1
1 ab

(A) ( 1, 0)
10.

(C)

(B) [0, 1]

2 ] [ 2 , )

All possible values of p and q for which cos1

(B) ( , 2 ) ( 2 , )
(D) none of these
p + cos1

1 p + cos1

1 q =

1
1
1
(B) q > 1, p =
(C) 0 p 1, q =
(D) none of these
2
2
2
1
1
If [cot x] + [cos x] = 0, where [.] denotes the greatest integer function, then complete set of values of x is
(A) (cos1, 1]
(B) (cot 1, cos 1)
(C) (cot1, 1]
(D) none of these
The complete solution set of the inequality [cot 1x]2 6 [cot1 x] + 9 0, where [.] denotes greatest integer
function, is
(A) ( , cot 3]
(B) [cot 3, cot 2]
(C) [cot 3, )
(D) none of these

(A) p = 1, q =

12.
13.

14.

1
1
1
1
tan + cos x + tan cos x , x 0 is equal to
4
2
4
2

(A) x
15.

3
holds, is
4

If

(B) 2x

(C)

2
x

(D)

x
2

1 1 3 sin 2
= , then tan is equal to
sin
5 + 4 cos 2 4
2

(A) 1/3

(B) 3

(D) 1

(C) 1
30

16.

(A)
17.

tan

19.

20.

(B)

cot

(C) tan

(D) cot

1 sin x + 1 + sin x
The value of cot 1
, < x < , is:
1 sin x 1 + sin x 2

(A)
18.

u
tan tan1 tan , then tan is equal to
4 2

If u = cot 1

x
2

(B)

x
+
2
2

(C)

x
2

(D) 2

x
2

The number of solution(s) of the equation, sin1x + cos1 (1 x) = sin1 ( x), is/are
(A) 0
(B) 1
(C) 2
(D) more than 2
1
1
2
+ tan 1
= tan 1 2 is
The number of solutions of the equation tan 1
x
2x + 1
4x + 1
(A) 0
(B) 1
(C) 2
(D) 3

If tan1

1
1
1
1
+ tan1
+ tan1
+ .......+ tan1
= tan1 , then is equal to
1 + n(n + 1)
1 + 2 .3
1 + 3 .4
1+ 2

n
(A) n + 2

n
(B) n + 1

(C)

n +1
n

(D)

1
n

n
> , n N, then the maximum value of n is:
6

21.

If cot1

22.

(A) 1
(B) 5
(C) 9
(D) none of these
The number of real solutions of (x, y) where, y = sin x, y = cos1 (cos x), 2 x 2, is:
(A) 2
(B) 1
(C) 3
(D) 4

1
1
The value of cos cos 1 is equal to
8
2
(B) 3/4
(C) 1/16
(A) 3/4
Part : (B) May have more than one options correct

23.

24.

, and are three angles given by

= 2tan1 ( 2 1), = 3sin1


25.

26.

(A) >
cos1x = tan1x then

The sum

n =1

tan 1

(A) tan 1 2 + tan 1 3


28.
29.

30.

1
2

+ sin1 and = cos1

(B) >

1
. Then
3

(C) <

(D) >

5 + 1
5 1
5 1

(B) x 2 =
(C) sin (cos1x) =
(D) tan (cos1x) =
(A) x 2 =

2
2
2

For the equation 2x = tan (2 tan 1 a) + 2 tan (tan 1 a + tan 1 a3), which of the following is invalid?
(A) a2 x + 2a = x
(B) a2 + 2 ax + 1 = 0 (C) a 0
(D) a 1, 1

27.

(D) 1/4

4n
n 4 2n 2 + 2

is equal to:

(D) sec 1 2

(C) /2

(B) 4 tan 1 1
1

If the numerical value of tan (cos (4/5) + tan (2/3)) is a/b then
(A) a + b = 23
(B) a b = 11
(C) 3b = a + 1
If satisfies the inequation x 2 x 2 > 0, then a value exists for
(A) sin1
(B) cos1
(C) sec 1

x
2

If f (x) = cos1x + cos1 +

(D) 2a = 3b
(D) cosec1

3 3x 2 then:
2

2
(A) f =
3 3

2
(B) f = 2 cos1
3
3

3

(C) f =
3
3

1
(D) f = 2 cos1 m
3
3

3
31

5 1

EXERCISE8
1.

Find the value of the following :


(i)

1 1
sin sin
2
3

(iii)

1 3

sin1 cos sin
2

(ii)

2.

Solve the equation : cot1 x + tan1 3 =

3.

x 1
x +1

+ tan1
=
Solve the equation : tan1
x 2
x +2
4
Solve the following equations :

4.

5.

6.
7.

(i)

1 x
1
=
tan1x , (x > 0)
tan1
1+ x
2

(ii)

1
1
tan1 = tan1
3tan1

x
2+ 3

9.

1

3

2
1

1
1 2 x
1 1 y
+ cos
Find the value of tan sin
, if x > y > 1.
2
2
1+ x 2
2
1 + y

1
1 4
then find the relation between x and y .
If x = sin (2 tan12) and y = sin tan
2
3

If arc sinx + arc siny + arc sinz = then prove that:(x, y, z > 0)

(i)
8.

1
1

tan cos 1 + tan 1

2
3

x 1 x 2 + y 1 y 2 + z 1 z 2 = 2xyz

(ii)
x 4 + y4 + z4 + 4 x 2y2z2 = 2 (x 2 y2 + y 2 z 2 + z2x 2)
Solve the following equations :
x
x
sec1
= sec 1b sec1a a 1; b 1, a b .
a
b

(i)

sec 1

(ii)

sin 1

(iii)

Solve for x, if (tan1x)2 + (cot 1 x)2 =

x
x 1
1
sin 1
= sin 1
1+ x
x +1
1+ x
5 2
8

1+ x
& = sin1 1 x for 0 < x < 1, then prove that + = . What the value of + will be if
If = 2 tan1
1+ x2
1 x

x>1?
10.
11.

12.

If X = cosec tan1 cos cot 1 sec sin1 a & Y = sec cot 1 sin tan1 cosec cos1 a; where 0 a 1. Find the relation
between X & Y. Express them in terms of 'a'.
Solve the following inequalities:
(i)
cos 1 x > cos 1 x 2
(ii)
sin1 x > cos1 x
1
1
(iii)
tan x > cot x.
(iv)
sin1 (sin 5) > x 2 4x.
(v)
tan2 (arc sin x) > 1
(vi)
arccot2 x 5 arccot x + 6 > 0
(vii)
tan 1 2 x 2 tan 1 x
Find the sum of each of the following series :
(i)

cot 1

31
139
319
2 5
+ cos1
+ cot 1
+ ... + cot1 3n .
12
12
12
12

(ii)

tan1

1
2
2n 1
+ tan1 + ..... + tan1
+ .....................
3
9
1 + 2 2n1

1
.
32
1
1
Find all positive integral solutions of the equation, tan x + cot y = tan1 3.
If 'k' be a positive integer, then show that the equation:
32 solution.
tan1 x + tan1 y = tan1 k has no nonzero integral

13.

Prove that the equation, (sin1x)3 + (cos1x)3 = 3 has no roots for <

14.

(i)
(ii)

FUNCTIONS (ASSERTION AND REASON)


Some questions (AssertionReason type) are given below. Each question contains Statement 1 (Assertion)
and Statement 2 (Reason). Each question has 4 choices (A), (B), (C) and (D) out of which ONLY ONE is
correct. So select the correct choice :
Choices are :
(A) Statement 1 is True, Statement 2 is True; Statement 2 is a correct explanation for Statement 1.
(B) Statement 1 is True, Statement 2 is True; Statement 2 is NOT a correct explanation for
Statement 1.
(C) Statement 1 is True, Statement 2 is False.
(D) Statement 1 is False, Statement 2 is True.
1.
Let f(x) = cos3x + sin 3x .
Statement 1 : f(x) is not a periodic function.
Statement 2 : L.C.M. of rational and irrational does not exist
2.
Statement 1: If f(x) = ax + b and the equation f(x) = f 1(x) is satisfied by every real value of x, then
aR and b = 1.
Statement 2: If f(x) = ax + b and the equation f(x) = f 1(x) is satisfied by every real value of x, then a
= 1 and bR.
x2
3.
Statements-1: If f(x) = x and F(x) =
, then F(x) = f(x) always
x
Statements-2: At x = 0, F(x) is not defined.
1
4.
Statement1 : If f(x) =
, x 0, 1, then the graph of the function y = f (f(f(x)), x > 1 is a straight
1 x
line
Statement2 : f(f(x)))) = x
5.
Let f(1 + x) = f(1 x) and f(4 + x) = f(4 x)
Statement1 : f(x) is periodic with period 6
Statement2 : 6 is not necessarily fundamental period of f(x)
6.
Statement1 : Period of the function f(x) = 1 + sin 2x + e{x} does not exist
Statement2 : LCM of rational and irrational does not exist
1
7.
Statement1 : Domain of f(x) =
is (, 0) Statement2:| x | x > 0 for x R
| x | x
8.
9.
10.
11.
12.

13.
14.

Statement1 : Range of f(x) = 4 x 2 is [0, 2]


Statement2 : f(x) is increasing for 0 x 2 and decreasing for 2 x 0.
a+x
Let a, b R, a b and let f(x) =
.
b+x
Statement1 : f is a oneone function.
Statement2 : Range of f is R {1}
Statement1 : sin x + cos (x) is a nonperiodic function.
Statement2 : Least common multiple of the periods of sin x and cos (x) is an irrational number.
Statement1: The graph of f(x) is symmetrical about the line x = 1, then, f(1 + x) = f(1 x).
Statement2 : even functions are symmetric about the y-axis.
x
x
Statement1 : Period of f(x) = sin
+ cos
is 2(n)!
n!
( n 1)!
Statement2 : Period of |cos x| + |sin x| + 3 is .
Statement2 : ?
Statement1 : Number of solutions of tan(|tan1x|) = cos|x| equals 2
Statement1 : Graph of an even function is symmetrical about yaxis
Statement2 : If f(x) = cosx has x (+)ve solution then total number of solution of the above equation is
2n. (when f(x) is continuous even function).
34
33

15.

16.
17.

18.

19.
20.
21.

22.

23.

24.

25.
26.

27.

28.

If f is a polynomial function satisfying 2 + f(x).f(y) = f(x) + f(y) + f(xy) x, yR


Statement-1: f(2) = 5 which implies f(5) = 26
Statement-2: If f(x) is a polynomial of degree 'n' satisfying f(x) + f(1/x) = f(x). f(1/x), then
f(x) = 1 xn + 1
Statement-1: The range of the function sin-1 + cos-1x + tan-1x is [/4, 3/4]
Statement-2: sin-1x, cos-1x are defined for |x| 1 and tan-1x is defined for all 'x'.
0 where x is rational
A function f(x) is defined as f(x) =
1 where x is irrational
Statement-1 : f(x) is discontinuous at xll xR
Statement-2 : In the neighbourhood of any rational number there are irrational numbers and in the
vincity of any irrational number there are rational numbers.
Let f(x) = sin 2 3 x + cos 3 3 x

Statement-1 : f(x) is a periodic function


Statement-2: LCM of two irrational numbers of two similar kind exists.
Statements-1: The domain of the function f(x) = cos-1x + tan-1x + sin-1x is [-1, 1]
Statements-2: sin-1x, cos-1x are defined for |x| 1and tan-1x is defined for all x.
Statement-1 : The period of f(x) = = sin2x cos [2x] cos2x sin [2x] is 1/2
Statements-2: The period of x [x] is 1, where [] denotes greatest integer function.
Statements-1: If the function f : R R be such that f(x) = x [x], where [] denotes the greatest integer
less than or equal to x, then f-1(x) is equals to [x] + x
Statements-2: Function f is invertible iff is one-one and onto.
Statements-1 : Period of f(x) = sin 4 {x} + tan [x] were, [] & {} denote we G.I.F. & fractional part
respectively is 1.
Statements-2: A function f(x) is said to be periodic if there exist a positive number T independent of x
such that f(T + x) = f(x). The smallest such positive value of T is called the period or fundamental
period.
x +1
is one-one function
Statements-1: f(x) =
x 1
x +1
Statements-2:
is monotonically decreasing function and every decreasing function is one-one.
x 1
Statements-1: f(x) = sin2x (|sinx| - |cosx|) is periodic with fundamental period /2
Statements-2: When two or more than two functions are given in subtraction or multiplication form we
take the L.C.M. of fundamental periods of all the functions to find the period.
Statements-1: ex = lnx has one solution.
Statements-2: If f(x) = x f(x) = f1(x) have a solution on y = x.
Statements-1: F(x) = x + sinx. G(x) = -x
H(x) = F(X) + G(x), is a periodic function.
Statements-2: If F(x) is a non-periodic function & g(x) is a non-periodic function then h(x) = f(x)
g(x) will be a periodic function.
x + 1, x 0
Statements-1: f (x) =
is an odd function.
x 1, x < 0
Statements-2: If y = f(x) is an odd function and x = 0 lies in the domain of f(x) then f(0) = 0
x; x Q
is one to one and non-monotonic function.
Statements-1: f (x) =
C
x; x Q
Statements-2: Every one to one function is monotonic.

35
34

29.

30.

31.
32.

33.
34.

35.
36.

37.
38.

x + 4, x [1, 2]
then the
Statement1 : Let f : [1, 2] [5, 6] [1, 2] [5, 6] defined as f (x) =
x + 7, x [5, 6]
equation f(x) = f1(x) has two solutions.
Statements-2: f(x) = f1(x) has solutions only on y = x line.
px + q
Statements-1: The function
(ps qr 0) cannot attain the value p/r.
rx + s
q sy
is all real except a/c.
Statements-2: The domain of the function g(y) =
ry p
Statements-1: The period of f(x) = sin [2] xcos [2x] cos2x sin [2x] is 1/2
Statements-2: The period of x [x] is 1.
b
Statements-1: If f is even function, g is odd function then (g 0) is an odd function.
g
Statements-2: If f(x) = f(x) for every x of its domain, then f(x) is called an odd function and if f(x)
= f(x) for every x of its domain, then f(x) is called an even function.
Statements-1: f : A B and g : B C are two function then (gof)1 = f1 og1.
Statements-2: f : A B and g : B C are bijections then f1 & g1 are also bijections.

Statements-1: The domain of the function f (x) = log 2 sin x is (4n + 1) , n N.


2
Statements-2: Expression under even root should be 0

Statements-1: The function f : R R given f (x) = log a (x + x 2 + 1) a > 0, a 1 is invertible.


Statements-2: f is many one into.

Statements-1: (x) = sin (cos x) x 0, is a one-one function.
2

Statements-2: '(x) x 0,
2
Statements-1: For the equation kx2 + (2 k)x + 1 = 0 k R {0} exactly one root lie in (0, 1).
Statements-2: If f(k1) f(k2) < 0 (f(x) is a polynomial) then exactly one root of f(x) = 0 lie in (k1, k2).
1+ x2
Statements-1: Domain of f (x) = sin 1
is {1, 1}
2x

1
1
2 when x > 0 and x + 2 when x < 0.
x
x
Statements-1: Range of f(x) = |x|(|x| + 2) + 3 is [3, )
Statements-2: If a function f(x) is defined x R and for x 0 if a f(x) b and f(x) is even function
than range of f(x) f(x) is [a, b].
Statements-1: Period of {x} = 1.
Statements-2: Period of [x] = 1
1
Statements-1: Domain of f = . If f(x) =
[x] x
Statements-2: [x] x x R
Statements-1: The domain of the function sin1x + cos1x + tan1x is [1, 1]
Statements-2: sin1x, cos1x are defined for |x| 1 and tan1x is defined for all x
Statements-2: x +

39.

40.
41.

42.

ANSWER KEY
1. A
8. C
15. A

2. D
9. B
16. A

3. A
10. C
17. A

4. C
11. A
18. A

5. A
12. C
19. A
36
35

6. A
13. B
20. A

7. A
14. A
21. D

22. A
29. C
36. A

23. A
30. A
37. C

24. A
31. A
38. A

25. D
32. A
39. A

26. C
33. D
40. A

27. D
34. A
41. A

28. C
35. C
42. A

SOLUTIONS
4.

5.

6.
7.
8.

9.

1
1
x 1
=
=
1 f (x) 1 1
x
1 x
1
1
f(f(f(x))) =
Ans. C
=
=x
x
1 f (f (x)) 1 1
x
f(1 + x) = f(1 x)
... (1)
f(4 + x) = f(4 x)
... (2)
x 1 x in (1) f(1 x) = f(x)
... (3)
x 4 x in (2) f(2 x) f(8 x) = f(x) ... (4)
(1) and (4) f(2 x) = f(8 x)
.... (5)
Use x x x in (5), we get
f(x) = f(6 + x)
f(x) is periodic with period 6
Obviously 6 is not necessary the fundamental period.
Ans. A
L.C.M. of {, 1} does not exist
(A) is the correct option.
(a)
Clearly both are true and statement II is correct explantion of Statement I .
(c)
x
f (x) =
4 x2
f(x) is increasing for 2 x 0 and decreasing for 0 x 2.

f(f(x)) =

Suppose a > b. Statement II is true as f (x) =

ba

( b + x )2

, which is always negative and hence monotonic

in its continuous part. Also lim + f (x) = and lim f (x) = . Moreover
x b

x b

lim f (x) = 1 + and lim f (x) = 1 . Hence range of f is R {1}.


x

10.

11.

F is obviously oneone as f(x1) = f(x2) x1 = x2.


However statement II is not a correct reasoning for statement I
Hence (b) is the correct answer.
Statement I is true, as period of sin x and cos x are 2 and 2 respectively whose L.C.M does not exist.
Obviously statement II is false
Hence (c) is the correct answer.
Graph of f(x) is symmetric about the line x = 0 if f(- x) = f(x) i.e. if f(0 x) = f(0 + x)
Graph of y = f(x) is symmetric about x = 1, if f(1 + x) = f(1 x).
Hence (a) is the correct answer.

37
36

12.

13.

14.
19.
20.

21.

22.
23.

24.

27.
28.

x
= 2 ( n 1) !
( n 1)!
x
Period of cos
= 2 ( n )!
n!
Period of f(x) = L.C.M of 2(n 1)! And 2(n)! = 2(n!)
Now,
f(x) = | copsx | + | sin x | +3 = 1+ | sin 2x | + 3

f(x) is periodic function with period = .


2
Hence (c) is the correct answer.
tan(|tan1x|) = |x|, since |tan1x| = tan1|x|
Obviously cos|x| and |x| meets at exactly two points
(B) is the correct option.
(A)Since cos n is also even function. Therefore solution of cosx = f(x) is always sym. also out yaxis.
(a) Both A and R are obviously correct.
(a)
f(x) = x [x]
f(x + 1) = x + 1 ([x] + 1) = x [x]
So, period of x [x] is 1.
Let f(x) = sin (2x [2x])

1
1
1

f x + = sin 2 x + 2 x +
2
2
2


= sin (2x + 1 [2x] 1)
= sin (2x [2x])
So, period is 1/2
f(1) = 1 1 = 0 f(0) = 0
f is not one-one
f-1(x) is not defined Ans. (D)
Clearly tan [x] = 0 xR and period of sin 4 {x} = 1. Ans. (A)
(x 1) (x + 1)
2
x +1
f(x) =
f(x) =
=
<0
2
(x 1)
(x 1) 2
x 1
So f(x) is monotonically decreasing & every monotonic function is one-one.
So a is correct.
f(x) = sin2x (|sinx| -|cosx|) is periodic with period /2 because f(/2 + x) = sin 2 (/2 + x) (|sin (/2 + x)|
-|cos (/2 + x)|)
= sin ( + 2x) (|cosx| - |sinx|)
= -sin2x (|cosx| - |sinx|)
= sin2x (|sinx| - |cosx|)
Sometimes f(x + r) = f(x) where r is less than the L.C.M. of periods of all the function, but according to
definition of periodicity, period must be least and positive, so r is the fundamental period.
So f is correct.
(D) If f(x) is an odd function, then f(x) + f(x) = 0 x Df
(C) For one to one function if x1 x2
f(x1) f(x2) for all x1, x2 Df 3 > 1
Period of sin

but f ( 3) < f (1)


f(5) > f(1)

and 3 > 1
f(x) is one-to-one

38
37

but non-monotonic

29.

30.

31.

32.

33.
34.

35.

3 11
11 3
(C) , and , both lie on y = f(x) then they will also lie on y = f1(x) there are two
2 2
2 2
solutions and they do not lie on y = x.
q sx
px + q
then x =
x does not exist if y = p/r
If we take y =
rx + s
rx p
Thus statement-1 is correct and follows from statement-2 (A)

1

f(x) = sin(2x [2x]
f(x + 1/2) = sin 2x + 1 2 x +
2

= sin (2x + 1 [2x] 1]


= sin (2x [2x].) i.e., period is 1/2.
f(x) = x [x]
f(x + 1) = x + 1 ([x] + 1) = x [x]
i.e., period is 1.
(A)
f (x)
(A)
Let h(x) =
g(x)
f ( x) f (x)
f (x)
h(x) =
=
=
= h(x)
g( x) g( x) g(x)
f
h(x) =
is an odd function.
g
(D)
Assertion : f : A B, g : B C are two functions then (gof)1 f1 og1 (since functions need
not posses inverses. Reason : Bijective functions are invertibles.
(A) for f(x) to be real log2(sin x) 0

sin x 2
sin x = 1
x = (4n + 1) , n N.
2
(C) f is injective since x y (x, y R)

log a x + x 2 + 1 log a y + y 2 + 1
f(x) f(y)

f is onto because log a x + x 2 + 1 = y


40.

41.

Since {x} = x [x]


{x + 1} = x + 1 [x + 1]
= x + 1 [x] 1
= x [x] = [x]
Period of [x] = 1
1
f(x) =
[x] x 0
[x] x
[x] x [x] > x It is imposible or [x] x
So the domain of f is
because reason [x] x

x=

a y ay
.
2

Ans (A)

Ans. (A)

39
38

STUDY PACKAGE
Target: IIT-JEE (Advanced)
SUBJECT: MATHEMATICS
TOPIC: 19 XII M 2. Determinants and
Matrices
Index:
1. Key Concepts
2. Exercise I to X
3. Answer Key
4. Assertion and Reasons
5. 34 Yrs. Que. from IIT-JEE
6. 10 Yrs. Que. from AIEEE

1.

Definition:
Let us consider the equations a1x + b1y = 0, a2x + b2y = 0
a2
a1
a2
a
y

1 =
=

b2
b1
b2
b1
x
we express this eliminant as

a1b2 a2b1 = 0

a1 b1
=0
a2 b2

a1 b1
is called the determinant of order two.
a2 b2
Its value is given by:
D = a1 b2 a2 b1
E x p a n s io n of D et er m i n a n t :
a1 b1 c 1
The symbol a 2 b 2 c 2 is called the determinant of order three.

The symbol

2.

a3 b3 c 3
Its value can be found as:

D = a1

b2 c 2
b3 c 3

a2

b1

c1

b3

c3

+ a3

b1

c1

b2

c2

OR

a2 c 2
a2 b2
b2 c 2
b1
+ c1
... & so on.
a3 c 3
a3 b3
b3 c 3
In this manner we can expand a determinant in 6 ways using elements of ; R1, R2, R3 or C1, C2, C3.
Minors:
The minor of a given element of a determinant is the determinant of the elements which remain after
deleting the row & the column in which the given element stands. For example, the minor of a1 in
a1 b1 c 1
b2 c 2
a1 c 1
& the minor of b2 is
.
a 2 b 2 c 2 is
b3 c 3
a3 c 3
a3 b3 c 3
Hence a determinant of order two will have 4 minors & a determinant of order three will have 9
minors.
Cofactor:
Cofactor of the element aij is Cij = (1) i+j. Mij ; Where i & j denotes the row & column in which the
particular element lies.
Note that the value of a determinant of order three in terms of Minor & Cofactor can be written as:
D = a11M11 a12M12 + a13M13
OR
D = a11C11 + a12C12 + a13C13 & so on.
Tran s pose of a D et erm in an t :
The transpose of a determinant is a determinant obtained after interchanging the rows & columns.
a1 b1 c 1
a1 a 2 a 3

D = a1

3.

4.

5.

6.

7.

DT =
b1 b 2 b 3
D = a2 b2 c 2
a3 b 3 c 3
c1 c 2 c 3
Sy m m et ri c , S kew -S y m m e t r ic , A sy m m et ri c D et er m i n a nt s:
(i)
A determinant is symmetric if it is identical to its transpose. Its i th row is identical to its i th
column i.e. aij = aji for all values of ' i ' and ' j '
(ii)
A determinant is skew-symmetric if it is identical to its transpose having sign of each element
inverted i.e. aij = aji for all values of ' i ' and ' j '. A skew-symmetric determinant has all elements
zero in its principal diagonal.
(iii)
A determinant is asymmetric if it is neither symmetric nor skew-symmetric.
Prop ert ies of D et erm in a n t s:
(i)
The value of a determinant remains unaltered, if the rows & columns are inter changed,
a1 b1 c 1
a1 a 2 a 3
i.e.
D = a 2 b 2 c 2 = b1 b 2 b 3 = D
a3 b3 c 3
c1 c 2 c 3
(ii)
If any two rows (or columns) of a determinant be interchanged, the value of determinant
is changed in sign only. e.g.
a2 b2 c 2
a1 b1 c 1

Then D = D.
Let D = a 2 b 2 c 2 & D = a1 b1 c 1
a3 b3 c 3
a3 b3 c 3
NOTE : A skew-symmetric deteminant of odd order has value zero.
2

54
DET. & MATRICES/Page : 2 of

Determinant

D = a 2 b 2 c 2 = 0.
a3 b3 c 3
If a determinant has any two rows (or columns) identical, then its value is zero,
a1 b1 c1

i.e.
(iv)

D = a1 b1 c 1 = 0.
a3 b3 c 3
If all the elements of any row (or column) be multiplied by the same number, then the determinant
is multiplied by that number, i.e.
a1 b1 c 1
Ka1 Kb1 Kc 1
c2
and D = a 2 b 2
Then D= KD
D = a2 b2 c 2
i.e.

(v)

(vi)

(vii)

a3 b3 c 3
a3
b3
c3
If each element of any row (or column) can be expressed as a sum of two terms then the
determinant can be expressed as the sum of two determinants, i.e.
a1+x b1+y c 1+z
a1 b1 c 1
x
y
z
a2
b2
c 2 = a2 b2 c 2 + a2 b2 c 2
a3
b3
c3
a3 b3 c 3
a3 b3 c 3
The value of a determinant is not altered by adding to the elements of any row (or column) a
constant multiple of the corresponding elements of any other row (or column),
a1 + ma 2 b1 + mb 2 c 1 + mc 2
a1 b1 c 1
a2
b2
c2
. Then D = D.
i.e. D = a 2 b 2 c 2
and D =
a 3 + na1 b 3 + nb1 c 3 + nc 1
a3 b3 c 3
a b c

Example :

b c a

Simplify

c a b
a+b+c a+b+c a+b+c

Solution.

R1 R 1 + R 2 + R 3

Let

1 1 1

Example :

Solution.

= (a + b + c) b c a
c a b
Apply C1 C1 C2, C2 C2 C3
0
0
1
b

c
c

a
a
= (a + b + c)
c a ab b
= (a + b + c) ((b c) (a b) (c a)2)
= (a + b + c) (ab + bc ca b2 c2 + 2ca a2)
= (a + b + c) (ab + bc + ca a2 b2 c2)
= 3abc a3 b3 c3
a b c
a2 b2 c 2
Simplify
bc ca ab

Given detereminant is equal to


a2

abc
=
abc

b2

c2

a3 b3 c 3
1 1 1
Apply C1 C1 C2,

1
=
abc

C2 C2 C 3

a 2 b2
a 3 b3

b2 c 2
b3 c 3

c2
c3

1
3

a2

b2

c2

a3

b3

c3

abc abc abc

54

If a determinant has all the elements zero in any row or column then its value is zero,
0 0 0

DET. & MATRICES/Page : 3 of

(iii)

b+c

54

c2

3
= (a b) (b c) a + ab + b b + bc + c c
0
0
1
= (a b) (b c) [ab2 + abc + ac2 + b3 + b2C + bc2 a2b a2c ab2 abc b3 b2c]
= (a b) (b c) [c(ab + bc + ca) a(ab + bc + ca)]
= (a b) (b c) (c a) (ab + bc + ca)
Use of factor theorem.
USE OF FACTOR THEOREM TO FIND THE VALUE OF DETERMINANT
If by putting x = a the value of a determinant vanishes then (x a) is a factor of the determinant.
a b c
2
2
2
= (a b) (b c) (c a) (ab + bc + ca) by using factor theorem.
Example :
Prove that a b c
bc ca ab
Solution.
Let a = b
a b c
2
2
2
=0
Hence (a b) is a factor of determinant

D= a b c
bc ac ab
Similarly, let b = c, D = 0
c = a, D = 0
Hence, (a b) (b c) (c a) is factor of determinant. But the given determinant is of fifth
order so
a b c
a 2 b 2 c 2 = (a b) (b c) (c a) ( (a2 + b2 + c2) + (ab + bc + ca))
bc ca ab
Since this is an identity so in order to find the values of and . Let
a = 0, b = 1, c = 1
2 = (2) (2 )
(2 ) = 1.
........(i)
Let a = 1, b = 2, c = 0
1 2 0
1 4 0 = (1) 2 ( 1) (5 + 2)
0 0 2

5 + 2 = 2
.......(ii)
from (i) and (ii) = 0 and = 1
a b c
2

Hence a 2 b 2 c 2 = (a b) (b c) (c a) (ab + bc + ca).


bc ca ab
Self Practice Problems
0
ba c a
0
c b .
1.
Find the value of = a b
ac bc

Ans.

Ans.

b 2 ab b c bc ac

2.

2
2
Simplify ab a a b b ab .
bc ac c a ab a 2

abc

2a

2a

2b

bc a

2b

2c

2c

c ab

= (a + b + c)3.

3.

Prove that

4.

1 a bc
Show that 1 b ca = (a b) (b c) (c a) by using factor theorem .

8.

1 c ab
M u l t i p li c a t io n O f T wo D e t e r m i n a n t s :
a1

b1

a2

b2

 1 m1
a  +b 
= 1 1 1 2
 2 m2
a 2  1+b 2  2

a1m1+b1m 2
a 2 m1+b 2 m 2

a1

b1

c1

1

m1

n1

a1 1 + b1 2 + c 1 3

a1m1 + b1m 2 + c 1m3

a2

b2

n2 = a 2  1 + b 2  2 + c 2 3
a3  1 + b 3  2 + c 3  3
n3

a 2m1 + b 2m 2 + c 2m 3

a 2n1 + b 2n2 + c 2n3

b3

c2  2
c3
3

m2

a3

a 3m1 + b3m 2 + c 3m 3

a3 n1 + b3 n2 + c 3n3

m3

a1n1 + b1n2 + c 1n3

DET. & MATRICES/Page : 4 of

a+b
2

Solution.

1 3

3 0
1 4

1 3 2 1

1 0 + 2 4

1 3 + 3 ( 1) 1 0 + 3 4

1
8
6 12

= 60
a1x1 + b1y1

Example :
Solution.

a1x 2 + b1y 2

a 3 x1 + b 3 y1 a 3 x 2 + b 3 y 2

Solution.

(a1 b1 )2

( a1 b 2 ) 2

(a1 b 3 )2

(a 2 b1 )2

(a 2 b 2 ) 2

(a 2 b 3 )2

(a 3 b1 )2

(a 3 b 2 ) 2

(a 3 b 3 )2

a3

b3

c3

a1 + b1 2a1b1 a1 + b 2 2a1b 2
2
2
2
2
a 2 + b1 2a 2b1 a 2 + b 2 2a 2b 2
a 3 2 + b12 2a 3b1 a 3 2 + b 2 2 2a 3b 2
2

a1 1 2a1
2
= a 2 1 2a 2
2
a3 1 2a 3

1
1
2
2
b1 b 2
b1 b 2

1 a12

a1

1 b12

b1

a2
a3

1 b2
2
1 b3

b2
b3

= 2 1 a2
2
1 a3

Example :

a3 x 3 + b3 y 3

(a1 b1 )
(a1 b 2 )
( a1 b 3 )
Prove that (a 2 b1 )2 (a 2 b 2 )2 (a 2 b 3 )2
( a 3 b1 ) 2 ( a 3 b 2 ) 2 ( a 3 b 3 ) 2
= 2(a1 a2) (a2 a3) (a3 a1) (b1 b2) (b2 b3) (b3 b1).
2

Example :

a1x 3 + b1y 3

Prove that a 2 x1 + b 2 y1 a 2 x 2 + b 2 y 2 a 2 x 3 + b 2 y 3 = 0
a 3 x1 + b 3 y1 a 3 x 2 + b 3 y 2 a 3 x 3 + b 3 y 3
Given determinant can be splitted into product of two determinants
x1 x 2
a1x1 + b1y1 a1x 2 + b1y 2 a1x 3 + b1y 3
a1 b1 c 1
a 2 x 1 + b 2 y 1 a 2 x 2 + b 2 y 2 a 2 x 3 + b 2 y 3 = a 2 b 2 c 2 y1 y 2
i.e.

a1 + b 3 2a1b 3
2
2
a 2 + b 3 2a 2b 3
a 3 2 + b 3 2 2a 3b 3

1
2
b3
b3

= 2(a1 a2) (a2 a3) (a3 a1) (b1 b2) (b2 b3) (b3 b1)
cos( A P) cos( A Q) cos( A R)
Prove that cos(B P) cos(B Q) cos(B R) = 0
cos(C P) cos(C Q) cos(C R)
cos( A P) cos( A Q) cos( A R)
cos(B P) cos(B Q) cos(B R)
cos(C P) cos(C Q) cos(C R)

Solution.

cos A cos P + sin A sin P cos A cos Q + sin A sin Q cos A cos R + sin A sinR
cos B cos P + sinB sinP cos B cos Q + sinB sin Q cos B cos R + sinB sinR
cos C cos P + sin C sinP cos C cos Q + sin C sin Q cos C cos R + sin C sin R

cos A sin A 0
cos P cos Q cos R
cos B sinB 0
sinP sin Q sinR
=

cos C sin C 0
0
0
0

= 0 0 = 0.

x3
y3 = 0
0

54
DET. & MATRICES/Page : 5 of

We have multiplied here rows by rows but we can also multiply rows by columns, columns by rows and
columns by columns.
If = |aij| is a detereminant of order n, then the value of the determinant |Aij | = n 1. This is also known
as power cofactor formula.
1 2
1
8
3 0
Example :
Find the value of

.
and prove that it is equal to
1 3
6 12
1 4

2bc a 2
c2
b2

2.
9.

c2
2ca b 2
a2

b2
a2
2ab c 2

(3abc a3 b3 c3)2

Ans.

1
cos(B A ) cos(C A )
cos(
A

B
)
1
cos(C B) . Ans.0
If A, B, C are real numbers then find the value of =
cos( A C) cos(B C)
1
Su m m a t i on o f D et erm in a n ts
f(r) g(r ) h(r )

Let (r) = a1 a 2 a 3
b1 b 2 b 3
where a1, a2, a3, b1, b2, b3 are constants indepedent of r, then
n

f (r )

r =1

(r ) =
r =1

g(r )

r =1

h(r )

r =1

a1

a2

a3

b1

b2

b3

Here function of r can be the elements of only one row or column. None of the elements other then that
row or column should be dependent on r. If more than one column or row have elements dependent on
r then first expand the determinant and then find the summation.
2r 1

Example :

Evaluate

cos 2
y
2n 1 2n +1 2

x
n2

r =1

Solution :

r =1

r =1

r =1

Cr

cos 2

n2

2n 1

y
2n+1 2

=0

2n 1 2n+1 2
n 2

Dr =

n2

Cr 2

r =1

2n 1 2n+1 2
cos2
y

n2
= x
n2

Example :

(2r 1) n

2r

nCr

n2

Cr 1

Cr

evaluate

r =2

n 2
n

Solution :

n 2

n2

3
2

r =2

r=2

Cr 2

n 2

Cr 1
1
1

C 0 + n 2C1 + .... + n 2Cn2


3

Cr

1
0
n 2

C1 + n 2C 2 + .... + n 2Cn2
1

2
2n 2

n 2

C 2 + n2C 3 + .... + n 2Cn2


1

2 n 2 1 2 n 2 1 n

0
2n2 2n1 + 2 2n2 1 2n 2 1 n

C1 C1 2 C 2

DET. & MATRICES/Page : 6 of

54

Self Practice Problems


1.
Find the value of

Example :
Solution.

3 + r , find
r
r
=
1
r +1 1 2
On expansion of determinent, we get

If r =

Dr = (r 1) (3 r) + 7 + r2 + 4r = 8r + 4

= 4n (n + 2)

r =1

Self Practice Problem


r 1

(r 1)

4n 2

1.

Evaluate

Dr

r =1

10.

Ans.

(r 1) z 3n 3n
I n teg ra t ion of a d et erm in an t
3

f ( x ) g( x ) h( x )

Let

(x) =

a1

b1

c1

a2
b2
c2
where a1, b1, c1, a2, b2, c 2 are constants independent of x. Hence
b

( x ) dx =

f ( x ) dx

h( x) dx

g( x ) dx

a1
a2

b1
b2

c1
c2

Note : If more than one row or one column are function of x then first expand the determinant and then
integrate it.
/2
cos x
1
0
f ( x ) dx
1
2 cos x
1
, then find
Example :
If f(x) =

Solution.

/2

/2

so

cos 3 x dx =

sin 3 x
3 0

1
3

2 1 2 2 2 3

Example :

If =

x2

x3

Solution.

( x) dx
0

2 1
6

x dx

11.

0
1
2 cos x
Here f(x) = cos x (4 cos2x 1) 2 cos x
= 4 cos3x 3 cos x = cos 3x

, then find

2 2
4

x 2 dx

2 1 2 2 2 3
6
4
3
=
1
1
1
2
3
4
D i ff er en t i a t io n of D et er m i n a n t :
f1( x ) f2 ( x ) f3 ( x )

(x) dx
0

2 3
3

dx

1
12

2 1 2 2 2 3
4
3
6
6
4
3

Let (x) = g1( x ) g2 ( x ) g3 ( x )


h1( x ) h2 ( x ) h3 ( x )

=0

DET. & MATRICES/Page : 7 of

54

2n 2 2n1 + 2 2n2 1 n
1
1
= 2n 1 n 3
r 1 1
0
n

= (1)

f2 ( x )

f3 ( x )

Example :

3
2
If f(x) = 6 x
1

Solution.

3
2
2
f(x) = 12x 6 x
1
a

Example :

Solution.

2
2x 3
a

54

f1( x ) f2 ( x ) f3 ( x )
f1( x ) f2 ( x ) f3 ( x )
then (x) = g1( x ) g2 ( x ) g3 ( x ) + g1 ( x ) g2 ( x ) g3 ( x ) + g1( x ) g2 ( x ) g3 ( x )
h1( x ) h2 ( x ) h3 ( x )
h1 ( x ) h2 ( x ) h3 ( x )
h1( x ) h2 ( x ) h3 ( x )
1
x 4 , then find the value of f(a).
a2
1
4x 3
a2

3
2
1
3 2 1
2
2
12
12
x
12
x
f(x) =

f(a) = 12 1 a a = 0.
1
a
a2
1 a a2
Let be a repeated root of quadratic equation f(x) = 0 and A(x), B(x) and C(x) be polynomial of
degree 3, 4 and 5 respectively, then show that
A( x ) B( x ) C( x )
A( ) B( ) C( )
divisible by f(x).
A( ) B( ) C( )
A( x ) B( x ) C( x )
A
( ) B( ) C( )
g(x) =
A( ) B( ) C( )

Let

A( x ) B( x ) C( x )
A( ) B( ) C( )
A( ) B( ) C( )
Since g() = g() = 0

g(x) = (x ) 2 h(x) i.e. is the repeated root of g(x) and h(x) is any polynomial
expression of degree 3. Also f(x) = 0 have repeated root . So g(x) is divisible by f(x).
Prove that F depends only on x 1, x 2 and x 3
1
1
1
x1 + a1
x 2 + a1
x 3 + a1
F=
x12 + b1x1 + b 2 x 22 + b1x 2 + b 2 x 32 + b1x 3 + b 2

Example :

Solution :

g(x) =

and simplify F.
0
dF
x1 + a1
=
da1
x12 + b1x1 + b 2

x 2 + a1

x 3 + a1

x 22 + b1x 2 + b 2

x 32 + b1x 3 + b 2

x12 + b1x1 + b 2 x 22 + b1x 2 + b 2


Hence F is independent of a1.
dF
dF
Similarly
=
= 0.
db 2
db1
Hence F is independent of b1 and b2 also.
So F is dependent only on x 1, x 2, x 3

Put a1 = 0, b1 = 0, b2 = 0

x 32 + b1x 3 + b 2

Example :
Solution :

If

sin x

cos x n(1 + x )
Put x = 0 in
ex

sin x

cos x n(1 + x )

F = x1
x12

x2

x3

x 22

x 32

= A + Bx + Cx 2 + ....., then find the value of A and B.

= A + Bx + Cx 2 + .......
8

+ x1 + a1 x 2 + a1 x 3 + a1 = 0
0
0
0

= (x 1 x 2) (x 2 x 3) (x 3 x 1).
x

DET. & MATRICES/Page : 8 of

f1( x )

54

1 0
=A
A = 0.
1 0
Differentiating the given determinant w.r.t x, we get

ex
sin x
ex
cos x
1
+ sin x
cos x n(1 + x )
1+ x
Put x = 0, we get
1 1
1 0
+
=0
1 0
0 1

B = 1 + 1 = 0

A = 0, B = 0
Self Practice Problem
x
x 1 x

1.

12.

= B + 2 C x + ......

2x x + 1 1
= ax 3 + bx 2 + cx + d. Find
x +1 1
x
(i)
d
Ans. [ 1]
(ii)
a+b+c+d
Ans. [ 5]
(iii)
b
Ans. [ 4]
Cra m er' s Ru le: Sy st em of Linea r Eq u a t ion s
(i)
Two Variables
(a)
Consistent Equations: Definite & unique solution. [ intersecting lines ]
(b)
Inconsistent Equation: No solution. [ Parallel line ]
(c)
Dependent equation: Infinite solutions. [ Identical lines ]
Let a1x + b1y + c1 = 0 & a2x + b2y + c 2 = 0 then:
a1
b
c
= 1 1
&
Given equations are inconsistent
a2
b2
c2

If

(ii)

(iii)
(a)

(b)
(c)

(d)
(e)

DET. & MATRICES/Page : 9 of

a1
b
c
= 1 = 1
a2
b2
c2
Three Variables

Given equations are dependent

Let,
a1x + b1y + c 1z = d1............ (I)
a2x + b2y + c 2z = d2............ (II)
a3x + b3y + c 3z = d3............ (III)
D1
D2
D3
,Y=
,Z=
.
Then, x =
D
D
D
a1 b1 c 1
d1 b1 c 1
a1 d1 c 1
a1 b1 d1
a2 b2 c 2
d2 b 2 c 2
a 2 d2 c 2
a 2 b 2 d2
Where D =
; D1 =
; D2 =
& D3 =
a3 b3 c 3
d3 b 3 c 3
a 3 d3 c 3
a 3 b 3 d3
Consistency of a system of Equations
If D 0 and alteast one of D1, D2, D3 0, then the given system of equations are consistent and
have unique non trivial solution.
If D 0 & D1 = D2 = D3 = 0, then the given system of equations are consistent and have trivial
solution only.
If D = D1 = D2 = D3 = 0, then the given system of equations have either infinite solutions or no
solution.

(Refer Example & Self Practice Problem with*)


If D = 0 but atleast one of D1, D2, D3 is not zero then the equations are inconsistent and have no solution.
If a given system of linear equations have Only Zero Solution for all its variables then the given equations
are said to have TRIVIAL SOLUTION.
(iv)
Three equation in two variables :
If x and y are not zero, then condition for a1x + b1y + c1 = 0 ; a2x + b2y + c2 = 0 &
a1 b1 c 1
a3x + b3y + c 3 = 0 to be consistent in x and y is a 2 b 2
a3 b3
9

c 2 = 0.
c3

Solution.

1 2 3

Now, D1 = 3 3 4
0 4 5
C3 C3 C2
1 2 1
D1 = 3 3 1

*Example :

Solution.

*Example :

Solution.

Example :

Solution.

0 4 1
R1 R1 R 2 , R 2 R2 R3
2 1 0
D1 = 3 1 0 = 5
0
4 1
D = 0 But D1 0
Hence no solution
Solve the following system of equations
x+y+z=1
2x + 2y + 2z = 3
3x + 3y + 3z = 4
1 1 1

D= 2 2 2 =0
3 3 3
D1 = 0, D2 = 0, D3 = 0

Let z = t
x+y=1t
2x + 2y = 3 2t
Since both the lines are parallel hence no value of x and y Hence there is no solution of the
given equation.
Solve the following system of equations
x+y+z=2
2x + 2y + 2z = 4
3x + 3y + 3z = 6
1 1 1
2 2 2
=0
3 3 3
D1 = 0, D2 = 0, D3 = 0
All the cofactors of D, D1, D2 and D3 are all zeros, hence the system will have infinite solutions.
Let z = t1, y = t2

x = 2 t1 t2
where t1, t2 R.
Consider the following system of equations
x+y+z=6
x + 2y + 3z = 10
x + 2y + z =
Find values of and if such that sets of equation have
(i)
unique solution
(ii)
infinite solution
(iii)
no solution
x+y+z=6
x + 2y + 3z = 10
x + 2y + z =
1 1 1
1 2 3
D=
1 2
Here for = 3 second and third rows are identical hence D = 0 for = 3.

D=

10

54

Find the nature of solution for the given system of equations.


x + 2y + 3z = 1
2x + 3y + 4z = 3
3x + 4y + 5z = 0
1 2 3
Let D = 2 3 4
3 4 5
apply C1 C1 C2 , C2 C2 C3
1 1 3
D = 1 1 4 = 0 D = 0
1 1 5

DET. & MATRICES/Page : 10 of

Example:

D2 =

D3 =

1 10 3
1
1 1 6
1 2 10

1 2
If = 3 then D1 = D2 = D3 = 0 for = 10
(i)
For unique solution D 0
i.e.
3
(ii)
For infinite solutions
D=0

=3
D1 = D2 = D3 = 0

= 10.
(iii)
For no solution
D=0

=3
Atleast one of D1, D2 or D3 is non zero

10.
Self Practice Problems
*1.
Solve the following system of equations
x + 2y + 3z = 1
2x + 3y + 4z = 2
3x + 4y + 5z = 3
Ans.
x=1+t
y = 2t
z=t
where t R
2.
Solve the following system of equations
x + 2y + 3z = 0
2x + 3y + 4z = 0
xyz=0
Ans.
x = 0, y = 0, z = 0
3.
Solve: (b + c) (y + z) ax = b c, (c + a) (z + x) by = c a, (a + b) (x + y) cz = a b
where a + b + c 0.
ac
ba
c b
,y=
,z=
Ans. x =
a+b+c
a +b+c
a +b+ c
4.
Let 2x + 3y + 4 = 0 ; 3x + 5y + 6 = 0, 2x 2 + 6xy + 5y2 + 8x + 12y + 1 + t = 0, if the system of equations
in x and y are consistent then find the value of t.
Ans. t = 7
13.
Ap p lic a t io n of D et erm i n an t s:
Following examples of short hand writing large expressions are:
(i)
Area of a triangle whose vertices are (x r, yr); r = 1, 2, 3 is:
x1 y1 1
1 x2 y2 1
If D = 0 then the three points are collinear.
D=
2 x
y 1
3

(ii)
(iii)

(iv)

x
y 1
Equation of a straight line passing through (x 1, y1) & (x 2, y2) is x1 y1 1 = 0
x2 y2 1
The lines:
a1x + b1y + c 1 = 0........ (1)
a2x + b2y + c 2 = 0........ (2)
a3x + b3y + c 3 = 0........ (3)
a1 b1 c 1
are concurrent if,
a 2 b 2 c 2 = 0.
a3 b 3 c 3
Condition for the consistency of three simultaneous linear equations in 2 variables.
ax + 2 hxy + by + 2 gx + 2 fy + c = 0 represents a pair of straight lines if:
a h g
h
b f
abc + 2 fgh af bg ch = 0 =
g f c

11

DET. & MATRICES/Page : 11 of

54

6 1 1
10 2 3
D1 =
2

Any rectangular arrangement of numbers (real or complex) (or of real valued or complex valued
expressions) is called a matrix. If a matrix has m rows and n columns then the order of matrix is said
to be m by n (denoted as m n).
The general m n matrix is
a11 a12 a13 ...... a1j ..... a1n

a 21 a 22 a 23 ...... a 2 j ..... a 2n
..... ..... ..... ..... ..... ..... .....

A=
ai1 ai2 ai3 ...... aij ...... ain
..... ..... ..... ..... ..... ..... .....

a m1 a m2 am3 ..... a mj ..... amn


where aij denote the element of i th row & j th column. The above matrix is usually denoted as [aij ] m n .
Note : (i)
The elements a11, a22, a33,........ are called as diagonal elements. Their sum is called as
trace of A denoted as T r(A)
(ii)
Capital letters of English alphabets are used to denote a matrix.
1.
Basic Definitions
(i)
Row matrix : A matrix having only one row is called as row matrix (or row vector). General form of row
matrix is A = [a11, a12, a13, ...., a1n]
Column matrix : A matrix having only one column is called as column matrix. (or column vector)
(ii)
a11

a 21
Column matrix is in the form A =
...

am1
(iii)
Squ are
m atri x
:
A
m at ri x
in
whi c h
num ber
of
rows
&
col um ns
a re
equal
is
cal l e d
a
square
m at ri x .
G eneral
form of a square matrix is
a11 a12 ....... a1n

a 21 a 22 ........ a 2n
A =
....... ....... ....... .......
a

n1 an2 ....... ann


which we denote as A = [aij ]n.
(iv)
Zero matrix : A = [aij ] m n is called a zero matrix, if aij = 0 i & j.
(v)
Upper triangular matrix :
A = [aij ] m n is said to be upper triangular, if aij = 0 for i > j (i.e., all the elements below the
diagonal elements are zero).
(vi)
Lower triangular matrix : A = [aij ]m n is said to be a lower triangular matrix, if aij = 0 for
i < j. (i.e., all the elements above the diagonal elements are zero.)
(vii)
Diagonal matrix : A square matrix [aij ] n is said to be a diagonal matrix if aij = 0 for i j.
(i.e., all the elements of the square matrix other than diagonal elements are zero)
Note : Diagonal matrix of order n is denoted as Diag (a11, a22, ......ann).
(viii) Scalar matrix :Scalar matrix is a diagonal matrix in which all the diagonal elements are same
A = [aij ] n is a scalar matrix, if (i) aij = 0 for i j and (ii) aij = k for i = j.
(ix)
Unit matrix (Identity matrix) :
Unit matrix is a diagonal matrix in which all the diagonal elements are unity. Unit matrix of
order 'n' is denoted by n (or ).
i.e.
A = [aij ]n is a unit matrix when aij = 0 for i j & aii = 1
1 0 0
1 0

eg.
2 =
, 3 = 0 1 0 .
0
1

0 0 1
(x)
Comparable matrices : Two matrices A & B are said to be comparable, if they have
the same order (i.e., number of rows of A & B are same and also the number of columns).
(xi)
Equality of matrices : Two matrices A and B are said to be equal if they are comparable and
all the corresponding elements are equal.
Let
A = [aij ] m n &
B = [bij ] p q
A = B iff
(i)
m = p, n = q
aij = bij i & j.
(ii)
(xii)
Multiplication of matrix by scalar :
Let be a scalar (real or complex number) & A = [aij ]m n be a matrix. Thus the product A is
defined as
A = [bij ] m n where bij = aij i & j.
Note : If A is a scalar matrix, then A = , where is the diagonal element.
(xiii) Addition of matrices : Let A and B be two matrices of same order (i.e. comparable matrices).
12

54
DET. & MATRICES/Page : 12 of

Matrices

(xv)

(xvi)

Then AB = [cij ] m n where c ij =

ik b kj

, which is the dot product of i th row vector of A and j th

k =1

column vector of B.
Note - 1: The product AB is defined iff number of columns of A equals number of rows of B. A is
/ BA is defined.
called as premultiplier & B is called as post multiplier. AB is defined
Note - 2 : In general AB BA, even when both the products are defined.
Note - 3 : A (BC) = (AB) C, whenever it is defined.
(xvii)

Properties of matrix multiplication :


Consider all square matrices of order 'n'. Let Mn (F) denote the set of all square matrices of
order n. (where F is Q, R or C). Then
(a)
A, B Mn (F) AB Mn (F)
(b)
In general AB BA
(c)
(AB) C = A(BC)
(d)
n, the identity matrix of order n, is the multiplicative identity.
An = A = n A A Mn (F)
(e)
For every non singular matrix A (i.e., |A| 0) of Mn (F) there exist a unique (particular)
matrix B Mn (F) so that AB = n = BA. In this case we say that A & B are multiplicative
inverse of one another. In notations, we write B = A1 or A = B1.
(f)
If is a scalar (A) B = (AB) = A(B).
(g)
A(B + C) = AB + AC A, B, C Mn (F)
(h)
(A + B) C = AC + BC A, B, C Mn (F).
Note : (i)
Let A = [aij ] m n. Then An = A & m A = A, where n & m are identity matrices of order
n & m respectively.
(ii)
For a square matrix A, A2 denotes AA, A3 denotes AAA etc.
Solved Example # 1
sin 1/ 2
1/ 2 sin

Let A = 1/ 2 cos & B = cos cos . Find so that A = B.


cos tan
cos 1

Solution.

By definition A & B are equal if they have the same order and all the corresponding elements are equal.
1
1

Thus we have sin =


, cos =
& tan = 1

= (2n + 1) .
4
2
2
Solved Example # 2
f(x) is a quadratic expression such that
a 2 a 1 f (0)
2a + 1
2

b b 1 f (1) = 2b + 1 for three unequal numbers a, b, c. Find f(x).


c 2 c 1 f ( 1)
2c + 1


Solution.
The given matrix equation implies
a 2 f (0) + af (1) + f ( 1)
2a + 1
2

b
f
(
0
)
+
bf
(
1
)
+
f
(

1
)

= 2b + 1
c 2 f (0) + cf (1) + f ( 1)
2c + 1

x 2 f(0) + xf(1) + f(1) = 2x + 1 for three unequal numbers a, b, c


.....(i)
13

54
DET. & MATRICES/Page : 13 of

(xiv)

Then A + B is defined to be.


A + B = [aij ] m n + [bij ]m n.
= [cij ]m n where c ij = aij + bij i & j.
Substraction of matrices : Let A & B be two matrices of same order. Then A B is defined as
A + B where B is ( 1) B.
Properties of addition & scalar multiplication :
Consider all matrices of order m n, whose elements are from a set F
(F denote Q, R or C).
Let Mm n (F) denote the set of all such matrices.
Then
(a)
A Mm n (F) & B Mm n (F)

A + B Mm n(F)
(b)
A+B= B+A
(c)
(A + B) + C = A + (B + C)
(d)
O = [o]m n is the additive identity.
For every A Mm n(F), A is the additive inverse.
(e)
(f)
(A + B) = A + B
(g)
A = A
(h)
(1 + 2) A = 1A + 2A
Multiplication of matrices : Let A and B be two matrices such that the number of columns of
A is same as number of rows of B. i.e., A = [aij ] m p & B = [bij ]p n.

Let A =[aij ] m n. Then the transpose of A is denoted by A( or AT ) and is defined as


A = [bij ]n m where bij = aji i & j.
i.e. A is obtained by rewriting all the rows of A as columns (or by rewriting all the columns of A as
rows).
(i)
For any matrix A = [aij ] m n, (A) = A
(ii)
Let be a scalar & A be a matrix. Then (A) = A
(iii)
(A + B) = A + B & (A B) = A B for two comparable matrices A and B.
(iv)
(A1 A2 ..... An) = A1 A2 ..... A n, where Ai are comparable.
(v)
Let A = [aij ] m p & B = [bij ]p n , then (AB) = BA
(vi)
(A1 A2 .......An) = An. An 1 ...........A2 . A1, provided the product is defined.
(vii)
Symmetric & skew symmetric matrix : A square matrix A is said to be symmetric if A = A
i.e.
Let A = [aij ]n. A is symmetric iff aij = aji i & j.
A square matrix A is said to be skew symmetric if A = A
i.e.
Let A = [aij ]n. A is skew symmetric iff aij = aji i & j.
a h g

e.g.
A = h b f is a symmetric matrix.
g f c
x y
o

x
o
z is a skew symmetric matrix.

B=
y z 0
Note-1 In a skew symmetric matrix all the diagonal elements are zero. ( aii = aii
aii = 0)
Note-2 For any square matrix A, A + A is symmetric & A A is skew symmetric.
Note- 3 Every square matrix can be uniqualy expressed as sum of two square matrices of which one is symmetric
and other is skew symmetric.
1
1
A = B + C, where B =
(A + A) & C =
(A A).
2
2
Solved Example # 3 Show that BAB is symmetric or skew symmetric according as A is symmetric or
skew symmetric (where B is any square matrix whose order is same as that of A).
Solution.
Case -
A is symmetric

A = A
(BAB) = (B)AB = BAB

BAB is symmetric.
Case -
A is skew symmetric

A = A
(BAB) = (B)AB
= B ( A) B
= (BAB)

BAB is skew symmetric


Self Practice Problems :
1.
For any square matrix A, show that AA & AA are symmetric matrices.
2.
If A & B are symmetric matrices of same order, than show that AB + BA is symmetric and AB BA is
skew symmetric.
3.
Submatrix, Minors, Cofactors & Determinant of a Matrix
(i)
Submatrix : Let A be a given matrix. The matrix obtained by deleting some rows or columns
of
A is called as submatrix of A.
a b c d

eg.
A = x y z w
p q r s

Then

a c

a b d
x z , p q s ,

p r

a b c

x y z are all submatrices of A.


p q r
14

54

(i) is an identity
f(0) = 0, f(1) = 2 & f( 1) = 1

f(x) = x (ax + b)
2 = a + b & 1 = a + b.
1
3
3
1

b=
&a=

f(x) = x 2 +
x.
2
2
2
2
Self Practice Problems :
cos sin
1.
If A() =
, varify that A() A() = A( + ).
sin cos
Hence show that in this case A(). A() = A() . A().
2 4
4 6 1

2 , B = 0 1 and C = [3 1 2].
2.
Let A = 3 0
1 2
1 2 5
Then which of the products ABC, ACB, BAC, BCA, CAB, CBA are defined. Calculate the product
whichever is defined. Ans. only CAB is defined. CAB = [25 100]
2.
Transpose of a Matrix

DET. & MATRICES/Page : 14 of

5 3
A=
, |A| = 23
1 4
Minors & Cofactors :
Let A = [aij ]n be a square matrix. Then minor of element aij , denoted by Mij is defined as the
determinant of the submatrix obtained by deleting i th row & j th column of A. Cofactor of element
aij, denoted by Cij (or Aij ) is defined as Cij = ( 1) i + j Mij .
a b
e.g. 1
A=

c d
M11 = d = C11
M12 = c, C12 = c
M21 = b, C21 = b
M22 = a = C22
a b c

e.g. 2
A = p q r
x y z

e.g.

(iii)

M11 =
M23 =

q r
= qz yr = C111.
y z

a b
x y

= ay bx, C23 = (ay bx) = bx ay etc.

(iv)

Determinant of any order :


Let A = [aij ] n be a square matrix (n > 1). Determinant of A is defined as the sum of products of
elements of any one row (or any one column) with corresponding cofactors.
a11 a12 a13

e.g.1 A = a 21 a 22 a 23
a 31 a 32 a 33
|A| = a11C11 + a12 C12 + a13C13 (using first row).
a 22 a 23
a 21 a 22
a 21 a 23
a12
+ a13
= a11
a 32 a 33
a 31 a 32
a 31 a 33
|A| = a12 C12 + a22 C22 + a32C32 (using second column).
a 21 a 23
a11 a13
a11 a13
= a12
+ a22
a32
.
a 31 a 33
a 31 a 33
a 21 a 23

(v)

Some properties of determinant


(a)
|A| = |A| for any square matrix A.
(b)

If two rows are identical (or two columns are identical) then |A| = 0.

(c)

Let be a scalar. Than |A| is obtained by multiplying any one row (or any one column)
of |A| by
Note : |A| = n |A|, when A = [aij ] n.

(d)

Let A = [aij ] n. The sum of the products of elements of any row with corresponding
cofactors of any other row is zero. (Similarly the sum of the products of elements of
any column with corresponding cofactors of any other column is zero).

(e)

If A and B are two square matrices of same order, then |AB| = |A| |B|.
Note : As |A| = |A|, we have |A| |B| = |AB| (row - row method)
|A| |B| = |AB| (column - column method)
|A| |B| = |AB| (column - row method)

(vi)

Singular & non singular matrix : A square matrix A is said to be singular or non singular
according as |A| is zero or non zero respectively.

(vii)

Cofactor matrix & adjoint matrix :Let A = [aij ]n be a square matrix. The matrix obtained by
replacing each element of A by corresponding cofactor is called as cofactor matrix of A, denoted
as cofactor A. The transpose of cofactor matrix of A is called as adjoint of A, denoted as adj A.

15

54

Determinant of a square matrix :


Let A = [a] 11 be a 11 matrix. Determinant A is defined as |A| = a.
e.g.
A = [ 3]11
|A| = 3
a b
Let A =
, then |A| is defined as ad bc.
c d

DET. & MATRICES/Page : 15 of

(ii)

(viii)

Properties of cofactor A and adj A:


(a)
A . adj A = |A| n = (adj A) A where A = [aij ] n.
(b)
|adj A| = |A|n 1, where n is order of A.
In particular, for 3 3 matrix, |adj A| = |A| 2
(c)
If A is a symmetric matrix, then adj A are also symmetric matrices.
(d)
If A is singular, then adj A is also singular.

(ix)

Inverse of a matrix (reciprocal matrix) : Let A be a non singular matrix. Then the matrix
1
adj A is the multiplicative inverse of A (we call it inverse of A) and is denoted by A1.
|A|
We have A (adj A) = |A| n = (adj A) A

adj A = n =
A
| A |

1
A1 =
adj A.
|A|

adj A A, for A is non singular


| A |

Remarks :
1.

The necessary and sufficient condition for existence of inverse of A is that A is non singular.

2.

A1 is always non singular.

3.

If A = dia (a11, a12, ....., ann) where aii 0 i, then A1 = diag (a11 1, a221, ...., ann1).

4.

(A1) = (A)1 for any non singular matrix A. Also adj (A) = (adj A).

5.

(A1) 1 = A if A is non singular.

6.

Let k be a non zero scalar & A be a non singular matrix. Then (kA)1 =

7.
8.

1
|A1| = | A | for |A| 0.
Let A be a nonsingular matrix. Then AB = AC B = C &

9.

A is non-singular and symmetric A1 is symmetric.

10.

In general AB = 0 does not imply A = 0 or B = 0. But if A is non singular and AB = 0, then B = 0.


Similarly B is non singular and AB = 0 A = 0. Therefore, AB = 0 either both are singular or one of
them is 0.

1 1
A .
k

BA = CA B= C.

Solved Example # 4
For a 33 skew symmetric matrix A, show that adj A is a symmetric matrix.
Solution.
c 2 bc ca
a b
0

2
ab
A = a 0 c
cof A = bc b
ca ab a 2
b c 0

2
c
bc ca

2
ab which is symmetric.
adj A = (cof A) = bc b
ca ab a 2

Solved Example # 5
For two nonsingular matrices A & B, show that adj (AB) = (adj B) (adj A)
Solution.
We have (AB) (adj (AB)) = |AB| n
= |A| |B| n
A1 (AB)(adj (AB)) = |A| |B| A1
1

B adj (AB) = |B| adj A


(
A1 = | A | adj A)

B1 B adj (AB) = |B| B1 adj A

adj (AB) = (adjB) (adj A)

16

DET. & MATRICES/Page : 16 of

54

i.e.
if A = [aij ] n
then cofactor A = [c ij ] n when c ij is the cofactor of aij i & j.
Adj A = [dij ] n where dij = cji i & j.

4.

( n 1)
For any square matrix A, show that |adj (adj A) | = | A |
.
1
If A and B are nonsingular matrices, show that (AB) = B1 A1.
2

System of Linear Equations & Matrices


Consider the system
a11 x 1 + a12x 2 + .......... + a1nx n = b1
a21x 1 + a22 x 2 + ..........+ a2n x n = b2
.................................................
am1x 1 + am2x 2 + ..........+ amnx n = bn.
b1
x1


b 2
x
2

Let
& B = ... .
....


...
x
n
b
n
Then the above system can be expressed in the matrix form as AX = B.
The system is said to be consistent if it has atleast one solution.
(i)
System of linear equations and matrix inverse:
If the above system consist of n equations in n unknowns, then we have AX = B where A is a
square matrix. If A is nonsingular, solution is given by X = A1B.
If A is singular, (adj A) B = 0 and all the columns of A are not proportional, then the system has
infinite many solution.
If A is singular and (adj A) B 0, then the system has no solution
(we say it is inconsistent).
(ii)
Homogeneous system and matrix inverse:
If the above system is homogeneous, n equations in n unknowns, then in the matrix form it is
AX = O. ( in this case b1 = b2 = ....... bn = 0), where A is a square matrix.
If A is nonsingular, the system has only the trivial solution (zero solution) X = 0
If A is singular, then the system has infinitely many solutions (including the trivial solution) and
hence it has non trivial solutions.
(iii)
Rank of a matrix :
Let A = [aij]mn. A natural number is said to be the rank of A if A has a nonsingular submatrix
of order and it has no nonsingular submatrix of order more than . Rank of zero matrix is
regarded to be zero.
3 1 2 5

eg.
A = 0 0 2 0
0 0 5 0

a11 a12

a 21 a 22
A =
..... .....

a
m1 a m2

a1n

.......... a 2n
,X=
.......... .....

.......... a mn
..........

3 2
we have
as a non singular submatrix.
0 2
The square matrices of order 3 are
3 1 2 3 1 5 3 2 5 1 2 5



0 0 2 , 0 0 0 , 0 2 0 , 0 2 0
0 0 5 0 0 0 0 5 0 0 5 0


and all these are singular. Hence rank of A is 2.

(iv)

Elementary row transformation of matrix :


The following operations on a matrix are called as elementary row transformations.
(a)
Interchanging two rows.
(b)
Multiplications of all the elements of row by a nonzero scalar.
(c)
Addition of constant multiple of a row to another row.
Note : Similar to above we have elementary column transformations also.
Remark :
1.
Elementary transformation on a matrix does not affect its rank.
2.
Two matrices A & B are said to be equivalent if one is obtained from other using elementary
transformations. We write A B.
(v)
Echelon form of a matrix : A matric is said to be in Echelon form if it satisfy the followings:
(a)
The first non-zero element in each row is 1 & all the other elements in the corresponding
column (i.e. the column where 1 appears) are zeroes.
(b)
The number of zeroes before the first non zero element in any non zero row is less
than the number of such zeroes in succeeding non zero rows.
Result : Rank of a matrix in Echelon form is the number of non zero rows (i.e. number of rows with
atleast one non zero element.)
Remark :
1.
To find the rank of a given matrix we may reduce it to Echelon form using elementary row transformations
and then count the number of non zero rows.
17

DET. & MATRICES/Page : 17 of

3.
4.

54

Self Practice Problems :


1.
If A is nonsingular, show that adj (adj A) = |A| n 2 A.
2.
Prove that adj (A1) = (adj A)1.

(vii)

Solved Example # 6
x+y+z =6

Solve the system x y + z = 2 using matrix inverse.


2x + y z = 1
Solution.
6
1 1 1
x


Let A = 1 1 1 , X = y & B = 2 .
1
2 1 1
z
Then the system is AX = B.
|A| = 6. Hence A is non singular.
3
0 3

Cofactor A = 2 3 1
2 0 2
2
0 2

adj A = 3 3 0
3 1 2
2
1/ 3
1/ 3
0 2
0
1
1

1
/
2
1
/
3

3
0

2
0
= | A | adj A =
=
6
3 1 2
1/ 2 1/ 6 1/ 3
1/ 3
1/ 3 6
0
x
1




1
1
/
2
1
/

2
0
y
X=A B=
i.e.
2
= 2
1/ 2 1/ 6 1/ 3 1
z
3

x = 1, y = 2, z = 3.

A1

Solved Example # 7
x y + 2z = 1
x + y +z=3
Test the consistancy of the system
. Also find the solution, if any..
x 3y + 3z = 1
2x + 4y + z = 8.
Solution.
1
1 1 2
x

3
1
1
1
X = y , B =
A =

1
1 3 3
z

8
2 4 1
1 1

1 1
[AB] =
1 3

2 4

2 1

1 3
3 1

1 8

18

54

System of linear equations & rank of matrix:


Let the system be AX = B where A is an m n matrix, X is the n-column vector & B is the
m-column vector. Let [AB] denote the augmented matrix (i.e. matrix obtained by accepting
elements of B as n + 1th column & first n columns are that of A).
(A) denote rank of A and ([AB]) denote rank of the augmented matrix.
Clearly (A) ([AB]).
(a)
If (A) < ([AB]) then the system has no solution (i.e. system is inconsistent).
(b)
If (A) = ([AB]) = number of unknowns, then the system has unique solution.
(and hence is consistent)
(c)
If (A) = ([AB]) < number of unknowns, then the systems has infinitely many solutions
(and so is consistent).
Homogeneous system & rank of matrix :
Let the homogenous system be AX = 0, m equations in 'n' unknowns. In this case B = 0 and so
(A) = ([AB]).
Hence if (A) = n, then the system has only the trivial solution. If (A) < n, then the system has
infinitely many solutions.

DET. & MATRICES/Page : 18 of

(vi)

1.

2.

0 1 2

A = 1 2 3 . Find the inverse of A using |A| and adj A. Also find A1 by solving a system of equations.
3 1 1

Find real values of and so that the following systems has


(i)
unique solution
(ii)
infinite solution
x+y+z=6
x + 2y + 3z = 1
x + 2y + z =
Ans. (i) 3, R
(ii)
= 3, = 1

(iii)

No solution.

(iii)

= 3, 1

3.

Find so that the following homogeneous system have a non zero solution
x + 2y + 3z = x
3x + y + 2z = y
2x + 3y + z = z
Ans. = 6

5.

More on Matrices
(i)

of
Remark :

Characteristic polynomial & Characteristic equation :


Let A be a square matrix. Then the polynomial | A x | is called as characteristic polynomial
A & the equation | A x | = 0 is called as characteristic equation of A.

Every square matrix A satisfy its characteristic equation (Cayley - Hamilton Theorem).
i.e.
a0 x n + a, x n 1 + ........ + an 1x + an = 0 is the characteristic equation of A, then
a0An + a1An 1 + ......... + an 1 A + an = 0
(ii)

More Definitions on Matrices :


(a)
Nilpotent matrix:
A square matrix A is said to be nilpotent ( of order 2) if, A2 = O.
A square matrix is said to be nilpotent of order p, if p is the least positive integer such
that Ap = O.
(b)

Idempotent matrix:
A square matrix A is said to be idempotent if, A2 = A.

19

54
DET. & MATRICES/Page : 19 of

1
1 1 2

R 2 R 2 R1

0
2
1
2
R 3 R 3 R1

0 2 1 2

R 4 R 4 2R1
0 6 3 6
1
2
1 R 2 R 2
1 1
2
0 1 1/ 2 1
1

R3

3
0 1 1/ 2 1
2

1
0 1 1/ 2 1 R 4 R 4
6
1 0 3 / 2 2

R1 R1 + R 2
0 1 1/ 2 1
R3 R3 R 2

0 0
0
0

R4 R4 R2
0
0
0 0
This is in Echelon form.
(AB) = 2 = (A) < number of unknowns
Hence there are infinitely many solutions n = 1.
Hence we can take one of the variables any value and the rest in terms of it.
Let z = r, where r is any number.
Then x y = 1 2r
x+y=3r
4 3r
2+r

x=
&y=
2
2
4 3r 2 + r
,
,r .

Solutions are (x, y, z) =


2
2

Self Practice Problems:

(c)

(d)
(e)

Involutory matrix:
A square matrix A is said to be involutory if A2 = , being the identity matrix.
1 0
e.g. A =
is an involutory matrix.
0 1
Orthogonal matrix:
A square matrix A is said to be an orthogonal matrix if,
A A = = AA.
Unitary matrix:
A square matrix A is said to be unitary if A( A ) = , where A is the complex conjugate
of A.

Solved Example # 8
1 2 0

If A = 2 1 0 , show that 5A1 = A2 + A 5.


0 0 1
Solution.
We have the characteristic equation of A.
| A x | = 0
2
0
1 x

2
1
x
0 =0

i.e.

0
0
1 x
i.e.
x 3 + x 2 5x 5 = 0.
Using cayley - Hamilton theorem.
A3 + A2 5A 5 = 0

5 = A3 + A2 5A
Multiplying by A1, we get
5A1 = A2 + A 5
Solved Example # 9
Show that a square matrix A is involutory, iff ( A) ( + A) = 0
Solution.
Let A be involutory
Then
A2 =
( A) ( + A) = + A A A2
= + A A A2
= A2
=0
Conversly, let ( A) ( + A) = 0

+ A A A2 = 0

+ A A A2 = 0

A2 = 0

A is involutory
Self Practice Problems
1.

If A is idempotent, show that B = A is idempotent and that AB = BA = 0.

2.

If A is a nilpotent matrix of index 2, show that A ( + A)n = A for all n N.

3.

A is a skew symmetric matrix, such that A2 + = 0. Show that A is orthogonal and is of even order.

4.

c b
0

a . If A3 + A = 0, find .
Let A = c 0
b a 0
Ans. a2 + b2 + c 2.

20

54
DET. & MATRICES/Page : 20 of

1 0
e.g.
is an idempotent matrix.
0 1

SHORT REVISION
1.

2.

a1

b1

The symbol a b is called the determinant of order two .


2
2
Its value is given by :
D = a1 b2 a2 b1
a1
The symbol a 2
a3

c1
c2 is called the determinant of order three .
c3
b 2 c2
b1 c1
b1
Its value can be found as : D = a1 b c a2 b c + a3 b
3
3
3
3
2
b2

b1
b2
b3

c2

a2

c2

a2

c1
c2

OR

b2

b1 a c + c1 a b ....... and so on .
D = a1
b 3 c3
3
3
3
3
In this manner we can expand a determinant in 6 ways using elements of ; R1 , R2 , R3 or C1 , C2 , C3 .
3.
Following examples of short hand writing large expressions are :
(i)
The lines :
a1x + b1y + c1 = 0........ (1)
a2x + b2y + c2 = 0........ (2)
a3x + b3y + c3 = 0........ (3)
are concurrent if ,
(ii)

a1

b1

c1

a2

b2

c2 = 0 .

a3

b3

c3

Condition for the consistency of three simultaneous linear equations in 2 variables.


ax + 2 hxy + by + 2 gx + 2 fy + c = 0 represents a pair of straight lines if :
a

h g

abc + 2 fgh af bg ch = 0 = h b f
g f c
(iii)

Area of a triangle whose vertices are (xr , yr) ; r = 1 , 2 , 3 is :


D=

x1
1
x2
2
x3

y1 1
y2 1
y3 1

If D = 0 then the three points are collinear .


x

(iv)
4.

6.

x2

y2 1

MINORS :
The minor of a given element of a determinant is the determinant of the elements
which remain after deleting the row & the column in which the given element stands . For example,
b2

5.

Equation of a straight line passsing through (x1 , y1) & (x2 , y2) is x1 y1 1 = 0

c2

a1

c1

& the minor of b2 is a c .


the minor of a1 in (Key Concept 2) is
b 3 c3
3
3
Hence a determinant of order two will have 4 minors & a determinant of order three will
have 9 minors .
COFACTOR :If Mij represents the minor of some typical element then the cofactor is defined as :
Cij = (1)i+j . Mij ; Where i & j denotes the row & column in which the particular element lies.
Note that the value of a determinant of order three in terms of Minor & Cofactor can be
written as : D = a11M11 a12M12 + a13M13
OR D = a11C11 + a12C12 + a13C13 & so on .......
PROPERTIES OF DETERMINANTS : P
1 :The value of a determinant remains unaltered , if the
a1

b1

c1

a1

a2

a3

rows & columns are inter changed . e.g. if D = a 2


a3

b2
b3

c 2 = b1
c3
c1

b2
c2

b 3 = D
c3

D & D are transpose of each other . If D = D then it is SKEW SYMMETRIC


determinant but D = D 2 D = 0 D = 0 Skew symmetric determinant of
third order has the value zero .
P
2 : If any two rows (or columns) of a determinant be interchanged , the value
of determinant is changed in sign only . e.g.

21

a1
Let D = a 2
a3

b1
b2
b3

a2

c1
c2
c3

b2

c2

& D = a 1 b1 c1 Then D = D .
a3

b3

c3

P
3 : If a determinant has any two rows (or columns) identical , then its value is
a1

b1

c1

zero . e.g. Let D = a 1 b1 c1


a3

b3

then it can be verified that D = 0.

c3

P
4 : If all the elements of any row (or column) be multiplied by the same number ,
then the determinant is multiplied by that number.
e.g.

a1

b1

c1

If D = a 2
a3

b2
b3

c2
c3

Ka 1 Kb1 Kc1

and

D =

a2

b2

a3

b3

c2 Then D= KD
c3

P
5 : If each element of any row (or column) can be expressed as a sum of two terms
then the determinant can be expressed as the sum of two determinants . e.g.
a 1 + x b1 + y c1 + z
a2

b2

c2

a3

b3

c3

a1

b1

c1

= a2
a3

b2

c2 + a 2

b2

c2

b3

c3

b3

c3

a3

P
6 : The value of a determinant is not altered by adding to the elements of any
row (or column) the same multiples of the corresponding elements of any
a1

other row (or column). e.g.

7.

c1

Let D = a 2 b 2 c2
a3

D =

b1

a 1 + ma 2

b1 + m b 2

c1 + m c2

a2

b2

c2

a 3 + n a1

b 3 + n b1

c3 + n c1

b3

and

c3

. Then D = D .

Note : that while applying this property ATLEAST ONE ROW (OR COLUMN)
must remain unchanged .
P 7 : If by putting x = a the value of a determinant vanishes then (x a) is a factor
of the determinant .
MULTIPLICATION OF TWO DETERMINANTS :
(i)

a1
a2

m1
a l +b l
= 1 1 1 2
m2
a 2 l1 + b 2 l 2

b1
l
x 1
b2
l2

a 1 m1 + b 1 m 2
a 2 m1 + b 2 m 2

Similarly two determinants of order three are multiplied.


a1

(ii)

b1

A1

c1

B1

C1

If D = a 2 b 2 c2 0 then , D = A 2 B2 C 2
a3

b3

PROOF :

A3

c3

Consider

a1
a2

b1
b2

c1
c2

a3

b3

c3

B3

C3

A1

A2

where Ai , Bi , Ci are cofactors


A3

B1 B2 B3 = 0 D 0
C1

C2

C3

Note : a1A2 + b1B2 + c1C2 = 0 etc.


A1 A 2

A3

A1 A 2

A3

therefore , D x B1 B2 B3 = D B1 B2 B3 = D
3

C1

8.

C2

C3

C1

C2

C3

OR

A1

B1

C1

A2

B2

CA 3

B3

C 2 = D
C3

SYSTEM OF LINEAR EQUATION (IN TWO VARIABLES) :


(i)
Consistent Equations : Definite & unique solution . [ intersecting lines ]
(ii)
Inconsistent Equation : No solution . [ Parallel line ]
(iii)
Dependent equation : Infinite solutions . [ Identical lines ]
Let a1x + b1y + c1 = 0 & a2x + b2y + c2 = 0 then :
a1
b
c
= 1 1
a2
b2
c2

Given equations are inconsistent

22

&

a1
b
c
= 1 = 1
a2
b2
c2

9.

Given equations are dependent

CRAMER'S RULE :
[ SIMULTANEOUS EQUATIONS INVOLVING THREE U NKNOWNS ]
Let ,a1x + b1y + c1z = d1 ...(I) ; a2x + b2y + c2z = d2 ...(II) ; a3x + b3y + c3z = d3 ...(III)
Then ,

D
D1
D
, Y= 2 , Z= 3
D
D
D
d1
a 1 b1 c1
D = a 2 b 2 c2 ; D1 = d 2
d3
a 3 b 3 c3

x=

Where

.
b1

c1

b2
b3

c2
c3

a1
; D2 = a 2
a3

d1
d2
d3

c1
c2
c3

a1
& D3 = a 2
a3

NOTE : (a)
If D 0 and alteast one of D1 , D2 , D3 0 , then the given system of
equations are consistent and have unique non trivial solution .
(b)
If D 0 & D1 = D2 = D3 = 0 , then the given system of equations are
consistent and have trivial solution only .
(c)
If D = D1 = D2 = D3 = 0 , then the given system of equations are
consistent and have infinite solutions .

(d)
10.

b1
b2

d1
d2

b3

d3

a1x + b1y + c1z = d1


a
In case
2 x + b 2 y + c 2 z = d 2 represents these parallel planes then also
a 3 x + b3 y + c 3z = d 3
D = D1 = D2 = D3 = 0 but the system is inconsistent.
If D = 0 but atleast one of D1 , D2 , D3 is not zero then the equations are inconsistent and have no
solution .
If x , y , z are not all zero , the condition for a1x + b1y + c1z = 0 ; a2x + b2y + c2z = 0 &
a1

b1

c1

a3x + b3y + c3z = 0 to be consistent in x , y , z is that a 2 b 2 c2 = 0.


a3

b3

c3

Remember that if a given system of linear equations have Only Zero Solution for all its
variables then the given equations are said to have TRIVIAL SOLUTION.

EXERCISE-1
Q 1.

Without expanding the determinant prove that :


(a)

(b) q p

=0

rp

a x by cz
a
b
c
2
2
2
x
y
z
x
y
z
=
(d)
1
1
1
y z zx xy

Q 2.

pq pr

0
rq

qr = 0
0

(c) 5 3i

8
2
+ 4i 4 5i
3

2a +1

a +2

x +1 x

If y + 1 y

y =0
z

z3 + 1 z2

1 c c2 a b

1 = (a 1)
1

(b)

z = [(xy) (yz) (zx) (x+y+z)]


z3

and x , y , z are all different then , prove that xyz = 1 .


89
198 .
89 198 440

Using properties of determinants or otherwise evaluate 40


a bc

Q 6.

2
(e) 1 b b ca = 0

18

Q 5.

is real

Without expanding as far as possible , prove that :


(a)

Q 4.

2
4i
3
4 + 5i

a2 b c

1 a

a 2 + 2a 2 a +1 1

Q 3.

5 + 3i

2a

2b
b c a
2c
2c
a b c
If D = c a b and D =
b c a

Prove that

40
89

2a

= (a + b + c)3 .
2b
c a b
b + c c+ a a + b
a + b b+ c c+a
c+ a

then prove that D = 2 D .

a +b b+c

23

2a a + b a + c

Q 7.

Q 8.

Q 9.

b + a 2 b b + c = 4 [(a+b) (b+c) (c+a)]


c+ a c+ b 2c
1+ a 2 b2
2 ab
2b
2
2
2
ab
1
a
b
2a

+
Prove that
= (1 + a + b)3 .
2
2
2b
2a
1 a b

Prove that

Prove that

a
b c c+ b
a+c
b
c a
a b b+a
c

= (a + b + c) (a + b + c) .
tan(A + P)

tan(B + P)

tan(C + P)

Q 10. Show that the value of the determinant tan(A + Q) tan(B + Q) tan(C + Q) vanishes for all values
tan(A + R)

tan(B + R)

tan(C + R)

of A, B, C, P, Q & R where A + B + C + P + Q + R = 0
Q 11. Factorise the determinant

b c b c + b c b c
ca ca + c a c a
a b a b + a b a b

( + ) 4 ( + )2
4
2
Q 12. Prove that ( + ) ( + )
( + ) 4 ( + ) 2
n!

Q 13. For a fixed positive integer n , if D =

1
1

= 64( ) ( )( ) ( ) ( ) ( )

1
( n + 1)! (n + 2)!

(n + 1)! (n + 2)! (n + 3)!

4
(n!)

then show that

(n + 2)! (n + 3)! ( n + 4)!

is divisible by n .
x+2

2x + 3

3x + 4
4x + 5

Q 14. Solve for x 2 x + 3 3 x + 4

=0 .

3 x + 5 5 x + 8 10 x + 17
ax

Q 15. If a + b + c = 0 , solve for x :

bx

cx

=0 .

Q 16. If a2 + b2 + c2 = 1 then show that the value of the determinant


a 2 + (b 2 + c 2 ) cos
ba (1 cos )
ca (1 cos )
2
2
2
ab(1 cos )
b + (c + a ) cos
cb(1 cos )
2
ac(1 cos )
bc(1 cos )
c + (a 2 + b 2 ) cos
pa qb

rc

Q 17. If p + q + r = 0 , prove that q c r a p b = pqr


rb
a

pc qa
a3

a b c
c a b
b c a

simplifies to cos2.

a4 1

Q 18. If a , b , c are all different & b b 3 b 4 1 = 0 , then prove that :


c

c3

c4 1

abc (ab + bc + ca) = a + b + c .


Q 19. Show that
a2 +
ab
ac
2
ab
b +
bc
is divisible by 2 and find the other factor..
2
ac
bc
c +

24

a2

bc a

1 a2

a3

Without expanding prove that ca b b 2 = 1 b 2 b 3 .

Q 20. (a)

ab

(b)

a2

b2

c2

(a + 1) 2
(a 1) 2

(b + 1) 2
(b 1) 2

c2

a2

b2

c2

(c + 1) 2 = 4 a
( c 1) 2
1

b
1

c
1

1 c2

c3

.
x2 + x

x +1

x2

Q 21. Without expanding a determinant at any stage, show that 2 x + 3 x 1 3 x 3 x 3 = Ax + B where


x2 + 2 x + 3 2 x 1 2 x 1
2

A & B are determinants of order 3 not involving x .


Q 22. Prove that

bc
2
a + ac

b 2 + b c c2 + b c
a c
c2 + a c

a + ab b + ab
2

x2 a 2
Q 23. Solve (x a )3
(x + a )3

x2 b2
(x b ) 3
(x + b ) 3
x2

= (ab + bc + ca)3 .

a b

x 2 c2
(x c)3 = 0 where a , b , c are non zero and distinct .
(x + c)3

2x 3

3x 4

Q 24. Solve for x : x 4 2 x 9 3 x 16 = 0 .


x 8 2 x 27 3 x 64

Q 25. If

1
a +x
1
a +y
1
a+z

1
b+x
1
b+y
1
b+z

1
c+ x
P
1
c+ y = Q
1
c+ z

where Q is the product of the denominator , prove that

P = (a b) (b c) (c a) (x y) (y z) (z x)

Q 26. If Dr =

( ) 4 (5 )

2r 1

2 3r 1

r 1

x
2n 1

y
3n 1

z
5n 1

then prove that

r=1

a2
2
Q 27. If 2 s = a + b + c then prove that (s b)2

(s c)

Dr = 0 .

(s a ) 2

(s a ) 2

b2

(s b) 2

(s c)

= 2 s3 (s a) (s b) (s c) .

cot A2

Q 28. In a ABC, determine condition under which tan + tan


B
2

cot B2
C
2

tan

1
b 2 c2

(
ca ( b

)
+c )

Q 29. Show that b a b + c


2

a b c2 + a 2

c a

c b c2 + a 2

2 2

(
b c (a

)
+b )

C
2

+ tan

cot C2
A
2

tan

A
2

+ tan B2

=0

a c a 2 + b2
2

= (ab + bc + ca)3 .

a 2b2

Q 30. Prove that


bc a2

ca b 2

a b c2

b c + ca + a b b c ca + a b b c + ca a b
(a + b) (a + c)

(b + c) (b + a )

= 3 . (b c) (c a) (a b) (a + b + c) (ab + bc + ca)

( c + a ) ( c + b)
cos (A P ) cos (A Q) cos (A R)

Q 31. For all values of A , B , C & P , Q , R show that cos (B P) cos (B Q) cos (B R) = 0.
cos (C P)

25

cos (C Q)

cos (C R)

a 1 l1 + b1 m1

Q 32. Show that

a 1 l 2 + b1 m 2

a 1 l 3 + b1 m 3

a 2 l1 + b 2 m1 a 2 l 2 + b 2 m 2

a 2 l 3 + b 2 m3

a 3 l1 + b 3 m1

a 3 l 3 + b 3 m3

a 3 l 2 + b3 m2

=0 .

( a 1 b1 ) 2 ( a 1 b 2 ) 2 ( a 1 b 3 ) 2
2
2
2
Q 33. Prove that ( a 2 b1 ) (a 2 b 2 ) (a 2 b 3 ) = 2 (a1 a2) (a2 a3) (a3 a1) (b1 b2) (b2 b3) (b3 b1)
( a 3 b1 ) 2 (a 3 b 2 ) 2 ( a 3 b 3 ) 2
++ +

Q 34. Prove that + + +

2 ( + )( + )
( + ) + ( + )
( + ) + ( + )
2

=0 .

Q 35. If ax + 2 hxy + by + 2 gx + 2 fy + c (l1x + m1y + n1) (l2x + m2y + n2) , then prove that
a

h g

h b f = 0 .
g f c

Q 36. Prove that


cos 2 (A B) cos 2 (A C)

1
cos 2 (B A)
2
2
cos (C A) cos (C B)

cos 2 (B C)
= 2sin2(A B)sin2(B C)sin2(C A)
1

ax1 + by1 + cz12 = ax22 + by22 + cz22 = ax32 + by32 + cz32 = d and
ax2x3 + by2y3 + cz2z3 = ax3x1 + by3y1 + cz3z1 = ax1x2 + by1y2 + cz1z2 = f , then prove that

Q 37. If
x1
x2

y1
y2

z1
z2

x3

y3

z3

d + 2f
= (d f) abc

1/ 2

(a , b , c 0)

Q 38. If (x1x2)2 + (y1 y2)2 = a2 , (x2x3)2 + (y2 y3)2 = b2 and (x3x1)2 + (y3 y1)2 = c2
x1
prove that 4 x 2
x3

y1 1
y 2 1 = (a + b + c) (b + c a) (c + a b) (a + b c) .
y3 1
S0

S1

S2

S3

S2

Q 39. If Sr = r + r + r then show that S1 S2 S3 = ( )2 ( )2 ( )2 .


2

S4
2

Q 40. If u = ax + 2 bxy + cy , u = ax + 2 bxy + cy . Prove that


y2

xy x 2

c =

ax + by
bx + cy
u
u
1
.
=
a x + b y b x + c y
y ax + by a x + b y

EXERCISE-2
4
3
+
= 1
x+5 y+7

Solve using Cramers rule :

Q 2.

Solve the following using Cramers rule and state whether consistent or not.
(a)

x + 2y + z = 1
3x + y + z = 6
x + 2y = 0

(b)

x 3y + z = 2
3x + y + z = 6

7 x 7 y + 5z = 3

(c)

3x + y + 5z = 7

5x + y + 3z = 3

2 x + 3 y + 5z = 5

z + a y + a x + a = 0

Solve the system of equations ; z + b y + b 2 x + b 3 = 0


z + cy + c2 x + c3 = 0
2

Q 3.

&

6
6

= 5 .
x+5 y+7

Q 1.

26

Q 4.

Q 5.
Q 6.
Q 7.

Q 8.

Q 9.
Q 10.
Q 11.
Q 12.
Q 13.

Q 14.

For what value of K do the following system of equations possess a non trivial
(i.e. not all zero) solution over the set of rationals Q ?
x + Ky + 3 z = 0 , 3 x + K y 2 z = 0 , 2 x + 3 y 4 z = 0 .
For that value of K , find all the solutions of the system .
Given x = cy + bz ; y = az + cx ; z = bx + ay where x , y , z are not all zero , prove that
a + b + c + 2 abc = 1 .
Given a =

y
x
z
;c=
; b=
where x , y , z are not all zero , prove that :
zx
yz
xy

1 + ab + bc + ca = 0 .
If sin q cos q and x, y, z satisfy the equations
x cos p y sin p + z = cos q + 1
x sin p + y cos p + z = 1 sin q
x cos(p + q) y sin (p + q) + z = 2
then find the value of x2 + y2 + z2.
If A, B and C are the angles of a triangle then show that
sin 2Ax + sin Cy + sin Bz = 0
sin Cx + sin 2By + sin Az = 0
sin Bx + sin Ay + sin 2Cz = 0
possess non-trivial solution.
Investigate for what values of , the simultaneous equations x + y + z = 6 ;
x + 2 y + 3 z = 10 & x + 2 y + z = have ; (a) A unique solution .
(b) An infinite number of solutions .
(c) No solution .
For what values of p , the equations : x + y + z = 1 ; x + 2 y + 4 z = p &
x + 4 y + 10 z = p have a solution ? Solve them completely in each case .
Solve the equations : K x + 2 y 2 z = 1 , 4 x + 2 K y z = 2 , 6 x + 6 y + K z = 3
considering specially the case when K = 2 .
Solve the system of equations :
x + y + z = m , x + y + z = n and x + y + z = p
Find all the values of t for which the system of equations ;
(t 1) x + (3 t + 1) y + 2 t z = 0
(t 1) x + (4 t 2) y + (t + 3) z = 0
2 x + (3 t + 1) y + 3 (t 1) z = 0
has non trivial solutions and in this context find the ratios
of x : y : z , when t has the smallest of these values.
Solve : (b + c) (y + z) ax = b c , (c + a) (z + x) by = c a and
(a + b) (x + y) cz = a b where a + b + c 0.
ap a

Q 15. If bc + qr = ca + rp = ab + pq = 1 show that

bq b q
cr

=0 .

Q 16. If x, y, z are not all zero & if ax + by + cz = 0, bx + cy + az = 0 & cx + ay + bz = 0, then prove


that x : y : z = 1 : 1 : 1 OR 1 : : OR 1 : : , where is one of the complex cube root of unity.
Q 17. If the following system of equations (a t)x + by + cz = 0 , bx + (c t)y + az = 0 and
cx + ay + (b t)z = 0 has nontrivial solutions for different values of t , then show that we can
express product of these values of t in the form of determinant .
Q18. Show that the system of equations
3x y + 4z = 3 , x + 2y 3z = 2 and 6x + 5y + z = 3
has atleast one solution for any real number . Find the set of solutions of = 5.

EXERCISE-3
Q.1
Q.2

For what values of p & q, the system of equations 2 x + p y + 6 z = 8 ; x + 2 y + q z = 5 &


x + y + 3 z = 4 has ; (i) no solution
(ii) a unique solution (iii) infinitely many solutions
(i)
Let a , b , c positive numbers . The following system of equations in x , y & z.
x2
y2
z2
x2 y2 z2
x2
y2
z2
+

+
=
1
;
= 1 ; 2 + 2 + 2 = 1 has:
2
2
2
2
2
2
a
b
c
a
b
c
a
b
c
(B) unique solution
(A) no solution
(C) infinitely many solutions (D) finitely many solutions
1+ i + 2

(ii)

If ( 1) is a cube root of unity , then 1 i


i

(A) 0

(B) 1

1
2 1 equals :
i + 1
1

(D)

(C) i

27

[ IIT '95 , 1 + 1 ]

Q.3

Let a > 0, d > 0 . Find the value of determinant


1
a
1
(a + d )
1
(a + 2 d)

Q.4

Q.5

1
a (a + d)
1
(a + d ) (a + 2 d )
1
(a + 2 d) (a + 3 d)

1
(a + d) (a + 2 d )
1
(a + 2 d ) (a + 3 d )
1
(a + 3 d ) (a + 4 d )

[ IIT 96 , 5 ]

Find those values of c for which the equations :


2 x+3y = 3
(c + 2) x + (c + 4) y = c + 6
(c + 2) x + (c + 4) y = (c + 6) are consistent .
[ REE 96 , 6 ]
Also solve above equations for these values of c .
For what real values of k , the system of equations x + 2y + z = 1 ; x + 3y + 4z = k ;
x + 5y + 10z = k2 has solution ? Find the solution in each case.
[ REE ' 97, 6 ]
1

Q.6

The parameter, on which the value of the determinant cos(p d) x cos px cos(p + d) x does not
sin (p d) x

depend upon is :
(A) a
3i

If 4

3i

20

6i

Q.7

(A) x = 3 , y = 1
Q.8

(ii)

a2

(i)

(B) p
= x + iy ,

sin px

sin (p + d) x

(C) d

(D) x

(C) x = 0 , y = 3

(D) x = 0 , y = 0

then :

(B) x = 1 , y = 3

1
x
If f(x) =
2x
x ( x 1)
3x ( x 1) x ( x 1) ( x 2)

x +1
then f(100) is equal to :
(x + 1) x
(x + 1) x (x 1)

(A) 0
(B) 1
(C) 100
(D) 100
Let a, b, c, d be real numbers in G.P. If u, v, w satisfy the system of equations,
u + 2v + 3w = 6
4u + 5v + 6w = 12
6u + 9v = 4
then show that the roots of the equation,
1 1 1 2
2
2
2
+ + x + [(b c) + (c a) + (d b) ] x + u + v + w = 0
u v w

and

Q.9

20 x2 + 10 (a d)2 x 9 = 0 are reciprocals of each other .


If the system of equations x Ky z = 0, Kx y z = 0 and x + y z = 0 has a non zero solution,
then the possible values of K are
(A) 1, 2
(B) 1, 2
(C) 0, 1
(D) 1, 1

Q.10

sin
2
Prove that for all values of , sin + 3
2
sin 3

cos
2
cos + 3
2
cos 3

Q.11

sin 2
4
sin 2 + 3 = 0
4
sin 2 3

Find the real values of r for which the following system of linear equations has a non-trivial solution
. Also find the non-trivial solutions :
2 rx 2y + 3z = 0
x + ry + 2z = 0
2x + rz = 0
Q.12 Solve for x the equation
a2
a
1
sin(n + 1)x sin nx sin(n 1)x
=0
cos(n + 1)x cos nx cos(n 1)x

28

Q.13 Test the consistency and solve them when consistent, the following system of equations for all
values of :
x+y+z =1
x + 3y 2z =
3x + ( + 2)y 3z = 2 + 1
[ REE 2001 (Mains) , 5 out of 100 ]
Q.14 Let a, b, c be real numbers with a2 + b2 + c2 = 1 . Show that the equation
ax by c
bx + ay
cx + a
bx + ay
cy + b
ax + by c
= 0
represents a straight line.
cx + a
cy + b
ax by + c
Q.15 The number of values of k for which the system of equations
(k + 1)x + 8y = 4k
kx + (k + 3)y = 3k 1
has infinitely many solutions is
(A) 0
(B) 1
(C) 2
(D) inifinite
for which the system of equations 2x y z = 12, x 2y + z = 4, x + y + z = 4
has no solution is
(A) 3
(B) 3
(C) 2
(D) 2
Q

. 1

h e

v a l u e

o f

ANSWER KEY [EXERCISE-1]


Q 4. 1

Q 11. (ab ab) (bc bc) (ca ca)

Q 15. x = 0 or x =

3 2
a + b 2 + c2
2

Q 14. x = 1 or x = 2

Q19. 2 ( a2 + b2 + c2 + )

Q 23. If ab + bc + ca 0 , then x = 0 is the only real root ; If ab + bc + ca > 0 ,

then x = 0 or x =
Q 24. x = 4

ab + bc + ca
3

Q 28. Triangle ABC is isosceles .

EXERCISE-2
Q 1. x = 7 , y = 4
Q 2. (a) x = 2 , y = 1 , z = 1 ; consistent
(b) x =

13
7
35
, y= ,z=
; consistent
3
6
6

(c) inconsistent

Q 3. x = (a + b + c) , y = ab + bc + ca , z = abc
Q 4. K =

33
15
, x: y: z = : 1 : 3
2
2

Q7. 2

Q 9. (a) 3 (b) = 3, =10 (c) = 3, 10


Q 10. x = 1 + 2 K , y = 3 K , z = K , when p = 1 ; x = 2 K , y = 1 3 K , z = K when p = 2 ;
where K R
Q 11. If K 2 ,

x
y
z
1
=
=
=
2
2 (K + 6) 2 K + 3 6 (K 2) 2 K + 2 K + 15

1 2
and z = 0 where R
If K= 2 , then x = , y =
2
;
Q 12. If 1 or 2 , unique solution
If = 2 & m + n + p = 0 , infinite solution ;
If = 2 & m + n + p 0 , no solution ;
If = 1 , infinite solution if m = n = p ;

29

If = 1 , no solution if m n or n p or p m
Q 13. t = 0 or 3 ; x : y : z = 1 : 1 : 1 Q 14. x =
a

c b
ba
ac
, y=
, z=
a
+
b
+
c
a + b+c
a +b+c

Q 17. b c a
c a

Q18.

4
9
; y = and z = 0 ;
7
7
13K 9
4 5K
If = 5 then x =
;y=
and z = K
7
7
If 5 then x =

where K R

EXERCISE-3

Q 1.
Q 2.
Q 3.

(i) p 2 , q = 3 (ii) p 2 & q 3 (iii) p = 2


(i) d (ii) a
4 d4
a (a + d) (a + 2 d) 3 (a + 3 d) 2 (a + 4 d)
2

1
4
, y=
2
3

Q 4.

for c = 0 , x = 3 , y = 3 ; for c = 10 , x =

Q 5.

k = 1 : ( 5t+1, 3t, t) ; k = 2 : (5t 1, 1 3 t, t) for t R ; no solution

Q 6.

Q 11.

r=2 ; x=k ; y=

Q 7. D

Q 8. (i) A

k
; z=k
2

Q9. D

where k R {0}

Q 12.

x = n, n I

1
Q 13. If = 5, system is consistent with infinite solution given by z = K, y = (3K + 4) and
2
1
x = (5K + 2) where K R
2
1
1
If 5, system is consistent with unique solution given by x = (1 ); x = ( + 2) and y =
3
3
0.
Q15.

Q.16 D

30

SHORT REVISION
1.

Definition :

Rectangular array of m n numbers . Unlike determinants it has no value.


a 11 a 12
a
21 a 22
A = :
:

a
a
m2
m1

a 1n
...... a 2 n
:
:

...... a m n
......

or

a 11 a 12

a 21 a 22
:
:

a m1 a m 2

...... a 1 n

...... a 2 n
:
:

...... a mn

Abbreviated as : A = [ a i j ] 1 i m ; 1 j n, i denotes the row and


j denotes the column is called a matrix of order m n.

2.
(a)
(b)
(c)

Special Type Of Matrices


:
a
11

a1n ]
Row Matrix : A = [ a11 , a12, a......
having one row . (1 n) matrix.(or row vectors)
21
Column Matrix :
A = :
having one column. (m 1) matrix (or column vectors)
Zero or Null Matrix : (A = Om n)
a m1 are zero .
An m n matrix all whose entries

0 0
A = 0 0 is a
0 0

(d)

3 2 null matrix

0 0 0
& B = 0 0 0
0 0 0

is 3 3 null matrix

Horizontal Matrix : A matrix of order m n is a horizontal matrix if n > m.

1 2 3 4
2 5
2 5 1 1

1 1

(e)
Verical Matrix :
A matrix of order m n is a vertical matrix if m > n.
3 6
(f)
Square Matrix : (Order n) If number of row = number of column
a square matrix.
4
Note (i)
In a square matrix the pair of elements aij & aj i are called Conjugate2Elements
.
e.g.
(ii)

a 11 a 12

a 21 a 22

The elements a11 , a22 , a33 , ...... ann are called Diagonal Elements . The line along which
the diagonal elements lie is called " Principal or Leading " diagonal.
The qty ai i = trace of the matrice written as , i.e. t r A

Square Matrix
Triangular Matrix

Diagonal Matrix denote as


ddia (d1 , d2 , ....., dn) all elements
except the leading diagonal are zero
diagonal Matrix
Unit or Identity Matrix
Note: Min. number of zeros in a diagonal matrix of order n = n(n 1)
"It is to be noted that with square matrix there is a corresponding determinant formed by the elements of A in the
same order."
3.
Equality Of Matrices :
Let
A = [a i j ] & B = [b i j ] are equal if ,
(i)
both have the same order .
(ii)
ai j = b i j for each pair of i & j.
4.
Algebra Of Matrices :
Addition :

A + B = a i j + bi j

where A & B are of the same type. (same order)

31

(a)
(b)
(c)
5.

Addition of matrices is commutative.


i.e.
A+B = B+A
Matrix addition is associative .
(A + B) + C = A + (B + C)
Additive inverse.
If A + B = O = B + A

A=mn

B=mn

Note : A , B & C are of the same type.


A = m n

Multiplication Of A Matrix By A Scalar :

a
A = b
c

Multiplication
AB exists if ,

b c
ka k b k c
;
k A = k b kc ka
c a
kc ka k b
a b

Of Matrices : (Row by Column)


A= m n
& B= np
23
33
AB exists , but BA does not AB BA

If

6.

A = prefactor

B = post factor

Note : In the product AB ,

A = (a1 , a2 , ...... an)

&

b1
b
2
B= :

b n

1n
n1
A B = [a1 b1 + a2 b2 + ...... + an bn]

[ ]

If A = a i j

[ ]

m n & B = bi j

n p matrix , then

(A B)i j =

r =1

ai r . br j

Properties Of Matrix Multiplication :


1.

Matrix multiplication is not commutative .


1 0
1 0
1 1
B = 0 0 ; AB = 0 0 ;
A = 0 0 ;

AB BA (in general)

2.

3.

4.

1 1
BA = 0 0

1 1 1 1
0 0
AB = 2 2 1 1 = 0 0
AB = O
A = O or B = O
/

Note: If A and B are two non- zero matrices such that AB = O then A and B are called the divisors of
zero. Also if [AB] = O | AB | | A | | B | = 0 | A | = 0 or | B | = 0 but not the converse.
If A and B are two matrices such that
(i)
AB = BA A and B commute each other
(ii)
AB = BA A and B anti commute each other
Matrix Multiplication Is Associative :
If A, B & C are conformable for the product AB & BC, then
(A . B) . C = A . (B . C)
Distributivity :
A (B + C ) = A B + A C
Provided A, B & C are conformable for respective products
(A + B) C = A C + BC

5.

POSITIVE INTEGRAL POWERS OF A SQUARE MATRIX :


For a square matrix A , A2 A = (A A) A = A (A A) = A3 .
Note that for a unit matrix I of any order , Im = I for all m N.

32

6.

(a)
(b)
(c)

(d)
7.

MATRIX POLYNOMIAL :
If f (x) = a0xn + a1xn 1 + a2xn 2 + ......... + anx0 then we define a matrix polynomial
f (A) = a0An + a1An1 + a2An2 + ..... + anIn
where A is the given square matrix. If f (A) is the null matrix then A is called the zero or root of the
polynomial f (x).
DEFINITIONS :
Idempotent Matrix : A square matrix is idempotent provided A2 = A.
Note that An = A n > 2 , n N.
Nilpotent Matrix: A square matrix is said to be nilpotent matrix of order m, m N, if
Am = O , Am1 O.
Periodic Matrix : A square matrix is which satisfies the relation AK+1 = A, for some positive integer K,
is a periodic matrix. The period of the matrix is the least value of K for which this holds true.
Note that period of an idempotent matrix is 1.
Involutary Matrix : If A2 = I , the matrix is said to be an involutary matrix.
Note that A = A1 for an involutary matrix.
The Transpose Of A Matrix : (Changing rows & columns)
Let A be any matrix . Then , A = ai j
of order m n
T
A or A = [ aj i ] for 1 i n & 1 j m of order n m
Properties of Transpose : If AT &
(A B)T = AT BT
(a)
IMP. (b)
(A B)T = BT AT
(c)
(AT)T = A
(d)
(k A)T = k AT

8.

BT denote the transpose of A and B ,


; note that A & B have the same order.
A & B are conformable for matrix product AB.
k is a scalar .

General :
(A1 , A2 , ...... An)T = A Tn , ....... , A 2T , A1T
Symmetric & Skew Symmetric Matrix :

[ ]

(reversal law for transpose)

A square matrix A = a i j is said to be ,


symmetric if ,
ai j = aj i i & j
(conjugate elements are equal) (Note A = AT)
n ( n + 1)
Note: Max. number of distinct entries in a symmetric matrix of order n is
.
2
and skew symmetric if ,
ai j = aj i i & j (the pair of conjugate elements are additive inverse of each other)
(Note A = AT )
Hence If A is skew symmetric, then
ai i = ai i
ai i = 0
i
Thus the digaonal elements of a skew symmetric matrix are all zero , but not the converse .
Properties Of Symmetric & Skew Matrix :
P 1 A is symmetric if
AT = A
A is skew symmetric if
AT = A
P 2 A + AT is a symmetric matrix
A AT is a skew symmetric matrix .
Consider (A + AT)T = AT + (AT)T = AT + A = A + AT
A + AT is symmetric . Similarly we can prove that A AT is skew symmetric .
P 3 The sum of two symmetric matrix is a symmetric matrix and
the sum of two skew symmetric matrix is a skew symmetric matrix .
Let
AT = A ;
BT = B where A & B have the same order .
T
(A + B) = A + B
Similarly we can prove the other

33

P 4 If A & B are symmetric matrices then ,


(a)
A B + B A is a symmetric matrix
AB BA is a skew symmetric matrix .
(b)
P 5 Every square matrix can be uniquely expressed as a sum of a symmetric and a skew symmetric matrix.
A=

1
1
(A + AT) +
(A AT)
2
2

P
Symmetric
9.

Q
Skew Symmetric

Adjoint Of A Square Matrix :


Let

A=

[ ]
aij

a11 a12

= a 21 a 22
a
31 a 32

a13

a 23
a 33

be a square matrix and let the matrix formed by the

C11 C12 C13

A
is = C 21 C 22 C 23 .
cofactors of [ai j ] in determinant
C

31 C32 C33
C11 C 21 C31

Then (adj A) = C12 C 22 C32


C

13 C 23 C33
V. Imp. Theorem : A (adj. A) = (adj. A).A = |A| In , If A be a square matrix of order n.
Note : If A and B are non singular square matrices of same order, then
| adj A | = | A |n 1
(i)
(ii)
adj (AB) = (adj B) (adj A)
(iii)
adj(KA) = Kn1 (adj A), K is a scalar
Inverse Of A Matrix (Reciprocal Matrix) :
A square matrix A said to be invertible (non singular) if there exists a matrix B such that,
AB = I = BA

B is called the inverse (reciprocal) of A and is denoted by A 1 . Thus


A 1 = B A B = I = B A .
We have ,

A . (adj A) = A In
A 1 A (adj A) = A 1 In ||
In (adj A) = A 1 A In

A 1 =

(adj A)
|A |

Note : The necessary and sufficient condition for a square matrix A to be invertible is that A 0.
Imp. Theorem : If A & B are invertible matrices ofthe same order , then (AB) 1 = B 1 A 1. This is reversal
law for inverse.
Note :(i)
If A be an invertible matrix , then AT is also invertible & (AT) 1 = (A 1)T.
(ii)
If A is invertible, (a) (A 1) 1 = A ; (b) (Ak) 1 = (A 1)k = Ak, k N
(iii)
If A is an Orthogonal Matrix. AAT = I = ATA
(iv)

A square matrix is said to be orthogonal if , A 1 = AT .

(v)

| A1 | =

1
|A|

SYSTEM OF EQUATION & CRITERIAN FOR CONSISTENCY


GAUSS - JORDAN METHOD

x+y+z = 6
xy+z= 2

34

2x + y z = 1

or

x + y+ z
6
x y+ z

= 2
1
2x + yz
1 1 1
1 1 1

2 1 1

AX = B

x
6
y
2
=

1

z
A 1 A X = A 1 B

(adj. A).B
.
|A|
Note :(1)
If A 0,
system is consistent having unique solution
(2)
If A 0 & (adj A) . B O (Null matrix) , system is consistent having unique non trivial solution.
(3)
If A 0 & (adj A) . B = O (Null matrix) ,system is consistent having trivial solution .
(4)
If
A = 0 , matrix method fails
X = A 1 B =

If (adj A) . B = null matrix = O

If (adj A) . B O

Consistent (Infinite solutions)

Inconsistent (no solution)

EXERCISE-4
Q1.

Q2.

1 2 2
2 1 1
2
2
3

Given that A =
, C = 2 2 1 , D =
1 1 3
1 1 1
Ax = b.
Find the value of x and y that satisfy the equations.

10
13 and that Cb = D. Solve the matrix equation
9

3 3
3 2
y
y

3
0

= 3 y 3 y
2 4 x x 10 10

Q 3.

Q 4.

0 1 0
0 0 0
0
0
1

If, E =
and F = 1 0 0 calculate the matrix product EF & FE and show that
0 0 0
0 1 0
E2F + FE2 = E .
If A is an orthogonal matrix and B = AP where P is a non singular matrix then show that the matrix
PB1 is also orthogonal.

Q 5.

cost sin t
The matrix, R(t) is defined by R(t) =
. Show that, R(s) R(t) R(s + t) .
sint cost

Q 6.

cos 2
sin cos
Prove that the product of two matrices, cos sin
sin 2 &

matrix when & differ by an odd multiple of

.
2

35

cos2 sincos

is a null
sin 2
cossin

Q 7.
Q.8

1 0 2
If, A = 0 2 1 , then show that the maxtrix A is a root of the polynomial f (x) = x3 6x2 + 7x + 2.
2 0 3
For a non zero , use induction to prove that : (Only for XII CBSE)
n

n (n 1) n 2
n
n 1

n
2

n
nn 1
= 0
, for every n N
n
0
0

1 , then (aI + bA)n = anI + nan 1 b A, where I is a unit matrix of order 2, n N.


0

(a)

1 0
0 1
0 0

(b)

If, A = 0
0

Q9.

Find the number of 2 2 matrix satisfying

2 + 2 = 2 + 2 = 2 ; (iii) a a + a a = 0
(i) aij is 1 or 1 ; (ii) a 11
a12
a 21
a 22
11 21
12 22
T
T
T
Q 10. Prove that (AB) = B . A , where A & B are conformable for the product AB . Also verify the result

1 2
2 3 5
for the matrices, A = 2 3 and B = 1 2 3 .

1 2
1 2 5
Q 11 Express the matrix 2 3 6 as a sum of a lower triangular matrix & an upper triangular matrix with zero
1 0 4
in its leading diagonal. Also Express the matrix as a sum of a symmetric & a skew symmetric matrix.
Q 12. Find the inverse of the matrix :

(i)

(iii)

cos sin 0

A = sin cos 0 ;
0
0
1

1 1
(ii) 1 w2
1 w

1
w 2 where w is the cube root of unity..
w

a 0 0
0 b 0

A=
0 0 c

2 1
3 2 2 4
Q 13. Find the matrix A satisfying the matrix equation, 3 2 . A . 5 3 = 3 1 .

Q 14. A is a square matrix of order n.


l = maximum number of distinct entries if A is a triangular matrix
m = maximum number of distinct entries if A is a diagonal matrix
p = minimum number of zeroes if A is a triangular matrix
If l + 5 = p + 2m, find the order of the matrix.
Q 15. If A is an idempotent matrix and I is an identity matrix of the same order, find the value of n, n N ,
such that ( A + I )n = I + 127 A.
a b
Q.16 If A = c d then prove that value of f and g satisfying the maxtrix equation A2 + f A + g I = O are

equal to tr (A) and determinant of A respectively. Given a, b, c, d are non zero reals and
1 0
0 0
I = 0 1 ; O = 0 0 .

36

Q17.
(i)
(ii)
Q18.

2 1
Matrices A and B satisfy AB = B1 where B = 2 0 . Find

1
1
without finding B , the value of K for which KA 2B + I = O
Without finding A1, the matrix X satisfying A1XA = B
(iii)

the matrix A, using A1

4 4 5
For the matrix A = 2 3 3 find A2.
3 3 4

1 1 1
2 3
1 0 1
Q19. Given A = 2 4 1 , B = 3 4 . Find P such that BPA = 0 1 0

2 3 1
Q 20. Use matrix to solve the following system of equations.
x + y+z=3
(i) x +2 y+3z=4
x +4 y+9z=6

x + y + z =6
x + y+z=3
(ii) x y+ z=2 (iii) x +2 y+3z=4
2 x + yz=1
2 x +3y+4z=7

x + y+ z=3
(iv) x + 2 y+3z=4
2 x +3y+4z=9

EXERCISE-5
Q1.

Q 2.

2 1
9 3
Given A = 2 1 ; B = 3 1 . I is a unit matrix of order 2. Find all possible matrix X in the following

cases.
(i)
AX = A
(ii)
XA = I
(iii)
XB = O but BX O.
If A & B are square matrices of the same order & A is symmetrical, show that B AB is also symmetrical.
tan 2

tan 2
cos sin

1 = sin cos .

Q 3.

1
Show that, tan
2

Q.4

a b
1 2
If the matrices A = 3 4 and B = c d

tan 2

(a, b, c, d not all simultaneously zero) commute, find the value of

db
. Also show that the
a +cb

2 3
matrix which commutes with A is of the form

Q 5.

If the matrix A is involutary, show that

1
1
(I + A) and
(I A) are idempotent and
2
2

1
1
(I + A) (I A)=O.
2
2

Q 6.

Prove that
(ii)

Q 7.

(i)

adj (adj A) = A

( n 1) 2

| adj (adj A) | = | A |
n2

, where A is a non-singular matrix of order 'n'.

. A, where | A | denotes the determinant of co-efficient matrix.

5 1 3

Find the product of two matrices A & B, where A = 7 1 5 & B =


1

1
1

solve the following system of linear equations,


x + y + 2z = 1 ; 3x + 2y + z = 7 ; 2x + y + 3z = 2 .

37

1 1 2
3 2 1 and use it to
2 1 3

Q 8.

1 2
If A = 2 4 then, find a non-zero square matrix X of order 2 such that AX = O. Is XA = O.

1 2
If A = 2 3 , is it possible to find a square matrix X such that AX = O. Give reasons for it.

Q 9.

cos sin
cos 2 sin 2
If A = sin cos ; B= sin 2 cos 2 Where 0 < < then prove that BAB = A1. Also

2
4
1
find the least positive value of for which B A B = A .

b
a
Q 10. If c 1 a is an idempotent matrix. Find the value of f(a), where f(x) = x x2, when

bc = 1/4. Hence otherwise evaluate a.


Q 11. If A is a skew symmetric matrix and I + A is non singular, then prove that the matrix
0 5
B = (I A)(I + A)1 is an orthogonal matrix. Use this to find a matrix B given A = 5 0 .

cos x sin x 0
Q 12. If F(x) = sin x cos x 0 then show that F(x). F(y) = F(x + y)
0
0
1
Hence prove that [ F(x) ]1 = F( x).
x1
1 2
3 1
1 2
Q 13. If A = 3 4 ; B = 1 0 ; C = 2 4 and X = x

3
equation.
(a) AX = B I
(b) (B I)X = IC
(c) CX = A

x2
x 4 then solve the following matrix

3 2 1
Q 14. Determine the values of a and b for which the system 5 8 9
2 1 a

x b
y = 3
z 1
(i) has a unique solution ; (ii) has no solution and (iii) has infinitely many solutions

Q 15. Let X be the solution set of the equation

Ax = I, where A =

unit matrix and x N then find the minimum value of


Q16.

0 1 1
4 3 4 and I is the corresponding
3 3 4

(cos x + sin x ) , R.

Determine the matrices B and C with integral element such that


1 1
A=
= B3 + C3
0 2

Q17.

0 2
If A = is an orthogonal matrix, find the values of , , .

k m
Q18.If A =
and kn lm ; then show that A2 (k + n)A + (kn lm) I = O.Hence find A1.
l n

Q19.

Evaluate

1
Lim
x
n

x
n

38

Q.20

x 1
3 3 z
2 y ; B = 3 2 3
z 3 1
y 3
Obtain x, y and z if the matrix AB is symmetric.

1
Given matrices A = x
1

EXERCISE-6
Q.1

a b c

If matrix A = b c a where a, b, c are real positive numbers, abc = 1 and ATA = I, then find the
c a b
value of a3 + b3 + c3 .

Q.2

[JEE 2003, Mains-2 out of 60]

2
If A =
and | 3 | =125, then =
2
(A) 3
(B) 2

(C) 5

(D) 0[JEE 2004 (Screening)]

Q.3

If M is a 3 3 matrix, where MTM = I and det (M) = 1, then prove that det (M I) = 0.

Q.4

a 0 1

A = 1 c b , B =
1 d b

a 1 1
0 d c

, U=
f g h

f
g
,V=
h

a 2

0 .
0

If there is vector matrix X, such that AX = U has infinitely many solution, then prove that BX = V cannot
have a unique solution. If afd 0, then prove that BX = V has no solution.
Q.5

Q.6

1 0 0
A = 0 1 1 , I =
0 2 4
(A) 6, 11
3

2
If P = 1

1 0 0
0 1 0 and A1 =
0 0 1
(B) 6, 11

Q7.

(C) 6, 11

(D) 6, 11

1
2

1 1

APT and x = PTQ2005 P, then x is equal to


3 , A = 0 1 and Q = PAP
2

1 2005
(A) 0
1

(C)

1 2

6 ( A + cA + dI) , then the value of c and d are

1 2 + 3
1

2 3
4 1

4 + 2005 3
6015
(B) 2005
4 2005 3

(D)

1 2005 2 3
4 2 + 3 2005

If f(x) is a quadratic polynomial and a, b, c are three real and distinct numbers satisfying
4a 2
2
4b
4c 2

4a 1 f ( 1) 3a 2

4b 1 f (1) = 3b 2
4c 1 f ( 2) 3c 2

+ 3a

+ 3b
. Given f(x) cuts the x-axis at A and V is the point of mixima.
+ 3c

If AB is any chord which subtends right angle at V, find curve f(x) and area bounded by chord AB and
curve f(x).

39

1 0 0
A = 2 1 0
, if U1, U2 and U3 are columns matrices satisfying.
3 2 1
1
0
AU1 = , AU2 =
0
Q8.
Q9.

Q10.

2
3
, AU3 =
0

2
3
and U is 3 x 3 matrix whose columns are U1, U2, U3 then answer the
1

following questions.
The value of |U| is
(A) 3
(B) -3
The sum of the elements of U-1 is
(A) -1
(B) 0

[JEE 2006]
(C) 3/2

(D) 2

(C) 1

(D) 3

[JEE 2006]

2
3
The value of [3 2 0] U is
0
(A) 5

[JEE 2006]

(B) 5/2

(C) 4

(D) 3/2

ANSWER SHEET

EXERCISE-4
1 0 0
Q.3 EF = 0 1 0 , FE =
0 0 0

0 0 0
0 1 0
0 0 1

Q.1 x1 = 1, x2 = 1, x3 = 1

3
Q.2 x = , y = 2
2

Q.9

2 0 0 3
1 0 0 0 2 5 1 2

0
0
6

2
3
3 + 0 0 3
2
3
0

;
Q.11
+
1 0 4 0 0 0 2 3 4 3 3 0

cos sin 0
1

Q.12 (i) sin cos 0 , (ii)


3
0
1
0
Q.14 4
Q.17

1 1
1 w 2
1 w

1
a

1
0
w , (iii)
w 2
0

Q.15 n = 7

1 2 2
(i) K = 2, (ii) X = B, (iii) A = 4 2
4

0
1
b
0

1
c

Q.13

1 48 25
19 70 42

Q.16 f = (a + d) ; g = ad bc
4 19
17

10
0
13

Q.18
21 3 25

4 7 7
Q.19

3 5 5

Q.20 (i) x = 2, y = 1, z = 0 ; (ii) x = 1, y = 2, z = 3 ;


2k, z = k where k R ; (iv) inconsistent, hence no solution
( i i i )

40

EXERCISE-5
Q.1

b
a
(i) X = 2 2a 1 2b for a, b R ; (ii) X does not exist. ;

a 3a
(iii) X = c 3c a, c R and 3a + c 0 ; 3b + d 0

Q.4
Q.9

Q.7

2c 2d
Q.8 X = c
d , where c, d R {0}, NO

x = 2, y = 1, z = 1

2
3

Q.10 f (a) = 1/4, a = 1/2

Q.11

1 12 5
12
13 5

3 3
2
, (b) X = 1
Q.13(a) X= 5
2

2 , (c) no solution

Q.14 (i) a 3 , b R ; (ii) a = 3 and b 1/3 ; (iii) a = 3 , b = 1/3


0 1
1 0
Q.16 B = 0 1 and C = 0 1

Q.15 2
Q.17

1
1
1
, =
, =
2
6
3

Q.18

1
kn lm

n m
l k

cos x sin x
Q.19

sin x cos x
4 2 2
4 2 2

Q.20 3 , 3 , 2 2 , 3 , 3 , 2 2 , (3, 3, 1)

EXERCISE-6
Q.1

Q.2

Q9.

Q10.

Q.5

Q.6

Q7.

41

125
sq. units
3

Q.8

If a, b, c > 0 & x, y, z
(A)

axbycz

(a +a ) (a a )
(b +b ) (b b )
(c +c ) (c c )

then the determinant

(B)

axbyc z

(C)

x 2

x 2

y 2

y 2

z 2

z 2

bc

2.

If a, b & c are non-zero real

(B) a2 b2 c2

(A) abc

3.

4.
5.

6.

numbers,

c 1+ a1

The determinant b 2 +c 2
b 3 +c 3

c 2 +a 2

a 2 +b 2 =

c 3 +a 3

a 3 +b 3

1 =
1

(D) zero

bc b + c

2 2
then D = c a ca c + a =
a2b2 ab a + b
(C) bc + ca + ab

b1+ c 1

a2xb2yc2z

2 2

DET. & MATRICES/Page : 42 of

1.

R,

54

Exercise - 7
Part : (A) Only one correct option

(D) zero

a1+b1

a1 b1 c 1
a1 b1 c 1
a1 b1 c 1
(B) 2 a 2 b 2 c 2
(C) 3 a 2 b 2 c 2
(A) a 2 b 2 c 2
(D) none of these
a3 b3 c 3
a3 b3 c 3
a3 b3 c 3
The system of linear equations x + y z = 6, x + 2y 3z = 14 and 2x + 5y z = 9
( R) has a unique solution if
(A) = 8
(B) 8
(C) = 7
(D) 7
If the system of equations x + 2y + 3z = 4, x + py + 2z = 3, x + 4y + z = 3 has an infinite number of
solutions, then:
(A) p = 2, = 3
(B) p = 2, = 4
(C) 3 p = 2
(D) none of these
cos 2
cossin sin

cos

sin

sin 2
cos then f =
Let f () =
6
sin
cos
0
(A) 0
(B) 1
(C) 2

(D) none

cos(+) sin(+) cos2


sin

cos

sin

is:

7.

The determinant

8.

cos
sin
cos
(B) independent of
(A) 0
(C) independent of
(D) independent of & both
sin(2 ) sin( + ) sin( + )
Value of = sin( + ) sin(2) sin( + ) is
sin( + )

sin( + )

sin( 2 )

(A) = 0
(C) = 3/2
9.

(B) = sin2 + sin2 + sin2


(D) none of these
0 b c
If a, b, c are complex number and z = b 0 a is
c a
0
(B) purely imaginary
(C) 0
(D) none of these
(A) purely real
sin

10.

If A, B, C are angles of a triangle ABC, then

(A)

3 3
8

sin( A + B + C)
cos

equal to
(B)

1
8

A
2

( A + B + C)
2

(C) 2 2

42

B
2
B
sin
2

C
2
A
cos
2
C
tan( A + B + C) sin
2
sin

(D) 2

sin

is less than or

11.

2a

3a 12 sin A

12.

If

(B) 0
a2

b
c2

(C)

(A) 1

(D)

1
(cos2A + cos2B)
2

b2
= k abc (a + b + c) 3 then the value of k is
2
(a + b )
(B) 2
(C) 0
(D) ab + bc + ac
2r 1

13.

1
sin2A
2

a2

(c + a )
c2

is (where a, b, c are the sides opposite to angles A, B, C respectively in a

cos B

triangle)
1
cos2A
(A)
2
(b + c )2

54

cos A

m 1
2

Let m be a positive integer & Dr =


given by:
(A) 0

( )

sin2 m 2

Cr

m+1

(0 r m) , then the value of

sin2 (m) sin2 (m+1)

14.

15.

1
x
If f(x) =
2x
x ( x 1)
3x ( x 1) x ( x 1) ( x 2)

x +1
then f(100) is equal to:
(x + 1) x
(x + 1) x (x 1)
(D) 100

(A) 0
(B) 1
(C) 100
Part : (B) May have more than one options correct
1

17.

x 2y z

19.

20.

y 2x z

x2

y 2 y z 2x 2y z
(A) x y is a factor of
(C) (x y) 3 is a factor of
sin cos

18.

+ b1x + b2 and = 1( x1 ) 1( x 2 ) 1( x 3 ) , then


2 ( x1 ) 2 ( x 2 ) 2 ( x 3 )
(A) is independent of a1
(B) is independent of b1 and b2
(C) is independent of x 1, x 2 and x 3
(D) none of these

Let 1 (x) = x + a1, 2 (x) =

If =

, then

sin sin

(B) (x y)2 is a factor of


(D) is independent of z
cos

Let = cos cos cos sin sin , then


sin sin sin cos
0
(A) is independent of
(B) is indepedent of
d
=0
d = / 2

(C) is a constant

(D)

1 0
a 1 , then
2
ax ax a
(A) x + a is a factor of
(C) (x + a)3 is a factor of

(B) (x + a)2 is a factor of


(D) (x + a)4 is not a factor of

Let =

r=0

is

(B) m 2 1

(C) 2m
(D) 2m sin2 (2m)
a
1 + 2i
3 5i
b
7 3i then D is
If a, b, c, are real numbers, and D = 1 2i
3 + 5i 7 + 3i
c
(A) purely real
(B) purely imaginary
(C) non real
(D) integer

16.

Dr

a
ax

1
2
Let = x
x

x
1
x2

x2
x , then
1
43

[IIT 1999, 2]

DET. & MATRICES/Page : 43 of

4 sinB
b
8 sin A

21.

b
c

The determinent =
(A) b, c, d are in A.P.
(C) b, c, d are in H.P.

b + c
c + d

c
d

54

(B) (1 x 3)2 is factor of


(D) (1) = 0
is equal to zero if

b + c c + d a 3 c

(B) b, c, d are in G.P.


(D) is a root of ax 3 bx 2 3cx d = 0

Exercise - 8
1.

Using the properties of determinants, evalulate:


103 115 114
113 116 104
111 108 106 + 108 106 111 .

(i)

104 113 116

13 + 3
15 + 26
3 + 65

(ii)

115 114 103

2 5
5
15

5
10 .
5

2.

ax + b
b
x + c = 0.
Find the non zero roots of the equation, =
ax + b bx + c
c

3.

b2 + c 2
ab
Show that =
ca

4.

a
b

ab
c 2 + a2
cb

ac
bc
= 4a2b2c2
a2 + b 2

+++
2( + )( + )

2
Prove that, + + +

b
c

+
( + ) + ( + ) = 0.

( + ) + ( + )

5.

S0
If Sr = r + r + r then show that S1
S2

6.

Find the value of a if the three equations,


(a + 1)3 x + (a + 2)3 y = (a + 3) 3; (a + 1) x + (a + 2) y = (a + 3) & x + y = 1 are consistent.

7.

Investigate for what values of , the simultaneous equations


x + y + z = 6; x + 2 y + 3 z = 10 & x + 2 y + z = have;
(a)
A unique solution
(b)
An infinite number of solutions.
(c)
No solution.

8.

Find those values of c for which the equations:


2x+3y = 3
(c + 2) x + (c + 4) y = c + 6
(c + 2) x + (c + 4) y = (c + 6) are consistent.
Also solve above equations for these values of c.

9.

'+'

S1
S2
S3

S2
S 3 = ( )2 ( )2 ( )2.
S4

' '

Prove that = '+ ' ' ' = (' ') (' ') (' ')
'+ ' ' '
a 2 + ( b 2 + c 2 )cos

10.

If

a2

b2

c2

= 1, then prove that

ab (1 cos )

ba (1 cos )
ca (1 cos )

ac (1 cos )

b 2 + ( c 2 + a 2 )cos
bc (1 cos )
2
cb (1 cos )
c + (a 2 + b 2 )cos

is independent of a, b, c

tan(A + P )
11.

tan(B + P )

Show that the value of the determinant tan(A + Q)

tan(C + P )

tan(B + Q) tan(C + Q) vanishes for all values of


tan(A + R) tan(B + R) tan(C + R)

A, B, C, P, Q & R where A + B + C + P + Q + R = 0.

44

DET. & MATRICES/Page : 44 of

(A) 1 x 3 is a factor of
(C) (x) = 0 has 4 real roots

12.

Prove that a + a c

a c

c2 + a c = (ab + bc + ca)3.

a 2 + a b b2 + a b
cos( x + x 2 )

13.

14.

15.

a b

sin( x + x 2 )

2
2
Show that, sin( x x ) cos( x x )
sin2x
0

cos( x + x 2 )
sin( x x 2 )
sin 2x 2

= sin (2 x + 2 x 2).

1
1
1
a + x b + x c+ x
P
1
1
1
where Q i s t he product of t he denom i nat ors, prov e t hat
If a + y b + y c+ y =
Q
1
1
1
a + z b + z c+ z
P = (a b) (b c) (c a) (x y) (y z) (z x)
If A1, B1, C1,................are respectively the cofactors of the elements a1, b1, c1,...........of the determinant

a1

b1

c1

= a2

b2

a3

b3

c2 then prove that


c3

B2

C2

B3

C3

(i)

16.

54

b 2 + b c c2 + b c

= a1.

b c a2
2
Show that, c a b
a b c2

(ii)

c a b2
a b c2
b c a2

A1

B1

C1

A2

B2

C2 = 2

A3

B3

C3

a b c2
a2
b c a 2 = 2 a b c2
c a b2
b2

c2
b2

2 a c b2
a2

2 b c a2

c2
ap a

17.

Using consistancy of equations, prove that if bc + qr = ca + rp = ab + pq =1 then b q b q = 0.


cr c r

sin

cos 1

18.

Show that : sin

19.

If ax + 2 hxy + by + 2 gx + 2 fy + c (l 1x + m 1y + n1) (l 2x + m 2y + n2), then prove that


a h g
h b f
= 0.
g f c

20.

Find all the values of t for which the system of equations;


(t 1) x + (3 t + 1) y + 2 t z = 0
(t 1) x + (4 t 2) y + (t + 3) z = 0
2 x + (3 t + 1) y + 3 (t 1) z = 0
has non trivial solutions and in this context find the ratios of x: y: z, when t has the smallest of these
values.

21.

Let a > 0, d > 0. Find the value of determinant

sin

1
a
1
(a + d)
1
(a + 2 d )

22.

cos 1 = sin ( ) + sin ( ) + sin ( ).


cos

1
a (a + d )
1
(a + d) (a + 2 d)
1
(a + 2 d) (a + 3 d)

1
(a + d ) (a + 2 d )
1
(a + 2 d) (a + 3 d) .
1
(a + 3 d) (a + 4 d )

[IIT 1996, 5]

Let a, b, c be real numbers with a2 + b2 + c2 = 1. Show that the equation


ax by c
bx + ay
cx + a
bx + ay
ax + by c
cy + b
= 0 represents a straight line
cx + a
cy + b
ax by + c
45

[IIT 2001, 6]

DET. & MATRICES/Page : 45 of

bc
2

54

Exercise - 8

1. D

2. D

3. B

4. B

5. D

6. B

7. B

8. A

9. B

10. B

11. B

12. B

13. A

14. A

15. A

16. AB 17. AB 18. BD 19. ABD

1. (i) 0

20. ABD

(ii) 5(3 2 5 3 )

6. a = 2
7. (a) 3

21. BD

2. x = 2 b/a

(b) = 3, = 10

(c) = 3, 10

8. for c = 0, x = 3, y = 3; for c = 10, x =

1
4
,y=
2
3

20. t = 0 or 3; x: y: z = 1: 1: 1
4 d4

21.

a (a + d) 2 (a + 2 d) 3 (a + 3 d) 2 (a + 4 d)

Exercise - 9
Part : (A) Only one correct option
1.
Let a, b, c, d, u, v be integers. If the system of equations ax + by = u, cx + dy = v has a unique solution in
integers, then
(A) ad bc = 1
(B) ad bc = 1
(C) ad bc 0
(D) ad bc need not be equal to 1
2.
If AB = O for the matrices

3.

cos 2
cos sin
and B =
A=
sin2
cos sin

(A) an odd multiple of


2

(C) an even multiple of


2
3 4
If X =
, then value of Xn is
1 1
3n 4n
(A)

n
n

4.

5.
6.
7.
8.
9.

10.

cos 2
cos sin

then is
sin2
cos sin

2 + n 5 n
(B)

n
n

(B) an odd multiple of


(D) 0

3n ( 4)n
(C) n
n
1 ( 1)

(D) none of these

0 2


If the matrix
is orthogonal, then

1
1
1
(A) =
(B) =
(C) =
(D) all of these
2
6
3
If A, B are two n n non-singular matrices, then
(A) AB is non-singular
(B) AB is singular
(C) (AB)1 = A1 B1
(D) (AB)1 does not exist
If B is a non-singular matrix and A is a square matrix, then det (B1 AB) is equal to
(A) det (A1)
(B) det (B1)
(C) det (A)
(D) det (B)
If A is a square matrix of order n n and k is a scalar, then adj (kA) is equal to
(A) k adj A
(B) kn adj A
(C) kn 1 adj A
(D) kn + 1 adj A
Let A be a matrix of rank r. Then
(A) rank (AT) = r
(B) rank (AT) < r
(C) rank (AT) > r
(D) none of these
If A = dig (2, 1, 3), B = dig (1, 3, 2), then A2B =
(A) dig (5, 4, 11)
(B) dig (4, 3, 18)
(C) dig (3, 1, 8)
(D) B

1 1

If is a cube root of unity and A = 1


1 2

2 , then A1 =

46

DET. & MATRICES/Page : 46 of

Exercise - 7

11.

1 1
1
2
(B) 1
3
1

2
(C) 1
1 1

(A) 1

13.
14.

15.

16.

17.
18.

19.

20.

1
(D)
2

2
1
1 1

If the system of equations ax + y + z = 0,. x + by + z = 0 and x + y + cz = 0, where


a, b, c 1, has a nontrivial solution, then the value of

12.

(B) 0

(C) 1

1
1
1
+
+
is:
1 a 1 b 1 c

(D) None of these

If A is a square matrix of order 3, then the true statement is (where I is unit matrix).
(A) det ( A) = det A
(B) det A = 0
(C) det (A + I) = 1 + det A
(D) det 2A = 2 det A
Which of the following is incorrect
(B) (AT )T = A
(A) A2 B2 = (A + B) (A B)
(C) (AB) n = AnBn, where A, B commute
(D) (A I) ( I + A) = O A2 = I
The value of a for which system of equations, a3x + (a + 1)3y + (a + 2)3z = 0,
ax + (a + 1) y + (a + 2) z = 0, x + y + z = 0, has a nonzero solution is:
(A) 1
(B) 0
(C) 1
(D) none of these
3 0 0
a1 a 2 a 3

0
3
0
and B = b1 b 2 b3 then AB is equal to
If A
0 0 3
c 1 c 2 c 3
(B) 3B
(C) B3
(D) A + B
(A) B
a b
satisfies the equation x 2 (a + d) x + k = 0, then
If A =
c d
(A) k = bc
(B) k = ad
(C) k = a2 + b2 + c2 + d2
(D) ad bc
x
x
x+

x
x
+

x
Let A =
, then A1 exists if
x
x
x +
(A) x 0
(B) 0
(C) 3x + 0, 0
(D) x 0, 0
Identity the correct statement
(A)
If system of n simultaneous linear equations has a unique solution, then coefficient matrix is
singular
(B)
If system of n simultaneous linear equations has a unique solution, then coefficient matrix is
non singular
(C)
If A1 exists, (adj A)1 may or may not exist
cos x sin x 0

sin x cos x 0
F(x) =
, then F(x) . F(y) = F(x y)
(D)
0
0
0
x 3 2

1 y 4
Matrix A =
, if x y z =
2 2 z
64 0 0
88

0 64 0
0
(A)
(B)
0 0 64
0
3

2
If P = 1
2

2
3,A=
2

1 2 + 3

4 1
Comprehension

0
88
0

0
88

68 0 0

0 68 0
(C)
0 0 68

34 0 0

0 34 0
(D)
0 0 34

1 1
AP1 and x = P1Q2005P, then x is equal to

and Q = PAP
0 1

4 + 2005 3
(B)
2005

1 2005
(A)

1
0

(C)

60 and 8x + 4y + 3z = 20, then A(adj A) is equal to

2 3

(D)

1 2005

4 2 + 3

[IIT JEE - 2005 ]

4 2005 3
6015

2 3

2005
[IIT JEE - 2006 ]

47

54

1
2

DET. & MATRICES/Page : 47 of

1
2
(A)

22.

23.
Part
24.
25.

26.

a b (a b)

Matrix b c ( b c) is non invertible if

2 1
0
(A) = 1/2

27.

28.

29.

30.

(B) a, b, c are in A.P. (C) a, b, c are in G.P. (D) a, b, c are in H.P.

1
a
a2

The singularity of matrix cos (p d) x cos px cos (p + d) x depends upon which of the following

sin (p d) x sin px sin (p + d) x


parameter
(A) a
(B) p
(C) x
(D) d
Which of the following statement is true
(A)
Every skew symmetric matrix of odd order is non singular
(B)
If determinant of a square matrix is nonzero, then it non singular
(C)
Rank of a matrix is equal or higher than the order of the matrix
(D)
Adjoint of a singular matrix is always singular
a b
If A =
(where bc 0) satisfies the equations x 2 + k = 0, then
c d
(A) a + d = 0
(B) k = |A|
(C) k = |A|
(D) none of these
1 1 0

0 2 1
If A1 =
, then
0 0 1
(A) | A | = 2
(B) A is non-singular
0
1/ 2 1/ 2

1
1
/2
(C) Adj. A =
(D) A is skew symmetric matrix
0
0
1/ 2

Exercise - 10
1.
2.
3.
4.

1 2 3 1


4 5 6 2
=0
Find x so that [ 1 x 1 ]
3 2 5 3
If A and B are two square matrices such that AB = A & BA = B, prove that A & B are idempotent
3 1
If f (x) = x 2 5x + 7, find f (A) where A =
.
1 2
Prove that the product of matrices
48

54
DET. & MATRICES/Page : 48 of

21.

1
2
1 0 0

2 1 0
0
3
, if U1, U2, and U3 are columns matrices satisfying AU1 = , AU2 = and
A=
0
0
3 2 1
2

3
AU3 = . If U is 3 3 matrix whose columns are U1, U2, U3 then answer the following questions
1
The value of |U| is
[IIT JEE - 2006]
(A) 3
(B) 3
(C) 3/2
(D) 2
The sum of the elements of U1 is
[IIT JEE - 2006]
(A) 1
(B) 0
(C) 1
(D) 3
3

2
The value of [3 2 0] U is
[IIT JEE - 2006]
0
(A) 5
(B) 5/2
(C) 4
(D) 3/2
: (B) May have more than one options correct
5
1 2

4
a

4
The rank of the matrix
is
1 2 a + 1
(B) 2 if a = 1
(C) 1 if a = 2
(D) 1 if a = 6
(A) 2 if a = 6
Which of the following statement is always true
(A) Adjoint of a symmetric matrix is a symmetric matrix
(B) Adjoint of a unit matrix is unit matrix
(C) A (adj A) = (adj A) A
(D) Adjoint of a diagonal matrix is diagonal matrix

6.

7.

8.
9.

10.
11.
12.

13.

14.
15.

16.

1 2

2 3
Compute A1 for the following matrix A =
1 1
x + 2y + 5z = 2; 2x 3y + z = 15 & x + y + z

1
. Hence solve the system of equations;
1
= 3

1
tan / 2
1
cos sin
1
tan / 2
Show that

= sin cos
tan
/
2

1
1

tan / 2

Gaurav purchases 3 pens, 2 bags and 1 instrument box and pays Rs. 41. From the same shop Dheeraj
purchases 2 pens, 1 bag and 2 instrument boxes and pays Rs. 29, while Ankur purchases 2 pens, 2
bags and 2 instrument boxes and pays Rs. 44. Translate the problem into a system of equations. Solve
the system of equations by matrix method and hence find the cost of 1 pen, 1 bag and 1 instrument
box.

1 2 2

2 1 2
, then prove that A2 4A 5I = O.
If A =
2 2 1
(a)
using A1
(b)
without using A1
Having given equations x = c y + b z, y = a z + cx, z = bx + a y where x, y, z are not all zero, prove that
a2 + b2 + c2 + 2 abc 1 = 0.
Consider the system of linear equations in x, y, z:
(sin 3) x y + z = 0
(cos 2) x + 4y + 3z = 0
2x + 7y + 7z = 0
Find the values of for which this system has non trivial solution.
Solve the following systems of linear equations by using the principle of matrix.
(i)
2x y + 3z = 8
(ii)
x+y+ z=9
x + 2y + z = 4
2x + 5y + 7z = 52
3x + y 4z = 0
2x + y z = 0
3 2 3
3 0 3

2
1
1

if A =
. Hence solve the system of equations 2 1 0
4 3 2
4 0 2
Find the rank of the following matrices:
1 2 1 3
0 1
1 2 3 2
1 3 4 3
4 1 2 1

1 0
3
9
12
3
(i) 2 3 5 1
(ii)
(iii)
(iv)

3 1 1 2
3 1
1 3 4 5

1 3 4 1
1
2
0
1

1 1

Compute A1,

Determine the product 7


5
x y + z = 4; x 2 y 2 z =

x
8
2 y


.
y = 1 + z
z
4
3 y
3 1

1
1
0
2

2 0

4 1 1 1


1 3 1 2 2 and use it to solve the system of equations.
3 1 2 1
3
9; 2 x + y + 3 z = 1.
4

17.

a b c

If A = b c a , where a, b, c are real positive numbers, a b c = 1 and AT A = 1, then find the value of
c a b

18.

a3 + b3 + c3.
[IIT JEE - 2003, 2]
If M is 3 3 matrix M has its det.(M) = 1 and MMT = I. Prove that del (M ) = 0.
[IIT JEE - 2004, 2 ]
49

54
DET. & MATRICES/Page : 49 of

5.

cos 2
cos 2
cos sin
cos sin

is the null matrix, when and differ by an odd


and
2
sin
sin2
cos sin
cos sin
multiple of /2.
cos x sin x 0

Given F (x) = sin x cos x 0 . If x , . Then for what values of y,,
2 2
0
0
1
F (x + y) = F (x) F (y).
0 2y z

x y z
obeys the law At A = I.
Find the values of x, y, z if the matrix A =
x y z

19.

Exercise - 9
1. C

2. A

3. D

4. D

5. A

6. C

7. C

8. A

9. B

10. B

11. C

12. A

13. A

14. A

15. B

16. D

17. C

18. B

19. C

20. A

21. A

22. B

23. A

24. ABD

25. ABCD 26. AB

1
7. A1 =
7

4 3 17

3 4 11 & x = 2, y = 3, z = 2
1 1 1

9. Rs. 2, Rs. 15 & Rs. 5

12. = n, n +(1)n

;nI
6

27. CD 28. BD 29. AC 30. BC

Exercise - 10
1.

9
8

3. f(A) = 0

5. y R

13. (i) x = 2; y = 2; z = 2
14. x = 1; y = 2; z = 3
15. (i) 2

(ii) 3

(iii) 2

16. x = 3; y = 2; z = 1
6. x =

1
2

,y=

1
6

,z=

(ii) x = 1; y = 3; z = 5

1
3

50

(iv) 2
17. 4

54
DET. & MATRICES/Page : 50 of

a 2
x
f
a 1 0
a 1 1


If A = 1 b d , B = 0 d c U = g, V = 0 , X = y

0
z
h
1 b c
f g h

and AX = U has infinitely many solution. Prove that BX = V has no unique solution, also prove that if afd 0,
then BX = V has no solution.
[IIT JEE - 2004, 4]

Some questions (AssertionReason type) are given below. Each question contains Statement 1 (Assertion)
and Statement 2 (Reason). Each question has 4 choices (A), (B), (C) and (D) out of which ONLY ONE is
correct. So select the correct choice :Choices are :
(A)Statement 1 is True, Statement 2 is True; Statement 2 is a correct explanation for Statement 1.
(B)Statement 1 is True, Statmnt 2 is True; Statement 2 is NOT a correct explanation for Statement 1.
(C)
Statement 1 is True, Statement 2 is False.
(D)
Statement 1 is False, Statement 2 is True.

1
491.

1
x 2

Statement1 : = (2 + 2 )
x

(2x 2 x )2

(3 + 3 )

(5 + 5 x ) 2 = 0

(3x 3 x )2

(5x 5 x )2

1
Statement2 : = 4

492.

Let f(x) =

1
x 2

(2 2 )
1

1
x

1
x 2

2x
x(x 1)
x(x + 1)
3x(x 1) x(x 1) (x 2) x(x 2 1)
Statement2 : f(x) = 0

Statement2 : Inverse of A exists if R {8}

sin cos 1

sin 0
Let A = cos

0
1
0
Statement1 : A1 = adj (A)

495.

100(101)
2

0 4 1

Let A = 2 3

1 2 1
Statement1 : Inverse of A exists for all R

494.

x 2

(5 5 )
x

x +1

Statement1 : f(100) + f(99) + f(98) + ... + f(1) =

493.

=0

1
x 2

(3 3 )
x

Statement2 : |A| = 1

0 4 1
Statement1 : If A = 2 3 then A1 exist if 8.

1 2 1
Statement2 : A1 exists if | A | = 0.

496.

Let there be a system of equations


6x + 5y + z = 0
3x y + 4z = 0
x + 2y 3z = 0
Statement1 : System of equations has infinite number of nontrivial solution for 5.
6 5
Statement2 : It will have non trivial solution is 3 1 4 = 0 .

497.

Let , , be the roots of the equation x + ax + b = 0; a, b R.

1
3

51 of 54

51


Statement1 : = 0

498.

Statement2 : Any cubic equation over reals has at least one real root.
Let A be a square matrix of order 3 satisfying AA = I.
Statement1 : AA = I
Statement2

499.

500.

: (AB) = B A

p q p r
0

Statement1 : The determinant of a matrix q p


0
q r is zero.
r p r q
0
Statement2 : The determinant of a skew symmetric matrix of odd order is zero.
r 1
r
Statement1 : If Ar =
, where r is a natural number,
r 1 r
A1 + A 2 + . . . A 2006 = ( 2006 )

Statement2 : If A is a matrix of order 3 and |A| = 2, then adj A = 22 .


501.

1 1 0

Statement1 : If matrix A = 1 2 1 then A3 3A2 I = 0

2 1 0

502

Statement2 : If B is a symmetric matrix then B1 will also be symmetric.


Statement1 : Adjoint of a diagonal matrix is diagonal matrix
Statement2 : If |A| = 0 then (adj A) A = A(adjA) = 0

503.

504.

1 3 5

Statement-1: The inverse of the matrix 2 6 10 does not exist.

9 8 7
1 3 5

Statement-2: The matrix 2 6 10 is singular.

9 8 7
a 0 0

Statement-1: If A = 0 b 0 , then A-1 =

0 0 c

0
1/ a 0
0 1/ b 0

0
0 1/ k

Statement-2 : The inverse of a diagonal matrix is a diagonal matrix.


505.

0 2 3

Statement-1: The inverse of the matrix A = 2


0 4 does not exist.

3 4 0
Statement-2: The determinant of a skew-symmetric matrix is zero.

506.

5 8 0

Consider the following matrix A = 3


5 0

1 2 1
Statement-1: A is involutory matrix

507.

Statement-2: A2 = I (identity matrix)

Consider the following system of equation


52 of 54

52

then

Statement-1: Above system of equation will have infinitely many solution if

a
b
c
+
+
=2
1 a 1 b 1 c
a 1 1

Statement-2: Above system of equation will have infinitely many solution if D= 1

b 1=0

1 1 c
508.
509.
510.

511.

Statement-1:
Statement-2:
Statement-1:
Statement-2:
Statement-1:
Statement-2:

If A is a skew symmetric of order 3 then its determinant should be zero


If A is square matrix than detA = detA = det (-A).
If A and B are two matrices such that AB = B, BA = A then A2 + B2 = A + B
A and B are idempotent matrices
The possible dimensions of a matrix containing 32 elements is 6.
The number of ways of expressing 32 as a product of two positive integers is 6.

a b a c
0

Statement-1: The determinants of a matrix b a


0
b c is zero.

c a c b
0
Statement-2: The determinant of a skew symmetric matrix of odd order is zero.

512.

513.

Statement-1: Every square matrix can be uniquely expressed as the sum of a symmetric matrix and a skew
symmetric matrix.
Statement-2: The elements on the main diagonal of a skew symmetric matrix are all different.

1 + a 2 b2
2ab
Statement-1: =
2ab
1 a 2 + b2
2b

2a

2b
2a

27a 2 b 2

1 a 2 b2

Statement-2: A.M. G.M.

514.

2 3 5
Statement-1: The value of = 3 7 0 is 59
4 1 8
Statement-2: The sum of products of elements of a row (column) is zero.

515.

Statement-1: The system of linear equations x + y + z = 6, x + 2y 3z = 14 and 2x + 5y - z = 9( R) half


unique solution. If 8.
Statement-2: A homogenous system is always is consistent for homogenous system, x = y = z = 0 is a
always a solution where determinant 0 i.e., 0.

516.

Statement-1: If is a cube root of unity and A =

0
, then A100 is equal to A

Statement-2: If A, and B are idempotent matrices, then AB is idempotent if A and B commute


n

517.

Statement-1: If A = [aij] is a scalar matrix then trace of A is

ii

8 2 3 3 5
x + y
the value of x = y; y = 1
=
+
x y 1 5 1 2
0

Statement-2: If

53 of 54

53

Answer
491. A

492. D

493. D

494. A

495. C

496. D

497. B

498. B

499. A

500. B

501. B

502. B

503. A

504. B

505. A

506. A

507. A

508. C

509. A

510. C

511. A

512. C

513. A

514. B

515. A

516. B

517. A

518. A

519. A

520. D

521. C

522. C

523. C

524. D

525. A

526. A

527. D

528. C

529. C

530. D

531. B

532. A

533. B

534. A

535. A

536. C

537. A

538. A

539. A

540. A

541. D

542. A

543. C

544. A

545. A

546. A

547. A

548. A

549. A

550. B

551. B

552. B

553. D

554. D

555. B

556. B

557. C

558. D

559. D

560. D

561. A

562. A

563. A

564. D

565. D

566. D

567. D

568. A

569. C

570. C

571. A

54 of 54

54

STUDY PACKAGE
Target: IIT-JEE (Advanced)
SUBJECT: MATHEMATICS
TOPIC: 20 XII M 3. Continuity &
Differentiability
Index:
1. Key Concepts
2. Exercise I to VIII
3. Answer Key
4. Assertion and Reasons
5. 34 Yrs. Que. from IIT-JEE
6. 10 Yrs. Que. from AIEEE

Continuity
1.

A function f(x) is said to be continuous at x = c,


if Limit f(x) = f(c). Symbolically f is continuous at
x c

Limit
x = c if Limit
h0 f(c h) = h0 f(c+h) = f(c).
i.e. LHL at x = c = RHL at x = c equals value of f at x = c.

If a function f (x) is continuous at x = c, the graph of f (x) at the corresponding point { c, f (c) } will not be
broken. But if f (x) is discontinuous at x = c , the graph will be broken at the corresponding point.

(i)

(ii)

(iii)
((i), (ii) and (iii) are discontinuous at x = c)
((iv) is continuous at x = c)

(iv)

A function f can be discontinuous due to any of the following three reasons:


Limit f(x) does not exist i.e. Limit f(x) Limit+ f (x)
(i)
[figure (i)]
x c

(ii)

x c

x c

f (x) is not defined at x = c


[figure (ii)]

Limit f(x) f (c)


[figure (iii)]
x c
Geometrically, the graph of the function will exhibit a break at x= c.
Solved Example # 1 Find whether f(x) is continuous or not at x = 1
x
f(x) = sin
; x<1
2
= [x]
x 1
x
sin
x <1
for continuity at x = 1, we determine, f(1), xlim
f(x) and xlim
Solution
f(x) =
f(x).
2
1
1+
[ x]
x 1
Now, f(1) = [1] = 1
lim f(x) = lim sin x = sin = 1
x 1
x 1
2
2
lim
lim
and
+ f(x) =
+ [x] = 1

(iii)

x 1

x 1

so
f(1) = xlim
f(x) = xlim
f(x)
1
1+
Self practice problems :
1.

f(x) is continuous at x = 1

If possible find value of for which f(x) is continuous at x =


1 sin x
,
1 + cos 2x

x=
=
2
2x

x>
=
Answer
2
4 + 2x 2
Find the values of a and b such that the function

0x<
f(x) = x + a 2 sin x ;
4

x
= 2x cot x + b
4
2

f(x) =

2.

x<

discontinuous

= a cos 2x b sin x
a=

Answer

,b=
6
12

<x
2

is continuous at

and
4
2

3.

If f(x) = (1 + ax) x
= b

x<0
x=0

(x + c) 3 1
=
x

x>0

The find the values of a, b, c, f(x) is continuous at x = 0

2.

Answer

a = In 3, b =

Types of Discontinuity :
(a)

1
,c=1
3

Removable Discontinuity
In case Limit
x c f(x) exists but is not equal to f(c) then the function is said to have a removable

f(x) = f(c) & make it


discontinuity. In this case we can redefine the function such that Limit
x c
continuous at x = c.
Removable type of discontinuity can be further classified as :
(i)
Missing Point Discontinuity :
Where Limit f(x) exists finitely but f(a) is not defined.
x a

e.g. f(x) =

(1 x ) (9 x 2 )
has a missing point discontinuity at x = 1.
(1 x )

(ii)

Isolated Point Discontinuity:


Limit
Where x a f(x) exists & f(a) also exists but;

x 2 16
, x 4 & f (4) = 9 has a break at x = 4.
x a
x4
Irremovable Discontinuity: In case Limit f(x) does not exist then it is not possible to make
Limit f(x)

(b)

f(a). e.g. f(x) =

x c

the function continuous by redefining it. However if both the limits (i.e. L.H. L. & R.H.L.) are
finite, then discontinuity is said to be of first kind otherwise it is nonremovable discontinuity of
second kind.
Irremovable type of discontinuity can be further classified as:
(i)
Finite discontinuity e.g. f(x) = x [x] at all integral x.
1
1
Infinite discontinuity e.g. f(x) =
or g(x) =
at x = 4.
(ii)
(
x

4) 2
x4
1
(iii)
Oscillatory discontinuity e.g. f(x) = sin at x = 0.
x
In all these cases the value of f (a) of the function at x = a (point of discontinuity) may or may not exist
but Limit does not exist.
x a

Discontinuity of st kind
If L.H.L. and R.H.L both exist finitely then discontinuity is said to be of st kind
(d)
Discontinuity of nd kind
If either L.H.L. or R.H.L does not exist then discontinuity is said to be of nd kind
(e)
Point functions defined at single point only are to be treated as discontinuous.
eg. f(x) = 1 x + x 1 is not continuous at x = 1.
Solved Example # 2
If f(x) = x
x<1
= x2
x>1
then check if f(x) is continuous at x = 1 or not if not, then comment on the type of discontinuity.
x x <1
Solution
f(x) = 2
x >1
x
lim f(x) = lim x = 1
(c)

x 1

and

x 1

lim f(x) = lim+ x 2 = 1


x 1+
x 1

lim f(x) = lim+ f(x) = finite

x 1
x 1
and
f(1) is not defined.
So f(x) is discontinuous at x = 1 and this discontinuity is removable missing point discontinuity
Self practice problems :
4.
f(x) = x,
x<1
= x2
x>1
=2
x = 1 which type of discontinuity is there
Answer isolated point discontinuity
5.
f(x) = x
;
x<1
= 2x
1 x
Find which type of discontinuity
it is.
3
Answer
non removable of Ist kind

3.

Jump of discontinuity
ln case of nonremovable discontinuity of the first kind the nonnegative difference between the value
of the RHL at x = c & LHL at x = c is called, the Jump of discontinuity.
Jump of discontinuity = RHL LHL

NOTE : A function having a finite number of jumps in a given interval is called a Piece Wise Continuous
or Sectionally Continuous function in this interval. For e.g. {x} , [x]

Solved Example # 3 f(x) = cos1 {cot x} x <


2

x
Find jump of discontinuity.
= [x] 1
2

Ans. =
1
2

1
cos {cot x} if x < 2
Sol.
f(x) =

[x] 1
if x
2

lim
lim
f(x) =

cos1 {cot x}
x
x
2

= cos1 {0+}
= cot 1 0 =

lim+

f(x) =

lim+

[x] 1 = 1

jump of discontinuity

=1
=

4.

1
2

Continuity in an Interval :
(a)

A function f is said to be continuous in (a, b) if f is continuous at each & every point (a, b).

(b)

A function f is said to be continuous in a closed interval [ a, b ] if:


(i)
f is continuous in the open interval (a, b) &
(ii)
f is right continuous at a i.e. Limit+ f(x) = f(a) = a finite quantity..
x a

(iii)
(c)

f is left continuous at b i.e. Limit


f(x) = f(b) = a finite quantity..
x b

All Polynomials, Trigonometrical functions, Exponential and Logarithmic functions are continuous
in their domains.
(d)
Continuity of {f(x)} and [f(x)] should be checked at all points where f(x) becomes integer.
(e)
Continuity of sgn (f(x)) should be checked at the points where f(x) = 0 (if f(x) is constanly equal
to 0 when x a then x = a is not a point of discontinuity)
(f)
Continuity of a function should be checked at the points where definition of a function changes.
Solved Example # 5 If f(x) = [sin x]
0 x <1
5
2

= Sgn x x 1 x 2 ,
where { . } represents fractional function
4
3

then comment on the continuity of function in the interval [0, 2].


Solution (i)
Continuity should be checked at the end-points of intervals of each definition i.e. x = 0, 1, 2
(ii)
For [sin x], continuity should be checked at all values of x at which sin x
1
i.e.
x = 0,
2
5
2

5
= 0 (as sgn (x) is
(iii)
For sgn x x , continuity should be checked when x
4
3

4
2
5
discontinuous at x = 0)
i.e.
x=
and when x

3
4
5
i.e.
x=
(as {x} is discontinuous when x )
3
1
5 5

overall discontinuity should be checked at x = 0, , 1, ,


and 2
4
2
4 3
check the discontinuity your self.

1
5 5
,1 ,
2
4 3

1
Self practice problems :
6.
If f(x) = sgn x [In x] 1 < x 3
2

= {x 2}
3 < x 3.5
Find the point where the continuity of f(x) should be checked.
3 5
Ans. { 1, , , e, 3 , 10 , 11 , 12 , 3.5 }
2 2
If f & g are two functions which are continuous at x = c then the functions defined by:
5.
F1(x) = f(x) g(x) ; F2(x) = K f(x), K any real number ; F3(x) = f(x).g(x) are also continuous at x = c.
f(x)
Further, if g (c) is not zero, then F4(x) = g( x ) is also continuous at x = c.
Note : (i)
If f(x) is continuous & g(x) is discontinuous at x = a then the product function
(x) = f(x). g(x) may be continuous but sum or difference function (x) = f(x) g(x) will
necessarily be discontinuous at x = a. e.g.
sin x 0
x
f (x) = x & g(x) =
x=0
0
(ii)
If f (x) and g(x) both are discontinuous at x = a then the product function (x) = f(x). g(x) is not
necessarily be discontinuous at x = a. e.g.
1 x0
f (x) = g(x) =
1 x < 0
Solved Example # 6 If f(x) = [sin(x1)] {sin(x1)}

Comment on continuity of f(x) at x = + 1


2
Solution
f(x) = [sin (x 1)] {sin (x 1)}
Let g(x) = [ sin (x 1)] + {sin (x 1} = sin (x 1)

which is continuous at x =
+1
2

+1
as [sin (x 1)] and { sin (x 1)} both are discontinuous at x =
2

At most one of f(x) or g(x) can be continuous at x =


+1
2

+ 1, there fore, f(x) must be discontinuous


As g(x) is continuous at x =
2
lim +

Alternatively, check the continuity of f(x) by evaluling x f(x) and f + 1
2

Answer

6.

discontinuous at x =

Continuity of Composite Function :

If f is continuous at x = c & g is continuous at x = f(c) then the composite g[f(x)] is continuous at


x sin x
xsinx
x = c. eg. f(x) = 2
& g(x) = x are continuous at x = 0, hence the composite (gof) (x) = 2
x +2
x +2
will also be continuous at x = 0.
x +1
1
Solved Example # 7 If f(x) =
and g(x) =
, then discuss the continuity of f(x), g(x) and fog (x).
x 1
x2
x +1
Sol.
f(x) =
x 1
f(x) is a rational function it must be continuous in its domain
and f is not defined at x = 1

f is discontinuous at x = 1
1
g(x) =
x2
g(x) is also a rational function. It must be continuous in its domain and fog is not defined at x = 2

g is discontinuous at x = 2
Now fog (x) will be discontinuous at
(i)
x=2
(point of discontinuity of g(x))
(ii)
g(x) = 1
(when g(x) = point of discontinuity of f(x))
if
g(x) = 1
1
=1

x=3

x2

discontinuity of fog(x) should be checked at x = 2 and x = 3


at x = 2
1
+1
x2
fog (x) = 1
fog (2) is not defined
1
x2
5

1
+1
x

2
1+ x 2
lim fog (x) = lim
= xlim
=1
1
x 2
x 2
2 1 x + 2
1
x2

fog (x) is discontinuous at x = 2 and it is removable discontinuity at x = 3


fog (3) = not defined
1
+1
x

2
lim+ fog (x) = lim+
=
1
x 3
x 3
1
x2
1
+1
x

2
lim fog (x) = lim
=
1
x 3
x 3
1
x2

fog (x) i s discontinuous at x = 3 and it is non removable discontinuity of nd kind.


Self practice problems :
1

( x 1) 3 , x < 0
1 + x 3 , x < 0
g( x ) =
f ( x) = 2
1
7.
x 1 , x 0

2
+
(
x
1
)
, x0

Then defined fog (x) and comment the continuity of gof(x) at x = 1


Ans. [fog(x) = x, x R gof(x) is discontinous at x = 0, 1]

7.

Intermediate Value Theorem :

A function f which is continuous in [a , b] possesses the following properties:


(i)
If f(a) & f(b) possess opposite signs, then there exists at least one solution of the equation
f(x) = 0 in the open interval (a, b).
(ii)
If K is any real number between f(a) & f(b), then there exists at least one solution of the
equation f(x) = K in the open inetrval (a, b).
Solved Example # 7 Given that a > b > c > d then prove that the equation (x a) (x c) + 2(x b) (x d) =
0 will have real and distinct roots.
Solution
(x a) (x c) + 2 (x b) (x d) = 0
f(x) = (x a) (x c) + 2 (x b) (x d)
f(a) = (a a) (a c) + 2 (a b) (a d) = + ve
f(b) = (b a) (b c) + 0 = ve
f(c) = 0 + 2 (c b) (c d) = ve
f(d) = (d a) (d c) + 0 = +ve

hence (x a) (x c) + 2(x b) (x d) = 0
have real and distinct root
Self practice problems :
f(x) = xex 2 then show that f(x) = 0 has exactly one root in the interval (0, 1).
8.
Solved Example # 8

Let f(x) = Lim


n

Sol.

Let f(x) = nlim

f
4

Now

, then find f and also comment on the continuity at x = 0


1 + n sin x
4
Ans. [Discontinous, removable discontinuity of Isolated type]
1

1 + n sin2 x
1
= nlim

1 + n . sin 2
4
1
lim
= n
=0
1
1+ n
2

f(0)

= nlim

1
n . sin (0) + 1
2

1
=
=1
1+ 0

1
lim f(x) = lim lim

x 0
x 0 n 1 + n sin2 x

1
=

6
1 +
2
{here sin x is very small quantity but not zero and very small quantity when multiplied

with becomes }
Self practice problems :
9.
f(x) = Lim (1 + x)n

f(x) is not continuous at x = 0

(1 + x ) x = e
Comment on the continuity of f(x) at 0 and explain Lim
x 0
Ans. Discontinous (non-removable)
Solved Example # 9
f(x) = maximum (sin t, 0 t x), 0 x 2 discuss the continuity of this function at x=
Solution

f(x) = maximum (sin t, 0 t x), 0 x 2


if x 0, , sin t is increasing function
2
Hence if t [0, x], sin t will attain its maximum value at t = x.

f(x) = sin x if x 0,
2

if x , 2 and t [0, x]
2

then sin t will attain its maximum value when t =

Now

= 1 if x , 2
2


sin x , if x 0,
2

f(x) =

, if x , 2
1
2

f(x) = sin

f =1
2
lim +
lim+

f(x) = x sin x = 1
x
2

lim +

as

f(x) =

lim+

1=1

f(x) = L.H.S. = R.H.S.

f(x) is continuous at x =

Short Revesion (CONTINUITY)


THINGS TO REMEMBER :
1.
A function f(x) is said to be continuous at x = c, if Limit f(x) = f(c). Symbolically
x c

2.
(i)

f is continuous at x = c if Limit f(c - h) = Limit f(c+h) = f(c).


h 0
h 0
i.e. LHL at x = c = RHL at x = c equals Value of f at x = c.
It should be noted that continuity of a function at x = a is meaningful only if the function is defined in the
immediate neighbourhood of x = a, not necessarily at x = a.
Reasons of discontinuity:
Limit f(x) does not exist
x c

i.e. Limit f(x) Limit f (x)


+
x c

x c

(ii)
(iii)

f(x) is not defined at x= c


Limit f(x) f (c)
x c
Geometrically, the graph of the function will exhibit a break at x= c. The graph as shown is discontinuous
at x = 1 , 2 and 3.
3.
Types of Discontinuities :
Type - 1: ( Removable type of discontinuities)
In case Limit f(x) exists but is not equal to f(c) then the function is said to have a removable discontinuity
x c

or discontinuity of the first kind. In this case we can redefine the function such that Limit f(x) = f(c) &
x c

(a)

make it continuous at x= c. Removable type of discontinuity can be further classified as :


M ISSING POINT DISCONTINUITY : Where Limit f(x) exists finitely but f(a) is not defined.
x a

2
sin x
has a missing point
e.g. f(x) = (1 x )(9 x ) has a missing point discontinuity at x = 1 , and f(x) =
x
(1 x )
discontinuity at x = 0

(b)

ISOLATED POINT DISCONTINUITY : Where Limit f(x) exists & f(a) also exists but ; Limit f(a).
x a

x a

x 16
, x 4 & f (4) = 9 has an isolated point discontinuity at x = 4.
x4
2

e.g. f(x) =

0 if x I

has an isolated point discontinuity at all x I.


Similarly f(x) = [x] + [ x] =
1 if x I

Type-2: ( Non - Removable type of discontinuities)


In case Limit f(x) does not exist then it is not possible to make the function continuous by redefining it.
x c

Such discontinuities are known as non - removable discontinuity or discontinuity of the 2nd kind.
Non-removable type of discontinuity can be further classified as :
(a)

Finite discontinuity e.g. f(x) = x [x] at all integral x ; f(x) = tan 1


( note that f(0+) = 0 ; f(0) = 1 )

(b)
(c)

Infinite discontinuity e.g. f(x) =

1
at x = 0 and f(x) =
x

1
1
1+ 2 x

at x = 0

cosx

1
1
or g(x) =
at x = 4 ; f(x) = 2tanx at x = and f(x) =
2
2
x
(x 4)
x4

at x = 0.
Oscillatory discontinuity e.g. f(x) = sin 1 at x = 0.
x
In all these cases the value of f(a) of the function at x= a (point of discontinuity) may or may not exist but
Limit does not exist.
x a

Note: From the adjacent graph note that


f is continuous at x = 1
f has isolated discontinuity at x = 1
f has missing point discontinuity at x = 2
f has non removable (finite type)
discontinuity at the origin.
4.

In case of dis-continuity of the second kind the non-negative difference between the value of the RHL at
x = c & LHL at x = c is called THE JUMP OF DISCONTINUITY. A function having a finite number of jumps
in a given interval I is called a PIECE WISE CONTINUOUS or SECTIONALLY CONTINUOUS function in this
8
interval.
5.All Polynomials, Trigonometrical functions, exponential & Logarithmic functions are continuous in their domains.

6.

If f & g are two functions that are continuous at x= c then the functions defined by :
F1(x) = f(x) g(x) ; F2(x) = K f(x) , K any real number ; F3(x) = f(x).g(x) are also continuous at
x= c. Further, if g (c) is not zero, then F4(x) =

7.

f (x)
g(x)

is also continuous at x= c.

The intermediate value theorem:


Suppose f(x) is continuous on an interval I , and a
and b are any two points of I. Then if y0 is a number
between f(a) and f(b) , their exists a number c
between a and b such that
f(c) = y0.

NOTE VERY CAREFULLY THAT :


(a)
If f(x) is continuous & g(x) is discontinuous at x = a then the product function (x) = f(x). g(x) is not
necessarily be discontinuous at x = a. e.g.
sin x

f(x) = x & g(x) =

x0
x=0

(b)

If f(x) and g(x) both are discontinuous at x = a then the product function (x) = f(x). g(x) is not necessarily
be discontinuous at x = a. e.g.

(c)

f(x) = g(x) =
1 x < 0
Point functions are to be treated as discontinuous. eg. f(x) = 1 x + x 1 is not continuous at x = 1.
A Continuous function whose domain is closed must have a range also in closed interval.
If f is continuous at x = c & g is continuous at x = f(c) then the composite g[f(x)] is continuous at x = c.

(d)
(e)

eg. f(x) =

x sin x
x +2
2

x0

& g(x) = x are continuous at x = 0 , hence the composite (gof) (x) =

x sin x
x2 + 2

7.
(a)
(b)
(i)

be continuous at
x=0.
CONTINUITY IN AN INTERVAL :
A function f is said to be continuous in (a , b) if f is continuous at each & every point (a , b).
A function f is said to be continuous in a closed interval[a , b] if :
f is continuous in the open interval (a , b)
&

(ii)

f(x) = f(a) = a finite quantity..


f is right continuous at a i.e. Limit
x a +

(iii)

f is left continuous at b i.e. Limit


f(x) = f(b) = a finite quantity..
x b

(i)
(ii)
8.

will also

Note that a function f which is continuous in [a , b] possesses the following properties :


If f(a) & f(b) possess opposite signs, then there exists at least one solution of the equation f(x) = 0 in the
open interval (a , b).
If K is any real number between f(a) & f(b), then there exists at least one solution of the equation
f(x) = K in the open inetrval (a , b).
SINGLE POINT CONTINUITY:
Functions which are continuous only at one point are said to exhibit single point continuity
e.g. f(x) =

x if x Q
x if x Q

and g(x) =

x if x Q
are both continuous only at x = 0.
0 if x Q

EXERCISE1
ln cos x

Q 1.

Let f(x) =

1 + x2 1

if x > 0

e sin 4 x 1
if x < 0
ln(1 + tan 2 x)
Is it possible to define f(0) to make the function continuous at x = 0. If yes what is the value of f(0), if not
then indicate the nature of discontinuity.

Q 2.

Q 3.

Suppose that f(x)

= x3

f (x )
x3

- 3x2 - 4x + 12 and h(x) =

, x3
, x=3

then

(a) find all zeros of f (x)


(b) find the value of K that makes h continuous at x = 3
(c) using the value of K found in (b), determine whether h is an even function.
x2
x2
x2
+
+
............
+
9
Let yn(x) = x2 +
1 + x 2 (1 + x 2 )2
(1 + x 2 ) n 1

Q 4.

Q 5.

Q 6.

Q 7.
Q 8.

y (x )
and y(x) = Limit
n n
Discuss the continuity of yn(x) (n = 1, 2, 3..........n) and y(x) at x = 0
Draw the graph of the function f(x) = x x x, 1 x 1 & discuss the continuity or discontinuity of
f in the interval 1 x 1.
1 sin x
, x < 12
1+ cos 2 x
, x = 12 . Determine the value of p , if possible, so that the function is
Let f(x) = p
2x 1

, x > 12
4 + 2 x 1 2

continuous at x = 1/2.
Given the function g (x) = 6 2x and h (x) = 2x2 3x + a. Then
g ( x ), x 1

, find 'a' so that f is continuous.


(b) If f (x) =
(a) evaluate h ( g(2) )
h ( x ), x > 1
1 + x , 0 x 2

. Determine the form of g(x) = f [f(x)] & hence find the point of
Let f(x) =
3 x , 2 < x 3
discontinuity of g , if any.
Let [x] denote the greatest integer function & f(x) be defined in a neighbourhood of 2 by
[ x +1]

4 16
(exp {( x + 2) n4})
, x <2
x
4 16
f (x) =
.
1cos( x 2)

, x >2
A ( x 2)tan ( x 2)
Find the values of A & f(2) in order that f(x) may be continuous at x = 2.
6x
6 tan
tan 5 x
5)
The function f(x) = b + 2

1 + cos x

Q 9.

tan x
b

if
if

0 < x < 2
x = 2

if

<x<

Determine the values of 'a' & 'b' , if f is continuous at x = /2.


1
x 2 sin , if x 0

x
Q.10 Let f (x) =

0
if x = 0
Use squeeze play theorem to prove that f is continuous at x = 0.
Q.11

Let f (x) = x + 2,
4x0
= 2 x2,
0<x4
then find f ( f (x ) ) , domain of f ( f ( x) ) and also comment upon the continuity of f ( f ( x ) ) .
1 + x 3 , x < 0
x 2 1 , x 0

Q 12. Let f(x) =

Q.13

Q.14

(x 1)1/ 3 , x < 0
. Discuss the continuity of g (f (x)).
1/ 2
, x0
(x + 1)

; g(x) =

1 sin 3 x if x <
2
3 cos 2 x

if x = 2
Determine a & b so that f is continuous at x = . f(x) = a
b (1 sin x )
2

( 2 x ) 2 if x > 2
sin (a +1) x + sin x

x
Determine the values of a, b & c for which the function f (x) = c
1/ 2
1/ 2
2
(x+ bx ) x
3/ 2
bx

for x < 0
for x = 0
for x > 0

is continuous at x = 0.

Q.15 If f(x) = sin 3 x + A sin 2 x + B sin x (x 0) is cont. at x = 0. Find A & B. Also find f(0).
x5

Do not use series expansion or L' Hospital's rule.


Q.16

1
x 3 1
Discuss the continuity of the function f defined as follows : f(x) = x + 1
x +1
x 5

for 0 x 2
for 2 < x 4 and draw the
for 4 < x 6

graph of the function for x [0, 6]. Also indicate the nature of discontinuities if any.
Q.17 If f(x) = x + {-x} + [x] , where [x] is the integral part & {x} is the fractional part of x. Discuss the
continuity of f in [ -2 , 2 ].
10

Q.18
Q.19
Q.20
Q.21
Q.22

Q.23

Q.24
(b)

ax b for
x 1
Find the locus of (a, b) for which the function f (x) = 3x
for 1 < x < 2
2
x2
bx a for
is continuous at x = 1 but discontinuous at x = 2.
2
Prove that the inverse of the discontinuous function y = (1 + x ) sgn x is a continuous function.
sin 2 (2 x )
x n f (x ) + h(x ) + 1
Lim
, x 1 and g (1) = x 1
Let g (x) = Lim
be a continuous function
ln sec(2 x )
n
2 x n + 3x + 3
at x = 1, find the value of 4 g (1) + 2 f (1) h (1). Assume that f (x) and h (x) are continuous at x = 1.
If g : [a, b] onto [a, b] is continous show that there is some c [a, b] such that g (c) = c.
2 + cos x 3
is not defined at x = 0. How should the function be defined at
The function f(x) = 3
x sin x x 4
x = 0 to make it continuous at x = 0. Use of expansion of trigonometric functions and L Hospitals rule
is not allowed.
a sin x a tan x
for x > 0
f (x) =
tan x sin x
ln (1 + x + x 2 ) + ln (1 x + x 2 )
for x < 0, if f is continuous at x = 0, find 'a'
=
sec x cos x
x

now if g (x) = ln 2 cot (x a) for x a, a 0, a > 0. If g is continuous at x = a then show that


a

g(e1) = e.
(a)
Let f(x + y) = f(x) + f(y) for all x , y & if the function f(x) is continuous at x = 0 , then show that
f(x) is continuous at all x.
If f(x. y) = f(x). f(y) for all x , y and f(x) is continuous at x = 1. Prove that f(x) is continuous for all x except
at x = 0. Given f(1) 0.

x
x
sec r 1 ; r , n N
r
2
2
r=1
n
x
 n f (x) + tan n f (x) + tan xn . sin tan x
2
2
2
Limit

Q.25 Given f (x) =


g (x) =

tan

) (
) [ ( )]
1 + (f (x) + tan x )
2

= k for x =
and the domain of g (x) is (0 , /2).
4
where [ ] denotes the greatest integer function.
Find the value of k, if possible, so that g (x) is continuous at x = /4. Also state the points of discontinuity
of g (x) in (0 , /4) , if any.
f (x )
where h is a function such that
Q.26 Let f (x) = x3 x2 3x 1 and h (x) =
g( x )
1
(a) it is continuous every where except when x = 1, (b) Lim h ( x ) = and (c) Lim h ( x ) = .
x 1
2
x
Find Lim (3h ( x ) + f ( x ) 2g ( x ) )
x 0

Q.27 Let f be continuous on the interval [0, 1] to R such that f (0) = f (1). Prove that there exists a point c in
1
1

0, 2 such that f (c) = f c + 2

Q.28

1 ax + xax  na
for x < 0

a x x2
Consider the function g(x) = x x
where a > 0.
2 a xn2 xna 1
for x > 0

x2

Without using , L 'Hospital's rule or power series , find the value of 'a' & 'g(0)' so that the function g(x) is
continuous at x = 0.

Q.29

))

sin 1 1 {x}2 . sin 1 (1 {x})


2
for x 0
2 {x} {x}3
where {x} is the fractional part of x.
Let f(x) =

for x = 0
2

Consider another function g(x) ; such that


g(x) = f(x) for x 0
11 of the functions f(x) & g(x) at x = 0.
= 2 2 f(x) for x < 0
Discuss the continuity

4 x 5 [x] for x > 1


; where [x] is the greatest
for x 1
[ cos x]

Q.30 Discuss the continuity of f in [0,2] where f(x) =


integer not greater than x. Also draw the graph.

EXERCISE2
(OBJECTIVE QUESTIONS)

Q 1.

State whether True or False.

(a).

If f(x) =

tan ( 4 x)

for x

cot 2x

, then the value which can be given to f(x) at x =

so that the function

becomes continuous every where in (0,/2) is 1/4.


(b)

The function f, defined by f(x) =

(c)

f(x) = Limit
n

(d)

2 x + 1 if
The function f(x) = x 1 if
x + 2 if

1
1 + n sin 2 x

is continuous for real x.

1 + 2 tan x

is continuous at x = 1.
3 < x < 2
2 x < 0

is continuous everywhere in (3 , 1).

0 x <1
x

for x 0 & f(0) = 1 is continuous at x = 0.

(e)

The function defined by f(x)=

(f)

The function f(x) = 2 2

(g)
(h)

The function f(x) = 2x (x 3 1) + 5 x (1 x 4 ) + 7x 2 (x 1) + 3x + 2 is continuous at x = 1.


There exists a continuous function f: [0, 1] [0, 10], but
there exists no continuous function g : [0, 1] (0, 10).
Fill in the blanks
1
1 cos( cx )
Given f(x) =
, x 0 & f (0) = . If f is continuous at x = 0, then the value of c is ______.
2
xsin x
1
The function f(x) =
has non removable discontinuity at x = ______ & removable discontinuity at x

Q 2.
(a)
(b).

1 /(1 x )

x + 2x 2

if x 1 & f(1) = 1 is not continuous at x = 1.

1n x

= ______ respectively.
1

(c).

If f(x) is continuous in [0 , 1] & f(x) = 1 for all rational numbers in [0 , 1] then f

(d).

x + a 2 sin x
,

,
The values of 'a' & 'b' so that the function f (x) = 2 x cot x + b
a cos 2 x b sin x ,

is continuous for 0 x are _______ & _______.


2 cos x 1
cot x 1

(e).

If f(x)=

is continuous at x =

Q3.

Indicate the correct alternative(s):

(a)

The function defined as f(x) = Limit


n

= ______.

0x<
4
x
4
2
<x
2


then f = ______.
4
4

cos x x 2 n sin (x 1)
1 + x 2n +1 x 2 n

(A) is discontinuous at x = 1 because f(1+) f(1)


(B) is discontinuous at x = 1 because f(1) is not defined
(C) is discontinuous at x = 1 because f(1+) = f(1) f(1)

(D) is continuous at x = 1
n2

(b)

Let f be a continuous function on R. If f (1/4n) = sin e n e n + 2


then f(0) is :
n +1
(A) not unique
(B) 1
(C) data sufficient to find f(0)
(D) data insufficient to find f(0)

(c)

Indicate all correct alternatives if, f (x) =

(d)

f is a continuous function on the real line. Given that


x + (f(x) 2) x 3 . f(x) + 2 3 3 = 0. Then the12value of f( 3 )

x
1 , then on the interval [ 0, ]
2
1
1
(A) tan ( f (x) ) &
are both continuous
(B) tan ( f (x) ) &
are both discontinuous
f (x)
f (x)
1
(C) tan ( f (x) ) & f 1 (x) are both continuous (D) tan ( f (x) ) is continuous but
is not
f (x )

(A) can not be determined

(B) is 2 (1 3 )

(C) is zero

(D) is

(e)

(f)

(g)

32
3

If f (x) = sgn (cos 2 x 2 sin x + 3) , where sgn ( ) is the signum function , then f (x)
(A) is continuous over its domain
(B) has a missing point discontinuity
(C) has isolated point discontinuity
(D) has irremovable discontinuity.
[x ]
Let g(x) = tan1|x| cot1|x|, f(x) =
{x}, h(x) = g(f (x)) where {x} denotes fractional part and
[x + 1]
[x] denotes the integral part then which of the following holds good?
(A) h is continuous at x = 0
(B) h is discontinuous at x = 0

(C) h(0) =
(D) h(0+) =
2
2
x n sin x n
Consider f(x) = Lim it n
for x > 0, x 1
n x + sin x n
f(1) = 0
then
(A) f is continuous at x = 1
(B) f has a finite discontinuity at x = 1
(C) f has an infinite or oscillatory discontinuity at x = 1.
(D) f has a removable type of discontinuity at x = 1.

[{| x|}] e {[x + {x}]}


Given f(x) =
x2

(h)

for x 0
12

e x 1 sgn (sin x)

=0
for x = 0
where {x} is the fractional part function; [x] is the step up function and sgn(x) is the signum function of x
then, f(x)
(A) is continuous at x = 0
(B) is discontinuous at x = 0
(C) has a removable discontinuity at x = 0
(D) has an irremovable discontinuity at x = 0
2
x[x] log(1+x ) 2 for 1 < x < 0

(i)

Consider f(x) =

(j)

(k)

ln e x + 2
2

{x} )

for 0 < x < 1


tan x
where [ * ] & {*} are the greatest integer function & fractional part function respectively, then
(B) f(0) = 2 f is continuous at x = 0
(A) f(0) = ln2 f is continuous at x = 0
(D) f has an irremovable discontinuity at x = 0
(C) f(0) = e2 f is continuous at x = 0
1+ x 1 x
Consider
f(x) =
x0
{x}

g(x) = cos2x
<x<0
4
1
f (g(x))
for x < 0
2
for x = 0
h(x) 1
f (x)
forx > 0
then, which of the following holds good.
where {x} denotes fractional part function.
(A) 'h' is continuous at x = 0
(B) 'h' is discontinuous at x = 0
(C) f(g(x)) is an even function
(D) f(x) is an even function
2x 1
The function f(x) = [x]. cos
, where [] denotes the greatest integer function, is discontinuous at
2
(A) all x
(B) all integer points
(C) no x
(D) x which is not an integer

EXERCISE3

Q.1

, where [] denotes the greatest integer function. The domain of f is_____ & the
[ x + 1]
points of discontinuity of f in the domain are ______.
[ JEE 96, 2 ]

Let f(x) = [x] sin

13

Q.2

Let f (x) be a continuous function defined for 1 x 3. If f (x) takes rational values for all x and f (2) = 10,
then f (1.5) = _______.
[ JEE 97, 2 ]

Q.3

The function f(x) = [x]2 [x2] (where [y] is the greatest integer less than or equal to y), is discontinuous
at :
(A) all integers
(B) all integers except 0 & 1
(C) all integers except 0
(D) all integers except 1
[ JEE '99, 2 (out of 200) ]

Q.4

Q.5

1/ x
for x < 0
(1 + ax)
Determine the constants a, b & c for which the function f(x) = b
for x = 0 is continuous at
(x + c)1/3 1

for x > 0
1/ 2
(x + 1) 1

x = 0.
Discuss the continuity of the function
e1/(x 1) 2

, x1
1/( x 1)
+2
f(x) = e
1,
x =1
at x = 1.

[ REE '99, 6 ]

[ REE 2001 (Mains) , 3 out of 100 ]

Short Revesion (DIFFERENTIABILITY)


THINGS TO REMEMBER :
1.
Right hand & Left hand Derivatives ;
f ( a + h ) f ( a )
if it exist
By definition : f (a) = Limit
h 0
h
(i)
The right hand derivative of f at x = a
denoted by f (a+) is defined by :
f ( a + h ) f ( a )
f ' (a+) = Limit
,
h 0 +
h
provided the limit exists & is finite.
(ii)
The left hand derivative : of f at x = a
denoted by f (a+) is defined by :
f ( a h ) f ( a )
f ' (a) = Limit
,
h 0 +
h
Provided the limit exists & is finite.
We also write f (a+) = f +(a) & f (a) = f _(a).
* This geomtrically means that a unique tangent with finite slope can be drawn at x = a as shown in the
figure.
(iii)
Derivability & Continuity :
(a)
If f (a) exists then f(x) is derivable at x= a f(x) is continuous at x = a.
If a function f is derivable at x then f is continuous at x.
(b)
f ( x + h ) f ( x )
For : f (x) = Limit
exists.
h 0
h
f ( x + h ) f ( x )
Also f ( x + h ) f ( x ) =
.h [ h 0 ]
h
Therefore :
f (x + h ) f (x )
Limit
Limit
.h = f '( x ).0 = 0
=
[
f
(
x
h
)
f
(
x
)
]
+

h 0
h 0
h
Therefore Limit [f ( x + h ) f ( x )] = 0 Limit f (x+h) = f(x) f is continuous at x.
h 0

h 0

Note : If f(x) is derivable for every point of its domain of definition, then it is continuous in that domain.
The Converse of the above result is not true :
IF f IS CONTINUOUS AT x , THEN f IS DERIVABLE AT x IS NOT TRUE.
e.g. the functions f(x) = x & g(x) = x sin 1 ; x 0 & g(0) = 0 are continuous at
x
x = 0 but not derivable at x = 0.
NOTE CAREFULLY :
Let f +(a) = p & f _(a) = q where p & q are finite then :
(a)
(i)
p = q f is derivable at x = a f is continuous at x = a.
(ii)
p q f is not derivable at x = a.
It is very important to note that f may be still continuous at x = a.
In short, for a function f :
Differentiability Continuity
;
Continuity
/ derivability ;
14
Non derivibality
discontinuous
;
But
discontinuity
Non derivability
/

(b)
3.
(i)
(ii)
NOTE
2.
3.
4.

5.

If a function f is not differentiable but is continuous at x = a it geometrically implies a sharp corner at


x = a.
DERIVABILITY OVER AN INTERVAL :
f (x) is said to be derivable over an interval if it is derivable at
each & every point of the interval f(x) is said to be derivable over the closed interval [a, b] if :
for the points a and b, f (a+) & f (b ) exist &
for any point c such that a < c < b, f (c+) & f(c ) exist & are equal.
:1.
If f(x) & g(x) are derivable at x = a then the functions f(x) + g(x), f(x) g(x) , f(x).g(x)
will also be derivable at x = a & if g (a) 0 then the function f(x)/g(x) will also be derivable at x = a.
If f(x) is differentiable at x = a & g(x) is not differentiable at x = a , then the product function F(x) = f(x).
g(x) can still be differentiable at x = a e.g. f(x) = x & g(x) = x.
If f(x) & g(x) both are not differentiable at x = a then the product function ;
F(x) = f(x). g(x) can still be differentiable at x = a e.g. f(x) = x & g(x) = x.
If f(x) & g(x) both are non-deri. at x = a then the sum function F(x) = f(x) + g(x) may be a differentiable
function. e.g. f(x) = x & g(x) = x.
If f(x) is derivable at x = a
/ f (x) is continuous at x = a.
x 2 sin x1

e.g. f(x) =

if x 0
if x = 0

6.

A surprising result : Suppose that the function f (x) and g (x) defined in the interval (x1, x2) containing
the point x0, and if f is differentiable at x = x0 with f (x0) = 0 together with g is continuous as x = x0 then
the function F (x) = f (x) g (x) is differentiable at x = x0
e.g. F (x) = sinx x2/3 is differentiable at x = 0.

Q.1

Discuss the continuity & differentiability of the function f(x) = sinx + sinx , x R. Draw a rough sketch
of the graph of f(x).
Examine the continuity and differentiability of f(x) = x + x 1 + x 2 x R.
Also draw the graph of f(x).
Given a function f (x) defined for all real x, and is such that
f (x + h) f (x) < 6h2 for all real h and x. Show that f (x) is constant.

EXERCISE4

Q.2
Q.3

Q.4

A function f is defined as follows : f(x) = 1 + sin x

2 + x 2

for

<x<0

for

0 x < 2

for

x<+

Q.6

Discuss the continuity & differentiability at x = 0 & x = /2.


Examine the origin for continuity & derrivability in the case of the function f defined by
f(x) = x tan1(1/x) , x 0 and f(0) = 0.
Let f (0) = 0 and f ' (0) = 1. For a positive integer k, show that
1
x
x
1 1
1
Lim f ( x ) + f + ......f = 1 + + + ...... +
x 0 x
2
k
2 3
k

Q.7
Q.8

Let f(x) = xe x ; x 0 , f(0) = 0, test the continuity & differentiability at x = 0


If f(x)=x 1. ( [x] [x]) , then find f (1+) & f (1-) where [x] denotes greatest integer function.

Q.5

Q.9
Q.10

1 + 1x

a x 2 b if x < 1
is derivable at x = 1. Find the values of a & b.
1
if x 1
x
1 , 2 x 0
Let f(x) be defined in the interval [-2, 2] such that f(x) =
&
x 1 , 0<x2

If f(x) =

g(x) = f(x) + f(x). Test the differentiability of g(x) in ( 2, 2).

Q.11

2 [x ]
where sgn (.) denotes the signum function & [.] denotes the greatest
3x [x ]

Given f(x) = cos1 sgn

integer function. Discuss the continuity & differentiability of f (x) at x = 1.


Q.12 Examine for continuity & differentiability the points x = 1 & x = 2, the function f defined by
, 0x<2
x [x ]
where [x] = greatest integer less than or equal to x.
(x 1) [x] , 2 x 3
e [x ]+ x 2
, x 0 & f(0) = 1 where [x] denotes greatest integer less than or equal to x.
f(x) = x .

[
]
x
+
x

f(x) =

Q.13

Test the differentiability of f(x) at x = 0.

2 x 3 [ x ] for x 1

Q.14 Discuss the continuity & the derivability in [0 , 2] of f(x) =

sin

x
2

for x < 1

where [ ] denote greatest integer function .


Q.15 If f(x) = 1 + x 1, 1 x 3 ; g(x) =152 x + 1, 2 x 2 , then calculate
(fog) (x) & (gof) (x). Draw their graph. Discuss the continuity of (fog) (x) at x = 1 & the differentiability

of (gof) (x) at x = 1.

Q.16
(i)
Q.17
Q.18

Q.19

ax( x 1) + b when x < 1

f ( x) = x 1
when 1 x 3
The function :
2
px + qx + 2 when x > 3

Find the values of the constants a, b, p, q so that


f(x) is continuous for all x
(ii)
f ' (1) does not exist
(iii)
f '(x) is continuous at x = 3
1/ x
1/ x
a a
Examine the function , f (x) = x. 1/ x
, x 0 (a > 0) and f (0) = 0 for continuity and existence of
a + a 1/ x
the derivative at the origin.
Discuss the continuity on 0 x 1 & differentiability at x = 0 for the function.
1
1
where x 0 , x 1/ r & f(0) = f (1/ r) = 0 ,
f(x) = x .sin .sin
x
x .sin x1
r = 1, 2, 3,.........
1 x , ( 0 x 1)
f(x) = x + 2 , (1< x < 2 ) Discuss the continuity & differentiability of
4 x , ( 2 x 4 )

y = f [f(x)] for 0 x 4.

2
x cos 2 x
Q.20 Consider the function, f (x) =

(a)
(c)

if x 0
if x = 0

Show that f ' (0) exists and find its value


For what values of x, f ' (x) fails to exist.

(b)

1
Show that f ' does not exist.
3

Q.21 Discuss the continuity & the derivability of 'f' where f (x) = degree of (ux + u + 2u 3) at x = 2.
Q.22 Let f (x) be a function defined on (a, a) with a > 0. Assume that f (x) is continuous at x = 0 and
f ( x ) f (kx )
Lim
= , where k (0, 1) then compute f ' (0+) and f ' (0), and comment upon the
x 0
x
differentiability of f at x = 0.
Q.23 A function f : R R satisfies the equation f(x + y) = f(x). f(y) for all x, y in R &
f(x) 0 for any x in R. Let the function be differentiable at x = 0 & f(0) = 2. Show that f(x) = 2f(x) for
all x in R. Hence determine f(x).
Q.24 Let f(x) be a real valued function not identically zero satisfies the equation,
f(x + yn) = f(x) + (f(y))n for all real x & y and f (0) 0 where n (> 1) is an odd natural number. Find
f(10).
Q.25 A function f : R R where R is a set of real numbers satisfies the equation
x+y

f (x) + f (y) + f (0)

f(
)=
for all x ,y in R. If the function is differentiable at x = 0 then show that it is
3
3
differentiable for all x in R.

EXERCISE5

Fill in the blanks :


f (3 + h 2 ) f ((3 h 2 )

Q.1

If f(x) is derivable at x = 3 & f (3) = 2 , then Limit


h 0

Q.2

If f(x) = sin x & g(x) = x3 then f[g(x)] is ______ & ______ at x = 0. (State continuity and derivability)

Q.3

Let f(x) be a function satisfying the condition f( x) = f(x) for all real x. If f (0) exists, then its value is
______.

Q.4

For the function f(x) = 1 + e1/ x

Q.5

from the left, f(0) = _______.


The number of points at which the function f(x) = max. {a x, a + x, b}, < x < , 0 < a < b cannot
be differentiable is ______.
Select the correct alternative : (only one is correct)

Q.6

, x0
, x=0

2h 2

= _______.

, the derivative from the right, f(0+) = _____ & the derivative

Let f(x) =
for x 0 & f(0) = 1 then ,
sin x
(A) f(x) is conti. & diff. at x = 0
(C) f(x) is discont. & not diff. at x = 0

(B) f(x) is continuous & not derivable at x = 0


(D) 16
none

Q.7

Q.8

log a [x] + [ x]
a
Given f(x) =

[x ] +2[ x ] 5
x

3+ax

for x 0 ; a > 1

where [ ] represents the integral


for x = 0

part function, then :


(A) f is continuous but not differentiable at x = 0
(B) f is cont. & diff. at x = 0
(C) the differentiability of 'f' at x = 0 depends on the value of a
(D) f is cont. & diff. at x = 0 and for a = e only.
For what triplets of real numbers (a, b, c) with a 0 the function
x 1
x
is differentiable for all real x ?
2
ax + bx + c otherwise

f(x) =

(B) {(a, 12a, c) a, c R, a 0 }


(A) {(a, 12a, a) a R, a 0 }
(C) {(a, b, c) a, b, c R, a + b + c = 1 }
(D) {(a, 12a, 0) a R, a 0}
Q.9 A function f defined as f(x) = x[x] for 1 x 3 where [x] defines the greatest integer x is :
(A) conti. at all points in the domain of f but non-derivable at a finite number of points
(B) discontinuous at all points & hence non-derivable at all points in the domain of f
(C) discont. at a finite number of points but not derivable at all points in the domain of f
(D) discont. & also non-derivable at a finite number of points of f.
Q.10 [x] denotes the greatest integer less than or equal to x. If f(x) = [x] [sin x] in (1,1) then f(x) is :
(A) cont. at x = 0
(B) cont. in (1, 0)
(C) differentiable in (1,1)
(D) none
Q.11 A function f(x) = x [1 + (1/3) sin (lnx)], x 0.[ ] = integral part f(0) = 0. Then the function :
(A) is cont. at x = 0
(B) is monotonic
(C) is derivable at x = 0
(D) can not be defined for x < -1
Q.12

if
x
x2
if
The function f(x) is defined as follows f(x) =
3
x x + 1 if

(A) derivable & cont. at x = 0


(C) neither derivable nor cont. at x = 1

x<0

0 x 1 then f(x) is :
x >1

(B) derivable at x = 1 but not cont. at x = 1


(D) not derivable at x = 0 but cont. at x = 1

x + {x} + x sin{x} for x 0


where {x} denotes the fractional part function, then :
for x = 0
0

Q.13 If f(x) =

(A) 'f' is cont. & diff. at x = 0


(B) 'f' is cont. but not diff. at x = 0
(C) 'f' is cont. & diff. at x = 2
(D) none of these
x
is differentiable is :
Q.14 The set of all points where the function f(x) =
1+ x

Q.15
Q.16
Q.17
Q.18

(A) ( , )
(B) [ 0, )
(C) ( , 0) (0, ) (D) (0, ) (E) none
Select the correct alternative : (More than one are correct)
If f(x) = 2x+1 + x 2 then f(x) is :
(A) cont. at all the points
(B) conti. at x = 2 but not differentiable at x = 1/2
(C) discontinuous at x = 1/2 & x = 2
(D) not derivable at x = 1/2 & x = 2
f(x) = [x]x in 1 x 2 , where [x] is greatest integer x then f(x) is :
(A) cont. at x = 0
(B) discont. x = 0
(C) not diff. at x = 2 (D) diff. at x = 2
f(x) =1 + x.[cosx] in 0 < x /2 , where [ ] denotes greatest integer function then ,
(A) It is continuous in 0 < x < /2
(B) It is differentiable in 0 < x < /2
(C) Its maximum value is 2
(D) It is not differentiable in 0 < x< /2
f(x) = (Sin-1x). Cos (1/x) if x 0 ; f(0) = 0 , f(x) is :
(A) cont. no where in 1 x 1
(B) cont. every where in 1 x 1
(C) differentiable no where in 1 x 1
(D) differentiable everywhere in 1 < x < 1

Q.19 f(x) = x + sinx in 2 , 2 . It is :


(A) Conti. no where
(B) Conti. every where
(C) Differentiable no where
(D) Differentiable everywhere except at x = 0
Q.20 If f(x) = 3(2x+3)2/3 + 2x+3 then ,
(A) f(x) is cont. but not diff. at x = 3/2
(B) f(x) is diff. at x = 0
(C) f(x) is cont. at x = 0
(D) f(x) is diff. but not cont. at x = 3/2
Q.21 If f(x) = 2 + sin1 x, it is :
(A) continuous no where
(B) continuous everywhere in its domain
(C) differentiable no where in its domain
(D) Not differentiable at x = 0
Q.22 If f(x) = x. sin (1/x) , x 0 and f(0) = 0 then ,
(A) f(x) is continuous at x = 0
(B) f(x) is derivable at x = 0
(C) f(x) is continuous at x = 0
(D) f(x) is not derivable at x = 0
Q.23 A function which is continuous & not differentiable at x = 0 is :
(A) f(x) = x for x < 0 & f(x) = x for x 0
(B) 17
g(x) = x for x < 0 & g(x) = 2x for x 0
(C) h(x) = xx x R
(D) K(x) = 1+x, x R

Q.24 If Sin-1x + y = 2y then y as a function of x is :


(A) defined for -1 x 1
(B) continuous at x = 0
(C) differentiable for all x

(D) such that


Min [ f ( t ) / 0 t x] for 0 x 2

Q.25 Let f(x) = Cosx & H(x) =

2 x

for

(A) H (x) is cont. & deri. in [0, 3]


(C) H(x) is neither cont. nor deri. at x = /2

<x3

dy
1
=
for 1 < x < 0
dx 3 1 x 2

, then

(B) H(x) is cont. but not deri. at x = /2


(D) Max. value of H(x) in [0,3] is 1

EXERCISE6
Q.1

Determine the values of x for which the following function fails to be continuous or differentiable
,
x <1
1 x

f (x) = (1 x) (2 x) , 1 x 2 . Justify your answer..


3x
,
x>2

Q.2

Q.3
Q.4
Q.5

Q.6

[JEE97, 5]

Let h(x) = min {x, x2}, for every real number of x. Then :
(A) h is cont. for all x
(B) h is diff. for all x
(C) h (x) = 1, for all x > 1
(D) h is not diff. at two values of x.

2 + 1 x 2
Discuss the continuity & differentiability of the function f(x) =
(1 x ) 2
2 e

(b)
(c)

Q.8
Q.9

, x >1

[ REE '98, 6 ]
The function f(x) = (x2 1) x2 3x + 2 + cos (x) is NOT differentiable at :
(A) 1
(B) 0
(C) 1
(D) 2
[ JEE '99, 2 (out of 200) ]
Let f : R R be any function. Define g : R R by g (x) = f (x) for all x. Then g is
(A) onto if f is onto
(B) one one if f is one one
(C) continuous if f is continuous
(D) differentiable if f is differentiable.
[ JEE 2000, Screening, 1 out of 35 ]
Discuss the continuity and differentiability of the function,
x
, |x| 1
1 + |x|
.
f (x) = x

, |x| < 1
1

|
x
|

Q.7
(a)

, x 1

[JEE'98, 2 ]

[ REE, 2000 (3) ]

[ JEE 2001 (Screening) ]


Let f : R R be a function defined by , f (x) = max [ x , x3 ]. The set of all points where
f (x) is NOT differentiable is :
(A) { 1 , 1}
(B) { 1 , 0}
(C) {0 , 1}
(D) { 1 , 0 , 1}
The left hand derivative of , f (x) = [ x ] sin ( x) at x = k , k an integer is :
(B) ( 1)k 1 (k 1)
(A) ( 1)k (k 1)
k
(C) ( 1) k
(D) ( 1)k 1 k
Which of the following functions is differentiable at x = 0 ?
(A) cos (x) + x
(B) cos (x) x
(C) sin (x) + x
(D) sin (x) x
Let R. Prove that a function f : R R is differentiable at if and only if there is a function
g : R R which is continuous at and satisfies f(x) f() = g(x) (x ) for all x R.
[JEE 2001, (mains) 5 out of 100]
The domain of the derivative of the function
tan 1 x if | x| 1

f(x) = 1 (| x|1) if | x| > 1 is


2
(A) R {0}
(B) R {1}
(C) R {1}
(D) R {1, 1}
[JEE 2002 (Screening), 3]
1/ x

f (1 + x)
Q.10 Let f: R R be such that f (1) = 3 and f (1) = 6. The Limit
equals
x0 f (1)
(A) 1
(B) e1/2
(C) e2
(D) e3
[JEE 2002 (Screening), 3]

Q.11

x + a if x < 0
f (x) =
| x 1| if x 0

and

if x < 0
x + 1
g (x) = 18 2
( x 1) + b if x 0

Where a and b are non negative real numbers. Determine the composite function gof. If (gof) (x) is
continuous for all real x, determine the values of a and b. Further, for these values of a and b, is gof
differentiable at x = 0? Justify your answer.
[JEE 2002, 5 out of 60]
Q.12 If a function f : [ 2a , 2a] R is an odd function such that f (x) = f (2a x) for x [a, 2a] and the left
hand derivative at x = a is 0 then find the left hand derivative at x = a.
[JEE 2003, Mains-2 out of 60]
Q.13(a) The function given by y = | x | 1 is differentiable for all real numbers except the points
(A) {0, 1, 1}
(B) 1
(C) 1
(D) 1
1
(b) If f (x) is a continuous and differentiable function and f = 0, n 1 and n I, then
n
(B) f (0) = 0, f ' (0) = 0
(A) f (x) = 0, x (0, 1]
(C) f '(x) = 0 = f ''(x), x (0, 1]
(D) f (0) = 0 and f ' (0) need not to be zero
[JEE 2005 (Screening), 3 + 3]
(c) If | f(x1) f(x2) | (x1 x2)2, for all x1, x2 R. Find the equation of tangent to the curve y = f (x) at the
point (1, 2).
[JEE 2005 (Mains), 2]
Q.14 If f (x) = min. (1, x2, x3), then
(A) f (x) is continuous x R
(B) f (x ) > 0 , x > 1
(C) f(x) is not differentiable but continuous x R
(D) f(x) is not differentiable for two values of x
[JEE 2006, 5 (1)]

EXERCISE7(Continuity)
Part : (A) Only one correct option
1.

(a2 ax + x 2 ) (a 2 + ax + x 2 )

The value of f (0), so that the function, f (x) =

(a + x ) (a x )

(a > 0) becomes

continuous for all x, is given by :


(A) a a
2.

(C) a

(D) a a

sin (1 / x) , x 0
continuous at x = 0 is:
, x=0
R

The value of R which makes f (x) =


(A) 8

3.

(B)

(B) 1

A function f(x) is defined as below f(x) =

(C) 1
cos(sin x ) cos x
x2

f(x) is continuous at x = 0 if a equals


(A) 0
(B) 4

(C) 5

(D) None of these


, x 0 and f(0) = a
(D) 6

4.

1
then f is
Let f(x) = (sin x ) 2 x , x . If f(x) is continuous at x =
2
2
2
(A) e
(B) 1
(C) 0
(D) none of these

5.

(1 + px ) (1 px )

x
f (x) =
2x + 1

x2

(A) 1

6.

7.

8.
9.

10.

, 1 x < 0
, 0 x 1

is continuous in the interval [ 1, 1 ], then ' p' is equal to:

(B) 1/2

(C) 1/2

(D) 1

Let f(x) = x + 2 [ x ] when 2 x 2. where [ . ] represents greatest integer function. Then

(A) f(x) is continuous at x = 2


(B) f(x) is continuous at x = 1
(C) f(x) is continuous at x = 1
(D) f(x) is discontinuous at x = 0
The set of all points for which
1
| x 3|
f(x) =
+
where [.] represents greatest integer function is continuous is
| x 2|
[1 + x]
(A) R
(B) R [ 1, 0]
(C) R ( {2} [ 1, 0])
(D) R {(1, 0) n, n }
(2x 1)
The function f (x) = [ x ] cos
, ( [.] denotes the greatest integer function) is dicontinuous at:
2
(A) all x
(B) x = n/2, n {1} (C) no x
(D) x which is not an integer
Let [x] denote the integral part of x R and g(x) = x [x]. Let f(x) be any continuous function with
f(0) = f(1) then the function h(x) = f(g(x)) :
(A) has finitely many discontinuities
(B) is continuous on R
(C) is discontinuous at some x = c
(D) is a constant function.
2
log( 4 x 3) x 2x + 5 if 34 < x < 1 & x > 1
The function f(x) is defined by f(x) =
19

4
if x = 1
(A)
is continuous at x = 1

is discontinuous at x = 1 since f(1+) does not exist though f(1) exists


is discontinuous at x = 1 since f(1) does not exist though f(1+ ) exists
is discontinuous since neither f(1) nor f(1+) exists.

n (sin x)
1 sinx
Let f(x) =
x . The value of f so that the function is continuous
.
2
2
2
2
2
( 2x )
(B)
(C)
(D)

11.

12.

13.

14.
15.

16.

17.

18.

n 1 + 4 x + 4 x

at x = /2 is:
(A) 1/16

Let f(x) =

(B) 1/32

(C) 1/64

(D) 1/128

if x is irrational
then:
if x is rational

(A)
f(x) is discontinuous for all x
(B)
discontinuous for all x except at x = 0
(C)
discontinuous for all x except at x = 1 or 1
(D)
none of these
Let f(x) = [x 2 ] [x] 2 , where [ . ] denotes the greatest integer function. Then
f(x) is discontinuous for all integeral values of x
(A)
(B)
f(x) is discontinuous only at x = 0, 1 (C)
f(x) is continuous only at x = 1
(D)
none of these
Let f(x) be a continuous function defined for 1 x 3. If f(x) takes rational values for all x and f(2) = 10
then the value of f(1.5) is
(A) 7.5
(B) 10
(C) 8
(D) none of these
Let f(x) = Sgn(x) and g(x) = x (x 2 5x + 6). The function f(g(x)) is discontinuous at
(A) infinitely many points
(B) exactly one point
(C) exactly three points
(D) no point
2 1
The function f(x) = x 2 , x 0, is [ . ] represents the greatest integer less than or equal to x
x
(B) continuous at x = 1
(A) continuous at x = 1
(C) discontinuous at infinitely many points
(D) continuous at x = 1
(1 + sin x ) t 1

The function f defined by f(x) = t lim


.
(1 + sin x ) t + 1 is

(A) everywhere continuous


(B) discontinuous at all integer values of x
(C) continuous at x = 0
(D) none of these
a 2|x| + { x } 1
If [x] and {x} represent integral and fractional parts of a real number x, and f(x) =
, x 0,
2 [ x ] + { x}
f(0) = loge a, where a > 0, a 1, then
(A) f(x) is continuous at x = 0
(B) f(x) has a removable discontinuity at x = 0
lim
(C)
f(x) does not exist
(D) none of these
x 0

Part : (B) May have more than one options correct


If f(x) = x and g(x) = x 1, then
19.
(A) fog is continuous on [0, )
(B) gof is continuous on [0, )
(C) fog is continuous on [1, )
(D) none of these
1
m
x sin , x > 0
x
is continuous at x = 0 if
20.
The function f(x) =
0
, x=0
(A) m 0
(B) m > 0
(C) m < 1
(D) m 1
1
21.
Let f(x) = [sin x ] ( [ . ] denotes the greatest integer function) then
(A)
domain of f(x) is (2n + , 2n + 2) {2n + /2}
(B)
f(x) is continuous when x (2n + , 2n + 2)
(C)
f(x) is continuous at x = 2n + /2
(D)
f(x) has the period 2
22.

23.

Let f(x) = [x] + x [ x ] , where [x] denotes the greatest integer function. Then
(A) f(x) is continuous on R+
(B) f(x) is continuous on R
(C) f(x) is continuous on R
(D) discontinuous at x = 1
x
0 ,
(where [ . ] denotes the greatest
Let f(x) and g(x) be defined by f(x) = [x] and g(x) = 2
x , x R
integer function) then
lim g(x) exists, but g is not continuous at x = 1
(A)
x 1

(B)
(C)
24.

lim f(x) does not exist and f is not continuous at x = 1


gof is continuous for all x
(D)
fog is continuous for all x
x 1

Which of the following function(s) defined below has/have single point continuity.

1
0
x
(C) h(x) =
0
(A) f(x) =

if x Q
if x Q
if x Q
if x Q

if x Q
x
1 x if x Q

(B) g(x) =

x if x Q
x if x Q

(D) k(x) =
20

EXERCISE8
1.

Discuss the continuity of the function, f (x) at x = 3, if


, if 0 x < 3
x [x]
where [.] denotes greatest integer function.
f (x) =
(
x

1
)
[
x
]
, if 3 x 4

2.

3.

4.
5.

1 sin3 x
,x < / 2
3 cos2 x

a
, x = / 2 is continuous at
Find the values of ' a ' & ' b ' so that the function, f (x) =
b (1 sinx )
( 2x )2 , x > / 2

x = /2.
e x 1
n (1 + 2x ) , x 0

at x = 0. If discontinuous, find the
Discuss the continuity of the function, f (x) =
7
,x = 0
nature of discontinuity ?

If f(x) = x + { x} + [x], where [x] is the integral part & {x} is the fractional part of x. Discuss the
continuity of f in [ 2, 2 ]. Also find nature of each discontinuity.
1 + x ,0 x 2
Let f(x) =
. Determine the form of g(x) = f (f(x)) & hence find the point of discontinuity
3 x , 2 < x 3
,
of g if any.

6.

Examine the continuity at x = 0 of the sum function of the infinite series:


x
x
x
+
+
+ .............. .
x +1
( x + 1)( 2x + 1)
(2 x + 1)(3 x + 1)

7.

If f(x) =

8.

9.
10.
11.

12.

13.

sin3 x + A sin 2 x + B sin x

(x 0) is continuous at x = 0. Find A & B. Also find f (0).


x5
Let [x] denote the greatest integer function & f(x) be defined in a neighbourhood of 2 by

exp ( x + 2) 4 [x + 1] ln 4 16

,x < 2
f(x) =
.
4 x 16

1 cos( x 2)
, x>2

A
( x 2) tan ( x 2)
Find the values of A & f(2) in order that f(x) may be continuous at x = 2.
Discuss the continuity of the function f (x) = Limit
n

(1 + sinx )n + ln x

.
2 + (1 + sinx )n
Let f(x + y) = f(x) + f(y) for all x , y and if the function f(x) is continuous at x = 0, then show that f(x) is
continuous at all x.
If f(x y) = f(x) f(y) for all x, y and f(x) is continuous at x = 1. Prove that f(x) is continuous for all x except
at x = 0. Given f(1) 0.
x + 2y f ( x )+ 2f ( y )
=
If f
x, y R and f(x) is continuous at x = 0. Prove that f (x) is continuous for all
3
3
x R.
max m {f ( t ) ; 0 t x , 0 x 2
If f(x) = sin x and g(x) =
, then discuss the continuity of g(x) x > 0 .
3x 4
; x>2

21

CON TINU ITY

EXERCISE1
Q1.
Q 2.
Q 3.
Q 4.

f(0+) = 2 ; f(0) = 2 hence f(0) not possible to define


(a) 2, 2, 3 (b) K = 5 (c) even
yn(x) is continuous at x = 0 for all n and y(x) is dicontinuous at x = 0
f is cont. in 1 x 1
Q 5. P not possible.

Q 6.
Q 7.

(a) 4 3 2 + a , (b) a = 3
g(x) = 2 + x for 0 x 1, 2 x for 1 < x 2, 4 x for 2 < x 3 ,
g is discontinuous at x = 1 & x = 2
A = 1 ; f(2) = 1/2
Q 9. a = 0 ; b = 1
f ( f ( x ) ) is continuous and domain of f ( f ( x ) ) is [ 4, 6 ]
gof is dis-cont. at x = 0, 1 & -1
a = 1/2, b = 4
Q14. a = 3/2, b 0, c = 1/2
A = 4 , B = 5, f(0) = 1
Q 16. discontinuous at x = 1, 4 & 5
discontinuous at all integral values in [ 2 , 2]
locus (a, b) x, y is y = x 3 excluding the points where y = 3 intersects it.

Q 8.
Q.11
Q 12.
Q 13.
Q 15.
Q 17.
Q 18.

Q 20. 5

1
60

Q22.
 n (tan x)

Q 25. k = 0 ; g (x) =

if 0 < x <
4 . Hence g (x) is continuous everywhere.
if x <
4
2

Q 26. g (x) = 4 (x + 1) and limit =


Q 28. a =

1
2

, g(0) =

39
4

( n 2)2
8

Q 29. f(0 ) = ; f(0) =


f is discont. at x = 0 ;
2
4 2
+

g(0+) = g(0) = g(0) = /2 g is cont. at x = 0

Q 30. the function f is continuous everywhere in [0 , 2] except for x = 0 , 1 , 1 & 2.

EXERCISE2

Q 1.

(a) false; (b) false ; (c) false ; (d) false ; (e) false ; (f) true ; (g) false ; (h) true

Q 2.
Q 3.

(a) c = 1 ; (b). x 1, 1 & x = 0 ; (c). 1 ; (d). a = , b =


(e). 1/2
12
6
(a) D (b). B, C (c). C, D (d). B (e). C (f). A (g). B (h) A (i) D (j) A (k) C

EXERCISE3
Q.1

R [ 1, 0 ) ; discontinuous for all integral values in domain except at zero

Q.2

10

Q.5

Discontinuous at x = 1; f(1+) = 1 and f(1) = 1


*************************

Q.3 D

Q.4

a = ln

2
2
; b= ; c=1
3
3

DIFFERENTI ABI LITY

EXERCISE4
Q 1. f(x) is conti. but not derivable at x = 0

Q 2. conti. x R, not diff. at x = 0,1 & 2

Q 4. conti. but not diff.at x = 0 ; diff. & conti. at x = /2


Q 5. conti. but not diff. at x = 0
Q 7. f is cont. but not diff. at x = 0
Q 8. f(1+) = 3 , f(1) = 1
Q 9. a= 1/2 , b = 3/2
Q 10. not derivable at x = 0 & x = 1
22
Q 11. f is cont. & derivable at x = 1 but f is neither cont. nor derivable at x = 1

Q 12. discontinuous & not derivable at x = 1, continuous but not derivable at x = 2


Q 13. not derivable at x = 0
Q 14. f is conti. at x = 1 , 3/2 & disconti. at x = 2, f is not diff. at x =1, 3/2 , 2
Q15. (fog)(x) = x+1 for 2 x 1, (x + 1) for 1 < x 0 & x 1 for 0 < x 2.
(fog)(x) is cont. at x = 1, (gof)(x) = x+1 for 1 x 1 & 3 x for 1 < x 3.
(gof)(x) is not differentiable at x = 1
1
Q 16. a 1, b = 0, p = and q = 1
3
Q 17. If a (0, 1) f (0+) = 1 ; f (0) = 1 continuous but not derivable
a = 1 ; f (x) = 0 which is constant continuous and derivable
If a > 1 f (0) = 1 ; f (0+) = 1 continuous but not derivable
Q  conti in  x
not diff at x  
Q.19 f is conti. but not diff. at x = 1, disconti. at x = 2 & x = 3. cont.& diff.at all other points
1

Q.20 (a) f ' (0) = 0, (b) f ' = and f '


2
3

1+
1
= , (c) x =
nI
3 2
2n + 1

Q.21 continuous but not derivable at x = 2


Q.22 f ' (0) =
1 k
2x
Q.23 f(x) = e
Q.24 f(x) = x f(10) = 10

EXERCISE5
Q.1 2
Q.5 2
Q.9 D
Q.13 D
Q.17 A, B
Q.21 B, D
Q.25 A, D

Q.2 conti. & diff.


Q.6 C
Q.10 B
Q.14 A
Q.18 B, D
Q.22 A, B , D

Q.3 0
Q.7 B
Q.11 A
Q.15 A, B, D
Q.19 B, D
Q.23 A, B, D

Q.4 f (0+) = 0 , f (0) = 1


Q.8 A
Q.12 D
Q.16 A, C
Q.20 A, B, C
Q.24 A, B, D

EXERCISE6
Q.1
Q.3
Q.5
Q.7
Q.12

Q.2 A, C, D
f(x) is conti. & diff. at x = 1 ; f(x) is not conti. & not diff. at x = 2
conti. but not derivable at x = 1, neither cont. nor deri. at x = 1
Q.4 D
C
Q.6
Discont. hence not deri. at x = 1 & 1. Cont. & deri. at x = 0
(a) D, (b) A, (c) D
Q.9
D
Q.10 C
Q.11 a = 1; b = 0(gof)'(0) = 0
Q.14 A, C
f (a) = 0
Q.13 (a) A, (b) B, (c) y 2 = 0

Continuity
EXERCISE 7

EXERCISE 8
1. continuous at x = 3

1. C

2. D

3. A

4. B

5. B

6. D

7. D

8. B

9. B

10. D

11. C

12. C

13. D

14. B

15. C

16. C

17. B

18. C

19. BC 20. BD

21. ABD

22. ABC

23. ABC

24. BCD

2. a =

3. Removable isolated point

1
,b=4
2

4. discontinuous at all integral values in [ 2, 2]


5. g(x)= 2 + x ; 0 x 1,
= 2 x ; 1 < x 2,
= 4 x ; 2 < x 3,
g is discontinuous at x = 1 & x = 2
6. Discontinuous

7. A = 4, B = 5, f(0) = 1

8. A = 1; f(2) = 1/2
9. f (x) is discontinuous at natural multiples of
13. continuous for all x 0 except at x = 2
23

LIM ITS, CONTINUITY & DIFFERENTIABILITY


Some questions (AssertionReason type) are given below. Each question contains Statement 1 (Assertion) and
Statement 2 (Reason). Each question has 4 choices (A), (B), (C) and (D) out of which ONLY ONE is correct. So select
the correct choice :
Choices are :
(A)
Statement 1 is True, Statement 2 is True; Statement 2 is a correct explanation for Statement 1.
(B)
Statement 1 is True, Statement 2 is True; Statement 2 is NOT a correct explanation for Statement 1.
(C)
Statement 1 is True, Statement 2 is False.
(D)
Statement 1 is False, Statement 2 is True.
43.

44.

x=0
0,

is differentiable is (, ).
Statements-1: The set of all points where the function f(x) = x
1 + e1/ x , x 0
1
1 + e1/ x x(e1/ x 2
x , which exists x 0.
Statements-2: Lf(0) = 1, Rf(0) = 0 and f(x) =
(1 + e1/ x ) 2
3 x 2 , x > 2
Statements-1: f(x) = 3
then f(x) is differentiable at x = 1
x + 1, x 2
Statements-2: A function y = f(x) is said to have a derivative if

lim

h 0+

f (x + h) f (x)
f (x + h) f (x)
= lim
h

0
h
h

45.

Consider the function f(x) = (|x| |x 1|)2


Statement 1: f(x) is continuous everywhere but not differentiable at x = 0 and 1.
Statement 2: f (0) = 0, f (0+) = 4, f (1) = 4, f (1+) = 0.

46.

Statement 1: lim

47.

Statement 2: L.H.L. = 1 and R.H.L. = 1


Statement1 : lim cos1 (cos2x) does not exist

48.

49.

50.

e1/ x 1
does not exist
x 0 e1/ x + 1

x 0

Statement2 : cosec1x is well defined for |x| 1.


Let f : [0, 2] [0, 2] be a continuous function
Statement1 : f(x) = x for at least one 0 x 2
Statement2 : f(x) = x for at least one 0 x 2
Let h(x) = f(x) + g(x) and f(a), g(a) are finite and definite
Statement1 : h(x) is continuous at x = 9 and hence h(x) = x2 + 1 cosx| is continuous at x = 0
Statement2 : h(x) is differentiable at x = a and hence h(x) = x2 + |cosx| is differentiable at x = 0
Statement1
: f(x) = e|x| is non differentiable at x = 0.
Statement2
: Left hand derivative of f(x) is 1 and right hand derivative of f(x) is 1.

Statement2

: lim cos x = 0 , where [x] = G.I.F

Statement2

: as x 0, cos x lies between 0 and 1.

52.

Statement1

53.

Statement2
: sec1 t is defined for those t, whose modulus value is more than or equal to 1.
Suppose [] and {} denotes the greatest integer function and fractional part function respectively. Let f(x) = {x} +

51.

x 0

x
lim sec1
does not exist.
x
x +1

{x} .

54.

Statement1
Statement2

: f is not differentiable at integrable points.


: f is not continuous at integral points.

Statement1

21/ x
: lim
= 1 . Statement2
x 0 1 + 21/ x

24

lim
x 0

cos 1 (1 x )
x

= 2.

55.

t sin t .

Statement1

: The number of points of discontinuity of f(x) is all 0. Where f(x) =

Statement2

: The function h(x) = max {- x, 1, x2} b x R, is not differnetiable at two values of x.

56.

Statement1 : If p, q, r all are positive , then lim 1 +


x

Statement2 : lim (1 + x )

1/ x

x 0

57.
58.

p + qx

r + sx

is es/q

= e.

Statement1 : For f(x) = ||x2| 4|x||, the number of points of non differentiability is 3.
Statement2 : A continuous function is always differentiable
Statement1 : If f(x) = x (1 logx) then for 0 < a < c < b
(a b logc = b (1 log b) a (1 loga)
Statement2 : If f(x) is diff. (a , b) and cont. in [a, b] then for at least one a < c < b f(c) =

59.

Statement 1 : Let {x} denotes the fractional part of x. Then lim


x 0

f (b) f (a)
ba

tan{x}
=1
{x}

tan x
=1
x 0
x

Statement 2 : lim
t

60.

Statement 1 :

sin x dx = 1 cost Statement 2 : sinx is continuous in any closed interval [0, t]


0

sin x

61.

Statement 1 : lim
= 0 where [] G.I.F.
x 0
x

62.

Statement 1 : The function f(x) =

sin x

=1
x 0 x

Statement 2 : lim

1
is continuous at a point x = a 4.
x4

Statement 2 : For x = a, f(x) has a definite value and as x a, f(x) has a limit which is also equal to its definite
value of x = a 4.

1
1
= 1 Statements-2: lim y sin = 1
y
y
x

63.

Statements-1: lim+ x sin

64.

Statements-1: f(x) =

65.

Statements-2: Since -1 < sinx < 1, as n , (sinx)2n 0, sinx = 1 (1)2n 1, n .


Statements-1: f(x) = lim (cosx)2n, then f is continuous everywhere in (-, )

66.

x 0

lim (sinx)2n , then the set of points of discontinuities of f is {(2n + 1) /2, nI}
n

Statements-2: f(x) = cosx is continuous everywhere i.e., in (-, )


Statements-1: For the graph of the function y = f(x) the valid statement is
(0, 1)

f(x) is differentiable at x = 0
Statements-2: Lf (c) = R f (c), we say that f (c) exists and Lf (c) = Rf (c) = f (c).
67.

sin x

Statements-1: lim
=1
x 0
x
Statements-2:

68.

lim f(g(x)) = f(L) where lim g(x) = L . Also function f must be continuous at L.
x a

x a

Statements-1: f(x) = max (1, x2, x3) is differentiable xR except x = -1, 1


Statements-2: Every continuous function is differentiable

25

69.

Statements-1: lim

sin(2x + 2)
=2
x

Statements-2: Since sinx has a range of [-1, 1] xR

70.

71.

72.

73.
74.
75.

lim

sin x
=0
x

| sin x |
, x>0
x

Statements-1: f(x) =
1
, x = 0 , is a continuous function at x = 0
| sin x |

, x<0
x

Statements-2: If left hand limit = right hand limit & both the limits exists finitely then function can be made
continuous.
Statements-1: f(x) = x|x| is differentiable at every point in its domain.
Statements-2: If f(x) is as a derivative at every point & g(x) has a derivative at every point in their domains, then
h(x) = f(x).g(x) is differentiable at every point in its domain.
Statements-1: x = cosx for some x (0, /2)
Statements-2: If f(x) is a continuous in an interval I and f(a) & f(b) are two values at a & b & c is any value in
between f(a) & f(b), then there is some x in (a, b) where f(x) = c.
Statements-1: f : R R and f(x) = ex ex the range of f(x) is R
Statements-2: If f(x) is a continuous function in [a, b] then f(x) will take all values in between f(a) and f(b).
Statements-1: If a < b < c < d then (x a) (x c) (x b) (x d) = 0 will have real for all R.
Statements-2: If f(x) is a function f(x1) f(x2) < 0 then f(x) = 0, for at least one x (x1, x2).

1
=
x 0 x 2
1
Statements-2: If lim 2 = , then for every positive number G arbitrarily assign (however large) there exist a
x a x

Statements-1: lim

> 0 such that for all x (a , a) (a, a + ) f(x) a > 0.

ex + e x
, 1 x 3, exists.
2

76.

Statements-1: The maximum and the minimum values of the function f (x) =

77.

Statements-2: If domain of a continuous function is in closed interval then its range is also in a closed interval.
Statements-1: For any function y = f(x) lim f (x) = f (a)
x a

Statements-2: If f(x) is a continuous function at x = a then


1/ x

78.

Statements-1: lim

( n)
x

lim f (x) = f (a)


x a

1
e
b

1 r
f = f (x)dx.

n
r =1 n n
a
n

Statements-2: If y = f(x) is continuous in (a, b) then lim


79.
80.
81.

82.

Statements-1: If f is finitely derivable at c, then f is continuous at c.


Statements-2: If at x = c both LHD and RHD exist finitely but LHD RHD then f(x) is continuous at x = c.
Statements-1: If f(x + y) = f(x) + f(y), then f is either differentiable everywhere or not differentiable everywhere
Statements-2: Any function is either differentiable everywhere or not differentiable everywhere.
Statements-1: The function f(x) = |x 3| is differentiable at x = 0
Statements-2: At x = 0 f(x) = 0

log(x + 2) x 2n cos x
= log(x + 2)
n
x 2n + 1

Statements-1: : When |x| < 1 lim

Statements-2: For 1 < x < 1, as n ,x2n 0.


83.

Statements-1: : f(x) =
function.

1
is discontinuous for integral values of x. where [.] denotes greatest integer
x [x]

Statements-2: For integral values of x, f(x) is undefined.


3

26

84.

1
is differentiable for all real values of x (n 2)
x

Statements-1: : f(x) = xn sin

Statements-2: for n 2 right hand derivative = Left hand derivative (for all real values of x).
85.

e1/ x 1
, when x 0

is discontinuous at x = 0.
Statements-1: The function f (x) = e1/ x + 1
0 , when x = 0

Statements-2: f(0) = 0.

86.

x
for x < 1

Statements-1: The function f(x) defined by 2 x


for 1 x 2 is differentiable at x = 2.
2 + 3x x 2 for x > 2

Statements-2: L.H.D. at x = 2 = R.H.D. at x = 2

87.

88.

sin x
= 0 [.] denotes greatest integer function.

x 0
x
1 tan x
Statements-2: lim sec
= 0 [.] denotes greatest integer function.
x 0
x
2x + 1 x < 1
2
Statements-1: f (x) = x + x + 1
1 x < 2 is continuous
x 3 1
x2
at x = 1, 2

Statements-1: lim sec

Statements-2: f (1) = 2 f (1 +) = 3, f (1 ) = 5 f (2+) = 6.

e1/ x
does not exist
x0 x

89.

Statements-1: Lim

90.

Statements-1: lim (1 + 3x)

91.

Statements-1: sinx = 0 has atleast one roots between ( /2, /2)


Statements-2: Since sinx is continuous in [-/2, /2] and sin (-/2) = -1, sin (/2 = 1 i.e. sinx has opposite sign is
at x = -/2, x = /2, by intermediate theorem

Statements-2: Right hand limit as x 0 does not exist

92.

1/ x

x 0

= e3

Statements-2: since lim (1 + x)1/x = e


x 0

e1/ x e 1/ x
, x 0 = 0, x = 0 then f(x) has a jump discontinuity at x = 0.
e1/ x + e1/ x
Statements-2: Since lim f(x) = 1
Statements-1: Let f(x) =

x 0

and lim+ f(x) = 1


x 0

93.

Statements-1: The set of all points where the function

f(x) =

, x=0

x
, x0
1 + e1/ x

is differentiable (-, ) {0}

Statements-2: Lf(0) = 1, Rf(0) = 0 is

e1/ x + e1/ x
. which exists x0
(1 + e1/ x ) 2
[x]
Statements-1: f(x) =
, x 0 , where [] denotes greatest integer function, then f(x) is differentiable at x = 1
x

f(x) =
94.

27

Statements-2: L f (1) lim


x 1

= lim
x 1

f (x) f (1)
[x]
= lim
1
x 1 | x |
x 1
x 1

0
1
1= lim
=
|x|
x 1 x 1
x 1

f(1) does not exist.

ANSWER KEY
43.
50.
57.
64.
71.
78.
85.
92.

D
A
A
A
C
A
B
A

44.
51.
58.
65.
72.
79.
86.
93.

D
A
A
D
A
A
A
A

45.
52.
59.
66.
73.
80.
87.
94.

A
A
D
D
A
C
D
A

46.
53.
60.
67.
74.
81.
88.

C
C
A
D
C
A
A

47.
54.
61.
68.
75.
82.
89.

A
B
B
C
A
A
A

48.
55.
62.
69.
76.
83.
90.

A
B
A
D
A
A
A

49.
56.
63.
70.
77.
84.
91.

C
A
D
B
D
A
A

SOLUTION
43.
47.
48.

49.

(D)
Statement-1 is wrong
Statement-2 is true.
Clearly cos2x < 1 in the neighbourhood of the point x = 0 cosec1 (cos2x) is well defined at x = 0 but not in the
neightbourhood of the point x = 0 limit does not exist. Hence (A) is the correct option.
Clearly 0 f(0) 2 and 0 f(2) 2
As f(x) is continuous, f(x) attains all values between f(0) and f(2), and the graph will have no breaks. So graph will
all the line y = x at are point x at least where 0 x 2.
Since f(a) and g(a) are finite and definite h(a) is also finite and definite
h(x) is differentiable at x = 0
h(x) is continuous at x = a.

e x ,

x0

L.H.D = 1

50.

(a)

e|x| =
x

51.

(a)

Clearly statement I is true and statement II is the correct explnation of statement I.

52.

Statement II is true and correct reasoning for statement I, because lim

53.

e , x < 0

x
= 1 .
x x + 1

Hence (a) is the correct answer.


Statement II is false, as for any n I,
f(n +) = n, f(n ) = n 1 + 1 = n, f(n) = n
However statement I is true, as for any n I

{n + h}
f (n + h) f (n)
1
Hence (c) is the correct answer.
= lim
= .
= lim
h 0+ h
h 0 +
h 0 +
h
h
21/ x
1
lim
= lim
=1
x 0 1 + 21/ x
x 0 1 + 2 1/ x
cos 1 (1 x )

(let, cos-1(1 x) = 1 x = cos )


lim
= lim+
x 0
x

0
x
1 cos

Hence (b) is the correct answer.


= 2.
lim
x 0+

2 sin
2
f (n +) = lim

54.

R.H.D = 1.

28

55.

f(x) =

t sin t dt
0

1
f (x) = x sin
x
clearly, f (x) is a finite number at all x (0, ).

f(x) is differentiable at all x (0, ).

x 2;

h(x) = 1;
x2;

x 1
1 x 1
x 1

from graph it is clear that h(x) is continuous at all x and it is not differentiable at x = - 1, 1.
Hence (b) is the correct answer.
56.

(A)

Required limit lim

1
(r + dx)
ep + qx

es/q
57.

Graph of f(x) = ||x2 4|x|| is

-4

So no of points of nondiff. is 3.
58.

63.

64.
65.

Ans. : A

f (b) f (a)
ba
b(1 log b) a(1 log a)
(a b) logc = b(1 logb) a (1 loga)
logc =
ba
1
The Statements-1: is false sin as x O+, the function xsin = a qtyt. apron. zero) (finite number between 0 &
x
1
1). Thus lim+ x sin 0 .
x 0
x
sin x = 1
The Statement-2 is true since it is equivalent to standard limit lim
x 0
x
(A)

f(c) =

option (d) is correct.


Option (a) is correct.

Statements-1: is the solution of Statement-2.

0 | x | < 1
lim x2n =
n
1 | x | = 1
0, if | cos x |< 1
f(x) = lim (cosx)2n =
n
1 if | cos x |= 1
f(x) is continuous at all x, except for those values of x for which |cosx| = 1
x = n nI.
Ans. (D)

66.

67.

from Questions figure clearly


Ans. (D)

sin x
lim
=0
x 0
x
because sinx < x when x > 0
6

29

sin x
< 1 for x > 0
x
sin x
sin x
So
is odd function so it is correct for x < 0.
= 0 for x > 0 because

x
x

So

68.

73.

y=x

y=1
(1, 1)

y=x
(1, 1)

So, d is correct.
The graph of max (1, x2, x3) is as under clearly function is NOT differentiable at x = 1, 1.
Every continuous function is not necessarily differentiable.
So, c is correct.
(A) lim f (x) =
x

lim f (x) = and f(x) is continuous in R then f(x) will take all values in between (, )
x

74.

80.

(C) A quadratic polynomial is always continuous f(b).f(d) < 0 then there exist one value of x (b, d) at which f(x)
= 0 if one root of a real equation is real then another real will also real. If f(x) is not continuous and f(x1).f(x2) < 0
then we cannot say that there is atleast one x (x1, x2) at which f(x) = 0.
The Statements-1: is true. If f is differentiable at c then f(c) exists.

f (c + h) f (c)
f (c) + f (h) f (c)
exists lim
exists
h

0
h
h
f (h)
f (p + h) f (p)
f (h)
= lim
exists. Now if p be some other point then f (0) = lim
= lim
h 0 h
h 0
h

0
h
h
lim
h 0

82.

which exists.
Now any function is either differentiable nowhere or differentiable atleast one point, then it is differentiable for all
x. Thus assertion is true.
The reason R is false since any function is either differentiable nowhere is differentiable at one point.
For x > 0, f(x) = x3 f(x) = 3x2 f(0) = 0
for x < 0, f(x) = -x3 f(x) = -3x2 f(0) = 0.
(A)
(a) Both Statements-1: and Statement-2 are true and Statement-2 is the correct explanation of Statements-1: .

83.

(a)

81.

f(x) =

For all integral values of x, x [x] = 0

1
, which is not defined.
0

Statements-1: and Statement-2 both are true and Statement-2 is the correct explanation of Statements-1: .

84.

85.

1
h n sin 0
f (0 + h) f (0)
h
(a)
f(0) = lim
= lim
h 0
h 0
h
h
1
= lim hn1 sin (n 2)
= 0 finite number = 0
h 0
h
Hence Statements-1: and Statement-2 both are true and Statement-2 is the correct explanation of Statements-1: .
(B) lim f (x) = 1
x 0

lim f (x) = 1

L.H.L. at x = 0, R.H.L. at x = 0.

x 0 +

86.

89.

(A) L.H.D. at x = 2

= (2 x) = 1
dx
x =2
d

= (2 + 3x x 2 ) = 1
dx
x =2
1 1 1
1+ +
+ ...
e1/ x
x 2! x 2
= lim+
lim +
x 0
x x 0
x

R.H.D. at x = 2

30

1
1 1

+ 2 + 3 + ... = (infinits)
x x 2x !

1/ x
e
lim+
does not exist
x 0
x

= lim+
x 0

90.

lim (1 + 3x)1/ x lim (1 + 3x1/ 3x )


x 0
x 0
= e3
because

91.

lim (1 + x )

1/ x

x 0

(Ans. A)

=e

Ans. (A)

f(x) = sinx continuous in [-/2, /2] by intermediate value theorem


f(-/2) = sin (-/2) = -1

f = sin = 1
2
2

f and f are of opposite sign is


2
2

by intermediate value theorem, a point


c [-/2, /2] such that f(x) = 0
s a point x [-/2, /2] such that f(x) = 0
i.e., sinx = 0


,
2 2

thus sinx = 0 has at least one root between


92.

e1/ x e1/ x
1 e2 / x
lim
x 0
x 0 e1/ x + e 1/ x x 0 1 + e 2 / x
lim+ f (x) = 1

lim f (x) = lim

lim f (x) = 1

x 0

x 0

x = 0, f(0) = 0

93.

Ans. (A)

Hence f(x) is discontinuous at x = 0 then Ans. (A)

x
0
1/ x
= lim 1 + e
Lf(0) = lim
x 0
x 0
x
f ( x )f (0)
x
Rf(0) = lim+ x 0 = lim+
0
1/ x
x 0
x 0 1 + e
x
1
= lim+
=0
1/ x
x 0 1 + e
f (x ) f (0)
x 0

L f(0) R f(0) so it is differentiable in (-, ) {0}

1 + e1/ x + e1/ x
x0
Ans. (A)
(1 + e1/ x )2
f (x) f (1)
[x]
Rf(1) = lim+
= x 1+
1
x 1
|
x|
x 1
x 1
1
1
1 x
1
|x|
= lim+
= lim+
= x 1+ = 1
x 1
x 1 x1 x(x 1)
x
f(x) =

94.

Lf(1) =

then f(1) does not exist.

then Ans. (A)

31

STUDY PACKAGE
Target: IIT-JEE (Advanced)
SUBJECT: MATHEMATICS
TOPIC: 21 XII M 4 .Applications of
Derivatives
Index:
1. Key Concepts
2. Exercise I to XV
3. Answer Key
4. Assertion and Reasons
5. 34 Yrs. Que. from IIT-JEE
6. 10 Yrs. Que. from AIEEE

A.

Tangent & Normal

Derivative as rate of change

If the quantity y varies with respect to another quantity x satisfying some relation y = f(x), then f(x) or
dy
represents rate of change of y with respect to x.
dx
Example :
The volume of a cube is increasing at rate of 7 cm 3/sec. How fast is the surface area increasing
when the length of an edge is 4 cm?
Solution.
Let at some time t, the length of edge is x cm.
dv
dx
dv
v = x3
= 3x 2
(but
= 7)
dt
dt
dt
7
dx
cm/sec.
=

dt
3x 2
Now
s = 6x 2
7
ds
dx
ds
28
= 12x

= 12x.
=
dt
dt
dt
3x 2
x
ds
= 7 cm 2/sec.
when x = 4 cm
dt
Example :
Sand is pouring from pipe at the rate of 12 cm 3/s. The falling sand forms a cone on the ground in
such a way that the height of the cone is always one - sixth of radius of base. How fast is the height
of the sand cone increasing when height is 4 cm?
1
r 2 h
Solution.
v=
3
r
but
h=
6
1

v=
(6h)2 h

v = 12 h3
3
dv
dh
= 36 h2.
dt
dt
dv
when,
= 12 cm 3/s
and
h = 4 cm
dt
12
dh
1
=
=
cm/sec.
36 .( 4)2
dt
48
Self practice problem :
1.
Radius of a circle is increasing at rate of 3 cm/sec. Find the rate at which the area of circle is increasing
at the instant when radius is 10 cm.
Ans. 60 cm 2/sec
2.
A ladder of length 5 m is leaning against a wall. The bottom of ladder is being pulled along the ground
away from wall at rate of 2cm/sec. How fast is the top part of ladder sliding on the wall when foot of ladder
8
is 4 m away from wall.
Ans.
cm/sec
3
3.
Water is dripping out of a conical funnel of semi-vertical angle 45 at rate of 2cm 3/s. Find the rate at
1
which slant height of water is decreasing when the height of water is 2 cm. Ans.
cm/sec.
2
4.
A hot air balloon rising straight up from a level field is tracked by a range finder 500 ft from the lift-off
point. At the moment the range finder's elevation angle is /4, the angle is increasing at the rate of 0.14
rad/min. How fast is the balloon rising at that moment. Ans. 140 ft/min.

Equation of Tangent and Normal

dy
= f (x 1) denotes the slope of tangent at point (x 1, y 1) on the curve y = f(x). Hence the equation
dx ( x1, y1 )
of tangent at (x 1, y1) is given by
(y y1) = f (x 1) (x x 1)
Also, since normal is a line perpendicular to tangent at (x 1 , y 1) so its equation is given by
1
(y y1) = f ( x ) (x x 1)
1

Example:

Find equation of tangent to y = ex at x = 0.

Solution

At x = 0

y = e0 = 1
Hence point of tangent is (0, 1)
dy
dy
= ex

dx x =0 = 1
dx
Hence equation of tangent is
1 (x 0) = (y 1)

y=x+1

1
Example :
Find the equation of all straight lines which are tangent to curve y =
and which are
x 1
parallel to the line x + y = 0.
Solution :
Suppose the tangent is at (x 1, y1) and it has slope 1.

dy
dx

= 1.
( x1 , y1 )

= 1.

( x1 1)2
x1 = 0

or

y1 = 1
or
1
Hence tangent at (0, 1) and (2, 1) are the required lines with equations
1(x 0) = (y + 1)
and
1 (x 2) = (y 1)

x+y+1=0
and
y+x=3

Find equation of normal to the curve y = |x 2 | x | | at x = 2.


In the neighborhood of x = 2, y = x 2 + x.
Hence the point of contact is ( 2, 2)
dy
dy
= 3.
= 2x + 1
dx
dx x = 2
1
So the slope of normal at ( 2, 2) is .
3
Hence equation of normal is
1
(x + 2) = y 2.

3y = x + 8.
3
Example :
Prove that sum of intercepts of the tangent at any point to the curve
coordinate is constant.
Solution :
Let P(x 1, y1) be a variable point on the curve x + y = a

Example :
Solution

equation of tangent at point p is

x
x1
+

x
x1
=

x1 =

y1

y1 =

(x x 1) = (y y1)

y1

x1 +

y
+

Hence point A is

C.

a on the

x1

x1

y =

y
+

y1

x +

y1

y1

x1 + y 1 = a )

( ax , 0) and coordinates of point B is (0,


1

a ( x1 +

y1 ) =

ay 1 ) . Sum of intercepts
s

a . a = a.

Tangent from an External Point


Given a point P(a, b) which does not lie on the curve y = f(x), then the equation of possible tangents to the
curve y = f(x), passing through (a, b) can be found by solving for the point of contact Q.

Example :

Find the equation of all possible normal to the parabola x 2 = 4y drawn from point (1, 2).

Solution

h2

Let point Q be h,
4

Now,

m PQ = slope of normal at Q.

Slope of normal =

dx
dy

=
x =h

h2
2
2
4
=
h 1
h
h3
2h = 2h + 2
4

2
h


h3 = 8

h=2
Hence coordinates of point Q is (2, 1) and so equation of required normal becomes x + y = 3.
Note : The equation gives only one real value of h, hence there is only one point of contact implying that
only one real normal is possible from point (1, 2).
Example :
Find value of c such that line joining points (0, 3) and (5, 2) becomes tangent to curve
c
.
y=
x +1
Solution.
Equation of line joining A & B is x + y = 3

Solving this line and curve we get


c
3x=

x 2 2x + (c 3) = 0
......(i)
x +1
For tangency, roots of this equation must be coincident. Hence D = 0

4 = 4 (c 3)

c=4
Note : If a line touches a curve then on solving the equation of line and tangent we get at least two
repeated roots corresponding to point of contact.
Putting c = 4, equation (i) becomes
x 2 2x + 1 = 0
x=1
Hence point of contact becomes (1, 2).
Example :
Tangent at P(2, 8) on the curve y = x 3 meets the curve again at Q. Find coordinates of Q.
Solution.

Equation of tangent at (2, 8) is


y = 12x 16
Solving this with y = x 3
x 3 12x + 16 = 0

this cubic must give all points of intersection of line and curve y = x 3 i.e., point P and Q.
But, since line is tangent at P so x = 2 will be a repeated root of equation x 3 12x + 16 = 0 and
another root will be x = h. Using theory of equations
sum of roots
2+2+h=0

h=4
Hence coordinates of Q are ( 4, 64)
Self Practice Problems :
1.
2.

3.
4.
5.

D.

Find the slope of the normal to the curve x = 1 a sin , y = b cos2 at = .


Ans.
2b
2
Find the equation of the tangent and normal to the given curves at the given points.
x3
(i)
y = x 4 6x 3 + 13x 2 10x + 5 at (1, 3)
(ii)
y2 =
at (2, 2).
4x
Ans. (i)
Tangent : y = 2x + 1, Normal :x + 2y = 7
Tangent : 2x + y = 2, Normal :x 2y = 6
(ii)
Prove that area of the triangle formed by any tangent to the curve xy = c2 and coordinate axes is constant.
How many tangents are possible from origin on the curve y = (x + 1)3. Also find the equation of these
tangents.
Ans. y = 0, 4y = 27x.
x+9
Find the equation of tangent to the hyperbola y =
which passes through (0, 0) origin
x+5
Ans. x + y = 0; 25y + x= 0

Length of Tangent, Normal

Let P (h, k) be any point on curve y = f(x). Let tangent drawn at point P meets x-axis at T & normal at point
P meets x-axis at N. Then the length PT is called the length of tangent and PN is called length of normal.

Projection of segment PT on x-axis, TM, is called the subtangent and similarly projection of line segment
PN on x axis is called sub normal.
dy
Let m = dx
= slope of tangent.
h, k
Hence equation of tangent is m (x h) = (y k)
k
putting y = 0 we get x - intercept of tangent x = h
m
similarly the x-intercept of normal is x = h + km
4

Now, length PT, PN etc can be easily evaluated using distance formula
(i)
(iii)
Example:
Solution.

PT = k 1 +

= Length of Normal

k
= Length of subtangent
(iv)
MN = |km| = Length of subnormal
m
3
Find the length of tangent for the curve y = x + 3x 2 + 4x 1 at point x = 0.
dy
Here m =
&
k = y(0)

k=1
dx x = 0

TM =

dy
= 3x 2 + 6x + 4
dx

= k 1 +
Example:
Solution

2
PN = k 1 + m

= Length of Tangent (ii)

m2

m2

m=4

=

1 1+

1
16

17
4

Prove that for the curve y = bex/a, the length of subtangent at any point is always constant.
y = bex/a
Let the point be (x 1, y1)
dy
y1
b.e x1 / a

m=
=
=
dx x1
a
a

y1
y1
=
=a
Hence proved.
y
m
1/a
2
2
Example :
For the curve y = a n (x a ) show that sum of lengths of tangent & subtangent at any point
is proportional to coordinates of point of tangency.
Solution.
Let point of tangency be (x 1, y1)

Now, length of subtangent =

m=

dy
dx

=
x1

2ax1
x 21

a2
1 +

tangent + subtangent = y1
= y1
= y1
=

1+

( x 21 a 2 )2
2

4a 2 x1

y1
m

y1( x12 a 2 )
2ax1
2

x14 + a 4 + 2a 2 x12

y 1( x 1 a 2 )
+
2ax1

2ax 1

2
y 1( x 1

1
m2

+ a2 )
y 1( x 1 a 2 )
+
2ax1
2ax1
2

y1( x1 )
x1 y 1
=
2ax1
2a

Hence proved.

Angle between the curves


Angle between two intersecting curves is defined as the acute angle between their tangents or the normals at the
point of intersection of two curves.

tan =

m1 m 2
1 + m1m 2

where m1 & m2 are the slopes of tangents at the intersection point (x1, y1). Note carefully that
The curves must intersect for the angle between them to be defined. This can be ensured by finding their point of
intersection or graphically.
(ii)
If the curves intersect at more than one point then angle between curves is written with references to the point of
intersection.
(iii)
Two curves are said to be orthogonal if angle between them at each point of intersection is right angle.
i.e. m1 m2 = 1.
Example :
Find angle between y2 = 4x and x2 = 4y. Are these two curves orthogonal?
y2 = 4x and x2 = 4y intersect at point (0, 0) and (4, 4)
Solution.
(i)

C1 : y2 = 4x
C2 : x2 = 4y
2
dy
dy
x
= y
=
dx
dx
2
dy
dy
=

dx 0, 0
dx 0, 0 = 0
Hence tan = 90 at point (0, 0)
5

dy
dx

Example:
Solution.

Example :
Solution.

dy
1
dx ( 4, 4 ) = 2
( 4, 4 )
2
1
2
2
3
tan =
1 =
1 + 2.
4
2
Two curves are not orthogonal because angle at (4, 4) is not 90.
Find the angle between curves y2 = 4x and y = ex/2
Let the curves intersect at point (x1, y1)

for y2 = 4x

dy
dx

and

for

y = ex/2

m1 m2 = 1

2
= y
( x 1, y 1 )
1
dy
1 x / 2
dx ( x1, y1 ) = 2 e 1
Hence = 90

y1
2

Note : here that we have not actually found the intersection point but geometrically we can see
that the curves intersect.
Find possible values of p such that the equation px2 = nx has exactly one solution.
Two curves must intersect at only one point. Hence

(i)

(ii)

I.
if p 0 then only one solution (see graph)
II.
if p > 0
then the two curves must only touch each other
i.e.
tangent at y = px2 and y = nx must have same slope at point (x1, y1)
1

2px1 = x
1

1
2p

x12 =

also

y1 = px12

y1 =

and

.........(i)

1
2

y1 = nx1
x1 = e1/2
1
Hence x12 =
2p

1
y1 = p
2p

........(ii)

1
= nx1
2
.........(iii)
1
e=
2p

p=

1
2e

1
Hence possible values of p are ( , 0]
2e
Self Practice Problems :
For the curve x m + n = am n y2n, where a is a positive constant and m, n are positive integers, prove that the
1.
m th power of subtangent varies as nth power of subnormal.

2.

3.
4.
5.

F.

a+ a 2 x 2
a
n
a 2 x 2 contained between
2
a a 2 x 2
the y-axis & the point of tangency has a constant length .
A curve is given by the equations x = at2 & y = at 3 . A variable pair of perpendicular lines through the origin
'O' meet the curve at P & Q . Show that the locus of the point of intersection of the tangents at P & Q is
4y2 = 3ax - a2 .
Find the length of the subnormal to the curve y2 = x 3 at the point (4, 8).
Ans. 24
Find the angle of intersection of the following curves:
x2 y2
+
(i)
y = x2 & 6y = 7 x3 at (1, 1) (ii)
x 2 y2 = 5 &
= 1.
18 8
Ans. (i)
/2
(ii)
/2

Prove that the segment of the tangent to the curve y =

Shor test distance between two cur ves

Shortest distance between two non-intersecting curves always along the common normal.
(Wherever defined)
6

Example:
Solution.

Find the shortest distance between the line y = x 2 and the parabola y = x2 + 3x + 2.
Let P(x1, y1) be a point closest to the line y = x 2

dy
dx ( x1 , y1 ) = slope of line

2x1 + 3 = 1

x1 = 1

y1 = 0
Hence point ( 1, 0) is the closest and its perpendicular distance from the line y = x 2 will give the
shortest distance
3
.

p=
2

then

Monotonocity
A.

Monotonocity about a point

1.

A function f(x) is called an increasing function at point x = a. If in a sufficiently small neighbourhood around
x = a.
f(a h) < f(a) < f(a + h)

2.

A function f(x) is called a decreasing function at point x = a if in a sufficiently small neighbourhood around
x = a.
f(a h) > f(a) > f(a + h)

Note :
If x = a is a boundary point then use the appropriate one sided inequality to test monotonocity of f(x).

Example :
Which of the following functions is increasing, decreasing or neither increasing nor decreasing
at x = a.

(ii)

(i)

(iii)

(iv)

3.

Test for increasing and decreasing functions at a point


(i)
If f (a) > 0 then f(x) is increasing at x = a.
(ii)
If f (a) < 0 then f(x) is decreasing at x = a.
(iii)
If f (a) = 0 then examine the sign of f (a+) and f (a).
(a)
If f (a+) > 0 and f (a) > 0 then increasing
(b)
If f (a+) < 0 and f (a) < 0 then decreasing
(c)
otherwise neither increasing nor decreasing.
Example :
Let f(x) = x 3 3x + 2. Examine the nature of function at points x = 0, 1, 2.
3
Solution :
f(x) = x 3x + 2
f (x) = 3(x 2 1)
(i)
f (0) = 3

decreasing at x = 0
f (1) = 0
(ii)
also, f (1+ ) = positive and f (1) = negative

neither increasing nor decreasing at x = 1.


(iii)
f (2) = 9

increasing at x = 2
Note : Above rule is applicable only for functions that are differentiable at x = a.

B.

Monotonocity over an interval

A function f(x) is said to be monotonically increasing for all such interval (a, b) where f (x) 0 and equality
may hold only for discreet values of x. i.e. f (x) does not identically become zero for x (a, b) or any sub
interval.
2.
f(x) is said to be monotonically decreasing for all such interval (a, b) where f (x) 0 and equality may hold
only for discrete values of x.
Note : By discrete, points, we mean that points where f (x) = 0 dont form an interval
For example.
Let
f(x) = x 3
f (x) = 3x 2
f (x) > 0 every where except at x = 0. Hence f(x) will be considered monotonically increasing function for x
R. also,
1.

Let

f(x) = x sinx
f (x) = 1 cosx
Now, f (x) > 0 every where except at x = 0, 2, 4 etc. but all these points are discrete and donot form
an interval hence we can conclude that f(x) is monotonically increasing for x R. In fact we can also see
it graphically.

Let us consider another function whose graph is shown for x (a, b).

Here also f (x) 0 for all x (a, b) but note that in this case equality of f (x) = 0 holds for all x (c, d) and
(e, b). Here f (x) become identically zero and hence the given function cannot be assumed to be
monotonically increasing for x (a, b).
Example :
Find the interval where f(x) = x 3 3x + 2 is monotonically increasing.
Solution.
f (x) = x 3 3x + 2
f (x) = 3(x 2 1)
f (x) = 3(x 1) (x + 1)
for M.. f (x) 0

3(x 1) (x + 1)8 0


x [ , 1] [1, )
Note :
(i)
A function is said to be monotonic if it's either increasing or decreasing.
(ii)
The points for which f (x) is equal to zero or doesn't exist are called critical points. Here it
should also be noted that critical points are the interior points of an interval.
(iii)
The stationary points are the points where f (x) = 0 in the domain.
Example :
Find the intervals of monotonicity of following functions.
(i)
f(x) = x 2 (x 2)2
(ii)
f(x) = x n x
(iii)
f(x) = sinx + cosx
;
x [0, 2]
Solution.
(i)
f(x) = x 2 (x 2)2
f (x) = 4x (x 1) (x 2)
observing the sign change of f (x)

Hence M. . for x [0, 1] [2, )


and
M.D. for x ( , 0] [1, 2]
Note : Closed bracket can be used for both M. . as well as M.D. In above example x = 1 is
boundary point for x [0, 1] and since f(1) > f(1 h). So we can say f(x) is M. . at
x = 1 for x [0, 1]. However also note that for the interval x [1, 2] again x = 1
becomes a boundary point and f(1) > f(1 + h). Hence f(x) is M.D. at x = 1 for x [1, 2]
(ii)
f(x) = x n x
f (x) = 1 + n x
1
f (x) 0

n x 1

x
e
1
1

M. . for x , and M.D for x 0 , .


e
e

f(x) = sinx + cosx


f (x) = cosx sinx
for M. . f (x) 0

cosx sinx
5

x 0 , , 2
4
4

(iii)

5
therefore M.D. for x ,

4 4

Exercise
1.
For each of the following graph comment whether f(x) is increasing or decreasing or neither increasing nor
decreasing at x = a.

(i)

(ii)

(iv)

(vi)

(iii)

(i)
neither M. . nor M.D.
(ii)
M.D.
(iii)
M.D
(iv)
M. .
3
2
Let f(x) = x 3x + 3x + 4, comment on the monotonic behaviour of f(x) at (i) x = 0 (ii) x = 1.
Ans. M. . both at x = 0 and x = 1.
x 0 x 1
Draw the graph of function f(x) =
. Graphically comment on the monotonic behaviour of f(x)
[ x ] 1 x 2
at x = 0, 1, 2. Is f(x) M. . for x [0, 2] ?
Ans. M. . at x = 0, 2 ; neither M.. nor M.D. at x = 1. No, f(x) is not M. . for x [0, 2].
Ans.

2.
3.

4.

Find the intervals of monotonicity of the following functions.


(i)
f(x) = x 3 + 6x 2 9x 2
Ans. in [1, 3] ; D in ( , 1] (3, )
1
(ii)
f(x) = x +
Ans. in ( , 2] [0, ) ; D in [ 2, 1) (1, 0]
x +1
1
1

2
(iii)
f(x) = x . e x x
Ans. in , 1 ; D in , [1, )
2
2

(iv)
f(x) = x cosx
Ans. I for x R
9

C.

Classification of functions

1.

Depending on the monotonic behaviour, functions can be classified into following cases.
Increasing functions
2.
Non decreasing functions

3.

Decreasing functions

Non-increasing functions

4.

However note that this classification is not complete and there may be function which cannot be classified
into any of the above cases for some interval (a, b).
Example :
f(x) = [x] is a step up function. Is it a monotonically increasing function for x R.
Solution.
No, f(x) = [x] is not M.I. for x R rather, it is a non-decreasing function as illustrated by its
graph.

Example :
If f(x) = sin4x + cos4x + bx + c, then find possible values of b and c such that f(x) is monotonic
for all x R
Solution.
f(x) = sin4x + cos4x + bx + c
f (x) = 4 sin3x cosx 4cos3x sinx + b = sin4x + b.
(i)
for M.I. f (x) 0
for all x R
b sin4x
for all x R
b1
for M.D. f (x) 0
for all x R
(ii)
b sin4x
for all x R
b1
Hence for f(x) to be monotonic b ( , 1] (1, ) and c R.
2x
Example :
Find possible values of a such that f(x) = e (a + 1) ex + 2x is monotonically increasing for
xR
Solution.
f(x) = e2x (a + 1) ex + 2x
f (x) = 2e2x (a + 1) ex + 2
Now, 2e2x (a + 1) ex + 2 0
for all x R
1
x
for all x R

2 e + x (a + 1) 0
e

1
x
(a + 1) < 2 e + x
for all x R
e

e x + x has min imum value 2

a+14
a3
e

Aliter
2e2x (a + 1) ex + 2 0
for all x R
putting ex = t ; t (0, )
2t 2 (a + 1) t + 2 0 for all t (0, )
Hence either
(i)
D0

(a + 1)2 4 0

(a + 5) (a 3) 0

a [ 5, 3]
or
(ii)
both roots are negative

D0

&

b
<0
2a

&

f(0) 0

a +1
a ( , 5] [3, )
&
<0
4

a ( , 5] [3, )
&
a<1

a ( , 5]
Taking union of (i) and (ii), we get a ( , 3].

10

&
&

20
aR

Exercise
1.
Let f(x) = x tan1x. Prove that f(x) is monotonically increasing for x R.
2.
If f(x) = 2ex aex + (2a + 1) x 3 monotonically increases for x R, then find range of values of a
Ans. a 0
3.
Let f(x) = e2x aex + 1. Prove that f(x) cannot be monotonically decreasing for x R for any value of 'a'.
4.
Find range of values of 'a' such that f(x) = sin2x 8(a + 1) sinx + (40 10)x is monotonically decreasing
x R Ans. a [ 4, 0]
5.
If f(x) = x 3 + (a + 2)x 2 + 5ax + 5 is a one-one function then find values of a.
Ans. a [1, 4]

D.

Proving Inequalities

Comparision of two functions f(x) and g(x) can be done by analysing their monotonic behavior or graph.

Example :
For x 0, prove that sin x < x < tan x
2
Solution.
Let
f(x) = x sin x

f (x) = 1 cos x

f (x) > 0 for x 0,
2

f(x) > f(0)

f(x) is M.I.

x sin x > 0

x > sin x
Similarly consider another function g(x) = x tan x

g(x) = 1 sec2x

g(x) is M.D.
g(x) < 0 for x 0,
2
Hence g(x) < g(0)
x tan x < 0

x < tan x
sin x < x < tan x
Hence proved
tan 1 x
x3
x3

For x (0, 1) prove that x


< tan1 x < x
hence or otherwise find xlim
Example :
0
3
6
x
Solution.

x3
tan1x
3
1
f(x) = 1 x 2
1+ x2
x4
f (x) =
1+ x2
f (x) < 0 for x (0, 1)
f(x) < f(0)

Let

f(x) = x

x3
tan1x < 0
3

x3
< tan1x
3
x3
Similarly g(x) = x
tan1x
6
1
x2

g(x) = 1
1+ x2
2
2
2
x (1 x )
g(x) =
2(1 + x 2 )
g(x) > 0
for x (0, 1)

g(x) > g(0)

f(x) is M.D.

...........(i)

g(x) is M.I.

x3
tan1x > 0
6
x3
x
> tan1x
........(ii)
6
from (i) and (ii), we get

Also,

x3
x3
< tan1x < x
3
6

x2
x2
tan 1 x
<
<1
3
6
x

Hence Proved

tan 1 x
= 1 but it must also be noted that
Hence by sandwich theorem we can prove that xlim
0
x
tan 1 x
tan 1 x
as x 0, value of
1 from left hand side i.e.
<1
x
x
tan1 x
lim
=0

x 0
x
11

NOTE : In proving inequalities, we must always check when does the equality takes place because the point of
equality is very important in this method. Normally point of equality will occur at the end point of intervals
or will be easy to be predicated by hit and trial

x3
Example :
For x 0, , prove that sin x > x
2
6

f(x) = sin x x +

x3
6

Solution

Let

Example :

x2
2
we cannot decide at this point wether f (x) is positive or negative, hence let us check for
monotonic nature of f (x)
f (x) = x sinx

Since f (x) > 0

f (x) is M.I.
for x 0,
2

f (x) > f(0)

f (x) > 0

f(x) is M.I.

f(x) > f(0)


x3
x3

sin x x +
>0

sin x > x
Hence proved
6
6
sin x tan x

Examine which is greater sin x tan x or x 2. Hence evaluate xlim
, where x 0, 2
2
0
x

Let
f(x) = sinx. tanx x 2
2
f (x) = cos x . tan x + sin x . sec x 2x

f (x) = sin x + sin x sec2x 2x

f (x) = cos x + cos x sec2x + 2sec2x sin x tan x 2

f (x) = (cos x + sec x 2) + 2 sec2x sin x tan x



2
Now
cos x + sec x 2 = cos x sec x and 2 sec2x tan x . sin x > 0 because x 0,
2

f (x) > 0

f (x) is M.I.
Hence f (x) > f (0)

f (x) > 0

f(x) is M.I.

f(x) > 0

sin x tan x x 2 > 0


Hence sin x tan x > x 2
sin x tan x
sin x tan x
lim

>1

=1
2
x 0
x2

x
f (x) = cos x 1 +

Solution

Prove that f(x) = 1 + is monotonically increasing in its domain. Hence or otherwise draw
x

graph of f(x) and find its range

Example :

Solution.

1
f(x) = 1 + , for Domain of f(x) 1 +
>0
x
x

x +1
>0

( , 1) (0, )

1
x

Consider
f (x) = 1 + n1 + +
2
1
x
x
1+
x

x
1
1
1

f (x) = 1 + n1 +

x
x
+ 1
x

1
1

Now 1 + is always positive, hence the sign of f (x) depends on sign of n 1 +


x

x 1+ x

1
i.e.
we have to compare n 1 + and
x

1+ x
1
1

So lets assume g(x) = n 1 +


x
+1
x

g(x) =

1 x
x
for x (0, ), g(x) < 0
g(x) is M.D. for x (0, )
g(x) > lim g(x)
1+

(i)

1
( x + 1)

g(x) > 0.

12

g(x) =

x( x + 1)2

and
(ii)

since g(x) > 0


f (x) > 0
for x ( , 1), g(x) > 0

g(x) is M.I. for x ( , 1)

g(x) > xlim


g(x)

g(x) > 0

f (x) > 0
Hence from (i) and (ii) we get f (x) > 0 for all x ( , 1) (0, )

f(x) is M.I. in its Domain


For drawing the graph of f(x), its important to find the value of f(x) at boundary points
i.e.
, 0, 1
x

lim

1 + = e
x

lim 1 + 1 = 1
x 0 +
x
so the graph of f(x) is

E.

and

lim

x 1

1 + =
x

Range is y (1, ) {e}

Proving inequalities using graph


Generally these inequalities involve comparison between values of two functions at some particular points.
e 2 x1 + e x 2
> e
3

2 x1 + x 2
3

Example :

Prove that for any two numbers x 1 & x2 ,

Solution.

Assume f(x) = ex and let x 1 & x 2 be two points on the curve y = ex.
Let R be another point which divides P and Q in ratio 1 : 2.

2 x1
x2
y coordinate of point R is e + e and y coordinate of point S is e
3
always concave up, hence point R will always be above point S.

2 x1 + x 2
3

. Since f(x) = ex is

2x + x

1
2
e 2 x1 + e x 2
< e 3
3
(above inequality could also be easily proved using AM and GM.)
x1 + x 2 + x 3
sin x1 + sin x 2 + sin x 3
>
Example :
If 0 < x 1 < x 2 < x 3 < then prove that sin
. Hence or
3
3

otherwise prove that if A, B, C are angles of a triangle then maximum value of

sinA + sinB + sinC is

3 3
.
2

Solution.

Let point A, B, C form a triangle y coordinate of centroid G is


x1 + x 2 + x 3
.
coordinate of point F is sin
3

13

sin x1 + sin x 2 + sin x 3


and y
3

x1 + x 2 + x 3
sin x1 + sin x 2 + sin x 3
>
Hence sin
.
3
3

if
A + B + C = , then
A +B+C
sin A + sin B + sin C
>
sin

3
3

3 3
> sin A + sin B + sin C
2
maximum value of (sinA + sinB + sinC) =

Exercise
Prove the following inequalities
1.
(i)
x < n(1 x)
(ii)
x > tan1(x)
(iii)
ex > x + 1
x
(iv)
n (1 + x) x
1+ x
2
sin x
<1
(v)
<

x
2.
3.

4.

5.

F.
(a)

(b)

sin A + sin B + sin C


>
3
3

3 3
2
Compare which of the two is greater (100)1/100 or (101)1/101.
Assume f(x) = x 1/x and let us examine monotonic nature of f(x)
1 nx

f (x) = x 1/x .
x2
f (x) > 0

x (0,e)
and f (x) < 0
x (e, )
Hence f(x) is M.D. for x e
and since 100 < 101

f(100) > f(101)

(100)1/100 > (101)1/101

Example :
Solution.

sin

for
for
for

x (0, 1)
x (0, )
x (0, )

for

x (0, )

for

x 0,
2

1+ e2
1 + 2
1 + e2
or
Ans.
e

e
If 0 < x 1 < x 2 < x 3 < , then prove that
2x1 + x 2 + x 3
2 sin x1 + sin x 2 + sin x 3
>
sin
4

4
If f(x) is monotonically decreasing function and f (x) > 0. Assuming f 1(x) exists prove that
x1 + x 2
f 1( x1 ) + f 1( x 2 )
.
> f 1
2

2
Using f(x) = x 1/x, identify which is larger e or e.
Ans. e

Identify which is greater

Mean Value of Theorems

Rolle's Theorem:
Let f(x) be a function of x subject to the following conditions:
(i)
f(x) is a continuous function of x in the closed interval of a x b.
(ii)
f (x) exists for every point in the open interval a < x < b.
f(a) = f(b).
(iii)
Then there exists at least one point x = c such that f (c) = 0 c (a,b).
LMVT Theorem:
Let f(x) be a function of x subject to the following conditions:
(i)
f(x) is a continuous function of x in the closed interval of a x b.
(ii)
f (x) exists for every point in the open interval a < x < b.
(iii)

Then there exists at least one point x = c such that a < c < b where f (c) =

f(a) f(b).

f (b) f (a )
ba

Geometrically, the slope of the secant line joining the curve at x = a & x = b is equal to the slope of the
tangent line drawn to the curve at x = c. Note the following:
* Rolle's theorem is a special case of LMVT since
f (b ) f ( a )
f(a) = f(b) f (c) =
= 0.
ba
(c)
Application Of Rolles Theorem For Isolating The Real Roots Of An Equation f(x) = 0
Suppose a & b are two real numbers such that;
(i)
f(x) & its first derivative f (x) are continuous for a x b.
f(a) & f(b) have opposite signs.
(ii)
(iii)
f (x) is different from zero for all values of x between a & b.
Then there is one & only one real root of the equation f(x) = 0 between a & b.
Example :
If 2a + 3b + 6c = 0 then prove that the equation ax 2 + bx + c = 0 has atleast one real root
between 0 and 1.
ax 3
bx 2
Solution.
Let f(x) =
+
+ cx
3
2
f(0) = 0
14

a
b
+
+ c = 2a + 3b + 6c = 0
3
2
If
f(0) = f(1) then f (x) = 0 for some value of x (0, 1)

ax 2 + bx + c = 0 for atleast one x (0, 1)


Verify Rolles throrem for f(x) = (x a)n (x b)m where m, n are natrual numbers for x [a, b].
Being a polynomial function f(x) is continuous as well as differentiable, f(a) = 0 and f(b) = 0

f (x) = 0 for some x (a , b)


n(x a)n1 (x b)m + m(x a) n (x b)m1 = 0

(x a)n1 (x b)m1 [(m + n) x (nb + ma)] = 0


nb + ma

x=
, which lies in the interval (a, b)
m+n
Verify LMVT for f(x) = x 2 + 4x 5 and x [1, 1]
f(1) = 2
;
f(1) = 10
f (1) f ( 1)

f (c) =
1 ( 1)

2c + 4 = 4

c=0
ba
ba
< tan1 b tan1 a <
Using mean value theorem, prove that if b > a > 0, then
1 + b2
1 + a2
Let f(x) = tan1 x ; x [a, b] applying LMVT
1
tan 1 b tan 1 a
,
f (c) =
for a < c < b and f (x) =
ba
1+ x 2
Now f (x) is a monotonically decreasing function
Hence if a < c < b

f (b) < f(c) < f(a)


1
1
tan 1 b tan 1 a

<
Hence proved
2 <
1+ b
1+ a2
ba

and

Example :
Solution.

Example :
Solution.

Example :
Solution.

f(1) =

Maxima - Minima

A.

st Fundamental Theorem

1.
A function f(x) is said to have a local maximum at x = a if f(a) > f(x) x (a h, a + h). Where h
is a very small positive arbitrary number.

Note : The local maximum of a function is the largest value only in neighbourhood of point x = a.
2.
A function f(x) is said to have local minimum at x = a
if f(a) < f(x) x (a h, a + h).

First fundamental theorem is applicable to all functions continuous, discontinuous, differentiable or


nondifferentiable at x = a.
| x | 0 < | x | 2
Example :
Let f(x) =
. Examine the behaviour of
f(x) at x = 0.
1 x=0
Solution.
f(x) has local maxima at x = 0.

Example :

Solution.

(b 3 b 2 + b 1)
x3 +
0 x <1
Let f(x) =
(b 2 + 3b + 2)
2x 3
1 x 3

Find all possible values of b such that f(x) has the smallest value at x = 1.
Such problems can easily solved using graphical approach.

15

Hence the limit value of f(x) from left of x = 1 should either be greater than or equal to the value
of function at x = 1.
lim f(x) f(1)
x 1

1+

(b 3 b 2 + b 1)

>1

(b 2 + 1)(b 1)
0
(b + 1) (b + 2)

(b 2 + 3b + 2)

b ( 2, 1) [1, )
Note : If x = a happens to be a boundary point of the function, then compare the value of f(a) with appropriate
values in either the left or right neighbourhood of x = a.

B.

From these figure we can see that boundary points are almost always points of local maxima/
minima.

Global Maxima/Minima

Global maximum or minimum value of f(x), x [a, b] basically refers to the greatest value and least value
of f(x) over that interval mathematically
(i)
If f(c) f(x) for x [a, b] then f(c) is called global maximum or absolute maximum value of
f(x).
(ii)
Similarly if f(d) f(x) x [a, b] then f(d) is called global minimum or absolute minimum value.
For example consider the graph of function

f(x) has local maxima at x = c, e, b and local minima at x = a, d, f. It can also be easily seen that f(b) is
the greatest value and hence global maximum and similarly f(d) is global minimum.
Also be careful about the fact that a function has global maximum or minimum value when it actually
achieves these values.
2x 1 1 x < 2
Let us take graph of function as f(x) =
4 x 2 x 4

This function has local minima at x = 1, 4 and at x = 2, it is a monotonically decreasing function and hence
neither maximum nor minimum.
f(4) = 0, which the global minimum value but global maximum value is not defined. The value of function can
be made as close to 3 as we may please.
3 2x 0 x < 1

1 x < 2
Also consider graph of another function as shown f(x) 1
x 1 2 x 3

f(x) has local maxima at x = 0, 3 and f(0) = 3 value 1 over this interval which is global minimum although
note that f(x) does not has local minima at x = 1, 2.
Self Practice Problems
1.
In each of following case identify if x = a is point of local maxima, minima or neither of them
16

(i)

(ii)

(iii)

(iv)

Ans.

(i)
(iii)

Maxima
Minima

(ii)
(iv)

Neither maxima nor minima


Neither maxima nor minima

( x + )2 x < 0
2. If f(x) =
, find possible values of such that f(x) has local maxima at x = 0. Ans. [ 1, 1)
cos x
x0
3.
Draw the graph of function f(x) = 2 |x 2| + 5 |x 3| (x R). Also identify points of local Maxima/Minima
and also global Maximum/Minimum values
Ans. Local minima at x = 3, Global minimum value 2 at x = 3, No point of local maximum, Global
maximum value is not defined.
4.
Examine the graph of following functions in each case identify the points of global maximum/minimum and
local maximum / minimum.

(i)

Ans.

C.

(ii)

(i)
(ii)
(iii)

(iii)

Local maxima at x = 2, Local minima at x = 3, Global maxima at x = 2


Local minima at x = 1, No point of Global minima, no point of local or Global maxima
Local & Global maxima at x = 1, Local & Global minima at x = 0.

nd Fundamental Theorem

Following points should be examined for maxima/minima in an interval.


1.
Points where f(x) = 0
2.
Points where f(x) does not exists
3.
Boundary points of interval (only when the interval is closed).
Example :
Find the possible points of Maxima/Minima for f(x) = |x 2 2x| (x R)
x 2 2x x 2

2
x<x<2
Solution.
f(x) = 2x x
x 2 2x x 0

2( x 1) x > 2

2
f(x) = 2(1 x ) 0 < x < 2
2( x 1) x < 0

f(x) = 0 at x = 1 and f(x) does not exist at x = 0, 2. Thus these are the possible critical points.

f(x) = |x 2 2x|

Example :
Solution.

from graph we can see that x = 1 is a point of local mixima where as x = 0, 2 are points of local
minima.
If f(x) = x 3 + ax 2 + bx + c has extreme values at x = 1 and x = 3. Find a, b, c.
Extreme values basically mean maximum or minimum values, since f(x) is differentiable function
so
f( 1) = 0 = f(3)
f(x) = 3x 2 + 2ax + b
f(3) = 27 + 6a + b = 0
f( 1) = 3 2a + b = 0

a = 3, b = 9, c R

Critical Points

All those points in the interior of an interval where f(x) is either equal to zero or does not exist are called
critical points.
Example:
Find the critical points of the function f(x) = 4x 3 6x 2 24x + 9 if (i) x [0, 3] (ii) x [3, 3]
(iii) x [ 1, 2].
Solution.
f(x) = 12(x 2 x 2)
= 12(x 2) (x + 1)
f(x) = 0

x = 1 or 2
17

Example :
Solution.

(i)
if x [0, 3] , x = 2 is the critical point.
(ii)
if x [ 3, 3], then we have two critical points x = 1, 2.
(iii)
If x [ 1, 2], then no critical point as both x = 1 and x = 2 become boundary points.
Note : Critical points are always interior points of an interval.
Find the number of critical points for f(x) = max (sinx, cosx) f, x (0, 2).

f(x) has three critical points x =

D.

Test for Maxima/Minima

5
, ,
.
4 2 4

Upto now we have been able to identity exactly which points should be examined for finding the extreme
values of a function. Let as now consider the various tests by which we can separate the critical points into
points of local maxima or minima.
1.
st derivative Test
(i)
If f(x) changes sign from negative to positive while passing through x = a from left to
right then x = a is a point of local maxima
(ii)
If f(x) changes sign from positive to negative while passing through x = a from left to
right then x = a is a point of local minima.
(iii)
If f(x) does not changes its sign about x = a then x= a is neither a point of maxima nor
minima.
Note : This test is applicable only for continuous functions. If f(x) is discontinuous at x = a, then use
of st fundamental theorem is advisable for investigating maxima/minima.
Example :
Find the points of maxima or minima of f(x) = x 2 (x 2) 2.
Solution.
f(x) = x 2 (x 2)2
f(x) = 4x (x 1) (x 2)
f(x) = 0

x = 0, 1, 2
examining the sign change of f(x)

Hence x = 1 is point of maxima, x = 0, 2 are points of minima.


Note : In case of continuous functions points of maxima and minima are alternate.
Example :
Find the points of Maxima/Minima of f(x) = x 3 12x also draw the graph of this functions.
Solution.
f(x) = x 3 12x
f(x) = 3(x 2 4) = 3(x 2) (x + 2)

x=2
f(x) = 0

For tracing the graph let us find maximum and minimum values of f(x).

f (x)

2 16
2 + 16

Example :
Solution.
Aliter

Find the greatest and least values of f(x) = x 3 12x


x [ 1, 3]
By graph of the function f(x) = x 3 12x we can easily see that minimum value of f(x) is 16 and
maximum value is 11.

We can use nd fundamental theorem. The possible points of maxima/minima are critical points
and the boundary points.
for
x [ 1, 3] and f(x) = x 3 12x
x = 2 is the only critical points.
Hence points of local maxima/minima are x = 1, 2, 3. Examining the value of f(x) at these
points we can find greatest and least values.
x f ( x)
1 11
2 16
3 9

Minima f(x) = 16 & Maxima f(x) = 11.


Example:
Show that f(x) = (x 3 6x 2 + 12x 8) does not have any point of local maxima or minima.
3
Solution.
f(x) = x 6x 2 + 12x 8
f(x) = 3(x 2 4x + 4)
f(x) = 3(x 2)2
f(x) = 0

x=2
18

but clearly f(x) does not change sign about x = 2. f(2+ ) > 0 and f(2) > 0. So f(x) has no point
of maxima or minima. In fact f(x) is a monotonically increasing function for x R.

Example :
Solution.

Example :
Solution.

x 3 + x 2 10x x < 0
. Examine the behaviour of f(x) at x = 0.
Let f(x) =
3 sin x
x0
f(x) is continuous at x = 0.
3x 2 + 2x 10 x < 0
f(x) =
3 cos x
x>0
f(0+) = 3 and f(0) = 10 thus f(x) is non-diff. at x = 0
x = 0 is a critical point.
Also derivative changes sign from negative to positive. So x = 0 is a point of local minima.
3
2
2
Let f(x) = x + 3(a 7)x + 3(a 9) x 1. If f(x) has positive point of maxima, then find possible
value of 'a'.
f(x) = 3 [x 2 + 2(a 7)x + (a2 9)] = 0
Let , be roots of f(x) = 0 and let be the smaller root. Examining sign change of f(x).

Maxima occurs at smaller root which has to be positive. This basically implies that both of
roots f(x) = 0 must be positive.
Applying location of roots
29
(i)
D>0

a<
7
b
(ii)

>0

a<7
2a
(iii)
f(0) > 0

a ( , 3) (3, )
29
from (i), (ii) and (iii)

a ( , 3) 3,

7
Self Practice Problems :
1.
Let f(x) = 2x 3 9x 2 + 12x + 6
(i)
Find the possible points of Maxima/Minima of f(x) for x R.
(ii)
Find the number of critical points of f(x) for x [0, 2].
(iii)
Discuss absoluble Maxima/Minima value of f(x) for x [0, 2]
(iv)
Prove that for x (1, 3), the function does not has a Global maximum.
Ans. (i)
x = 1, 2
1 (x = 1)
(ii)
(iii)
f(0) = 6 is the global minimum, f(1) = 11 is global maximum
2.
Let f(x) = sinx (1 + cosx) ; x (0, 2). Find the number of critical points of f(x). Also identify which of these
critical points are points of Maxima/Minima.

5
Ans. 3 critical point x = , ,
3
3

5
Local maxima at x = , Local minima at x =
.
3
3
x
2
3.
Let f(x) =
+ . Find local maximum and local minimum value of f(x). Can you explain this discrepancy
2
x
of locally minimum value being greater than locally maximum value.
Ans. Local maxima at x = 2
f(2) = 2
Local minima at x = 2
f(2) = 2.
4.
Find the points of local Maxima or Minima of following functions
(i)
f(x) = (x 1)3 (x + 2) 2
(ii)
f(x) = sin 2x x
(iiI)
f(x) = x 3 + x 2 + x + 1.
Ans. (i)
Maxima at x = 2, Minima at x = 0

(ii)
Maxima at x = n +
; Minima at x = n
6
6
(iii)
No point of local maxima or minima.
nd
2.
derivative Test
If f(x) is continuous function in the neighbourhood of x = 0 such that f(x) = 0 and f(a) exists then
we can predict maxima or minima at x = 0 by examining the sign of f(a)
(i)
If f(a) > 0 then x = a is a point of local minima.
(ii)
If f(a) < 0 then x = a is a point of local maxima.
(iii)
If f(a) = 0 then second derivative test does not gives use conclusive results.
Example :
Find the points of local maxima or minima for f(x) = sin2x x, x (0, ).
Solution.
f(x) = sin2x x
f(x) = 2cos2x 1
5
1
19
f(x) = 0

cos 2x =
x= ,
6
6
2

f(x) = 4 sin 2x

f < 0
6

3.

Example :
Solution.

E.

Maxima at x =

5
5

Minima at x =
f > 0
6
6

n th derivative test
Let f(x) be function such that f(a) = f(a) = f(a) = ........= f n 1 (a) = 0 & f n (a) = 0, then

n = even,
n = odd
(i) f n (a) > 0

Minima
Neither Maxima nor Minima at x = a
(ii) f n (a) < 0

Maxima
Find points of local maxima or minima of f(x) = x 5 5x 2 + 5x 3 1
f(x) = x 5 5x 2 + 5x 3 1
f(x) = 5x 2 (x 1) (x 3)
f(x) = 0

x = 0, 1, 3
f(x) = 10x (2x 2 6x + 3)
Now, f(1) < 0

Maxima at x = 1
f(3) > 0

Minima at x = 3
and,
f(0) = 0

nd derivative test fails


f(x) = 30 (2x 2 4x + 1)
so,
f(0) = 30

Neither maxima nor minima at x = 0.


Note : It was very convenient to check maxima/minima at first step by examining the sign
change of f(x) no sign change of f(x) at x = 0
f(x) = 5x 2 (x 1) (x 3)

Application of Maxima/Minima to Problems

Example :
Solution.

Example :
Solution.

Find two positive numbers x and y such that x + y = 60 and xy3 is maximum.
x + y = 60

x = 60 y

xy3 = (60 y)y3


Let
f(y) = (60 y) y3
;
y (0, 60)
for maximizing f(y) let us find critical points
2
3
f(y) = 3y (60 y) y = 0
f(y) = y2 (180 4y) = 0

y = 45
f(45+) < 0 and f(45) > 0. Hence local maxima at y = 45.
So
x = 15 and y = 45.
Rectangles are inscribe inside a semi-circle of radius r. Find the rectangle with maximum area.
Let sides of rectangle be x and y.

A = xy.
Here x and y are not independent variables and are related by pythogoreas theorem with r.
x2
+ y 2 = r2
4

A(x) = x

A(x) =

r2

x2
4

x 2r 2

x4
4

y=

r2

x2
4

x4
;
x (0, r)
4
A(x) is maximum when f(x) is maximum
Hence f (x) = x(2r2 x 2) = 0

x=r 2
Let

f(x) = r 2x 2

also

f (r 2 + ) < 0

and

f (r 2 ) > 0

confirming at f(x) is maximum when x = r 2 & y =


Aliter

.
2
Let use choose coordinate system with origin as centre of circle
A = xy

A = 2 (rcos) (rsin)

A = r2 sin2

0,
2

20

Clearly A is maximum when =

x=r 2

and

y=

.
2
Example:
A sheet of area 40 m in used to make an open tank with square base. Find the dimensions of
the base such that volume of this tank is maximum.
Solution.
Let length of base be xm and height be ym.
2

v = x 2y
again x and y are related to surface area of this tank which is equal to 40 m 2.

x 2 + 4xy = 40
40 x 2
40 x 2

2
V(x) = x 4 x
y=
x (0, 40 )
4x

( 40x x 3 )
4
maximizing volume,
V(x) =

V(x) =

( 40 3 x 2 )
=0
4

x=

40
m
3

40

V 3 < 0.

40
confirming that volume is maximum at x =
m.
3
Example :
If a right circular cylinder is inscribed in a given cone. Find the dimensions of the cylinder such
that its volume is maximum.
Solution.
Let x be the radius of cylinder and y be its height
v = x 2y
x, y can be related by sing similar triangles
y
h
=
rx
r
h
(r x)

y=
r

and

V(x) =

3x
2

v(x) = x 2

h
(r x)
r

x (0, r)

h
(rx 2 x 3)
r
h
v (x) =
x (2r 3x)
r

v(x) =

2r
v > 0
3
2r
h
Thus volume is maximum at x = and y = .
3
3

Note : Following formulae of volume, surface area of important solids are very useful in problems of
maxima & minima.

v (x) = 0

6.

and

Useful Formulae of Measuration to Remember :

1.
3.
5.
7.
9.
11.
12.
13.
14.
15.

Volume of a cuboid = bh.


2.
Surface area of cuboid = 2(b + bh + h).
Volume of cube = a3
4.
Surface area of cube = 6a2
1
Volume of a cone = r2 h.
6.
Curved surface area of cone = r ( = slant height)
3
Curved surface of a cylinder = 2rh. 8.
Total surface of a cylinder = 2rh + 2r2.
4
10.
Surface area of a sphere = 4r2.
Volume of a sphere = r3.
3
1
Area of a circular sector = r2 , when is in radians.
2
Volume of a prism = (area of the base) (height).
Lateral surface of a prism = (perimeter of the base) (height).
Total surface of a prism = (lateral surface) + 2 (area of the base)
(Note that lateral surfaces of a prism are all rectangle).
1
Volume of a pyramid =
(area of the base) 21(height).
3

16.
Example :
Solution.

1
Curved surface of a pyramid =
(perimeter of the base) (slant height).
2
(Note that slant surfaces of a pyramid are triangles).
Among all regular square pyramids of volume 36 2 cm 3. Find dimensions of the pyramid having
least lateral surface area.
Let the length of a side of base be x cm and y be the perpendicular height of the pyramid
1
V=
area of base x height
3
1 2
x y = 36 2

V=
3

and

but

y=

108 2

x2
1
perimeter of base x slant height
S=
2

1
(Ax). 
2

=

x2
+ y2
4

S = 2x
S(x) =

Let

x2
+ y2 =
4
x4 +

f(x) = x 4 +

f (x) = 4x 3

x + 4x y
4

2 2

S=

108 2

x 4 + 4x 2
x2

8.(108)2

x2
8.(108 )2
x2

for minimizing f(x)

16(108 )2

=0
x3
( x 6 66 )

f (x) = 4
=0
x3

x = 6, which a point of minima


Hence x = 6 cm and y = 3 2 .
Example :
Let A(1, 2) and B( 2, 4) be two fixed points. A variable point P is chosen on the straight line
y = x such that perimeter of PAB is minimum. Find coordinates of P.
Solution.
Since distance AB is fixed so for minimizing the perimeter of PAB, we basically have to
minimize (PA + PB)
Let A be the mirror image of A in the line y = x.
F(P) = PA + PB
F(P) = PA + PB
But for PAB

PA + PB AB and equality hold when P, A and B comes collinear. Thus for minimum path
length point P is that special point for which PA and PB be come incident and reflected rays
with respect to the mirror y = x.
Equation of line joining A and B is y = 2x intersection of this line with y = x is the point P.
Hence P (0, 0).

Note : Above concept is very useful because such problems become very lengthily by making perim eter as a
function of position of P and then minimizing it.
Self Practice Problems :
1.
Find the two positive numbers x and y whose sum is 35 and the product x 2 y5 maximum.
Ans. x = 25, y = 10.
2.
A square piece of tin of side 18 cm is to be made into a box without top by cutting a square from each
22
corner and folding up the slops to form a box. What should
be the side of the square to be cut off such that

3.

volume of the box is maximum possible.


Ans. 3 cm
Prove that a fight circular cylinder of given surface area and maximum volume is such that the height is
equal to the diameter of the base.

4.

A normal is drawn to the ellipse

5.
6.

F.

x2
y2
+
= 1. Find the maximum distance of this normal from the centre.
25
16

Ans. 1 unit
A line is drawn passing through point P(1, 2) to cut positive coordinates axes at A and B. Find minimum
area of PAB. Ans. 4 units
Two towns A and B are situated on the same side of a straight road at distances a and b respectively
perpendiculars drawn from A and B meet the road at point c and d respectively. The distance between C
and D is C. A hospital is to be built at a point P on the road such that the distance APB is minimum. Find
ac
from c.
position of P.
Ans. P is at distance of
a+b

Points of Inflection

For continuous function f(x), If f (x 0) = 0 or doesnot exist at points where f (x 0) exists and if f (x) changes
sign when passing through x = x 0 then x 0 is called a point of inflection. At the point of inflection, the curve
changes its concavity i.e.
(i)
If f (x) < 0, x (a, b) then the curve y = f (x) is convex in (a, b)

(ii)

Example :
Solution.

If f (x) > 0, x (a, b) then the curve y = f (x) is concave in (a, b)

Find the points of inflection of the function f(x) = sin2x x [0, 2]


f(x) = sin2x
f (x) = sin2x
f (x) = 2 cos2x
3
f (0) = 0

x= ,
4
4
both these points are inflection points as sing of f (x) change but f (x) does not changes about
these points.

Example :
Find the inflection point of f(x) = 3x 4 4x 3. Also draw the graph of f(x) giving due importance to
maxima, minima and concavity.
Solution.
f(x) = 3x 4 4x 3
f (x) = 12x 3 12x 2
f (x) = 12x 2 (x 1)
f (x) = 0

x = 0, 1

examining sign change of f (x)


thus x = 1 is a point of local minima
f (x) = 12(3x 2 2x)
f (x) = 12x(3x 2)
2
f (x) = 0

x = 0, .
3
Again examining sign of f (x)
2
thus x = 0,
are the inflection points
3
Hence the graph of f(x) is

23

SHORT REVISION
TANGENT & NORMAL
THINGS TO REMEMBER :
I
The value of the derivative at P (x1 , y1) gives the
slope of the tangent to the curve at P. Symbolically
f (x1) =
II

= Slope of tangent at
1 y1

P (x1 y1) = m (say).


Equation of tangent at (x1, y1) is ;
y y1 =

III

dy
d x x

dy

d x x

(x x1).
1 y1

Equation of normal at (x1, y1) is ;


1
(x x1).
y y1 =
dy
d x x

1 y1

NOTE :
1.
The point P (x1 , y1) will satisfy the equation of the curve & the equation of tangent & normal line.
2.
If the tangent at any point P on the curve is parallel to the axis of x then dy/dx = 0 at the point P.
3.
If the tangent at any point on the curve is parallel to the axis of y, then dy/dx = or dx/dy = 0.
4.
If the tangent at any point on the curve is equally inclined to both the axes then dy/dx = 1.
5.
If the tangent at any point makes equal intercept on the coordinate axes then dy/dx = 1.
6.
Tangent to a curve at the point P (x1, y1) can be drawn even through dy/dx at P does not exist.
e.g. x = 0 is a tangent to y = x2/3 at (0, 0).
7.
If a curve passing through the origin be given by a rational integral algebraic equation, the equation of the tangent
(or tangents) at the origin is obtained by equating to zero the terms of the lowest degree in the equation.
e.g. If the equation of a curve be x2 y2 + x3 + 3 x2 y y3 = 0, the tangents at the origin are given by
x2 y2 = 0 i.e. x + y = 0 and x y = 0.
IV
Angle of intersection between two curves is defined as the angle between the 2 tangents drawn to the
2 curves at their point of intersection. If the angle between two curves is 90 every where then they are
called ORTHOGONAL curves.
y1 1 + [f ( x1 ) ]2
y1
V
(a) Length of the tangent (PT) =
(b) Length of Subtangent (MT) =

f ( x1 )
f ( x1 )

(d) Length of Subnormal (MN) = y1 f ' (x1)


(c) Length of Normal (PN) = y1 1 + [f ( x1 )]2
VI
DIFFERENTIALS :
The differential of a function is equal to its derivative multiplied by the differential of the independent variable.
Thus if, y = tan x then dy = sec2 x dx.
In general dy = f (x) d x.
Note that :
d (c) = 0 where 'c' is a constant.
d (u + v w) = du + dv dw
d (u v) = u d v + v d u
Note :1.
For the independent variable 'x' , increment x and differential d x are equal but this is not the case
with the dependent variable 'y' i.e. y d y.
dy
2.
The relation d y = f (x) d x can be written as
= f (x) ; thus the quotient of the differentials of 'y' and 'x'
dx
is equal to the derivative of 'y' w.r.t. 'x'.

EXERCISE1

Q.1
Q.2
Q.3
Q.4
Q.5

Q.6

Find the equations of the tangents drawn to the curve y2 2x3 4y + 8 = 0 from the point (1, 2).
Find the point of intersection of the tangents drawn to the curve x2y = 1 y at the points where it is
intersected by the curve xy = 1 y.
Find all the lines that pass through the point (1, 1) and are tangent to the curve represented parametrically as
x = 2t t2 and y = t + t2.
In the curve xa yb = Ka+b , prove that the portion of the tangent intercepted between the coordinate axes is
divided at its point of contact into segments which are in a constant ratio. (All the constants being positive).
A straight line is drawn through the origin and parallel to the tangent to a curve
a + a 2 y2
x + a 2 y2

= ln
at an arbitary point M. Show that the locus of the point P of intersection
y
a

of the straight line through the origin & the straight line parallel to the x-axis & passing through the point M
is x2 + y2 = a2.
2
2
a a+ a x
Prove that the segment of the tangent to the curve y =24 ln
2 a a2 x2

a 2 x 2 contained between

the y-axis & the point of tangency has a constant length.


A function is defined parametrically by the equations
1
1
2
sin t 2 if t 0
if t 0
2t + t sin

t
f(t) = x =
and g(t) = y = t
0
o
if t = 0
if t = 0
Find the equation of the tangent and normal at the point for t = 0 if exist.
Q.8 Find all the tangents to the curve y = cos (x + y), 2 x 2, that are parallel to the line x + 2y = 0.
Q.9 (a)
Find the value of n so that the subnormal at any point on the curve xyn = an + 1 may be constant.
(b)
Show that in the curve y = a. ln (x a), sum of the length of tangent & subtangent varies as the
product of the coordinates of the point of contact.
Q.10 Prove that the segment of the normal to the curve x = 2a sin t + a sin t cos2t ; y = a cos3t contained
between the co-ordinate axes is equal to 2a.
Q.11 Show that the normals to the curve x = a (cos t + t sin t) ; y = a (sin t t cos t) are tangent lines to the circle
x2 + y2 = a2.
1
at the point x = 2 and is bisected
Q.12 The chord of the parabola y = a2x2 + 5ax 4 touches the curve y =
1 x
by that point. Find 'a'.
Q.13 If the tangent at the point (x1, y1) to the curve x3 + y3 = a3 (a 0) meets the curve again in (x2, y2) then show
x
y
that 2 + 2 = 1.
x1 y1
1
Q.14 Determine a differentiable function y = f (x) which satisfies f ' (x) = [f(x)]2 and f (0) = . Find also the
2
equation of the tangent at the point where the curve crosses the y-axis.
Q.15 If p1 & p2 be the lengths of the perpendiculars from the origin on the tangent & normal respectively at any
dy
p = x sin y cos
point (x, y) on a curve, then show that 1
where tan = . If in the above case,

p 2 = x cos + y sin
dx
the curve be x2/3 + y2/3 = a2/3 then show that : 4 p12 + p22 = a2.
Q.16 The curve y = ax3 + bx2 + cx + 5 , touches the x - axis at P ( 2 , 0) & cuts the y-axis at a point Q where its
gradient is 3. Find a , b , c.
Q.17 The tangent at a variable point P of the curve y = x2 x3 meets it again at Q. Show that the locus of the
middle point of PQ is y = 1 9x + 28x2 28x3.
Q.18 Show that t he distance from the origin of the normal at any point o f the curve

x = a e sin + 2 cos & y = a e cos 2 sin is twice the distance of the tangent at the point from
2
2
2
2

the origin.
Q.19 Show that the condition that the curves x2/3 + y2/3 = c2/3 & (x2/a2) + (y2/b2) = 1 may touch if c = a + b.
Q.20 The graph of a certain function f contains the point (0, 2) and has the property that for each number 'p' the
line tangent to y = f (x) at (p, f (p) ) intersect the x-axis at p + 2. Find f (x).
Q.21 A curve is given by the equations x = at2 & y = at3. A variable pair of perpendicular lines through the origin
'O' meet the curve at P & Q. Show that the locus of the point of intersection of the tangents at P & Q is 4y2
= 3ax a2.
x2
y2
x2
y2
+ 2
+ 2
Q.22(a) Show that the curves 2
=1& 2
= 1 intersect orthogonally..
a + K1 b + K1
a + K2 b + K2
Q.7

(b) Find the condition that the curves

x 2 y2
x 2 y2
+
+
=1&
= 1 may cut orthogonally..
a
b
a b

y
Q.23 Show that the angle between the tangent at any point 'A' of the curve ln (x2 + y2) = C tan1 x and the line
joining A to the origin is independent of the position of A on the curve.
2

Q.24 For the curve x2/3 + y2/3 = a2/3, show that z + 3p2 = a2 where z = x + i y & p is the length of the
perpendicular from (0 , 0) to the tangent at (x , y) on the curve.
Q.25 A and B are points of the parabola y = x2. The tangents at A and B meet at C. The median of the triangle
ABC from C has length 'm' units. Find the area of the triangle in terms of 'm'.

EXERCISE2

RATE MEASURE AND APPROXIMATIONS


Q.1
Q.2

Water is being poured on to a cylindrical vessel at the rate of 1 m3/min. If the vessel has a circular base of
radius 3 m, find the rate at which the level of water is rising in the vessel.
A man 1.5 m tall walks away from a lamp post 4.5 m high at the rate of 4 km/hr.
(i)
how fast is the farther end of the shadow moving on the pavement ?
(ii)
how fast is his shadow lengthening ?
25

Q.3
Q.4
Q.5

Q.6
Q.7

Q.8
Q.9

Q.10

Q.11
Q.12

(a)
(b)
Q.13
Q.14
(a)
(b)
Q.15

A particle moves along the curve 6 y = x3 + 2. Find the points on the curve at which the y coordinate is
changing 8 times as fast as the x coordinate.
An inverted cone has a depth of 10 cm & a base of radius 5 cm. Water is poured into it at the rate of
1.5 cm3/min. Find the rate at which level of water in the cone is rising, when the depth of water is 4 cm.
A water tank has the shape of a right circular cone with its vertex down. Its altitude is 10 cm and the radius
of the base is 15 cm. Water leaks out of the bottom at a constant rate of 1cu. cm/sec. Water is poured into
the tank at a constant rate of C cu. cm/sec. Compute C so that the water level will be rising at the rate of 4 cm/
sec at the instant when the water is 2 cm deep.
Sand is pouring from a pipe at the rate of 12 cc/sec. The falling sand forms a cone on the ground in such a
way that the height of the cone is always 1/6th of the radius of the base. How fast is the height of the sand
cone increasing when the height is 4 cm.
An open Can of oil is accidently dropped into a lake ; assume the oil spreads over the surface as a circular
disc of uniform thickness whose radius increases steadily at the rate of 10 cm/sec. At the moment when the
radius is 1 meter, the thickness of the oil slick is decreasing at the rate of 4 mm/sec, how fast is it decreasing
when the radius is 2 meters.
Water is dripping out from a conical funnel of semi vertical angle /4, at the uniform rate of 2 cm3/sec
through a tiny hole at the vertex at the bottom. When the slant height of the water is 4 cm, find the rate of
decrease of the slant height of the water.
An air force plane is ascending vertically at the rate of 100 km/h. If the radius of the earth is R Km, how fast
the area of the earth, visible from the plane increasing at 3min after it started ascending. Take visible area A
2
= 2R h Where h is the height of the plane in kms above the earth.
R+h
A variable ABC in the xy plane has its orthocentre at vertex 'B' , a fixed vertex 'A' at the origin and the
7x 2
third vertex 'C' restricted to lie on the parabola y = 1 +
. The point B starts at the point (0, 1) at time t
36
= 0 and moves upward along the y axis at a constant velocity of 2 cm/sec. How fast is the area of the triangle
7
increasing when t = sec.
2
A circular ink blot grows at the rate of 2 cm2 per second. Find the rate at which the radius is increasing after
22
6
2 seconds. Use = .
7
11
Water is flowing out at the rate of 6 m3/min from a reservoir shaped like a hemispherical bowl of radius R =
2
13 m. The volume of water in the hemispherical bowl is given by V = y (3R y) when the water is y
3
meter deep. Find
At what rate is the water level changing when the water is 8 m deep.
At what rate is the radius of the water surface changing when the water is 8 m deep.
If in a triangle ABC, the side 'c' and the angle 'C' remain constant, while the remaining elements are changed
da
db
= 0.
slightly, show that
+
cos A cos B
At time t > 0, the volume of a sphere is increasing at a rate proportional to the reciprocal of its radius. At t
= 0, the radius of the sphere is 1 unit and at t = 15 the radius is 2 units.
Find the radius of the sphere as a function of time t.
At what time t will the volume of the sphere be 27 times its volume at t = 0.
Use differentials to a approximate the values of ; (a) 25.2 and (b) 3 26 .

EXERCISE3

Q.1
Q.2

Find the acute angles between the curves y = x2 1 and y = x2 3 at their point of intersection.
Find the equation of the straight line which is tangent at one point and normal at another point of the curve,
x = 3t2 , y = 2t3.
[ REE 2000 (Mains) 5 out of 100 ]

Q.3

If the normal to the curve , y = f (x) at the point (3, 4) makes an angle
3

3
with the positive xaxis. Then f
4
4
(C)
(D) 1
3

(3) =
(A) 1
(B)
4
3
2
Q.4 The point(s) on the curve y + 3x = 12y where the tangent is vertical, is(are) [JEE 2002 (Scr.), 3]
11
4

, 2
, 2
(B) 3 , 1
(C) (0, 0)
(D)
(A)

3
3
Q.5
Tangent to the curve y = x2 + 6 at a point P (1, 7) touches the circle x2 + y2 + 16x + 12y + c = 0
at a point Q. Then the coordinates of Q are
[JEE 2005 (Scr.), 3]
(A) ( 6, 11)
(B) (9, 13)
(C) ( 10, 15)
(D) (6, 7)

EXERCISE4

PART - (A) Only one correct option


1.
Water is poured into an inverted conical vessel of which the radius of the base is 2 m and height 4 m, at the
rate of 77 litre/minute. The rate at which the water26level is rising at the instant when the depth is
70 cm is: (use = 22/7)
(A) 10 cm/min
(B) 20 cm/min
(C) 40 cm/min
(D) none

2.

The area of the triangle formed by the positive x-axis and the normal and the tangent to the circle
x 2 + y2 = 4 at (1, 3 ) is
(A) 3 3 sq. units

3.
4.

(B) 2 3 sq. units

(C) 4 3 sq. units

3 sq. units

The line which is parallel to x-axis and crosses the curve y = x at an angle of
is
4
(D)

(A) y = 1/2
(B) x = 1/2
(C) y = 1/4
(D) y = 1/2
If at any point on a curve the subtangent and subnormal are equal, then the tangent is equal to

(B) 2 ordinate
(C) 2 (ordinate )
(A) oridinate
(D) none of these
If curve y = 1 ax 2 and y = x 2 intersect orthogonally then the value of a is
(A) 1/2
(B) 1/3
(C) 2
(D) 3
2
(length of normal)
is equal to
6.
For a curve
(length of tan gent )2
(A) (subnormal) / (subtangent)
(B) (subtangent) / (subnormal)
(C) subnormal/(subtangent)2
(D) none of these
2 3
7.
If the tangent at each point of the curve y = x 2ax 2 + 2x + 5 makes an acute angle with the positive
3
direction of x-axis, then
(A) a 1
(B) 1 a 1
(C) a 1
(D) none of these
sinx 2
8.
Equation of normal drawn to the graph of the function defined as f(x) =
, x 0 and f(0) = 0 at the origin
x
is:
(A) x + y = 0
(B) x y = 0
(C) y = 0
(D) x = 0
x

9.
All points on the curve y2 = 4a x + a sin at which the tangents are parallel to the axis of x, lie on a
a

(A) circle
(B) parabola
(C) line
(D) none of these
10.
The point(s) of intersection of the tangents drawn to the curve x2y = 1 y at the points where it is intersected
by the curve xy = 1 y is/are given by:
(A) (0, 1)
(B) (0, 1)
(C) (1, 1)
(D) none of these
11.
The ordinate of y = (a/2) (ex/a + e-x/a) is the geometric mean of the length of the normal and the quantity:
(A) a/2
(B) a
(C) e
(D) none of these
12.
The curves x 3 + p xy2 = 2 and 3 x 2y y3 = 2 are orthogonal for:
(A) p = 3
(D) p = 3
(B) p = 3
(C) no value of p
13.
If the area of the triangle included between the axes and any tangent to the curve xn y = an is constant, then
n is equal to
(A) 1
(B) 2
(C) 3/2
(D) 1/2
14.
A curve with equation of the form y = ax 4 + bx 3 + cx + d has zero gradient at the point (0, 1) and also
touches the x axis at the point ( 1, 0) then the values of x for which the curve has a negative gradient are:
(B) x < 1
(C) x < 1
(D) 1 x 1
(A) x > 1
15.
If the tangent at P of the curve y2 = x 3 intersects the curve again at Q and the straight lines OP, OQ make
angles , with the x-axis, where 'O' is the origin, then tan /tan has the value equal to:
(A) 1
(B) 2
(C) 2
(D) 2
PART - (B) One or more than one correct options
Consider the curve f(x) = x 1/3 , then
16.
(A)
the equation of tangent at (0, 0) is x = 0
(B)
the equation of normal at (0, 0) is y = 0
(C) normal to the curve does not exist at (0, 0) (D) f(x) and its inverse meet at exactly 3 points.
5.

17.

18.
19.

20.
21.
22.

x y
The equation of normal to the curve + = 2 (n N) at the point with abscissa equal to 'a' can be:
a b
(A) ax + by = a2 b2
(B) ax + by = a2 + b2
(C) ax by = a2 b2
(D) bx ay = a2 b2
If the line, ax + by + c = 0 is a normal to the curve xy = 2, then:
(A) a < 0, b > 0
(B) a > 0, b < 0
(C) a > 0, b > 0
(D) a < 0, b < 0
In the curve x = t2 + 3t 8, y = 2t2 2t 5, at point (2, 1)
(B) slope of tangent = 6/7
(A) length of subtangent is 7/6.
(C) length of tangent = (85 ) / 6
(D) none of these
If y = f(x) be the equation of a parabola which is touched by the line y = x at the point where x = 1. Then
(A) f (1) = 1
(B) f (0) = f(1)
(C) 2f(0) = 1 f (0)
(D) f(0) + f (0) + f (0) = 1
If the tangent to the curve 2y3 = ax 2 + x 3 at the point (a, a) cuts off intercepts , on co-ordinate axes,
where 2 + 2 = 61, then the value of 'a' is equal to:
(A) 20
(B) 25
(C) 30
(D) 30
The curves ax 2 + by2 = 1 and Ax 2 + By2 = 1 intersect orthogonally, then
1 1 1 1
1 1 1 1
1 1 1 1
1 1 1 1
(B) =
(C) + =
(D) =
(A) + = +
a A b B
a A b B
a b B A
a b A B

EXERCISE5

1.
2.
3.

Find the parameters a, b, c if the curve y = a x 2 + b x + c is to pass through the point (1, 2) and is to be
tangent to the line y = x at the origin.
If the tangent at (1, 1) on y2 = x(2 x)2 meets the curve again at P, then find coordinates of P
If the relation between subnormal SN and subtangent ST at any point S on the curve by2 = (x + a)3 is
p
p(SN) = q (ST)2, then find value of
in terms of b and a.
q
27

4.
5.
6.
7.

In the curve x = a cos t + log tan 1 t , y = a sin t, show that the portion of the tangent between the point
2
of contact and the x axis is of constant length.
Find the angle of intersection of the following curves:
(i)
2 y2 = x 3 & y2 = 32 x
(ii)
y = 2sin2x and y = cos2x at x = /6
(iii)
y = 4 x2 & y = x 2
The length x of rectangle is decreasing at a rate of 3 cm/min and the width y is increasing at the rate of 2
cm/min. when x = 10 cm and y = 6 cm, find the rates of changes of (i) the perimeter, and (ii) the area of the
ractangle.
A particle mov es along the curve 6 y = x 3 + 2 . Find the points on the curv e at which the
y coordinate is changing 8 times as fast as the x coordinate.

x2

y2

8.

Prove that the straight line, x cos + y sin = p will be a tangent to the curve

9.

if p2 = a2 cos2 + b2 sin2 .
Show that the normal to any point of the curve x = a (cos t + t sin t), y = a (sin t t cos t) is at a constant
distance from the origin.

10.

Show that the condition, that the curves x 2/3 + y2/3 = c2/3 and

11.

if c = a + b.
Find the equation of axes of the conic 5x 2 + 4xy + 2y2 = 1.

12.
13.
14.
15.

x2

+ 2 = 1,
a2 b

y2

+ 2 = 1 may touch,
a2 b

Find the abscissa of the point on the curve, x y = (c + x)2 the normal at which cuts off numerically equal
intercepts from the axes of co-ordinates.
In the curve x a yb = Ka+b, prove that the portion of the tangent intercepted between the coordinate axes is
divided at its point of contact into segments which are in a constant ratio. (All the constants being positive).
The tangent to curve y = x x 3 at point P meets the curve again at Q. Prove that one point of trisection of
PQ lies on y-axis. Find locus of other point of trisection
A straight line is drawn through the origin and parallel to the tangent to a curve
x+

a2 y 2

a +

= n

a 2 y 2
at an arbitary point M. Show that the locus of the point P of intersection
y

of the straight line & the straight line parallel to the x-axis & passing through the point M is x 2 + y2 = a2 .
16.
Find the possible values of a such that the inequality 3 x 2 > |x a| has atleast one negative solution.
17.
Consider the family of circles x 2 + y2 = r2, 2 < r < 5. In the first quadrant, the common tangents to a circle
of this family and the ellipse 4x 2 + 25y2 = 100 meets the co-ordinate axes at A and B, then find the equation
of the locus of the mid-point of AB.
[IIT 1999]
18.
Let T1, T2 be two tangents drawn from (2, 0) onto the circle C : x 2 + y2 = 1. Determine the circles touching
C and having T1, T2 as their pair of tangents. Further; find the equations of all possible common tangents to
these circles, when taken two at a time.
[IIT 1999]
19.
An inverted cone of height H and radius R is pointed at bottom. It is filled with a volatile liquid completely.
If the rate of evaporation is directly proportional to the surface area of the liquid in contact with air (constant
of proportionality k > 0). Find the time in which whole liquid evaporates.
[IIT 2003, 4]
If |f(x 1) f(x 2)| < (x 1 x 2)2 , for all x 1, x 2 R. Find the equation of tengent to the curve y = f(x) at the point
20.
(1, 2).
[IIT 2005, 2]
____________________________________________________________________________________________

MONOTONOCITY
(Significance of the sign of the first order derivative)
DEFINITIONS :
1.
A function f (x) is called an Increasing Function at a point x = a if in a sufficiently small neighbourhood around

x = a we have

f (a + h) > f (a ) and
increasing;
f (a h) < f (a )

Similarly decreasing if
2.
3.
4.

f (a + h) < f (a ) and
decreasing.
f (a h) > f (a )

A differentiable function is called increasing in an interval (a, b) if it is increasing at every point within the
interval (but not necessarily at the end points). A function decreasing in an interval
(a, b) is similarly defined.
A function which in a given interval is increasing or decreasing is called Monotonic in that interval.
Tests for increasing and decreasing of a function at a point :
If the derivative f (x) is positive at a point x = a, then the function f (x) at this point is increasing. If it is
negative, then the function is decreasing. Even if f '(a) is not defined, f can still be increasing or decreasing.

Note : If f (a) = 0, then for x = a the function may be still increasing or it may be decreasing as shown. It has to be
identified by a seperate rule. e.g. f (x) = x3 is increasing at every point.
Note that, dy/dx = 3 x.

28

5.

Tests for Increasing & Decreasing of a function in an interval :


SUFFICIENCY TEST : If the derivative function f (x) in an interval (a , b) is every where positive, then the
function f (x) in this interval is Increasing ;
If f (x) is every where negative, then f (x) is Decreasing.
General Note :
If a function is invertible it has to be either increasing or decreasing.
(1)
(2)
If a function is continuous the intervals in which it rises and falls may be separated by points at which its
derivative fails to exist.
(3)
If f is increasing in [a, b] and is continuous then f (b) is the greatest and f (c) is the least value of f in [a, b].
Similarly if f is decreasing in [a, b] then f (a) is the greatest value and f (b) is the least value.
6.
ROLLE'S THEOREM :
(a)
Let f(x) be a function of x subject to the following conditions :
(i)
f(x) is a continuous function of x in the closed interval of a x b.
(ii)
f (x) exists for every point in the open interval a < x < b.
(iii)
f (a) = f (b).
Then there exists at least one point x = c such that a < c < b where f (c) = 0.
Note that if f is not continuous in closed [a, b] then it may lead to the adjacent
graph where all the 3 conditions of Rolles will be valid but the assertion will not
be true in (a, b).

(b)
(i)
(ii)
(iii)

LMVT THEOREM :
Let f(x) be a function of x subject to the following conditions :
f(x) is a continuous function of x in the closed interval of a x b.
f (x) exists for every point in the open interval a < x < b.
f(a) f(b).

Then there exists at least one point x = c such that a < c < b where f (c) =

f (b) f (a )
ba

Geometrically, the slope of the secant line joining the curve at x = a & x = b is equal to the slope of the
tangent line drawn to the curve at x = c. Note the following :
Rolles theorem is a special case of LMVT since
f (a) = f (b) f (c) =

f ( b ) f (a )
= 0.
ba

Note : Now [f (b) f (a)] is the change in the function f as x changes from a to b so that [f (b) f (a)] / (b a) is the
average rate of change of the function over the interval [a, b]. Also f '(c) is the actual rate of change of the
function for x = c. Thus, the theorem states that the average rate of change of a function over an interval is
also the actual rate of change of the function at some point of the interval. In particular, for instance, the
average velocity of a particle over an interval of time is equal to the velocity at some instant belonging to the
interval.
This interpretation of the theorem justifies the name "Mean Value" for the theorem.
(c)
APPLICATION OF ROLLES THEOREM FOR ISOLATING THE REAL ROOTS OF AN EQUATION f (x)=0
Suppose a & b are two real numbers such that ;
(i)
f(x) & its first derivative f (x) are continuous for a x b.
(ii)
f(a) & f(b) have opposite signs.
(iii)
f (x) is different from zero for all values of x between a & b.
Then there is one & only one real root of the equation f(x) = 0 between a & b.

EXERCISE6
Q.1

Find the intervals of monotonocity for the following functions & represent your solution set on the number line.

Q.2
Q.3

(a) f(x) = 2. e x 4 x
(b) f(x) = ex/x
(c) f(x) = x2 ex
(d) f (x) = 2x2 ln | x |
Also plot the graphs in each case.
Let f (x) = 1 x x3. Find all real values of x satisfying the inequality, 1 f (x) f 3(x) > f (1 5x)
Find the intervals of monotonocity of the function
(a)
f (x) = sin x cos x in x [0 , 2 ]
(b)
g (x) = 2 sinx + cos 2x in (0 x 2 ).
Show that, x3 3x2 9 x + 20 is positive for all values of x > 4.
2

Q.4

29

Q.5
Q.6
Q.7

Q.8

max {f ( t ) : 0 t x} , 0 x 1
Let f (x) = x3 x2 + x + 1 and g(x) =

3 x
,1 < x 2
Discuss the conti. & differentiability of g(x) in the interval (0,2).
Find the set of all values of the parameter 'a' for which the function,
f(x) = sin 2x 8(a + 1)sin x + (4a2 + 8a 14)x increases for all x R and has no critical points
for all x R.
Find the greatest & the least values of the following functions in the given interval if they exist.
x
1

(a) f (x) = sin1


ln x in , 3 (b) y = xx in (0, ) (c) y = x5 5x4 + 5x3 + 1 in [ 1, 2]
2
x +1
3

1
Find the values of 'a' for which the function f(x) = sin x a sin2x sin3x + 2ax increases throughout the
3
number line.
ex

Q.9

Prove that f (x) =

(9 cos

(2 ln t ) 25 cos(2 ln t ) + 17 dt is always an increasing function of x, xR

a 2 1
x3 + (a - 1) x2 + 2x + 1 is monotonic increasing for every x R then find the range of values
Q.10 If f(x) =

of a.
Q.11 Find the set of values of 'a' for which the function,

f(x) = 1

21 4 a a 2 3
x + 5x + 7 is increasing at every point of its domain.

a +1

Q.12 Find the intervals in which the function f (x) = 3 cos4 x + 10 cos3 x + 6 cos2 x 3, 0 x ; is monotonically
increasing or decreasing.
Q.13 Find the range of values of 'a' for which the function f (x) = x3 + (2a + 3)x2 + 3(2a + 1)x + 5 is monotonic
in R. Hence find the set of values of 'a' for which f (x) in invertible.
Q.14 Find the value of x > 1 for which the function
x2

t 1 is increasing and decreasing.


dt
32
x
Find all the values of the parameter 'a' for which the function ;
f(x) = 8ax a sin 6x 7x sin 5x increases & has no critical points for all x R.
If f (x) = 2ex aex + (2a + 1)x 3 monotonically increases for every x R then find the range of values of
a.
x2 9
2
Construct the graph of the function f (x) =
and comment upon the following
x+
x +3
x 1
(a) Range of the function,
(b) Intervals of monotonocity,
(c) Point(s) where f is continuous but not diffrentiable,
(d) Point(s) where f fails to be continuous and nature of discontinuity.
(e) Gradient of the curve where f crosses the axis of y.
Prove that, x2 1 > 2x ln x > 4(x 1) 2 ln x for x > 1.
3

Prove that tan2x + 6 ln secx + 2cos x + 4 > 6 sec x for x , 2 .


2

If ax + (b/x) c for all positive x where a > 0 & b > 0 then show that 27ab2 4c3.
If 0 < x < 1 prove that y = x ln x (x/2) + (1/2) is a function such that d2y/dx2 > 0. Deduce
that x ln x > (x2/2) (1/2).
Prove that 0 < x. sin x (1/2) sin x < (1/2) ( 1) for 0 < x < /2.
Show that x > (1 + x) [ln(1 + x)]2 x > 0.
Find the set of values of x for which the inequality ln (1 + x) > x/(1 + x) is valid.
If b > a, find the minimum value of (x a)3+ (x b)3, x R.

F (x) =

Q.15
Q.16
Q.17

Q.18
Q.19
Q.20
Q.21
Q.22
Q.23
Q.24
Q.25

t ln

EXERCISE7
Q.1
Q.2
Q.3

Verify Rolles throrem for f(x) = (x a)m (x b)n on [a, b] ; m, n being positive integer.
Let f : [a, b] R be continuous on [a, b] and differentiable on (a, b). If f (a) < f (b), then show that
f ' (c) > 0 for some c (a, b).
Let f (x) = 4x3 3x2 2x + 1, use Rolle's theorem to prove that there exist c, 0< c <1 such that f(c) = 0.
30

Q.4
Q.5
Q.6

Q.7
Q.8
Q.9
Q.10
Q.11
Q.12
Q.13

Q.14
Q.15
Q.16

Q.17
Q.18
Q.19
Q.20


Using LMVT prove that : (a) tan x > x in 0, ,
(b) sin x < x for x > 0
2
Prove that if f is differentiable on [a, b] and if f (a) = f (b) = 0 then for any real there is an x (a, b) such
that f (x) + f ' (x) = 0.
3
x=0
x 2 + 3x + a 0 < x < 1
For what value of a, m and b does the function f (x) =
mx + b
1 x 2
satisfy the hypothesis of the mean value theorem for the interval [0, 2].
Suppose that on the interval [2, 4] the function f is differentiable, f (2) = 1 and | f ' (x) | 5. Find the
bounding functions of f on [2, 4], using LMVT.
Let f, g be differentiable on R and suppose that f (0) = g (0) and f ' (x) g ' (x) for all x 0. Show that f (x)
g (x) for all x 0.
Let f be continuous on [a, b] and differentiable on (a, b). If f (a) = a and f (b) = b, show that there exist
distinct c1, c2 in (a, b) such that f ' (c1) + f '(c2) = 2.
Let f (x) and g (x) be differentiable functions such that f ' (x) g (x) f (x) g ' (x) for any real x. Show that
between any two real solutions of f (x) = 0, there is at least one real solution of g (x) = 0.
Let f defined on [0, 1] be a twice differentiable function such that, | f " (x) | 1 for all x [0, 1]
If f (0) = f (1), then show that, | f ' (x) | < 1 for all x [0, 1]
f (x) and g (x) are differentiable functions for 0 x 2 such that f (0) = 5, g (0) = 0, f (2) = 8, g (2) = 1. Show
that there exists a number c satisfying 0 < c < 2 and f ' (c) = 3 g' (c).
If f, , are continuous in [a, b] and derivable in ]a, b[ then show that there is a value of c lying between a
& b such that,
f (a ) f (b) f (c)
(a ) (b) (c) = 0
(a ) (b) (c)
Show that exactly two real values of x satisfy the equation x2 = x sinx + cos x.
Let a > 0 and f be continuous in [a, a]. Suppose that f ' (x) exists and f ' (x) 1 for all x (a, a). If f
(a) = a and f ( a) = a, show that f (0) = 0.
Let a, b, c be three real number such that a < b < c, f (x) is continuous in [a, c] and differentiable
in (a, c). Also f ' (x) is strictly increasing in (a, c). Prove that
(c b) f (a) + (b a) f (c) > (c a) f (b)
x 1
< ln x < x 1, x > 1
Use the mean value theorem to prove,
x
x +1 x .
Use mean value theorem to evaluate, Lim
x
Using L.M.V.T. or otherwise prove that difference of square root of two consecutive natural numbers
1
greater than N2 is less than
.
2N
x
Prove the inequality e > (1 + x) using LMVT for all x R0 and use it to determine which of the two
numbers e and e is greater.

EXERCISE8
Q.1

If f (x) =

sin x

& g (x) =

tan x

, where 0 < x 1, then in this interval :

(A) both f (x) & g (x) are increasing functions


(C) f (x) is an increasing function
Q.2

(B) both f (x) & g (x) are decreasing functions


(D) g (x) is an increasing function
[ JEE '97 (Scr), 2 ]

Let a + b = 4 , where a < 2 and let g (x) be a differentiable function . If


a

g (x) dx +

g (x) dx increases as (b a) increases.

dg
> 0 for all x, prove that
dx

[JEE 97, 5]

Q.3(a) Let h(x) = f(x) (f(x))2 + (f(x))3 for every real number x. Then :
(A) h is increasing whenever f is increasing
(B) h is increasing whenever f is decreasing
(C) h is decreasing whenever f is decreasing (D) nothing can be said in general.
x2 1

, for every real number x, then the minimum value of f :


(b) f(x) = 2
x +1
(A) does not exist because f is unbounded
(B) is not attained even though f is bounded
(C) is equal to 1
(D) is equal to 1.
[ JEE '98, 2 + 2 ]
Q.4(a) For all x (0, 1) :
(A) ex < 1 + x
(B) loge(1 + x) < x
(C) sin x > x
(D) loge x > x
(b) Consider the following statements S and R :
S : Both sin x & cos x are decreasing functions in the interval (/2, ).
R : If a differentiable function decreases in an interval (a, b), then its derivative also decreases in (a, b).
31

Which of the following is true ?


(A) both S and R are wrong
(B) both S and R are correct, but R is not the correct explanation for S
(C) S is correct and R is the correct explanation for S (D) S is correct and R is wrong.
(c) Let f (x) = ex (x 1) (x 2) d x then f decreases in the interval :
(A) ( , 2)
(B) ( 2, 1)
(C) (1, 2)
(D) (2, + )
[JEE 2000 (Scr.) 1+1+1 out of 35]
Q.5(a) If f (x) = xex(1 x), then f(x) is
1
(B) decreasing on R
(A) increasing on ,1
2
1
(C) increasing on R
(D) decreasing on , 1
2

1
(b) Let 1 < p < 1. Show that the equation 4x3 3x p = 0 has a unique root in the interval , 1 and identify
2
it.
[ JEE 2001, 1 + 5 ]
The length of a longest interval in which the function 3sinx 4sin3x is increasing, is

3
(A)
(B)
(C)
(D)
3
2
2
[JEE 2002 (Screening), 3]

Q.7(a) Using the relation 2(1 cosx) < x2 , x 0 or otherwise, prove that sin (tanx) > x , x 0, .
4
(b) Let f : [0, 4] R be a differentiable function.
(i) Show that there exist a, b [0, 4], (f (4))2 (f (0))2 = 8 f (a) f (b)
(ii) Show that there exist , with 0 < < < 2 such that
Q.6

f(t) dt = 2 ( f (2 ) + f (2) )

[JEE 2003 (Mains), 4 + 4 out of 60]

x lnx , x > 0

Q.8(a) Let f (x) =


. Rolles theorem is applicable to f for x [0, 1], if =
x=0
0,
1
(A) 2
(B) 1
(C) 0
(D)
2
f (x 2 ) f (x)
is equal to
(b) If f is a strictly increasing function, then Lim
x 0 f ( x ) f ( 0)
(A) 0
(B) 1
(C) 1
(D) 2
[JEE 2004 (Scr)]
101
100
Q.9 If p (x) = 51x 2323x 45x + 1035, using Rolle's theorem, prove that at least one root of p(x) lies
[JEE 2004, 2 out of 60]
between (451/100, 46).
Q.10(a) If f (x) is a twice differentiable function and given that f(1) = 1, f(2) = 4, f(3) = 9, then
(A) f '' (x) = 2, for x (1, 3)
(B) f '' (x) = f ' (x) = 2, for some x (2, 3)
(C) f '' (x) = 3, for x (2, 3)
(D) f '' (x) = 2, for some x (1, 3)
[JEE 2005 (Scr), 3]
(b) f (x) is differentiable function and g (x) is a double differentiable function such that | f (x) |
1 and f'(x)=g (x). If f2(0) +g2(0) = 9. Prove that there exists some c (3, 3) such that g (c) g"(c)<0.
[JEE 2005 (Mains), 6]

EXERCISE9

Only one correct options


| x 1|
1.
The function
is monotonically decreasing for
x2
(A) (2, )
(B) (0, 1)
(C) (0, 1) (2, )
(D) ( , )
p+4

1 x 5 3x + ln 5 decreases for all real x is


2.
The values of p for which the function f(x) =

1 p

(A) ( , )

3 21
(1, )
(B) 4,
2

5 27
(2, )
(C) 3,
2

(D) [1,32)

3.
4.

5.

6.
7.
8.

The set of all x for which n (1 + x) x is equal to


(A) x > 0
(B) x > 1
(C) 1 < x < 0
(D) null set
Let f(x) = x 3 + ax 2 + bx + 5 sin2 x be an increasing function in the set of real numbers R. Then a & b satisfy
the condition:
(A) a2 3b 15 > 0
(B) a2 3b + 15 < 0
(C) a2 3b 15 < 0
(D) a > 0 & b > 0

{a x sgnx}

a x

sgnx

If f(x) = a
; g(x) = a
for a > 1, a 1and x R, where { } & [ ] denote the fractional part and
integral part functions respectively, then which of the following statements holds good for the function h(x),
where (n a) h(x) = (n f(x) + n g(x)).
(A) h is even and increasing
(B) h is odd and decreasing
(C) h is even and decreasing
(D) h is odd and increasing
If f : [1, 10] [1, 10] is an non-decreasing function and g : [1, 10] [1, 10] is a non-increasing function.
Let h(x) = f(g(x)) with h(1) = 1, then h(2)
(A) lies in (1, 2)
(B) is more than 2
(C) is equal to 1
(D) is not defined
Let f be a differentiable function of x, x R. If f(1) = 4 and f(x) 2 x [1, 6], then minimum value of
f(6) is
(A) 6
(B) 2
(C) 4
(D) none of these
For what values of a does the curve f(x) = x(a2 2a 2) + cosx is always strictly monotonic x R.

(A) a R
(C) 1

(B) |a| < 2


2 <a<1+
2

(D) |a| < 2 1

2
2

x
x
; g(x) =
where 0 < x < 1, then
2 2 cos x
6 x 6 sin x
(A)
both 'f' and 'g' are increasing functions
(B)
'f' is decreasing & 'g' is increasing function
(C)
'f' is increasing & 'g' is decreasing function
(D)
both 'f' & 'g' are decreasing function
One or more than one correct options :
10.
The set of values of a for which the function f(x) = x 2 + ax + 1 is an increasing function on [1, 2] is 1 and
decreasing in [1, 2] is 2, then :
(A) 1 : a (2, )
(B) 2 : a ( , 4)
(C) 2 : a ( , 4]
(D) 1 ; a [ 2, )
11.
If f is an even function then
(A) f 2 increases on (a, b)
(B) f cannot be monotonic
(C) f 2 need not increases on (a, b)
(D) f has inverse
12.
Let g(x) = 2f(x/2) + f(1 x) and f(x) < 0 in 0 x 1 then g(x) :
2
2
(B) decreases , 1
(A) decreases in 0,
3
3

9.

13.
14.
15.

If f(x) =

2
2
(C) increases in 0,
(D) increases in , 1
3

3
On which of the following intervals, the function x 100 + sinx 1 is strictly increasing
(A) ( 1, 1)
(B) [0, 1]
(C) [/2, ]
(D) [0, /2]
2x 1
(x 2) :
The function y =
x2
(A) is its own inverse
(B) decreases for all values of x
(C) has a graph entirely above x-axis
(D) is bound for all x.
Let f and g be two functions defined on an interval such that f(x) 0 and g(x) 0 for all x and f is strictly
decreasing on while g is strictly increasing on then
(A)
the product function fg is strictly increasing on
(B)
the product function fg is strictly decreasing on I
(C)
fog(x) is monotonically increasing on (D)
fog (x) is monotonically decreasing on

EXERCISE10

max ( x, x ) x 0

Let f(x) =
. Draw the graph of f(x) and hence comment on the nature of monotonic
min ( x, x 2 2) x < 0

behaviour at x = 1, 0, 1.
x2
x0

Let f(x) =
. Find possible values of a such that f(x) is monotonically increasing at x = 0.
x | a | x < 0

Find the relation between the constants a, b, c & d so that the function,
f(x) = [a sinx + b cosx]@[c sin x + d cos x] is always increasing.
Find the intervals of monotonocity for the following functions :
2

1.

2.
3.
4.

f(x) =

5.

1+ x + x2

If p, q, r be real, locate the intervals in which, f(x) =


(a)

6.

1 x + x2

increase

(b)

x + p2

pq

pq

x+q

pr
decrease

qr

pr
2

qr

x + r2

Find the values of a for which the function f(x) = (a + 2) x 3 3ax 2 + 9ax 1 decreases for all real values
33
of x.

7.
8.
9.

10.
11.
12.
13.

14.

15.

16.
17.
18.
19.

Prove that for 0 < x < 1, the inequality, x < n (1 x) < x (1 x)1.

3x

For x 0, identify which is greater (2sinx + tanx) Or (3x) hence find xlim

0
2

2 sin x + tan x
sin x

Prove that f(x) =


is monotonically decreasing function for x 0, . Hence prove that
x
2

sin(sin x )
sin x

for x 0, , x cosec x <


(ii)
<
(i)
3
sin x
x
6


A (0, 1), B , 1 are two points on the graph given by y = 2 sinx + cos2x. Prove that there exists a point
2
P on the curve between A & B such that tangent at P is parallel to AB. Find the co-ordinates of P.
Using Rolles theorem prove that the equation 3x 2 + px 1 = 0 has at least one real root in the interval x
( 1, 1).
Show that xex = 2 has one & only one root between 0 & 1.

Find the interval in which the following function is increasing or decreasing :


4 sin x 2x x cos x
f(x) =
in [0, ].
2 + cos x

x sin
for x > 0
Show that the derivative of the function f(x) =
vanishes on an infinite set of points of the
x
0
for x = 0
interval (0, 1).
f (a )
f (b )
Assume that f is continuous on [a, b] a > 0 and differentiable in (a, b) such that
=
. Prove that
a
b
f(x0 )
there exists x 0 (a, b) such that f(x 0) =
.
x0
Find the greatest & least value of f(x) = sin1

1
,
n x in
3
x +1

3 .

Show that, 1 + x n x + x + 1 1 + x 2 for all x 0.

x2

tan x 2
Prove the inequality,
> x for 0 < x 1 < x 2 < .
2
tan x1
1

A function f is differentiable in the interval 0 x 5 such that f(0) = 4 & f(5) = 1. If g(x) =
that there exists some c (0, 5) such that g(x) =

f ( x)
, then prove
x +1

5
.
6

2
1+ e 2 x (1 + x) + 2 + 2x + x .

21.
Let f (sinx) < 0 and f (sin x) > 0, x 0, and g(x) = f(sin x) + f(cos x), then find the interval in which
2
g(x) is increasing and decreasing.
22.
If ax 2 + (b/x) c for all positive x where a > 0 and b > 0 then show that 27ab2 4c3.
23.
Prove that for 0 p 1 & for any a > 0, b > 0 the inequality (a + b)p ap + bp .
24.
Find the greatest and the least values of the function f(x) defined as below.
f(x) = minimum of {3t 4 8t3 6t2 + 24t ; 1 t x}, 1 x < 2.
1

maximum of 3t + sin 2 t + 2 ; 2 t x , 2 x 4.
4

25.
If a > b > 0, with the aid of Largrange's formula prove the validity of the inequailities
nbn 1 (a b) < an bn < nan 1 (a b), if n > 1. Also prove that the inequalities of the opposite sense if 0 <
n < 1.
____________________________________________________________________________________________

20.

Prove that for all x R ex +

MAXIMA - MINIMA
FUNCTIONS OF A SINGLE VARIABLE
HOW MAXIMA & MINIMA ARE CLASSIFIED
1.
A function f(x) is said to have a maximum at x
= a if f(a) is greater than every other value
assumed by f(x) in the immediate
neighbourhood of x = a. Symbolically
f (a ) > f (a + h)
x = a gives maxima for a
f (a ) > f (a h)

sufficiently small positive h.


Similarly, a function f(x) is said to have a minimum

34

value at x = b if f(b) is least than every other value


assumed by f(x) in the immediate neighbourhood
at x = b. Symbolically if

f ( b ) < f ( b + h)
x = b gives minima for a sufficiently small positive h.
f ( b) < f ( b h)

Note that :
the maximum & minimum values of a function are also known as local/relative maxima or
(i)
local/relative minima as these are the greatest & least values of the function relative to some neighbourhood
of the point in question.
(ii)
the term 'extremum' or (extremal) or 'turning value' is used both for maximum or a minimum value.
(iii)
a maximum (minimum) value of a function may not be the greatest (least) value in a finite interval.
(iv)
a function can have several maximum & minimum values & a minimum value may even be greater than a
maximum value.
(v)
maximum & minimum values of a continuous function occur alternately & between two consecutive maximum
values there is a minimum value & vice versa.
2.
A NECESSARY CONDITION FOR MAXIMUM & MINIMUM :
If f(x) is a maximum or minimum at x = c & if f (c) exists then f (c) = 0.
Note :
(i)
The set of values of x for which f (x) = 0 are often called as stationary points or critical points. The rate of
change of function is zero at a stationary point.
(ii)
In case f (c) does not exist f(c) may be a maximum or a minimum & in this case left hand and right hand
derivatives are of opposite signs.
(iii)
The greatest (global maxima) and the least (global minima) values of a function f in an interval [a, b] are f(a)
or f(b) or are given by the values of x for which f (x) = 0.
(iv)
3.

dy

Critical points are those where


= 0, if it exists , or it fails to exist either by virtue of a vertical tangent or
dx
by virtue of a geometrical sharp corner but not because of discontinuity of function.
SUFFICIENT CONDITION FOR EXTREME VALUES :
f (c- h) > 0
x = c is a point of local maxima, where f (c) = 0.
f (c+ h) < 0
f (c- h) < 0
x = c is a point of local minima, where f (c) = 0.
Similarly
f (c+ h) > 0

h is a sufficiently
small positive
quantity

Note : If f (x) does not change sign i.e. has the same sign in a certain complete neighbourhood of c, then
f(x) is either strictly increasing or decreasing throughout this neighbourhood implying that f(c) is not an
extreme value of f.
4.

(a)
(b)
5.

USE OF SECOND ORDER DERIVATIVE IN ASCERTAINING THE MAXIMA OR MINIMA:

f(c) is a minimum value of the function f, if f (c) = 0 & f (c) > 0.


f(c) is a maximum value of the function f, f (c) = 0 & f (c) < 0.
Note : if f (c) = 0 then the test fails. Revert back to the first order derivative check for ascertaning the
maxima or minima.
SUMMARYWORKING RULE :
FIRST :
When possible , draw a figure to illustrate the problem & label those parts that are important
in the problem. Constants & variables should be clearly distinguished.
SECOND : Write an equation for the quantity that is to be maximised or minimised. If this quantity is
denoted by y, it must be expressed in terms of a single independent variable x. his may require some
algebraic manipulations.
THIRD :
If y = f (x) is a quantity to be maximum or minimum, find those values of x for which
dy/dx = f (x) = 0.
FOURTH : Test each values of x for which f (x) = 0 to determine whether it provides a maximum or
minimum or neither. The usual tests are :
(a)
If dy/dx is positive when dy/dx = 0 y is minimum.
If dy/dx is negative when dy/dx = 0 y is maximum.
If dy/dx = 0 when dy/dx = 0, the test fails.
(b)

If

positive for x < x 0


dy

is zero
for x = x 0 a maximum occurs at x = x0.
dx
negative for x > x 0

But if dy/dx changes sign from negative to zero to positive as x advances through
xo there is a minimum. If dy/dx does not change sign, neither a maximum nor a minimum. Such points are
called INFLECTION POINTS.
FIFTH : If the function y = f (x) is defined for only a limited range of values a x b then examine x = a &
x = b for possible extreme values.
SIXTH : If the derivative fails to exist at some point, examine this point as possible maximum or minimum.
Important Note :

Given a fixed point A(x1, y1) and a moving point P(x, f (x)) on the curve y = f(x). Then AP will be
maximum or minimum if it is normal to the curve at P.
35

If the sum of two positive numbers x and y is constant than their product is maximum if they are
equal, i.e. x + y = c , x > 0 , y > 0 , then
1
xy = [ (x + y)2 (x y)2 ]
4
If the product of two positive numbers is constant then their sum is least if they are equal.
i.e. (x + y)2 = (x y)2 + 4xy

6.

USEFUL FORMULAE OF MENSURATION TO REMEMBER :



Volume of a cuboid = lbh.

Surface area of a cuboid = 2 (lb + bh + hl).

Volume of a prism = area of the base x height.

Lateral surface of a prism = perimeter of the base x height.

Total surface of a prism = lateral surface + 2 area of the base
(Note that lateral surfaces of a prism are all rectangles).
1
area of the base x height.
3
1
Curved surface of a pyramid = (perimeter of the base) x slant height.
2

Volume of a pyramid =

(Note that slant surfaces of a pyramid are triangles).

7.




Volume of a cone = r2h. 


Curved surface of a cylinder = 2 rh.
3
Total surface of a cylinder = 2 rh + 2 r2.

Volume of a sphere =

4
r3. 
Surface area of a sphere = 4 r2.
3
1
Area of a circular sector = r2 , when is in radians.
2


SIGNIFICANCE OF THE SIGN OF 2ND ORDER DERIVATIVE AND POINTS OF INFLECTION :
The sign of the 2nd order derivative determines the concavity of the
curve. Such points such as C & E on the graph where the concavity
of the curve changes are called the points of inflection. From the
graph we find that if:
(i)

d 2y
> 0 concave upwards
dx 2

(ii)

d 2y
< 0 concave downwards.
dx 2

d 2y
At the point of inflection we find that 2 = 0 &
dx
2
dy
changes sign.
dx 2

Inflection points can also occur if


as,
x 3/ 5

d 2y
fails to exist. For example, consider the graph of the function defined
dx 2

for x ( , 1)

f (x) = [ 2 x 2 for x (1 , )
Note that the graph exhibits two critical points one is a point of local maximum
& the other a point of inflection.

EXERCISE11

Q.1

A cubic f(x) vanishes at x = 2 &


1

If

f ( x ) dx =

has relative minimum/maximum at x = 1 and x =

14
, find the cubic f (x).
3
x

[2 (t 1) (t 2)3 + 3 (t 1)2 (t 2)2] dt

Q.2

Investigate for maxima & minima for the function, f (x) =

Q.3

Find the maximum & minimum value for the function ;


(a)
y = x + sin 2x , 0 x 2
(b)
y = 2 cos 2x cos 4x , 0 x
Suppose f(x) is real valued polynomial function of degree
36 6 satisfying the following conditions ;

Q.4

1
.
3

(a)
(c)

f has minimum value at x = 0 and 2


1
for all x, Limit
ln
x0
x

f (x)
x

0
1

Q.5
Q.6
Q.7
Q.8

(b)

1
x

1 = 2.

f has maximum value at x = 1


Determine f (x).

1
x

Find the maximum perimeter of a triangle on a given base a and having the given vertical angle .
The length of three sides of a trapezium are equal, each being 10 cms. Find the maximum area of such a
trapezium.
The plan view of a swimming pool consists of a semicircle of radius r attached to a rectangle of length '2r'
and width 's'. If the surface area A of the pool is fixed, for what value of 'r' and 's' the perimeter 'P' of the pool
is minimum.
For a given curved surface of a right circular cone when the volume is maximum, prove that the semi vertical
angle is sin1 1 .
3

Q.9

Of all the lines tangent to the graph of the curve y =

6
x +3
2

, find the equations of the tangent lines of

minimum and maximum slope.


Q.10 A statue 4 metres high sits on a column 5.6 metres high. How far from the column must a man, whose eye
level is 1.6 metres from the ground, stand in order to have the most favourable view of statue.
Q.11 By the post office regulations, the combined length & girth of a parcel must not exceed 3 metre. Find the
volume of the biggest cylindrical (right circular) packet that can be sent by the parcel post.
Q.12 A running track of 440 ft. is to be laid out enclosing a football field, the shape of which is a rectangle with
semi circle at each end. If the area of the rectangular portion is to be maximum, find the length of its sides.
22
Use : .
7
Q.13 A window of fixed perimeter (including the base of the arch) is in the form of a rectangle surmounted by a
semicircle. The semicircular portion is fitted with coloured glass while the rectangular part is fitted with clean
glass. The clear glass transmits three times as much light per square meter as the coloured glass does. What
is the ratio of the sides of the rectangle so that the window transmits the maximum light?
Q.14 A closed rectangular box with a square base is to be made to contain 1000 cubic feet. The cost of the
material per square foot for the bottom is 15 paise, for the top 25 paise and for the sides 20 paise. The
labour charges for making the box are Rs. 3/-. Find the dimensions of the box when the cost is minimum.
Q.15 Find the area of the largest rectangle with lower base on the x-axis & upper vertices on the
curve y = 12 x2.
Q.16 A trapezium ABCD is inscribed into a semicircle of radius l so that the base AD of the trapezium is a
diameter and the vertices B & C lie on the circumference. Find the base angle of the trapezium ABCD
which has the greatest perimeter.
Q.17 If y =

ax + b
has a turning value at (2, 1) find a & b and show that the turning value is a maximum.
(x 1) (x 4)

Q.18 Prove that among all triangles with a given perimeter, the equilateral triangle has the maximum area.
Q.19 A sheet of poster has its area 18 m. The margin at the top & bottom are 75 cms and at the sides
50 cms. What are the dimensions of the poster if the area of the printed space is maximum?
Q.20 A perpendicular is drawn from the centre to a tangent to an ellipse

x2 y2
+
= 1. Find the greatest value of
a 2 b2

the intercept between the point of contact and the foot of the perpendicular.
x

Q.21 Consider the function, F (x) =

(t

t ) dt , x R.

Q.22

Q.23
Q.24
Q.25

(a)
Find the x and y intercept of F if they exist.
(b)
Derivatives F ' (x) and F '' (x).
(c)
The intervals on which F is an increasing and the invervals on which F is decreasing.
(d)
Relative maximum and minimum points.
(e)
Any inflection point.
A beam of rectangular cross section must be sawn from a round log of diameter d. What should the width x
and height y of the cross section be for the beam to offer the greatest resistance (a) to compression; (b) to
bending. Assume that the compressive strength of a beam is proportional to the area of the cross section and
the bending strength is proportional to the product of the width of section by the square of its height.
What are the dimensions of the rectangular plot of the greatest area which can be laid out within a triangle of
base 36 ft. & altitude 12 ft ? Assume that one side of the rectangle lies on the base of the triangle.
The flower bed is to be in the shape of a circular sector of radius r & central angle . If the area is fixed &
perimeter is minimum, find r and .
The circle x2 + y2 = 1 cuts the x-axis at P & Q. Another circle with centre at Q and varable radius
intersects the first circle at R above the x-axis & the line segment PQ at S. Find the maximum area of the
triangle QSR.
37

EXERCISE12
Q.1
(a)
(c)
Q.2
Q.3

Q.4
Q.5
Q.6
Q.7

3
1 + 4e t
dM 1
Find Lim M( t ) and Lim M( t )
(b)
Show that
= M (3 M )
dt
3
t
t
Find the maximum rate of growth of M and also the vlaue of t at which occurs.
Find the cosine of the angle at the vertex of an isosceles triangle having the greatest area for the given
constant length l of the median drawn to its lateral side.
From a fixed point A on the circumference of a circle of radius 'a', let the perpendicular AY fall on the tangent
at a po int P on the circle, prove that t he greatest area which the APY can have
a2
sq. units.
is 3 3
8
Given two points A ( 2 , 0) & B (0 , 4) and a line y = x. Find the co-ordinates of a point M on this line so
that the perimeter of the AMB is least.
A given quantity of metal is to be casted into a half cylinder i.e. with a rectangular base and semicircular
ends. Show that in order that total surface area may be minimum , the ratio of the height of the cylinder to the
diameter of the semi circular ends is /( + 2).
Depending on the values of p R, find the value of 'a' for which the equation x3 + 2 px2 + p = a has three
distinct real roots.
Show that for each a > 0 the function eax. xa has a maximum value say F (a), and that F (x) has a minimum
value, ee/2.
The mass of a cell culture at time t is given by, M (t) =

1a

Q.8

For a > 0, find the minimum value of the integral


1

Q.9

Find the maximum value of the integral

(a

+ 4 x a 5 x 2 )e ax dx .

| x a |e

dx where | a | 1.

x ln x when x > 0

Q.10 Consider the function f (x) =

0
for x = 0
(a)
Find whether f is continuous at x = 0 or not. (b)
Find the minima and maxima if they exist.
(c)
Does f ' (0) ? Find Lim f ' ( x ) .
x 0

(d)
Q.11

Q.12
Q.13
Q.14
Q.15

Q.16

Q.17
Q.18
Q.19
Q.20
Q.21
Q.22

Find the inflection points of the graph of y = f (x)..


Consider the function y = f (x) = ln (1 + sin x) with 2 x 2. Find
(a)
the zeroes of f (x)
(b)
inflection points if any on the graph
(c)
local maxima and minima of f (x)
(d)
asymptotes of the graph 2
(e)
sketch the graph of f (x) and compute the value of the definite integral f ( x ) dx .
A right circular cone is to be circumscribed about a sphere of a given radius. Findthe
2 ratio of the altitude of
the cone to the radius of the sphere, if the cone is of least possible volume.
Find the point on the curve 4 x + ay = 4 a, 4 < a < 8 that is farthest from the point (0 , 2).
3
5
Find the set of value of m for the cubic x3 x2 + = log1 4 ( m) has 3 distinct solutions.
2
2
Let A (p2, p) , B (q2, q) , C (r2, r) be the vertices of the triangle ABC. A parallelogram AFDE is drawn
with vertices D, E & F on the line segments BC, CA & AB respectively. Using calculus, show that maximum
1
area of such a parallelogram is : (p + q) (q + r) (p r).
4
A cylinder is obtained by revolving a rectangle about the x axis , the base of the rectangle lying on the x
axis and the entire rectangle lying in the region between the curve
x
y = 2 & the x axis. Find the maximum possible volume of the cylinder..
x +1
For what values of a does the function f (x) = x3 + 3 (a 7) x2 + 3 (a2 9) x 1 have a positive point of
maximum.
Among all regular triangular prism with volume V, find the prism with the least sum of lengths of all edges.
How long is the side of the base of that prism?
A segment of a line with its extremities on AB and AC bisects a triangle ABC with sides a, b, c into two equal
areas. Find the length of the shortest segment.
What is the radius of the smallest circular disk large enough to cover every acute isosceles triangle of a given
perimeter L?
Find the magnitude of the vertex angle of an isosceles triangle of the given area A such that the radius
r of the circle inscribed into the triangle is the maximum.
Prove that the least perimeter of an isosceles triangle in which a circle of radius r can be inscribed
38
is 6 r 3 .

Q.23 The function f (x) defined for all real numbers x has the following properties
(i) f (0) = 0, f (2) = 2 and f ' (x) = k(2x x2)e x for some constant k > 0. Find
(a) the intervals on which f is increasing and decreasing and any local maximum or minimum values.
(b) the intervals on which the graph f is concave down and concave up.
(c) the function f (x) and plot its graph.
Q.24 Find the minimum value of | sin x + cos x + tan x + cot x + sec x + cosec x | for all real x.
2x

Q.25 Use calculus to prove the inequality, sin x


in 0 x .

x2
in 0 x
You may use the inequality to prove that, cos x 1
2

EXERCISE13

Q.1

A conical vessel is to be prepared out of a circular sheet of gold of unit radius. How much sectorial area is
to be removed from the sheet so that the vessel has maximum volume.
[ REE '97, 6 ]

Q.2(a) The number of values of x where the function f(x) = cos x + cos
(B) 1
(C) 2
(A) 0
(b) Suppose f(x) is a function satisfying the following conditions :
(i)

f has a minimum value at x =

5
and
2

for all x f (x) =

Q.4

(ii)

(D) infinite

2ax
2ax 1
2ax + b + 1
b
b +1
1
2(ax + b) 2ax + 2b + 1 2ax + b
Where a, b are some constants. Determine the constants a, b & the function f(x).
Find the points on the curve ax2 + 2bxy + ay2 = c ; c > b > a > 0, whose distance from the origin is minimum.
[ REE '98, 6]
(iii)

Q.3

f(0) = 2, f(1) = 1

( 2 x) attains its maximum is :

The function f(x) =

t (et 1) (t 1) (t 2)3 (t 3)5 dt has a local minimum at x =

Q.5
Q.6
Q.7
Q.8

(A) 0
(B) 1
(C) 2
(D) 3
Find the co-ordinates of all the points P on the ellipse (x2/a2) + (y2/b2) = 1 for which the area of the triangle
PON is maximum, where O denotes the origin and N the foot of the perpendicular from O to the tangent at
P.
[JEE '99, 10 out of 200]
2
2
Find the normals to the ellipse (x /9) + (y /4) = 1 which are farthest from its centre.
[REE '99, 6]
Find the point on the straight line, y = 2 x + 11 which is nearest to the circle,
16 (x2 + y2) + 32 x 8 y 50 = 0.
[REE 2000 Mains, 3 out of 100]
| x | for 0 < | x | 2
Let f (x) =
. Then at x = 0, ' f ' has :

[1

for x = 0

(A) a local maximum


(B) no local maximum
(C) a local minimum
(D) no extremum.
Q.9 Find the area of the right angled triangle of least area that can be drawn so as to circumscribe a rectangle of
sides 'a' and 'b', the right angles of the triangle coinciding with one of the angles of the rectangle.
Q.10(a) Let f(x) = (1 + b2)x2 + 2bx + 1 and let m(b) be the minimum value of f(x). As b varies, the range
of m (b) is
1
1
(A) [0, 1]
(B) 0,
(C) , 1
(D) (0, 1]
2
2
(b) The maximum value of (cos 1) (cos 2).......... (cos n), under the restrictions

O < 1, 2,..............., n < and cot 1 cot 2.......... cot n = 1 is


2
1
1
1
(A) n/ 2
(B) n
(C)
(D) 1
2n
2
2
Q.11(a) If a1 , a2 ,....... , an are positive real numbers whose product is a fixed number e, the minimum value of a1 +
a2 + a3 +....... + an1 + 2an is
(A) n(2e)1/n
(B) (n+1)e1/n
(C) 2ne1/n
(D) (n+1)(2e)1/n
(b) A straight line L with negative slope passes through the point (8,2) and cuts the positive coordinates axes at
points P and Q. Find the absolute minimum value of OP + OQ, as L varies, where O is the origin.
Q.12(a) Find a point on the curve x2 + 2y2 = 6 whose distance from the line x + y = 7, is minimum.
(b) For a circle x2 + y2 = r2, find the value of r for which the area enclosed by the tangents drawn from the
point P(6, 8) to the circle and the chord of contact is maximum.
[JEE-03, Mains-2 out of 60]
Q.13(a) Let f (x) = x3 + bx2 + cx + d, 0 < b2 < c. Then f
(A) is bounded
(B) has a local maxima
(C) has a local minima
(D) is strictly increasing
[JEE 2004 (Scr.)]
39


3x ( x + 1)
x 0, . (Justify the inequality, if any used).
2

Q.14 If P(x) be a polynomial of degree 3 satisfying P(1) = 10, P(1) = 6 and P(x) has maximum
at x = 1 and P'(x) has minima at x = 1. Find the distance between the local maximum and local
minimum of the curve.
[JEE 2005 (Mains), 4
out of 60]
Q.15(a) If f (x) is cubic polynomial which has local maximum at x = 1. If f (2) = 18, f (1) = 1 and
f '(x) has local maxima at x = 0, then
(A) the distance between (1, 2) and (a, f (a)), where x = a is the point of local minima is 2 5 .
(B) f (x) is increasing for x [1, 2 5 ]
(C) f (x) has local minima at x = 1
(D) the value of f(0) = 5
e x
0 x 1
x

x 1
1 < x 2 and g (x) = f (t ) dt , x [1, 3] then g(x) has
(b) f (x) = 2 e
x e
0
2<x3

(A) local maxima at x = 1 + ln 2 and local minima at x = e


(B) local maxima at x = 1 and local minima at x = 2
(C) no local maxima
(D) no local minima
(c) If f (x) is twice differentiable function such that f (a) = 0, f (b) = 2, f (c) = 1, f (d) = 2, f (e) = 0,

(b) Prove that sin x + 2 x

2
where a < b < c < d < e, then find the minimum number of zeros of g ( x ) = (f ' ( x ) ) + f ( x ).f " ( x ) in the
interval [a, e].
[JEE 2006, 6]

EXERCISE14

Only one correct option


1.
The greatest value of f(x) = (x + 1) 1/3 (x 1)1/3 on [0, 1] is:
(A) 1
(B) 2
(C) 3
(D) 21/3
2.
The function 'f' is defined by f(x) = x p (1 x)q for all x R, where p,q are positive integers, has a maximum
value, for x equal to:
pq
p
(B) 1
(C) 0
(D) p+q
(A) p+q
3.
The coordinates of the point on the curve x 2 = 4y, which is at least distance from the line y = x 4 is
(A) (2, 1)
(B) ( 2, 1)
(C) ( 2, 1)
(D) none
4.
Tangents are drawn to x 2 + y2 = 16 from the point P(0, h). These tangents meet the x-axis at A and B. If the
area of triangle PAB is minimum, then
(B) h = 6 2
(C) h = 8 2
(D) h= 4 2
(A) h = 12 2
5.
A function f is such that f(2) = f(2) = 0 and f has a local maximum of 17 at x = 2, then f(x) may be
(A) f(x) = 17 (x 2)n n N n 4
(B) f(x) = 17 (x 2)n n 3
(C) f(x) = 17 + (x 2)n n 3
(D) f(x) 171 (x 2)n n 4

1
tan x, | x | < 4
6.
f(x) =
, then
| x |, | x |
4
2
(B) f(x) has only one point of local maxima
(A) f(x) has no point of local maxima
(C) f(x) has exactly two points of local maxima (D) f(x) has exactly two points of local minimas
x 3 x 2 +10x 5 , x1
7.
Let f(x) =
the set of values of b for which f(x) have greatest value at x = 1 is given
2x +log2 b 2 2 , x >1
by :
(A) 1 b 2
(B) b = {1, 2}

(C) b ( , 1)
(D) 130 , 2 U 2 , 130

8.
A tangent to the curve y = 1 x 2 is drawn so that the abscissa x 0 of the point of tangency belongs to the
interval (0, 1]. The tangent at x 0 meets the xaxis and yaxis at A & B respectively. The minimum area of
the triangle OAB, where O is the origin is

(A)
9.

10.

11.

2 3
9

(B)

4 3
9

(C)

2 2
9

(D) none

The lower corner of a leaf in a book is folded over so as to just reach the inner edge of the page. The
fraction of width folded over if the area of the folded part is minimum is:
(A) 5/8
(B) 2/3
(C) 3/4
(D) 4/5
2
n
{a1, a2,....., a4,......} is a progression where an = 3
. The largest term of this progression is:
n +200
(A) a6
(B) a7
(C) a8
(D) none
1
1
(sin x + tan x )
40 is
If f(x) =
+ 2 x then the range of f(x)

3 5
(A) ,
4 4

12.

13.

11
3 7
7 11
(B) 0,
(C) ,
(D) ,

4
4 4
4 4
{x }
{ x}
Let f(x) = sin
+ cos
where a > 0 and { . } denotes the fractional part function. Then the set of
a
a
values of a for which f can attain its maximum values is
4
4

(A) 0,
(B) ,
(C) (0, )
(D) none of these

A and B are the points (2, 0) and (0, 2) respectively. The coordinates of the point P on the line
2x + 3y + 1 = 0 are
1 1
(A) (7, 5) if |PA PB| is maximum
(B) , if |PA PB| is maximum
5 5

1 1
(D) , if |PA PB| is minimum
5 5
14.
The maximum area of the rectangle whose sides pass through the angular points of a given rectangle of
sides a and b is
1
1 2
(a + b)2
(C)
(a + b2)
(D) none of these
(A) 2 (ab)
(B)
2
2
2
15.
Number of solution(s) satisfying the equation, 3x 2x 3 = log2 (x 2 + 1) log2 x is:
(A) 1
(B) 2
(C) 3
(D) none
2
2
16.
Least value of the function, f(x) = 2 x 1 + x 2
is:
2 +1
(A) 0
(B) 3/2
(C) 2/3
(D) 1
17.
A straight line through the point (h, k) where h > 0 and k > 0, makes positive intercepts on the coordinate
axes. Then the minimum length of the line intercepted between the coordinate axes is
(A) (h2/3 + k2/3) 3/2
(B) (h3/2 + k3/2)2/3
(C) (h2/3 k2/3) 3/2
(D) (h3/2 k3/2)2/3
x
x
18.
The value of a for which the function f(x) = (4a 3) (x + log 5) + 2(a 7) cot
sin2
does not posses
2
2
critical points is
(A) ( , 4/3)
(B) ( , 1)
(C) [1, )
(D) (2, )
1
1

1 +
is
19.
The minimum value of 1 +
sinn cos n
(A) 1
(B) 2
(C) (1 + 2n/2)2
(D) None of these
20.
The altitude of a right circular cone of minimum volume circumscribed about a sphere of radius r is
(A) 2 r
(B) 3 r
(C) 5 r
(D) none of these
One or more than one correct options
Let f(x) = 40/(3x 4 + 8x 3 18x 2 + 60), consider the following statement about f(x).
21.
(A)
f(x) has local minima at x = 0
(B)
f(x) has local maxima at x = 0
(C)
absolute maximum value of f(x) is not defined (D)
f(x) is local maxima at x = 3, x = 1
22.
Maximum and minimum values of the function,
1
2x
f(x) =
cos (x + 3) + 2 sin (x + 3) 0 < x < 4 occur at :

(A) x = 1
(B) x = 2
(C) x = 3
(D) x =
lim
lim
23.
If
f(x) =
[f(x)] ( [ . ] denotes the greater integer function) and f(x) is non-constant continuous

(C) (7, 5) if |PA PB| is minimum

x a

x a

function, then
(A) lim f(x) is integer

24.

25.

(B) xlim
f(x) is non-integer
x a
a
(C) f (x) has local maximum at x = a
(D) f (x) has local minima at x = a
If the derivative of an odd cubic polynomial vanishes at two different values of x then
(A)
coefficient of x 3 & x in the polynomial must be same in sign
(B)
coefficient of x 3 & x in the polynomial must be different in sign
(C)
the values of x where derivative vanishes are closer to origin as compared to the respective
roots on either side of origin.
(D)
the values of x where derivative vanishes are far from origin as compared to the respective
roots on either side of origin.
Let f(x) = l n (2x x 2) + sin
(A)
(C)

26.

x
. Then
2

graph of f is symmetrical about the line x = 1 (B)graph of f is symmetrical about the line x = 2
maximum value of f is 1
(D) minimum value of f does not exist

The curve y =

x+1

x2 + 1

has:

(A) x = 1, the point of inflection


27.

(B) x = 2 + 3 , the point of inflection

(C) x = 1, the point of minimum


(D) x = 2 3 , the point of inflection
If the function y = f (x) is represented as,
x = (t) = t3 5 t 2 20 t + 7
y = (t) = 4 t3 3 t 2 18 t + 3 ( 2 < t < 2),
then:
(A) ymax = 12
(B) ymax = 14
(C) ymin = 67/4
(D) ymin = 69/4
41

28.

ax 2 + 2bx + c

The maximum and minimum values of y =


(A)
(B)

are those for which


Ax 2 + 2Bx + C
ax + 2bx + c y (Ax + 2Bx + C) is equal to zero
ax 2 + 2 bx + c y (Ax 2 + 2Bx + C) is a perfect square
2

dy
d2 y
= 0 and
0
dx
dx 2
2
(D)
ax + 2bx + c y (Ax 2 + 2 Bx + C) is not a perfect square
f(x) is cubic polynomial which has local maximum at x = 1, If f(2) = 18, f(1) = 1 and f(x) has local minima at
x = 0, then
[IIT 2006, (5, 1)]

(C)

29.

1.
2.
3.
4.
5.
6.
7.
8.
9.

(A)

the distance between point of maxima and minima is 2 5 .

(B)
(C)
(D)

f(x) is increasing for x [1, 2 5 )


f(x) has local minima at x = 1
the value of f(0) = 5

EXERCISE15

Find the area of the largest rectangle with lower base on the x-axis & upper vertices on the curve
y = 12 x .
Find the cosine of the angle at the vertex of an isosceles triangle having the greatest area for the given
constant length  of the median drawn to its lateral side .
ax 3
Find the set of value(s) of 'a' for which the function f (x) =
+ (a + 2) x2 + (a 1) x + 2 possess a negative
3
point of inflection.
The fuel charges for running a train are proportional to the square of the speed generated in m.p.h. & costs
Rs. 48/- per hour at 16 mph. What is the most economical speed if the fixed charges i.e. salaries etc.
amount to Rs. 300/- per hour.
The three sides of a trapezium are equal each being 6 cms long, find the area of the trapezium when it is
maximum.
What are the dimensions of the rectangular plot of the greatest area which can be laid out within a triangle
of base 36 ft. & altitude 12 ft? Assume that one side of the rectangle lies on the base of the triangle.
A closed rectangular box with a square base is to be made to contain 1000 cubic feet. The cost of the
material per square foot for the bottom is 15 paise, for the top 25 paise and for the sides 20 paise. The
labour charges for making the box are Rs. 3/-. Find the dimensions of the box when the cost is minimum.
Find the point on the curve, 4 x2 + a2 y2 = 4 a2, 4 < a2 < 8, that is farthest from the point (0, 2).
A cone is circumscribed about a sphere of radius ' r '. Show that the volume of the cone is minimum when

1
3

its semi vertical angle is, sin 1 .


a 3
x + (a + 2) x2 + (a 1) x + 2 possess a negative point
3

10.

Find the values of 'a' for which the function f(x) =

11.

of minimum.
A figure is bounded by the curves, y = x 2 + 1, y = 0, x = 0 & x = 1. At what point (a, b), a tangent should
be drawn to the curve, y = x 2 + 1 for it to cut off a trapezium of the greatest area from the figure.
Prove that the least perimeter of an isosceles triangle in which a circle of radius ' r ' can be inscribed is

12.

6r 3.

f ( x)

x3

1/ x

13.

1+
Find the polynomial f (x) of degree 6, which satisfies Limit
x0

14.

x = 1 and local minimum at x = 0 & 2.


Two towns located on the same side of the river agree to construct a pumping station and filteration plant
at the rivers edge, to be used jointly to supply the towns with water. If the distance of the two towns from
the river are a & b and the distance between them is c, show that the pipe lines joining them to the
pumping station is atleast as great as c + 4ab .
Find the co-ordinates of all the points P on the ellipse (x 2/a2) + (y2/b2) = 1 for which the area of the triangle
PON is maximum, where O denotes the origin and N the foot of the perpendicular from O to the tangent at
P.
[IIT 1999, 10]
If p(x) be a polynomial of degree 3 satisfying p(1) = 10, p(1) = 6 and p(x) has maxima at
x = 1 and p(x) has minima at x = 1. Find the distance between the local maxima and local minima of the
curve.
[IIT 2005]
2

15.
16.

= e2 and has local maximum at

42

TANGENT & NORMAL


EXERCISE - 1
Q.1 2 3 x y = 2

3 1 or 2 3 x + y = 2

3 +1

Q.2 (0, 1)

Q.3

x = 1 when t = 1, m ; 5x 4y = 1 if t 1, m = 1/3]

Q.7

T : x 2y = 0 ; N : 2x + y = 0

Q.9 (a) n = 2

Q.12 a = 1

Q.8 x + 2 y = /2 & x + 2 y = 3 /2
Q.14

1
; x 4y = 2
x+2

Q.22 (b) a b = a b

Q.20 2ex/2

Q.16 a = 1/2 ; b = 3/4 ; c = 3

Q.23 = tan1

2
C

Q.25

m m
2

EXERCISE - 2
Q.1 1/9 m/min
Q.4 3/8 cm/min
Q.8

2
cm/s
4

Q.12 (a)

Q.2 (i) 6 km/h (ii) 2 km/hr


Q.3 (4 , 11) & ( 4, 31/3)
Q.5 1 + 36 cu. cm/sec
Q.6 1/48 cm/s
Q.7 0.05 cm/sec
Q.9 200 r3 / (r + 5) km / h

5
1
m/min., (b)
m/min.
24
288

Q.10

66
7

Q.11

1
cm/sec.
4

Q.14 (a) r = (1 + t)1/4, (b) t = 80 Q.15 (a) 5.02, (b)

80
27

EXERCISE - 3
Q.1 = tan1

4 2
7

Q.3 D

Q.2

2 x+y2

2 = 0 or

Q.4 D

Q.5

2 xy2 2 =0

EXERCISE - 4
1. B
9. B
17. AC

2. B
10. B
18. AB

3. D
11. B
19. ABC

4. B 5. B
12. B 13. A
20. AC 21. CD

6.
14.
22.

A
C
BD

7.
15.

B
B

8.
16.

A
ABD

2 ,2 , 2 ,2

EXERCISE - 5
1. a = 1, b = 1, c = 0 2. (9/4, 3/8)

5. (i)

3.

8b
27

1
at (0, 0); tan 1 at (8, 16), (8, 16) (ii) /3
2
2

6. (i) 2 cm/min

(ii) 2 cm 2/min.

11. 2x + y = 0, x = 2y 12.

17. 25y2 + 4x 2 = 4x 2y2 19. t =

c
2
H
k

(iii)

4 2

7 at

tan1

)(

7. (4 , 11) & ( 4, 31/3)


14. y = x 5x 3

16.

13
a , 3
4

20. y = 2

MONOTONOCITY
EXERCISE - 6
Q.1

(a) I in (2 , ) & D in ( , 2) (b) I in (1 , ) & D in ( , 0) (0 , 1)


(c) I in (0, 2) & D in ( , 0) (2 , )
(d) I for x >

Q.2
Q.3

1
1
1
1
or < x < 0 & D for x < or 0 < x <
2
2
2
2

(2, 0) (2, )
(a)
I in [0, 3/4) (7/4 , 2 ] & D in (3/4 , 7 /4)
(b)
I in [0 , /6) (/2 , 5/6) (3/2 , 2 ] &43 D in (/6 , /2) (5/6, 3 /2)]

Q.5

continuous but not diff. at x = 1

a < 2+ 5

Q.6

or a >

(c) 2 & 10
[ 7, 1) [2, 3]

(1/e)1/e, no greatest value,

Q.7 (a) (/6)+(1/2)ln 3, (/3) (1/2)ln 3, (b) least value is equal to


Q.8 [1, )
Q.10 a ( , 3] [1 , )
Q.11
Q.12 increasing in x (/2 , 2/3) & decreasing in [0 , /2) (2/3 , ]
Q.13 0 a

3
2

Q.14 in (3, ) and in (1, 3)

Q.15

(6, )

Q.16 a 0

5
5
5
Q.17 (a) ( , 0] ; (b) in 1, and in ( ,1) , { 3} ; (c) x = ;
3
3

3
(d) removable discont. at x = 3 (missing point) and non removable discont. at x = 1 (infinite type)
(e) 2
Q.25 (b a)3/4
Q.24 ( 1, 0) (0, )

EXERCISE - 7
mb + na
which lies between a & b
m+n

Q.1

c=

Q.7

y = 5x 9 and y = 5x + 11

Q.6

a = 3, b = 4 and m = 1

Q.18 0

EXERCISE - 8
Q.1 C

Q.3 (a) A, C ; (b) D

1
1
Q.5 (a) A, (b) cos cos p
3

Q.4 (a) B ; (b) D ; (c) C


Q.8

Q.6 A

(a) D ; (b) C

Q.10 (a) D

EXERCISE - 9
1. C
9. C

2. B
10. CD

3. B
11. BC

4. C 5. D
12. BC 13. ABD

C
AB

6.
14.

7.
15.

A
ABD

8.

EXERCISE - 10
1. Neither increasing nor decreasing at x = 1, increasing at x = 0, 1.
3. ad > bc
2. a 0
4. in ( , 1] [1, ) & D in [ 1, 1]
5. (a) x <

2
(p2 + q2 + r2), x > 0
3

(b) (

2
(p2 + q2 + r2), 0)
3

6. ( , 3]
8. 2sinx + tanx > 3x, limit = 0
3
13. increasing on [0, /2] and decreasing on [/2, ]
10. ,
6 2
16. (/6) + (1/2) n 3, (/3) (1/2) n 3


21. increasing when x , , decreasing when x 0, .
4
2

4
23. Prove that for 0 p 1 & for any a > 0, b > 0 the inequality (a + b)p ap + bp .

24. greatest = 14, least = 8

*************************************

MAXIMA - MINIMA
EXERCISE - 11
Q.1

f (x) = x3 + x2 x + 2 Q.2

Q.3

(a) Max at x = 2 , Max value = 2 , Min. at x = 0 , Min value = 0


(b) Max at x = /6 & also at x = 5 /6 and
Max value = 3/2 , Min at x = /2 , Min value = 3

Q.4 f (x) =

2 6
12 5
x
x + 2 x4
3
5

max. at x = 1 ; f(1) = 0 , min. at x = 7/5 ; f(7/5) = 108/3125

Q.5 Pmax =

a 1 + cos ec

2
44

Q.6 75 3 sq. units

Q.7

2A
,s=
+4

r=

2A
+4

Q.9 3x + 4y 9 = 0 ; 3x 4y + 9 = 0

Q.11 1/ cu m
Q.13 6/(6 + )
Q.16 = 600

Q.10 4 2 m
Q.12 110 ' , 70 '
Q.15 32 sq. units

Q.14 side 10', height 10'


Q.17 a = 1, b = 0

Q.19 width 2 3 m, length 3 3 m


Q.20 a b
Q.21 (a) (1, 0), (0, 5/6) ; (b) F ' (x) = (x2 x), F '' (x) = 2x 1, (c) increasing ( , 0) (1, ), decreasing (0, 1)
; (d) (0, 5/6) ; (1, 2/3) ; (e) x = 1/2
Q.22 (a) x = y =

d
d
2
, (b) x =
, y=
d
2
3
3

Q.24 r = A , = 2 radians

Q.23 6' 18'

Q.25

3 3

EXERCISE - 12
Q.1

(a) 0, 3, (c)

Q.6 p < a <


Q.9

3
, t = ln 4
4

Q.2 cos A = 0.8

32 p 3
32 p 3
+ p if p > 0 ;
+ p < a < p if p < 0
27
27

Q.8 4 when a =

Maximum value is (e + e1) when a = 1

Q.10 (a) f is continuous at x = 0 ; (b)


Q.11

Q.4 (0 , 0)

2
; (c) does not exist, does not exist ; (d) pt. of inflection x = 1
e

(a) x = 2, , 0, , 2, (b) no inflection point, (c) maxima at x =


3

and x = , (e) ln 2
2
2

(d) x =
Q.12 4

and
and no minima,
2
2

Q.13 (0 , 2) & max. distance = 4

1 1
Q.14 m ,
32 16

Q.16

1/ 3

4V
(c + a b)(a + b c)

Q.17 ( , 3) (3 , 29/7) Q.18 H = x =


Q.19
2
3

Q.20 L/4
Q.21
3
Q.23 (a) increasing in (0, 2) and decreasing in (, 0) (2, ), local min. value = 0 and local max. value = 2

(b) concave up for ( , 2 2 ) (2 + 2 , ) and concave down in (2 2 ), (2 +


1 2x 2
(c) f (x) = e x
Q.24 2 2 1
2

EXERCISE - 13

2
sq. units
Q.1 1
3

Q.3

,
2 (a + b)

Q.5

a2
a 2 + b2

Q.2 (a) B, (b) a =

c
&

2 (a + b )

b2
a 2 + b2

c
,
2 (a + b)

1
5
1
; b = ; f(x) = (x2 5x + 8)
4
4
4

2 (a + b)

Q.6 3 x 2 y =
45

Q.4 (a) B, D,

Q.7 (9/2 , 2)

2)

Q.8

Q.9

Q.13 (a) D Q.14

Q.10 (a) D ; (b) A

2ab

4 65 Q.15

Q.11 (a) A ; (b) 18

Q.12 (a) (2, 1) ; (b) 5

(a) B, C; (b) A, B, (c) 6 solutions

EXERCISE - 14
1. B
9. B
17. A

2. D
10. B
18. A

3. A
11. B
19. C

4. D
12. A
20. D

25. ACD

26. ABD

27. BD

28. BC

5. A
13. A
21. ACD

C
B
AC
BC

6.
14.
22.
29.

7.
15.
23.

EXERCISE - 15
1. 32 sq. units

2. cos A = 0.8

3. ( , 2) (0, )

4.

40 mph

5. 27 3 sq. cms

6. 6 18

7. side 10', height 10'

8.

(0, 2)

10. (1, )

11.

15.

a2
a 2 + b2

1 5
,
2 4

b2
a 2 + b2

13. f (x) = 2 x 4

12 5 2 6
x + x
5
3

16. 4 65

46

D
A
AD

8.
16.
24.

B
D
BC

95.

Statements-1: For the circle (x 1)2 + (y 1)2 = 1, the tangent at the point (1, 0) is the x-axis.
Statements-2: the derivative of a single valued function y = f(x) at x = a is the slope of the tangent drawn to the
curve at x = a.

96.

Statements-1: Both sin x, and cos x are decreasing functions in


,
2

[ Good ]

Statements-2: If a differentiable function decreases is an interval (a, b) then its derivative also decreases in (a, b).
97.

Statements-1: e >

[ Good ]
1
x

98.

Statements-2: The function x ( x > 0) has a local maximum at x = e


Statements-1: Conditions of LMVT fail in f(x) = |x 1| (x 1)
Statements-2: |x 1| is not differentiable at x = 1

99.

Let f(x) =

(x x )

i =1

Statement1 : Minimum value of f(x) occurs at x =


100.

102.

103.

105.

log e x
is a decreasing function for x > e.
x

Statement1 : Total number of critical points of f(x) = max. {1/2, sinx, cox} x are 5
Statement2 : Total number of critical points of f(x) = max {1/2, x, cosx} x are 2
Let f(x) = 5p2 + 4(x 1) x2, xR and p is a real constant
Statement1 : If the maximum values of f(x) is 20, then p = 2.
Statement2 : If the maximum value of f(x) is 20, then p = 2.
Let f(x) = sin1 x + cos1 x + tan1x and x [ 1, 1]
Statements-1:

104.

Statement2 : Minimum of f(x) = ax2 + bx + c (a > 0) occurs at x = b/2a.


Statement1 : > , for 2.91 < <
Statement2 : f(x) =

101.

Statements-2:
Let f(x) = x3
Statements-1:
Statements-2:
Suppose f(x) =

.
Range of f(x) is ,
4 4
f(x) is an increasing function.
x = 0, in the point of inflexion for f(x)
f (x) < 0 for x < 0 and f (x) > 0 for x > 0.

x2
+ n x + 2 cos x
2

Statements-1: f is an increasing function.


Statements-2: derivative of f(x) with respect to x is always greater than zero.
106.

Let 0 < x

sin x
and f(x) =
2
x

Statements-1:

The minimum value of f is

Statements-2: 0 < sin x < x, x 0,


107.
108.

109.

, attained at x = .

.
2

Statements-1: The equation x2 = x sin x + cos x has only one solution.


Statements-2: The derivative of the function x2 x sin x cos x is x(2- cos x).
Statement1 : Angle of intersects in between y = x2 and 6y = 7 x3 at (1, 1) is /4
Statement2 : Angle of intersection between any two curve is angle between the tangents at the point of
intersection.
Statement 1 : The curve y = x1/3 has a point of inflection at x = 0
Statement 2 : A point where y fails to exist can be a point of inflection

47 of 52
47

110.
Let f(x) and g(x) are two positive and increasing function
Statement 1 : If (f(x)) g(x) is decreasing then f(x) < 1
Statement 2 : If f(x) is decreasing then f(x) < 0 and increasing then f(x) > 0 for all x.
111.
112.
113.

114.

Statement 1 : If f(0) = 0, f(x) = ln (x + 1 + x ), then f(x) is positive for all xR0


Statements-2: f(x) is increasing for x > 0 and decreasing for x < 0.
Statements-1: The two curves y2 = 4x and x2 + y2 6x + 1 = 0 at the point (1, 2) intersect orthogonally.
Statements-2: Two curves y = f(x) & y=g(x) intersect orthogonally at (x1 y1) if (f (x1).g((x1)) = 1.
Statements-1: If 27a + 9b + 3c + d = 0, then the equation 4ax3 + 3bx2 + 2cx + d = 0 has atleast one real root lying
between (0, 3)
Statements-2: If f(x) is continuous in [a, b], derivable in (a, b), then at least one point c (a, b) such that f(c)=0 .
Statements-1: f(x) = {x} has local minima at x = 1.
Statements-2: x = a will be local minima for y = f(x) provided lim f (x) > f (a) also
2

x a

lim+ f(x) > f(a).

x a

115.

Statements-1: f(x) =

1
x ;
2

x<

1
2

1
1

= x ; x . Mean value theorem is applicable in the interval [0, 1].


2
2

116.

117.

118.

S-2: For application of mean value theorem, f(x) must be continuous in [0, 1] and differentiable in (0, 1).
Statements-1: For some 0 < x1 < x 2 < /2, tan-1x2 tan-1x1 < x2 x1
Statements-2: If f(x) >f(x1) x2 > x1
function is always increasing
Statements-1: The graph of a continuous function y = f(x) has a cusp at point x = c if f (x) has same sign on both
sides of c.
Statements-2: The concavity at any point x = c depends upon f (x). If f (x) < 0 or f (x) > 0 the function is
either concave up or concave down.
Statements-1: If f be a function defined for all x such that |f(x) f(y)| < (x y)2 then f is constant
Statements-2: If (x) < (x) < (x) for all x and lim (x) = lim (x) = L lim (x) = L
x a

119.
120.

121.

122.

x a

x a

Statements-1: f : R R be a function such that f(x) = x3 + x2 + 3x + sinx. Then f is one-one.


Statements-2: f(x) is neither increasing nor decreasing.
Statements-1: If & are any two roots of equation ex cosx = 1, then the equation
ex sinx 1 = 0 has at least one root in (, )
Statements-2: f is continuous in [, ]. f is derivable in (, ). f() = f() then these exists
x ( , )such that f(x) = 0
Statements-1: The minimum value of the expression x2 + 2bx + c is c b2.
Statements-2: The first order derivative of the expression at x = b is zero and second derivative is always
positive.


then (x) is decreasing in
2

Statements-1: Let (x) = sin (cosx) in 0,


0, 2

Statements-2: (x) 0 x 0,
123.

124.
125.

Statements-1: The function f(x) = x4 8x3 + 22x2 24x + 21 is decreasing for every
x (2, 3) (, 1)
Statements-2: f (x) > 0 for the given values of x.
Statements-1: For the function f(x) = xx, x = 1/e is a point of local minimum.
Statements-2: f (x) changes its sign from ve to positive in neighbourhood of x = 1/e.
Statements-1: Consider the function f(x) = (x3 6x2 + 12x 8) ex is neither maximum nor minimum let x = 2
Statements-2: f(x) = 0, f(x) = 0, f(x) 0 at x = 2

f (x1 + x 2 ) f (x1 ) + f (x 2 )
<
2
2

126.

Statements-1: Consider the function f(x)

127.

Statements-2: f(x) > 0, f(x) > 0 where x1 < x2


Consider the following function with regard to the function
48 of 52
48

f(x) = (x3 6x2 + 12x 8) ex


Statement-1: f(x) is neither maximum nor minimum at x = 2
Statement-2: f (x) = 0, f (x) = 0, f (x) 0 at x = 2.
128.
Statements-1: Equation f(x) = x3 + 9x2 + 2ax + a2 + a + 1 = 0 has at least one real negative root.
Statements-2: Every equation of odd degree has at least one real root whose sign is opposite to that of its constant
term.
ANSWER
95. B
96. C
97. A
98. D
99. A
100. A
101. A
102. A
103. A
104. A
105. A
106. B
107. D
108. D
109. A
110. A
111. A
112. D
113. A
114. A
115. D
116. A
117. A
118. A
119. C
120. A
121. A
122. A
123. C
124. A
125. A
126. A
127. A
128. A

Que. from Compt. Exams


1.

d
cos x sin x
tan 1
=
dx
cos x + sin x

(a)
2.

3.

1
2 (1 + x 2 )

[AISSE 1985, 87; DSSE 1982,84; MNR 1985; Karnataka CET 2002; RPET 2002, 03]

(b)

1
1 + x2

(c)

If y =

x
a2
dy
a2 + x 2 +
log( x + x 2 + a 2 ) ,then
=
2
2
dx

(a)

x 2 + a2

(b)

2
3

1/2

1
3

(a)

(d)

[AISSE 1983]
2 x 2 + a2

(c)

x 2 + a2

If y = cot 1 (cos 2 x )1 / 2 , then the value of

(d)

x 2 + a2

dy

at x = will be
dx
6

[IIT 1992]

1/2

(b)

(3)1 / 2

(c)

(6)1 / 2

(d)

4.

If f ( x + y ) = f (x ). f (y ) for all x and y and f (5 ) = 2 , f ' (0) = 3 , then f ' (5 ) will be [IIT 1981; Karnataka CET 2000;
UPSEAT 2002; MP PET 2002; AIEEE 2002]
(a) 2
(b) 4
(c)
6
(d)
8

5.

If xe xy = y + sin 2 x , then at x = 0,

6.

(a) 1
(b) 2
If u(x , y ) = y log x + x log y, then

dy
=
dx

[IIT 1996]
(c)

(d)

(d)

u x uy u x log x uy log y + log x log y = [EAMCET 2003]

(a) 0
7.

8.

(b) 1

dy
2x 1
If y = f 2
=
and f (x ) = sin x 2 , then
dx
x +1

(c)
[IIT 1982]

(a)

6 x 2 2x + 2
2x 1
sin 2

(x 2 + 1)2
x +1

(c)

2x 2 + 2x + 2
2x 1
sin 2 2

( x 2 + 1)2
x +1

(b)

6 x 2 2x + 2
2x 1
sin 2 2

(x 2 + 1)2
x +1

(d)

2x 2 + 2x + 2
2x 1
sin 2

( x 2 + 1)2
x +1

If x = sec cos and y = sec n cos n , then

[IIT 1989]

dy
= n 2 (y 2 + 4 )
dx

(a) (x 2 + 4 )

(b)

dy
(x 2 + 4 )
= x 2 (y 2 + 4 )
dx

(d)

None of these

dy
= (y 2 + 4 )
dx
x ......
dy
, then
If y = x x
=
dx

(c) (x 2 + 4 )
9.

[UPSEAT 2004; DCE 2000]

49 of 52
49

(a)
10.

y2
x (1 + y log x )

y2
x (1 y log x )

(b)

If y = (x log x )log log x , then

y
x (1 + y log x )

(c)

dy
=
dx

y
x (1 y log x )

(d)

[Roorkee 1981]

(a) (x logx)log log x 1 (logx + loglog x) + (log log x) 1 +


x log x

x log x

2
1
+
log
x
x

(b) (x log x )x log x log log x

( x log x )x log x

(c)

log log x
x

+ 1

x
log

(d) None of these


11.

d
dx

(a)
12.

If
(a)

13.

1 + x2 + 1 x2
tan 1

1 + x2 1 x2

x
1 x

[Roorkee 1980; Karnataka CET 2005]


x

(b)

1 x

1 x6

x2
y

1y

If y = sec 1

y2

(b)

(c)

2 1x

dy
=
dx

(1 x 6 ) + (1 y 6 ) = a 3 ( x 3 y 3 ) , then

(a) 1
14.

(c)

dy
2x
x 1
,then
is equal to
+ sin 1
x +1
dx
1 + x2
x 1
(b)
x +1

(c)

1
4

(b)

x2
y2

1 y6
1 x6

Does not exist

1 + x2 1
with respect to tan 1 2 x 1 x
2

x
1 2x

1
8

2 1 x4

(d)

None of these

[Pb. CET 2000]

The derivative of tan 1


(a)

[Roorkee 1994]

1 y6
1x

(d)

1
2

(c)
d
dx

(d)

None of these

at x = 0 , is

(d)

3 d 2 y
y . 2 =

dx
(c)
p( x ). p ( x )

15.

If y 2 = p(x ) is a polynomial of degree three, then 2

16.

(a) p ( x ) + p ' ( x )
(b) p ( x ). p ( x )
(d)
Constant
rd
Let f (x ) and g(x ) be two functions having finite non-zero 3 order derivatives f (x ) and g (x ) for all, x R . If
f (x )g(x ) = 1 for all x R , then
f

(a) 3
f
g
17.

If In =

dn
dx n

(b) 3
g
f

g" f "
3
g
g

(c)

(b) n 1

(c)

If x = sin t and y = sin pt , then the value of 1 x 2


(a) 0

19.

f g
is equal to

f g

(d)

f g
3
f
f

( x n log x ), then I n nI n 1 = [EAMCET 2003]

(a) n
18.

[IIT 1988; RPET 2000]

(d)

d y
dy
) dx
x
+ p y is equal to
dx
2

(b) 1
x

n!

(n 1)!

(c)

[Pb. CET 2002]

(d)

Let f : (0, + ) R and F(x ) = f (t) dt . If F(x ) = x (1 + x ) , then f (4 ) equals


2

[IIT Screening 2001]

(a)
20.

(b) 7

(c)

(d)

The volume of a spherical balloon is increasing at the rate of 40 cubic centrimetre per minute. The rate of change of
the surface of the balloon at the instant when its radius is 8 centimetre, is
[Roorkee 1983]
(a)

21.

5
4

5
sq cm/min
2

(b) 5 sq cm/min

10 sq cm/min

(c)

(d)

20 sq cm/min

A man of height 1.8 metre is moving away from a lamp post at the rate of 1.2 m / sec . If the height of the lamp post
be 4.5 metre, then the rate at which the shadow of the man is lengthening is
[MP PET 1989]
(a) 0 .4 m / sec
(b) 0 .8 m / sec
(c)
(d)
None of these
1 . 2 m / sec
50 of 52
50

22.

The radius of the cylinder of maximum volume, which can be inscribed in a sphere of radius R is
(a)

23.

24.

25.

2
R
3

2
R
3

(b)

3
R
4

(c)

3
R
4

(d)

The distance travelled s (in metre) by a particle in t seconds is given by, s = t 3 + 2 t 2 + t. The speed of the particle after
1 second will be
[UPSEAT 2003]
(a) 8 cm/sec
(b) 6 cm/sec
(c)
2 cm/sec (d)
None of these
If y = 4 x 5 is tangent to the curve y 2 = px 3 + q at (2, 3), then
[IIT 1994; UPSEAT 2001]
(b) p = 2, q = 7
(c)
(d)
(a) p = 2, q = 7
p = 2, q = 7
p = 2, q = 7
At what points of the curve y =

2 3 1 2
x + x , tangent makes the equal angle with axis
3
2

1
1
1 4

1 5

1 1
and 1, (b) , and (1, 0 ) (c) , and 3,
6
2
2 24

3 7
2 9

(a) ,
26.

[AMU 1999]

If the normal to the curve y = f (x ) at the point (3, 4 ) makes an angle

(d)

[UPSEAT 1999]
1
1 4

,
and 1,
3
3 47

3
with the positive x-axis then f ' (3) is equal
4

to
(a) 1
27.

(b)

11
(b)
,1

Let f (x ) =
0

(c) Minima when n = 0, 2, 4 ,....

30.

(d)

,2

(c)

(0 , 0 )

(d)

(b)

[Kurukshetra CEE 2002]


Maxima when n = 1, 3, 5, ....

(d)

Minima when n = 1,3,5 ....

cos t
dt , x > 0 then f (x ) has
t

(a) Maxima when n = 2, 4 , 6, .....


29.

4
3

(c)

The point(s) on the curve y 3 + 3 x 2 = 12 y where the tangent is vertical (parallel to y-axis), is (are)
[IIT Screening 2002]
4

(a)
,2

28.

3
4

If f (x ) = x + 2bx + 2 c and g(x ) = x 2cx + b such that min f (x ) > max g(x ) , then the relation between b and c is
[IIT Screening 2003]
(a) No real value of b and c (b) 0 < c < b 2
2

(c) | c | <| b | 2
(d) | c | >| b | 2
N characters of information are held on magnetic tape, in batches of x characters each; the batch processing time is
[MNR 1986]
+ x 2 seconds; and are constants. The optimal value of x for fast processing is
(a)

(b)

(c)

(d)

31.

On the interval [0, 1], the function x 25 (1 x )75 takes its maximum value at the point
(a) 0
(b) 1/2
(c)
1/3
(d)
1/4

32.

The function

[IIT 1995]

f (x ) =

t(e

1)(t 1)(t 2)3 (t 3)5 dt

[IIT 1999]

has a local minimum at x =

33.

34.

(a) 0
(b) 1
The maximum value of exp (2 + 3 cos x + sin x ) is

(d)
3
[AMU 1999]

(a) exp( 2)
(b) exp( 2 3 )
(c)
1
exp( 4 ) (d)
If the function f (x ) = 2 x 3 9 ax 2 + 12 a 2 x + 1, where a > 0 attains its maximum and minimum at p and q respectively
such that p 2 = q , then a equals
[AIEEE 2003]
(a) 3

35.

(c)

(b) 1

(c)

ln( + x )
is
[IIT 1995]
ln(e + x )
(a) Increasing on [0, )
(b) Decreasing on [0, )

The function f (x ) =

51 of 52
51

(d)

1
2

36.

(c) Decreasing on 0, and increasing on ,


e
e

(d) Increasing on 0, and decreasing on ,


e
e

The function f (x ) = sin 4 x + cos 4 x increases, if

[IIT 1999; Pb. CET 2001]

(a) 0 < x <

(c)

37.

3
(b) < x <
8
4
8
Let h(x ) = f (x ) ( f (x ))2 + ( f (x ))3 for every real number x. Then

38.

(a) h is increasing whenever f is increasing


(b)
(c) h is decreasing whenever f is decreasing
(d)
In [0, 1] Lagrange's mean value theorem is NOT applicable to

[IIT 1998]
h is increasing whenever f is decreasing
Nothing can be said in general
[IIT Screening 2003]

1
1
x<
2 x,
2

(b) f (x ) =
(a) f (x ) =
2
1 x , x 1
2
2

(c) f (x ) = x | x |
39.

3
5
5
3
(d)
<x<
<x<
8
8
8
4

sin x
, x0

x
1 , x =0

(d) f (x ) =| x |

If the function f (x ) = x 6 ax 2 + 5 x satisfies the conditions of Lagrange's mean value theorem for the interval [1, 2]
3

7
is parallel to the chord that joins the points of intersection of the curve
4
with the ordinates x = 1 and x = 2 . Then the value of a is

and the tangent to the curve y = f (x ) at x =

[MP PET 1998]


35
(a)
16

40.

35
48

(b)

7
16

(c)

5
16

(d)

x lnx , x > 0
, Rolles theorem is applicable to f for x [0,1] , if =
x = 0
0,

Let f (x ) =
(a) 2

(b) 1

1
6
11
16
21
26
31
36

d
c
a
b
b
d
d
b

(c)

Que. from Compt. Exams


2
a
3
a
4
c
7
d
8
a
9
b
12
c
13
c
14
b
17
d
18
a
19
c
22
b
23
a
24
a
27
d
28
b,d 29
d
32
b,d 33
c
34
c
37
a, c 38
a
39
b

52 of 52
52

[IIT Screening 2004]


1
2

(d)

5
10
15
20
25
30
35
40

c
a
c
c
a
c
b
d

STUDY PACKAGE
Target: IIT-JEE (Advanced)
SUBJECT: MATHEMATICS
TOPIC: 22 XII M 5. Integration
Index:
1. Key Concepts
2. Exercise I to VIII
3. Answer Key
4. Assertion and Reasons
5. 34 Yrs. Que. from IIT-JEE
6. 10 Yrs. Que. from AIEEE

Indefinite
1.

Integration

If f & g are functions of x such that g (x) = f(x) then,

f(x) dx = g(x) + c

d
{g(x)+c} = f(x), where c is called the constant of integration.
dx

2.

S t a nd ard

(i)

(ax + b)

(ii)

dx
1
= ln (ax + b) + c
ax + b a

(iii)

eax+b dx =

1 ax+b
e
+c
a

(iv)

apx+q dx =

1 a px+q
+ c; a > 0
p n a

(v)

sin (ax + b) dx =

(vi)

cos (ax + b) dx =

(vii)

tan(ax + b) dx =

1
ln sec (ax + b) + c
a

(viii)

cot(ax + b) dx =

1
ln sin(ax + b)+ c
a

(ix)

sec (ax + b) dx =

(x)

cosec(ax + b) dx =

(xi)

sec (ax + b). tan (ax + b) dx =

(xii)

cosec (ax + b). cot (ax + b) dx =

(xiii)

secx dx = ln (secx + tanx) + c

(xiv)

cosec x dx = ln (cosecx cotx) + c OR ln tan

Fo rmu l a:

(ax + b)n +1
dx =
a ( n + 1)

+ c, n 1

1
cos (ax + b) + c
a

1
sin (ax + b) + c
a

1
tan(ax + b) + c
a
1 cot(ax + b)+ c
a
1
sec (ax + b) + c
a
1 cosec (ax + b) + c
a
ln tan

OR

+ x + c
4 2

x
+ c OR ln (cosecx + cotx) + c
2

dx

x
+c
a

(xv)

(xvi)

(xvii)

(xviii)

(xix)

(xx)

a+x
dx
1
ln ax + c
2 =
a x
2a

(xxi)

xa
dx
1
ln x+a + c
2 =
x a
2a

(xxii)

(xxiii)

(xxiv)

(xxv)

a x
2

= sin1

1
dx
x
= tan1 + c
2
a
a
a +x
2

dx

x x a
2

dx

x +a
2

dx

x a
2

1
x
sec1
+c
a
a

= ln

= ln

[x +

x2 + a 2

[x +

x2 a2

OR

OR

sinh1

x
+c
a

cosh1

x
+c
a

a 2 x 2 dx =

x
2

a 2 x2 +

a2
x
sin1
+c
2
a

x
x + a dx =
2

x + x 2 + a2

a2
n
x +a +
a
2

+ c

x
x a dx =
2

x + x 2 a2

a2
n
x a
a
2

+c

eax. sin bx dx =

(xxvi)

3.

T heo r em s

e ax
(a sin bx b cos bx) + c
a 2 + b2

eax. cos bx dx =

e ax
(a cos bx + b sin bx) + c
a 2 + b2

on

i nteg ra t io n

(i)

c f ( x).dx

(ii)

(f ( x) g(x)) dx = f ( x)dx g(x) dx

=c

f (x).dx

(iii)
Note : (i)
(ii)

f (x)dx = g( x) + c

f (ax + b)dx =

every contineous function is integrable


the integral of a function reffered only by a constant.

f ( x).dx

= g(x) + c

= h(x) + c
g(x) = f(x)
&
g(x) h(x) = 0
means, g(x) h(x) = c
Example :

Evaluate :

Solution.

4x

Example :

Evaluate :

Solution.

4x

7
2
dx
+
x
x

7
2
dx
+
x
x
7
dx +
x

dx 4 . 1 . dx + 7 .

x1/ 2
x3
x4

4x + 7 log | x | + 2 1/ 2 + C
+5.
3
4

5
x4
+ x 3 4x + 7 log | x | + 4
3
4

dx +

dx + 5 .

Evaluate :

Solution.

We have,
x log a

4dx +

5x

+ 5x 2 4 +

Example :

Solution.

dx

+ 5x 2 4 +

h(x) = f(x)

4 6
2 6
x +C=
x + C.
6
3

dx =

Example :

g(ax + b)
+c
a

dx

x log a

+ e a log x + e a log a dx

+ e a log x + ea log a dx

(a

ax
x a +1
+
+ aa . x + C.
log a
a +1

log a x

dx

+ x a + aa ) dx

dx +

2x + 3x
5x

+ elog x + elog a

Evaluate :

x +C

dx +

2x + 3 x
5x

dx

dx

dx

2x 3x

+
5 x 5 x dx

dx

1
dx + 2 .
x

1 / 2

dx

x
x
2
3
( 2 / 5) x
(3 / 5 ) x
+ dx =
+
+C
loge 2 / 5
loge 3 / 5
5
5

Example:

Evaluate :

Solution.

Example :

Solution.

1
8
1
=
8

sin

x cos3 x dx

(2 sin x cos x)
sin

dx

2x dx

3 sin 2x sin 6x
dx
4

1
8

1
32

1 3
2 cos 2 x + 6 cos 6 x + C
32

(3 sin 2x sin 6x) dx

Evaluate :

x4
2

x4 1+ 1

(x

x +1
2

+1

dx

dx

+1

x4

dx =

Evaluate :

Solution.

We have

1) dx +

Example:

x4 1

4 + 9x

+1

1
2

+1

1
x2 + 1

dx =

dx

x3
x + tan1 x + C
3

dx

4 + 9x
=

1
9

4+x

dx
2

1
9

1
x
3x
1
1
+C=
.
tan1
tan1 + C
9 (2 / 3)
6
2/3
2

(2 / 3)

+ x2

Example :

cos x cos 2x dx

Solution.

cos x cos 2x dx
=

1
2

dx

2 cos x cos 2x dx
5

1
2

1 sin 3 x + sin x

+c
1
2 3

(cos 3x + cos x) dx

Self Practice Problems

1.

Evaluate :

tan

2.

Evaluate :

1 + sin x

4.

Integration by Subsitutions

x dx

dx

Ans.

tanx x + C

Ans.

tanx sec x + C

If we subsitute x = (t) in a integral then


(i)
everywhere x will be replaced in terms of t.
(ii)
dx also gets converted in terms of dt.
(iii)
(t) should be able to take all possible value that x can take.
Example :

Evaluate :

Solution.

We have

Example :
Solution.

L e t

Solution.

sin x 4 dx

sin x 4 dx

=t

d(x 4) = dt

4x 3 dx = dt dx =

(n x )2
dx
x

(n x )2
dx
x
Put
nx = t

Example :

1
dx = dt
x
dx
t 2.
x

t dt

t3
+c
3

(n x )3
+c
3

2
Evaluate (1 + sin x ) cos x dx

Put
sinx = t
cosx dx = dt

(1 + t 2 ) dt = t +

t3
+c
3

1
4x 3

dt

sin3 x
+c
3

= sin x +

Example :

Evaluate :

Solution.

We have,
=

+ x2 + 1

dx =

+ x +1

dx

(x

x
) + x2 + 1

2 2

Let x 2 = t, then, d (x 2) = dt
=

1
2

x
t + t +1

1
2

Note:

1 3

t + +
2 2
2

tan

(i)

(ii)

[f (x)]

(iii)

(iv)

(v)

dx =

dx

x x +1

( n1)

dx

x 1 + xn
n

2x 2 + 1

+ C.

( f ( x ))n +1
n +1

( f ( x ))1n
1 n

dx
nN
x (x n +1)
2

dt

2t + 1
1

+C=
tan1

3
3

[ f(x)]n f (x) dx =

f (x)

t+ 1

2 +C
3

tan1

3
2

dt

t2 + t + 1

1
=
.
2

2x dx = dt

dt
2x

dx

Take x n common & put 1 + x n = t.

n N, take x n common & put 1+x n = tn


n

1/ n

take x n common as x and put 1 + x n = t.

Self Practice Problems


1.
2.

sec 2 x
dx
1 + tan x

Ans.

n |1 + tan x| + C

sin(nx )
dx
x

Ans.

cos (n x) + C

dx =

dt
2x

5.

Integra t i o n

(f (x) g(x)) dx

by

= f(x)

Pa r t

(g( x)) dx

:
d

dx (f ( x)) (g(x)) dx dx

g( x) dx then it will not contain arbitarary constant.

(i)

when you find integral

(ii)

g(x) dx should be taken as same both terms.

(iii)

the choice of f(x) and g(x) is decided by ILATE rule.


the function will come later is taken an integral function.

Inverse function
L

Logrithimic function
A

Algeberic function
T

Trigonometric function
E

Exponential function

Example :

Evaluate :

Solution.

x tan

x tan

x dx

x dx

= (tan1 x)

x2

1+ x

x2
dx
2

x2 + 1 1

1
x2
tan1 x
2
2

1
x2
tan1 x
[x tan1 x] + C.
2
2

x +1
2

dx =

1
x2
tan1 x
2
2

Evaluate :

Solution.

x log(1 + x) dx
= log (x + 1) .

x2

1
x2
log (x + 1)
2
2

1
x2
log (x + 1)
2
2

1 ( x 1) + 1 dx
x2

log (x + 1)
x + 1
2
2

x2

2
1 x x + log | x + 1 |
x2

+C
log (x + 1)
2 2
2

x + 1 dx =

1
x2
log (x + 1)
2
2

1
x2 1
+
dx
x
+1
x +1

Solution.

Let =
=

e
2x

+1

dx

1
x2
.
dx
x +1
2

Evaluate :

1
2

x log(1 + x) dx

Example :

Example :

1 x

e
2x

2x

sin 3x dx

sin 3x dx. Then,

sin 3x dx

x2 1+ 1
dx
x +1

cos 3 x

= e2x
3

1 2x
2
e cos 3x +
3
3

sin 3 x

1 2x
2 2 x sin 3 x
2e 2 x
dx
e cos 3x +
e
3
3
3
3

1 2x
2
4
e cos 3x + e2x sin 3x
3
9
9

1 2x
2
4
e cos 3x + e2x sin 3x

3
9
9

13
e2x
(2 sin 3x 3 cos 3x)
=
9
9

2e

2x

cos 3 x

dx
3

2x

cos 3x dx

2x

sin 3 x dx

4
e2x
(2 sin 3x 3 cos 3x)
=
9
9

e2x
(2 sin 3x 3 cos 3x) + C
13

Note :
(i)
(ii)

ex [f(x) + f (x)] dx = ex. f(x) + c


[f(x) + xf (x)] dx = x f(x) + c

Example :

Solution.

Solution.

e
e

dx

x + 1 1

dx

( x + 1)2

Example :

x
( x + 1)2

1
1
ex
( x + 1) ( x + 1)2 dx

ex
+c
( x + 1)

1 sin x

dx
1 cos x

x
x

1 2 sin cos
2
2

dx
2 sin2

x
1
2
cos ec cot dx
2
2

Example :

n (nx) + (nx)

Solution.

put

x = et

= ex cot

dx

nt + 2 dt
t

x
+c
2

1 1 1

nt + + 2 dt
t t t

x n ( nx )
+c
nx


= et nt + c
t

Self Practice Problems

1.

x sin x dx

Ans.

x cosx + sin x + C

2.

x e

Ans.

x 2 ex 2xex + 2ex + C

6.

Integration of Rational Algebraic Functions by using Partial Fractions:

2 x

dx

PARTIAL FRACTIONS :
f ( x)
If f(x) and g(x) are two polynomials, then g( x ) defines a rational algebraic function of a rational function
of x.
f(x)
If degree of f(x) < degree of g(x), then g( x ) is called a proper rational function.
f(x)
If degree of f(x) degree of g(x) then g( x ) is called an improper rational function
f ( x)
f ( x)
If g( x ) is an improper rational function, we divide f(x) by g(x) so that the rational function g( x ) is
( x )
expressed in the form (x) + g( x ) where (x) and (x) are polynomials such that the degree of (x) is
f ( x)
less than that of g(x). Thus, g( x ) is expressible as the sum of a polynomial and a proper rational

function.
f ( x)
Any proper rational function g( x ) can be expressed as the sum of rational functions, each having a

simple factor of g(x). Each such fraction is called a partial fraction and the process of obtained them is
f ( x)
called the resolutions or decomposition of g( x ) into partial fractions.
f(x)
The resolution of g( x ) into partial fractions depends mainly upon the nature of the factors of g(x) as

discussed below.
CASE I When denominator is expressible as the product of non-repeating linear factors.
Let g(x) = (x a1) (x a2) .....(x an ). Then, we assume that
A1
A2
An
f(x)
=
+
+
.....
+
g( x )
x a1
x a2
x an

where A1, A2, ...... An are constants and can be determined by equating the numerator on R.H.S. to the

10

numerator on L.H.S. and then substituting x = a1, a2, ........,an .


3x + 2

Example :

Resolve

Solution.

We have,

x 6x 2 + 11x 6
3

into partial fractions.

3x + 2
x 3 6 x 2 + 11x 6

3x + 2
( x 1)( x 2)( x 3)

Let

3x + 2
B
A
B
+
+
=
. Then,
( x 1)( x 2)( x 3)
x3
x 1
x2

3x + 2
A( x 2)( x 3) + B( x 1)( x 3) + C( x 1)( x 2)
=
( x 1)( x 2)( x 3)
( x 1)( x 2)( x 3)

3x + 2 = A(x 2) (x 3) + B (x 1) (x 3) + C(x 1) (x 2)
Putting x 1 = 0 or x = 1 in (i), we get

5 = A(1 2) (1 3) A =

...........(i)

5
,
2

Putting x 2 = 0 or, x = 2 in (i), we obtain


8 = B (2 1) (2 3) B = 8.
Putting x 3 = 0 or, x = 3 in (i), we obtain
11 = C (3 1) (3 2) C =
3x + 2

x 6 x + 11x 6
3

11
.
2

3x + 2
5
11
8
=

+
( x 1)( x 2)( x 3)
2( x 1)
2
(
x
3)
x2

Note : In order to determine the value of constants in the numerator of the partial fraction corresponding to the
non-repeated linear factor px + q in the denominator of a rational expression, we may proceed as
follows :

Replace x =

q
(obtained by putting px + q = 0) everywhere in the given rational expression except in
p

the factor px + q itself. For example, in the above illustration the value of A is obtained by replacing x
by 1 in all factors of

A=

3x + 2
except (x 1) i.e.
( x 1)( x 2)( x 3)

3 1+ 2
5
=
(1 2)(1 3)
2

Similarly, we have
B=

3 2 +1
33 + 2
11
= 8 and, C =
=
(1 2)(2 3)
(3 1)(3 2)
2

x 3 6 x 2 + 10 x 2

into partial fractions.

Example :

Resolve

Solution.

Here the given function is an improper rational function. On dividing we get

x 2 5x + 6

x 3 6 x 2 + 10 x 2
x 5x + 6
2

=x1+

( x + 4)

...........(i)

( x 5 x + 6)
2

11

we have,

So, let

x + 4

x 5x + 6
2

x + 4
( x 2)( x 3)

x + 4
B
A
=
+
x + 4 = A(x 3) + B(x 2)
( x 2)( x 3)
x3
x2

...........(ii)

Putting x 3 = 0 or, x = 3 in (ii), we get


1 = B(1) B = 1.
Putting x 2 = 0 or, x = 2 in (ii), we get
2 = A (2 3) A = 2
x + 4
1
2
=
+
( x 2)( x 3)
x3
x2

x 3 6 x 2 + 10 x 2

Hence

=x1

x 2 5x + 6

2
2
+
x3
x2

CASE II When the denominator g(x) is expressible as the product of the linear factors such that some
of them are repeating.
1
1
Example g( x ) =
this can be expressed as
( x a)k ( x a1 )( x a 2 ).......( x a r )

A3
A2
Ak
B1
B2
Br
A1
+
2 +
3 + ....+
k + ( x a ) + ( x a ) + ...... + ( x a )
xa
( x a)
( x a)
( x a)
1
2
r

Now to determine constants we equate numerators on both sides. Some of the constants are determined
by substitution as in case I and remaining are obtained by
The following example illustrate the procedure.

Example :

Resolve

Solution.

Let

3x 2
( x 1)2 ( x + 1)( x + 2)
3x 2

( x 1) ( x + 1)( x + 2)
2

into partial fractions, and evaluate

(3x 2)dx

(x 1) ( x + 1)(x + 2)
2

A2
A3
A1
A4
+
+
+
( x 1)2
x 1
x +1
x+2

3x 2 = A1 (x 1) (x + 1) (x + 2) + A2 (x + 1) (x + 2)
+ A3 (x 1)2 (x + 2) + A4 (x 1)2 (x + 1) .......(i)
Putting x 1 = 0 or, x = 1 in (i) we get
1
1 = A2 (1 + 1) (1 + 2) A2 =
6
Putting x + 1 = 0 or, x = 1 in (i) we get
5
5 = A3 (2)2 (1 + 2) A3 =
4
Putting x + 2 = 0 or, x = 2 in (i) we get
8
8 = A4 (3)2 (1) A4 =
9
Now equating coefficient of x 3 on both sides, we get 0 = A1 + A3 + A4

8
13
5

=
9
36
4

A1 = A3 A4 =
3x 2

( x 1)2 ( x + 1)( x + 2)

and hence

1
13
5
8
+

+
36( x 1)
4( x + 1)
9( x + 2)
6( x 1)2

(3 x 2)dx

( x 1) ( x + 1)( x + 2)
2

1
13
8
5
n |x 1|

n |x + 1| +
n |x + 2| + c
6( x 1)
36
9
4
12

CASE III When some of the factors of denominator g(x) are quadratic but non-repeating. Corresponding

Ax + B

to each quadratic factor ax 2 + bx + c, we assume partial fraction of the type

, where A and
ax + bx + c
B are constants to be determined by comparing coefficients of similar powers of x in the numerator of

both sides. In practice it is advisable to assume partial fractions of the type

A( 2ax + b)
ax + bx + c
2

B
ax + bx + c
2

The following example illustrates the procedure

Example :

Resolve

Solution.

Let

2x 1

into partial fractions and evaluate

( x + 1)( x + 2)
2

2x 1
( x + 1)( x 2 + 2)
2x 1
( x + 1)( x 2 + 2)

2x 1

( x + 1)( x

+ 2)

dx

Bx + C
A
+ 2
. Then,
x +1
x +2

A( x 2 + 2) + (Bx + C)( x + 1)
( x + 1)( x 2 + 2)

2x 1 = A (x 2 + 2) + (Bx + C) (x + 1)
...(i)
Putting x + 1 = 0 or, x = 1 in (i), we get 3 = A(3) A = 1.
Comparing coefficients of the like powers of x on both sides of (i), we get
A + B = 0, C + 2A = 1 and C + B = 2

1 + B = 0, C 2 = 1 (Putting A = 1)

B = 1, C = 1
2x 1

( x + 1)( x 2 + 2)

Hence

2x 1

( x + 1)(x

+ 2)

x +1
1
+ 2
x +1
x +2

dx

= n |x + 1| +

1
x
1
n |x 2 + 1| +
tan1
+c
2
2
2

CASE IV When some of the factors of the denominator g(x) are quadratic and repeating fractions of the
A ( 2ax + b)
A2
A
A 0 (2ax + b )

1
+
+ 2 1
+ 2
form 2
2
2
ax + bx + c ax + bx + c
ax + bx + c
ax + bx + c

) (

A 2k
2k 1( 2ax + b)
+
+ .......+ 2
k
k
ax + bx + c
ax 2 + bx + c

) (

The following example illustrates the procedure.


Example:

Resolve

Solution.

Let

2x 3
( x 1)( x 2 + 1)2
2x 3

( x 1)( x + 1)
2

into partial fractions.

Dx + E
Bx + C
A
+ 2
+ 2
. Then,
( x + 1)2
x 1
x +1

2x 3 = A(x 2 + 1)2 + (Bx + C) (x 1) (x 2 + 1) + (Dx + E) (x 1)


Putting x = 1 in (i), we get 1 = A (1 + 1)2 A =

13

1
4

......(i)

Equation coefficients of like powers of x, we have


A + B = 0, C B = 0, 2A + B C + D = 0, C + E B D = 2 and A C E = 3.
Putting A =
B=

1
and solving these equations, we get
4

1
1
5
= C, D =
and E =
4
2
2
2x 3

( x 1)( x 2 + 1)2

2x

Example :

Resolve

Solution.

We have,

So, let

x3 1

x +1
x+5
1
+
+
4( x 1)
4( x 2 + 1)
2( x 2 + 1)2

into partial fractions.

2x
x 1
3

2x
( x 1)( x 2 + x + 1)

2x
( x 1)( x + x + 1)
2

Bx + C
A
. Then,
+ 2
x 1
x + x +1

2x = A (x 2 + x + 1) + (Bx + C) (x 1)

.......(i)

Putting x 1 = 0 or, x = 1 in (i), we get 2 = 3 A A =


Putting x = 0 in (i), we get A C = 0 C = A =

2
3

2
3

Putting x = 1 in (i), we get 2 = A + 2B 2 C.

2=

2
4
2
+ 2B
B=
3
3
3

2x

x3 1

2 / 3 x + 2 / 3
2x
2
2
2
1
1
+
or,, 3
=
+
.
3
3 x 1
3
x 1
x2 + x + 1
x 1

1 x
x2 + x + 1

Self Practice Problems


1.

7.

(i)

( x + 2)(x + 3) dx

(ii)

( x + 1)(x

dx

+ 1)

Int eg ra t i o n o f t y pe

x+2
+C
x+3

Ans.

n

Ans.

1
1
1
n |x + 1|
n (x 2 + 1) +
tan1 (x) + C
2
4
2

dx

ax 2 +bx +c ,

dx
2

ax +bx +c

ax 2 +bx + c

dx

Express ax 2 + bx + c in the form of perfect square & then apply the standard results.
Example :

Evaluate :

Solution.

We have,

x 2 + 2x + 5 dx

x 2 + 2x + 5

x 2 + 2x + 1 + 4 dx

14

1
(x + 1)
2

( x 1)2 + 2 2 +

1
(x + 1)
2

x 2 2x + 5 + 2 log |(x + 1) +

Example :

Evaluate :

Solution.

x
=

( x + 1)2 + 2 2 | + C

x 2 + 2x + 5 | + C

dx

x +1

dx

x +1

1
2

1
. (2)2 log |(x + 1) +
2

1
dx
1 1
x + +1
4 4
1

( x 1/ 2)

( x 1/ 2) + (

+ 3/4

dx

3 /2

x 1/ 2
+C
tan1

3 /2
3 /2
1

dx =

2x 1
+ C.
tan1

3
3
1
Evaluate :
dx
9 + 8x x 2
1
dx
9 + 8x x 2
2

Example :

Solution.

1
{ x 8 x 9}
2

dx

1
{ x 8 x + 16 25}
2

{( x 4)

5 }
2

dx =

dx

x4
+C
dx = sin1
5

5 ( x 4)2
2

Self Practice Problems


1.

2.

8.

2x

1
2

dx

+ x 1

Ans.

1
2x + 3 x 2
2

dx

Ans.

Int eg ra t i o n
px + q

ax 2 +bx +c

dx,

of

1
n
3

2x 1
2x + 2

+C

3
3

2
log x + 4 + x + 2 x 1 + C

t y pe

px +q
2

ax +bx +c

dx,

(px + q)

ax 2 + bx + c dx

Express px + q = A (differential coefficient of denominator) + B.

Example :

Evaluate :

Solution.

2x + 3
x + 4x + 1
2

dx

2x + 3
x + 4x + 1
2

dx
15

( 2x + 4 ) 1
x 2 + 4x + 1

dx

2x + 4
x + 4x + 1
2

dt

dx
1

( x + 2) 2

t log | (x + 2) +

=2

1
x + 4x + 1
2

( 3)

dx, where t = x 2 + 4x + 1

x 2 + 4x + 1 | + C

= 2 x 2 + 4 x + 1 log | x + 2 +

( x 5)

Example :

Evaluate :

Solution.

Let (x 5) = .

dx

x 2 + 4x + 1 | + C

x 2 + x dx

d
(x 2 + x) + . Then,
dx
x 5 = (2x + 1) + .
Comparing coefficients of like powers of x, we get
1
11
1 = 2 and + = 5 =
and =
2
2

( x 5)

x 2 + x dx

11

2 (2x + 1) 2

2 (2x + 1)

x 2 + x dx

11
2

x 2 + x dx

1
2

x 2 + x dx

11
2

x 2 + x dx

1
=
2

(2x + 1)

11
t dt
2

1
11
t3 / 2
.
=

2
2
3/2

x 2 + x dx

1
x +
2

dx where t = x 2 + x

2
2
1 x + 1 x + 1 1

2
2
2 2

2
2
2
x + 1 + x + 1 1
1 1

.
log
2
2 2 + C

2 2

2x + 1 2
1
1

x + x n x + + x 2 + x

8
2

1 3/2 11
t
3
2

1 2
11 2x + 1 x 2 + x 1 n

(x + x)3/2
8
3
2 4

+C

x + + x2 + x + C
2

Self Practice Problems


1.

2.

x +1
2

+x+3

dx

Ans.

1
log |x 2 + x + 3| +
2

Ans.

6x 5
3 x 2 5x + 1

dx

3x 2 5x + 1 + C
16

2x + 1
+ C
tan1
11
11
1

3.

(x 1)

1 + x + x 2 dx

Ans.

1
3
(x 2 + x + 1) 3/2
(2x + 1)
3
8

9
2
1 + x + x 2 16 log (2x +1 + 2 x + x + 1 ) + C

9.

Integration of trigonometric functions

(i)

dx
OR
a + b sin 2 x

dx
a + bcos 2 x

OR

dx
a sin x + b sin x cos x + c cos 2 x

dx
a + b sin x + c cos x

Multiply Nr & Dr by sec x & put tan x = t.

(ii)

dx
a + bsinx

dx
OR
a + bcosx

OR

Hint:
Convert sines & cosines into their respective tangents of half the angles and then,
put tan

(iii)

x
=t
2

a.cosx +b.sinx +c
d
dx. Express Nr A(Dr) + B
(Dr) + c & proceed.
.cosx +m.sinx +n
dx

Example :

Evaluate :

Solution.

1 + sin x + cos x

=
=

dx

dx

1
2 tan x / 2

1+

1 + tan2 x / 2

1 + tan

Putting tan
=

1 + sin x + cos x

1 tan2 x / 2 dx
1 + tan 2 x / 2

1 + tan 2 x / 2
2

x / 2 + 2 tan x / 2 + 1 tan x / 2
2

Evaluate :

Solution.

sec 2 x / 2
dx
2 + 2 tan x / 2

x
1
x
= t and
sec2
dx = dt, we get
2
2
2

t + 1 dt = log | t + 1| + C = log

Example :

dx =

3 sin x + 2 cos x

3 cos x + 2 sin x

3 sin x + 2 cos x

3 cos x + 2 sin x

tan

x
+1 + C
2

dx

dx

Let 3 sin x + 2 cos x = .

d
(3 cos x + 2 sin x ) + (3 cos x + 2 sin x)
dx

3 sin x + 2 cos x = (3 sin x + 2 cos x) + (3 cos x + 2 sin x )


Comparing the coefficients of sin x and cos x on both sides, we get

3 + 2 = 3 and 2 + 3 = 2

12
5
and =
13
13

( 3 sin x + 2 cos x ) + (3 cos x + 2 sin x )


dx
3 cos x + 2 sin x
17

1 . dx

3 sin x + 2 cos x

3 cos x + 2 sin x

=x+

dt
, where t = 3 cos x + 2 sin x
t
5
12
x+
n | 3 cos x + 2 sin x | + C
13
13

= x + n | t | + C =
Example :

Evaluate :

Solution.

We have,
=

dx

3 cos x + 2

sin x + 2 cos x + 3

3 cos x + 2

sin x + 2 cos x + 3

dx

dx

Let 3 cos x + 2 = (sin x + 2 cos x + 3) + (cos x 2 sin x) +


Comparing the coefficients of sin x, cos x and constant term on both sides, we get
2 = 0, 2 + = 3, 3 + = 2

6
3
8
,
and =
5
5
5

= x + log | sin x + 2 cos x + 3 | + 1, where

1 =

(sin x + 2 cos x + 3) + (cos x 2 sin x ) +


dx
sin x + 2 cos x + 3
cos x 2 sin x

dx + sin x + 2 cos x + 3
1

sin x + 2 cos x + 3

Putting, sin x =

1 =

2 tan x / 2
1 + tan x / 2

2 tan x / 2
1 + tan2 x / 2

, cos x =

2(1 tan2 x / 2)
1 + tan2 x / 2

1 tan 2 x / 2
1 + tan 2 x / 2

2 tan x / 2 + 2 2 tan
tan

=2

x / 2 + 2 tan x / 2 + 5

we get

dx

x / 2 + 3(1 + tan2 x / 2)

sec 2 x / 2
2

dx

+3

1 + tan2 x / 2

dx

dx

x
1
x
= t and
sec 2
= dt or
2
2
2

Putting tan
1 =

sin x + 2 cos x + 3

dx

dx +

sec 2

x
dx = 2 dt, we get
2

2dt
2

+ 2t + 5
dt

(t + 1)

+ 22

tan + 1
t + 1
2
2

= tan1
=
tan1
2

2
2

Hence, = x + log | sin x + 2 cos x + 3 | + tan1

18

tan + 1
2

+C
2

6
3
8
,=
and =
5
5
5

where =
dx

Example :

1 + 3 cos

Solution.

tan x
1
+C
tan1
2
2

sec 2 x dx
tan 2 x + 4

Self Practice Problems


1.

4 sin x + 5 cos x

5 sin x + 4 cos x

dx

Ans.
m

sin

1 0 . Integration of type

40
9
x+
log |5sinx + 4cosx| + C
41
41

x. cos n x dx

Case -
If m and n are even natural number then converts higher power into higher angles.
Case -
If at least m or n is odd natural number then if m is odd put cosx = t and vice-versa.
Case -
When m + n is a negative even integer then put tan x = t.
Example:

sin

Solution.

put

x cos 4 x dx
cos x = t

(1 t

(t

2t 2 + 1) t 4 dt

(t

2t 6 + t 4 ) dt

t9
t5
2t 7
+

+c
9
5
7

cos 9 x
cos 5 x
cos7 x
+2

+c
9
5
7

Example :

(sin x)

(cos x ) 7 / 3 dx

Solution.

(sin x)

(cos x ) 7 / 3 dx

1/ 3

1/ 3

(tan x)

1/ 3

cos 2 x
tanx = t

put
=

3 4/3
t +c
4

3
(tanx) 4/3 + c
4

1/ 3

sinx dx = dt
2 2

) . t 4 . dt

dx

sec2x dx = dt

dt

Ans.

19

Ans.

Example :

sin

Solution.

1
8

sin
1
8

2x(1 + cos 2x )dx

sin

1
=
16

x cos 4 x dx

1
8

2x dx +

sin

(1 cos 4 x ) dx +

2x cos 2x dx

sin3 2x

1
16

1
sin 4 x
sin 3 2x

+
+c
16
64
48

1 1 . Integration of type

x2 1
dx where K is any constant.
x 4 + Kx 2 + 1

Divide Nr & Dr by x & put x

Example :

Solution.

1 x2

1
= t.
x

dx

1+ x2 + x4

1 2 dx
x

1
x2 + 2 + 1
x

1
n
2

x+

1
=t
x

dt
t 1
2

t 1
t +1

+C

1
1
x
+C
1
x + +1
x
x+

1
n
2

Example :

Evaluate :

Solution.

We have,
1
=
dx
x4 + 1

+1

dx

x2
2

dx

x2
2

1
=
2

x2
2

dx

x2
20

Putting x

1
2

1
1 dx 2
+ 2
x
x

1
1
x2
dx
2
2
1

x + 2
x

1+

x2
1 dx
+ 2
x

x2
dx
2
1
x
+

1
1
= u in 1st integral and x +
= in 2nd integral, we get
x
x
du

u + ( 2)

1
2

1
1
1 2
x2
x
1
1 dx
x2 + 2 x2 + 2
x
x
1+

1+

1
=
2

1
=
2

1
2

( 2)

u
2
1
1
tan1
log
+C
+ 2
2 2
2 2 2
2
1

x 1/ x
x + 1/ x 2
1

tan1
log
+C
x + 1/ x + 2
2
2 2
4 2

x 2 1
x2 2 x + 1
1

tan1
log
+C

x2 + x 2 + 1
2 2
4 2
2x

Self Practice Problem :

x 1

1.

x 7x + 1

2.

tan x dx

1
3
x
1
x+ +3
x
x+

dx

Ans.

1
n
6

y
1
+
tan1
n
2
2 2
2

Ans.

1 2 . Int eg ra t i o n
dx

(ax + b) px + q

+C

of

t y pe

OR

(ax 2 +bx + c )

dx

px +q

y 2
y+ 2

+ C where y = tan x

; put px + q = t2.

Example:

Evaluate :

Solution.

Let =

( x 3)

x +1

dx

( x 3)

x +1

dx

Here, P and Q both are linear, so we put Q = t 2 i.e. x + 1 = t 2 and dx = 2t dt

(t

1
2

2t

1 3) t 2

dt

21

1
tan x

=2

Example :

Evaluate :

Solution.

Let =

(x

dt

x +1 2

1
log
2

(x

t2
1
log
+C
t+2
2( 2)

=2.

x +1+ 2

+ C.

x+2
+ 3x + 3) x + 1

dx

x+2
+ 3x + 3) x + 1

dx

{(t

Putting x + 1 = t 2, and dx = 2t dt, we get =

=2

=2

( t 2 + 1)
4

+ t2 + 1

dt = 2

1+
2

du

u + ( 3)

2
3

tan1

where t

( t 2 + 1) 2t dt
2

1)2 + 3( t 2 1) + 3} t 2

1
t2
1

t2

+1

dt

1
= u.
t

u
2

tan1
3 +C=
3

1
t t

3 +C

t 2 1
2

tan1
tan1
+C=

3
3
t 3

+C
3 ( x + 1)

1 3 . Integration of type

(ax

dx

+ b) px 2 + qx + r

, put ax + b =

dx

(ax 2 + b) px 2 +q , put x =

dx

Example :

(x + 1 ) x 2 + x + 1

Solution

dt
1
t2
t

1
1
1 +
t
t

dt
t t +1
2

dt

1 1
+1
t2 t

dt
1 3
t +
2 4

22

1
1
3

= n t + t + + C
4
2
2

dx

Example :

(1 + x

Solution.

Put x =

) 1 x 2

1
t

put

t 2 1 = y2

(y

(t

y dy
2

+ 2) y

1 x2

tan1
2x
2

dt
2

+ 1) t 2 1

1
2

y
+ C
tan1
2

+C

Self Practice Problems :


dx

1.

( x + 2) x + 1

2.

( x + 5x + 6) x + 1

3.

4.

5.

Ans.

2 tan1

Ans.

2 tan1

dx
2

dx

Ans.

( x + 1) 1 + x x 2

sin1

dx
(2x 2 + 1) 1 x 2

1 4 . Int eg ra t i o n

of

x +1 + C

x +1

x +1

1
+C
2 tan
2

1
2 x +1

+C
5

1 x 2

3 x2 + C

Ans.

x 2 + 2x 4 6 ( x + 1)

n 2
+C
2 6
x
+
2
x

4
+
6
(
x
+
1
)

tan

Ans.

dx
( x 2 + 2 x + 2) x 2 + 2 x 4

t y pe

x
dx or
x

(x )( x ) ;

put x = cos2 + sin 2

x
dx or
x

(x )(x ) ;

put x = sec2 tan 2

dx
;
(x )(x )

put x = t2 or x = t2.

23

1 5 . Red u c t io n

of

x tann2 x dx =

(sec

fo rmu l a

tann x dx ,

cot

x dx ,

cosec x dx
n

1.

2.

3.

n =

tan

n =

sec

n =

tann1 x
n 2
n 1

n =

cot

n =

n =

sec

x 1) tann 2x dx

x tann2 + dx n 2

cot

cos ec

n =

x dx =

. cot n2 x dx =

(cos ec x 1) cot
2

n 2

x dx

x cot n2 x dx n 2

cot n1 x
n 2
n 1

x dx =

sec

x sec n2 x dx

(tan x)(n 2) sec

n = tanx sec n 2x

n = tanx sec n 2 x dx (n 2) (sec2 x 1) sec n 2x dx


(n 1) n = tanx sec n 2x + (n 2) n 2
n =

4.

tan

x dx =

n =

n3

x. secx tanx dx.

n2
tan x sec n 2 x
+

n 1 n 2
n 1

cos ec dx = cos ec
n

x cosecn 2 x dx

(cot x)(n 2) ( cosec

n = cotx cosecn 2x +

cotx cosecn 2x (n 2)

n = cotx cosecn 2x (n 2)

(n 1) n = cotx cosecn 2 x + (n 2) 2n 2
n =

cot

n3

x cosec x cot x) dx

x cos ec n 2 x dx

(cos ec

x 1) cosec n 2 x dx

n2
cot x cos ec n2 x
+

n 1 n 2
n 1

Example :

Obtain reducation formula for n =

Solution.

n =

(sin x) (sin x)

n 1

sin x dx. Hence evaluate sin x dx


n

dx

= cos x (sin x)n1 + (n 1)


= cos x (sin x)n1 + (n 1)

(sin x)
(sin x)

n2

n 2

cos2x dx
(1 sin2x) dx

n = cos x (sin x)n1 + (n 1) n2 (n 1) n

24

sec

x dx ,

(n 1)
cos x(sin x )n1
+
n2
n
n

n =

Hence 4 =

(n 2)

3 cos x(sin x ) + 1 x
cos x(sin x )3

+C
+
2
2
4
4

Self Practice Problems :


1.

2.

x3
dx
x4
dx
[( x 1)(2 x )]

3/2

dx

3.

4.

Deduce the reduction formula for n =

[( x + 2)8 ( x 1)6 ]1/ 7

Ans.

n =

( x 3)( x 4 ) + n

Ans.

x
4(n 1)(1 + x 4 )n1

x 1
2 x

8 2 x x 1 + C

Ans.

x 1

7
x +2

dx
(1 + x 4 )n

1/ 7

+C

and Hence evaluate 2 =

m,n =

(sin x)

m,n =

n 1
(sin x )m+1(cos x )n1
+
.
m + n m,n2
m+n

4n 5

4(n 1) n1

x
x+
3 1
1
1
x

tan
n

2 =
+
4
4 (1 + x )
4 2 2
2 4 2

x+

If

x 3 + x 4 + C

Ans.

5.

(cos x)n dx then prove that

25

2
x
+ C
1

+ 2
x

dx
(1 + x 4 )2

PART A :
A Let f(x) be a continuous function defined on [a, b],

f ( x ) dx = F(x) + c. Then

f (x) dx = F(b) F(a) is called definite integral. This formula is known as Newtona

Leibnitz formula.
Note :
b

1.

The indefinite integral

f ( x ) dx is a function of x, where as definite integral

f ( x) dx is a number..
a

2.

Given

f ( x ) dx we can find

f ( x ) dx , but given

a
2

Illustration. 1

Evaluate

Sol.

dx
( x + 1)( x + 2)

1
1
1

=
( x + 1)( x + 2)
x +1
x+2
2

f (x) dx we cannot find f ( x) dx

(by partial fractions)

dx
2
( x + 1)( x + 2) = [log e ( x + 1) log e ( x + 2)]1
9

= loge3 loge4 loge2 + loge3 = loge 8


Self Practice Problems

Evaluate the following


2

1.

5x 2

dx

x + 4x + 3
2

2.

(2 sec

Ans.

x + x 3 + 2 dx Ans.

3.

x
dx
1 + sec x

Ans.

5
2

5
3

9 log e 4 log e 2

+
+2
1024
2

+ 2 loge
3 3
18

PART B :
Properties of definite integral
b

P1

i.e.

definite integral is independent of variable of integration.

f ( x ) dx =

f (t ) dt
a

P2

f ( x ) dx =

f (x) dx
b

26

Indefinite & Definite Itegration 26 of 89

Definite Integrals

P3

f ( x ) dx =

f ( x ) dx +

f (x ) dx
2

f ( x) dx

f ( x ) dx =

f ( x) dx, where c may lie inside or outside the interval [a, b].

x+3 : x <3
If f(x) = 2
, then find
3 x + 1 : x 3

Illustration 2

Sol.

f ( x ) dx +

(3 x

( x + 3 ) dx +

+ 1) dx

94
+ 3 (3 2) + 53 33 + 5 3
2

211
2

| x 5 | dx = (x + 5) dx + ( x + 5) dx

=9

Evaluate | x 5 | dx

Illustration 3

Sol.

Show that

Illustration 4

(2x + 1) dx =

Sol.

( 2x + 1) +

(2x + 1)
5

L.H.S. = x2 + x ]20 = 4 + 2 = 6
R.H.S. = 25 + 5 0 + (4 + 2) (25 + 5) = 6

L.H.S. R.H.S

Self Practice Problems

Evaluate the following


2

| x

1.

+ 2x 3 | dx

Ans.

Ans.

Ans.

13

0
3

[ x] dx , where [x] is integral part of x.

2.

[ t ] dt
9

3.

PART C :
a

P4

f ( x ) dx =

(f ( x) + f ( x)) dx
0
a

=2

f ( x) dx

if f(x) = f(x) i.e. f(x) is even

=0

if f(x) = f (x) i.e. f(x) is odd

27

Indefinite & Definite Itegration 27 of 89

ex + ex
1+ ex

1
1

Sol.

ex + ex
1+ ex

1
1

dx =

dx

e x + ex e x + e x

+
1+ ex
1 + ex

e x + e x e x (e x + e x )

+
1+ ex
dx =
ex + 1

dx

(e x + e x ) dx = e 1 +

e2 1
(e 1 1)
=
e
1

Illustration 6

Evaluate

cos x dx

Sol.

cos x dx = 2 cos x

Illustration 7

Evaluate

log

Sol.

dx = 2

( cos x is even function)

2 x
dx
2+x

2x

Let f(x) = loge


2+x

i.e.

2+x
= log
f(x) = loge
e
2x
f(x) is odd function
1

log

2x

= f(x)
2+ x

2 x
dx = 0
2+x

Self Practice Problems

Evaluate the following


1

1.

| x | dx

Ans.

Ans.

Ans.

2.

sin

3.

x dx

cos x
1+ ex

dx

PART D :

P5

f ( x) dx = f (a + b x) dx
a

Further

f ( x ) dx =

f (a x) dx
0

28

Indefinite & Definite Itegration 28 of 89

Evaluate

Illustration 5

Illustration 8

Prove that

Sol.

Let =

g (cos x )
dx =
4
g (sin x ) + g (cos x )

g (sin x )
dx
g (sin x ) + g (cos x )

g sin x

2
=

g sin x + g cos x

g (sin x )
dx =
g (sin x ) + g (cos x )

Indefinite & Definite Itegration 29 of 89

g (cosx)

g (cosx) + g (sinx)

dx

on adding, we obtain

2 =

Note : 1.
2.

g (sin x )
g (cos x )

dx =
+
g (sin x ) + g (cos x ) g (cos x ) + g (sin x )

The above illustration can be remembered as a formula


Other similar formulae are

0
a

g (tan x )
dx =
g (tan x ) + g (cot x )

g (cos ecx )
dx =
g (cos ecx ) + g (sec x )

g (cot x )
dx =
4
g (tan x ) + g (cot x )

g (sec x )
dx =
4
g (cos ecx ) + g (sec x )

g (x)
a
g ( x ) + g (a x ) dx = 2

Self Practice Problems

Evaluate the following

1.

1 + sin x

dx

Ans.

2.

sin x + cos x

dx

Ans.

3.

x sin x cos x

sin
0

4.

dx

1+

x + cos 4 x

dx

Ans.

2
16

Ans.

12

dx
tan x

2 2

loge 1 + 2

PART E :
29

P6

f ( x ) dx

(f (x) + f (2a x)) dx

0
a

=2

f ( x) dx

if f (2a x) = f(x)

=0

if f (2a x) = f(x)

Illustration 9

Evaluate

sin

x cos 3 x dx

Sol.

f( x) = f(x)

Let f(x) = sin x cos3x

sin

x cos 3 x dx = 0

Illustration 10 Evaluate

dx

1 + 2 sin

Sol.

1 + 2 sin2 x
f( x) = f(x)

dx

1 + 2 sin

=2

sec 2 x dx

1 + 3 tan

tan

dx

1 + 2 sin

=2

dx

Let f(x) =

2
3

=2

sec 2 x dx
1 + tan2 x + 2 tan2 x

[tan ( 3 tan x ) ]
1

2
0

is undefined, we take limit


2

1
1
Lt
tan
3
tan
x

tan
3
tan
0

3 x

=
3
2

2
3

Note : We can evaluate the integral without using this property

Alternatively :

dx

1 + 2 sin
0

cos ec 2 x
cos ec 2 x + 2

dx =

cos ec 2 x dx
cot 2 x + 3

Observe that we are not converting in terms of tan x as it is not continuous in (0, )
=

1 cot x
1
=
tan

3
3
3 0

cot x
1 cot x
Lt tan 1

Lt tan

3
+
x
3 x 0


2 2 =
3
3

Note : If we convert in terms of tan x, then we have to break integral using property P 3.

30

Indefinite & Definite Itegration 30 of 89

2a

log

Illustration 11 Prove that

log

sin x dx =

cos x dx =

log (sin 2x) dx = 2


e

Sol.

Let =

log

sin x dx

..........(i)

log sin 2 x dx

(by property P 5)

log (cos x) dx

..........(ii)

Adding (i) and (ii)

log (sin x . cos x) dx = log

2=

log (sin 2x) dx log

2=

sin 2x
dx
2

2
e

dx

2 = 1

log2e
2

..........(iii)

where

1 =

log (sin 2x) dx


e

dx =

L.L:x=0

U.L:x=
2

t=0

t=

1 =

log (sin t )
e

1
=
2
2

1
dt
2

put 2x = t

1 =

1
dt
2

log (sin t ) dt
e

(by using property P 6)

(iii) gives

log2e
2

Self Practice Problems

Evaluate the following

1.

2.

loge x +
x

1+ x2
dx

sin 1 x
dx
x

Ans :

loge2

Ans :

loge2
2

31

loge2 .

Indefinite & Definite Itegration 31 of 89

3.

x log e sin x dx

Ans :

2
loge2
2

PART F :

P7

If f(x) is a periodic function with period T, then


T

nT

(i)

f ( x) dx, n z

f ( x ) dx = n

0
a +nT

(ii)

f ( x) dx, n z, a R

f ( x ) dx = n

0
T

nT

(iii)

f ( x ) dx = (n m)

a +nT

(iv)

f ( x ) dx =

nT

f ( x) dx, n z, a R
0

b + nT

(v)

f ( x) dx, m, n z
0

mT

f ( x ) dx =

a + nT

f ( x) dx, n z, a, b R
a

Illustration 12 Evaluate

{ x}

dx

1
1+ 3

Sol.

e { x } dx =

e { x } dx = 3 e { x } dx = 3

n + v

| cos x | dx ,

Illustration 13 Evaluate

0
n + v

Sol.

| cos x | dx +

| cos x | dx

| cos x | dx

cos x dx + n

< v < and n z


2

cos x

dx = 3(e 1)

n + v

| cos x | dx =

{x}

= (1 0) (sin v 1) + 2n

cos x dx
0

= 2 sin v + 2n (1 0) = 2n + 2 sin v
Self Practice Problem

Evaluate the following


2

1.

{3 x }

dx

Ans.

32

3 (e 1)

Indefinite & Definite Itegration 32 of 89

dx

2.

5
4

3.

dx

1 + e sin x

sin 2x

sin 4 x + cos 4 x

dx

Ans.

1000

Ans.

Indefinite & Definite Itegration 33 of 89

2000

PART G :

P8

If (x) f(x) (x)


b

( x) dx

f ( x ) dx

then

( x ) dx

a x b,

for

P9

If m f(x) M for a x b, then m (b a)

f ( x) dx M (b a)
a
b

Further if f(x) is monotonically decreasing in (a, b) then f(b) (b a) <

f (x ) dx < f(a) (b a) and if f(x)


a

is monotonically increasing in (a, b) then f(a) (b a) <

f ( x) dx < f(b) (b a)
a

P 10

f ( x ) dx

f ( x ) dx

a
b

f (x) dx 0

P 11 If f(x) 0 on [a, b] then

Illustration 14 For x (0, 1) arrange f1(x) =

order and hence prove that


<
6

Sol.

4x

, f2(x) =

4x x
2

0<x <x

x < x + x < 2x
4 2x2 < 4 x2 x3 < 4 x2

4 2x 2 < 4 x 2 x 3 < 4 x 2
f1(x) < f3(x) < f2(x)
for x (0, 1)

f1( x ) dx <

sin

<
6

x
<
2 0
1

<

f ( x ) dx
2

0
1

dx
4 x 2 x3

dx
4x x
2

<

<

and f3(x) =

1
4 x2 x3

4 2

f3 ( x ) dx <

4 2x

dx

1
2

sin

2 0

4 2

33

2x2 < x2 x3 < x2

in ascending

Illustration 15 Estimate the value of

Let f(x) =

Sol.

f(x) =

sin x
dx
x

sin x
x

x cos x sin x

Lt

x 0 +

(cos x )( x tan x )

<0
x
x2
f(x) is monotonically decreasing function.
f(0) is not defined, so we evaluate
2

Lt + sin x = 1. Take f(0) = Lt + f(x) = 1


f(x) = x
x 0
0
x

2
f =

2
. 0 <
2

1<

sin x
dx < 1 . 0
2

sin x
dx <
2
x

Note : Here by making the use of graph we can make more appropriate approximation as in next illustration.
1

Illustration 16 Estimate the value of

x2

dx using (i) rectangle, (ii) triangle

Sol.

(i)

By using rectangle
1

Area OAED <

x2

dx < Area OABC

0
1

1<

x2

dx < 1 . e

1<

x2

dx < e

(ii) By using triangle


1

Area OAED <

x2

dx < Area OAED + Area of triangle DEB

0
1

x2
1
. 1. (e 1)
1 < e dx < 1 +
2
0

1<

e
0

34

x2

dx < e + 1
2

Indefinite & Definite Itegration 34 of 89

e x dx by using

e dx
x

For x (0, 1), e x < ex


2

Sol.

11<

e x dx <

e dx
x

1<

x2

dx < e 1

Exercise : Prove the following :


1

1.

e x cos 2 x dx <

0<

sin

n +1

x dx <

3.

cos 2 x dx

2.

x2

1
4

sin

x dx

<

x2 x

dx < 1

0
1

4.

x 3 cos x

dx <

2 + x2

5.

1<

sin x dx <

6.

0<

x dx

16 + x

<

1
2

1
6

PART - H
h( x )

Leibnitz Theorem :

If F(x) =

f (t ) dt , then
g( x )

dF( x )
= h(x) f(h(x)) g(x) f(g(x))
dx

35

Indefinite & Definite Itegration 35 of 89

Illustration 17 Estimate the value of

Indefinite & Definite Itegration 36 of 89

Proof :

f (t) dt

Let P(t) =

h( x )

f (t ) dt = P(h(x)) P(g(x))

F(x) =

g( x )

dF( x )
= P(h(x)) h(x) P(g(x)) g(x)
dx
= f(h(x)) h(x) f (g(x)) g(x)

x2

Illustration 18 If F(x) =

sin t dt , then find F(x)

Sol.

F(x) = 2x .

sin x 2 1 .

e3 x

Illustration 19 If F(x) =

dt , then find first and second derivative of F(x) with respect to logxe

log et

2x

sin x

at x = log2e
Sol.

dF( x )
dx
e3x
e 2x
dF( x )
3 . e 3 x .
2 . e2x
=
=
3
x
2x
d (log x )
d logex
dx

logee
logee

( )

d2F( x )
d

( )

2
logex

( )
logex

(e6x e4x) =

x = e6x e4x.

1
d
6x
4x
(e6x e4x)
x = (6 e 4 e ) x
d
log
dx
e
dx

First derivative of F(x) at x = log2e


(i.e. ex = 2) is 26 24 = 48
Second derivative of F(x) at x = log2e (i.e. ex = 2) is (6 . 26 4 . 24) . loge2 = 5 . 26 . log2e.
x 2
e t dt

Illustration 20 Evaluate xLt

2t 2

dt

x 2
e t dt

Lt 0

Sol.

form

2t 2

dt

Applying L Hospital rule


x

2 . e t dt . e x

Lt

1 . e 2x

2 . e t dt

Lt

ex

Lt

2 . ex

2x . e x

36

=0

Indefinite & Definite Itegration 37 of 89

Modified Leibnitz Theorem :


h( x )

If

f (x, t) dt, then

F(x) =

g( x )
h( x )

f ( x, t )
dt + f(x, h(x))h(x) f(x, g(x)) . g(x)
x

(x) =

g( x )

Illustration 21 If f(x) =

dt
x + t , then find f(x)

logex
x

Sol.

f(x) =

(x + t )2

logex

dt + 1 .

1
1

2x
x

1
1
1
1

x +
x
x
+
logex
2x
2x
x + log e

Alternatively : f(x) =

logex

(x + log )
x
e

dt
= log e ( x + t )
(treating t as constant)

x+t
log x
e

1
1

x
+
log ex
x

loge 1
1

1 + =
x
x x + logex

Definite Integrals dependent on parameters :


1

Illustration 23 Evaluate

0
1

Sol.

Let (b) =

d (b )
=
db

xb 1
log ex

, b being parameter

xb 1
log ex
x b log ex
log ex

dx

dx + 0 0

(using modified Leibnitz Theorem)


1

x b +1
1

x dx =
b + 1 = b + 1
0
b

(b) = loge (b + 1) + c
b=0
(0) = 0

c=0
(b) = loge (b+1)
1

Illustration 24 Evaluate

x +1
logex 1
1

x =
x x + loge
x x + logex
x

f(x) = loge2x loge (x + log xe)


f(x) =

1
1
1

= (x + t) x +
loge
x x + logex
2x

tan 1(ax )
x 1 x2

dx , a being parameter

37

Let (a) =

Sol.

tan 1(ax )

dx

x 1 x2

0
1

d ( a )
=
da

(1 + a x ) x 1 x
2 2

dx =

dx = cos t dt
t=0

U.L. : x = 1

t=

1
cos t dt =
2
2
cos
t
1 + a sin t

sec 2 t dt

1 + (1 + a
0

1
1+ a

(a) =

But (0) = 0
(a) =

dx
(1 + a x ) 1 x 2
2 2

Put x = sin t
L.L. : x = 0

d ( a )
=
da

Indefinite & Definite Itegration 38 of 89

) tan 2 t

1
1+ a2

1+ a

dt
2

sin 2 t

tan

1 + a 2 tan t 2

loge a + 1 + a + c

c=0

loge a + 1 + a

Self Practice Problems :


x3

1.

If f(x) =

cos t dt, find f(x).

Ans.

3x2

Ans.

2
17

Ans.

cos x 3

0
x

2.

g(x)

If f(x) = e

and g(x) =

1+ t

dt then find the value of f(2).

3.

If x =

dt
1 + 4t

and

d2 y
dx 2

= Ry then find R

x2

4.

If f(x) =

sin t dt then find f (x).

Ans.

x2 (2x sin x2 sin x) + (cos x cos x2) x

5.

If (x) = cos x

( x t ) (t) dt, then find the value of (x) + (x). Ans.

cos x

((log

6.

Find the value of the function f(x) = 1 + x +

t 2
e)

+ 2 log et dt where f (x) vanishes.

38

Ans. 1 +

2
e

cos t

7.

Evaluate xLt
0

dt

x sin x

Ans.

Ans.

1+ 1 b2

loge
2

8.

Evaluate log e (1 + b cos x ) dx, b being parameter..


0

PART -
Definite Integral as a Limit of Sum.
Let f(x) be a continuous real valued function defined on the closed interval [a, b] which is divided into n parts
as shown in figure.

y = f(x)

a a+h a+2h ................ a+(n-1)h a+nh=b

The point of division on x-axis are a, a + h, a + 2h ..........a + (n 1)h, a + nh, where

ba
= h.
n

Let Sn denotes the area of these n rectangles.


Then, Sn = hf(a) + hf(a + h) + hf(a + 2h) + ........+hf(a + (n 1)h)
Clearly, Sn is area very close to the area of the region bounded by curve y = f(x), xaxis and the ordinates
x = a, x = b.
b

Hence

f ( x) dx =

Lt

Sn

a
n1

n 1

f (x) dx =

h f (a + rh) =

Lt

r=0

Lt

b a
(b a ) r
a +

n f
n

n r =0

Note :
1.
We can also write
n

Sn = hf(a + h) + hf (a + 2h) + .........+ hf(a + nh) and

f (x) dx =
a

n 1

2.

If a = 0, b = 1,

f ( x ) dx = n Lt

Lt

ba
ba
a +
r
n f
n

n r =1

1 r

n f n

r =0

Steps to express the limit of sum as definte integral


Step 1. Replace

r
1
by
by x,
by dx and n Lt

n
n

r
by putting least and greatest values of r as lower and upper limits respectively..
Step 2. Evaluate n Lt

n
39

Indefinite & Definite Itegration 39 of 89

x2

r =1

1 r
f
n n =

r
r


( n Lt
= 0, n Lt
= p)
n
n
r =1
r = np

f( x) dx
0

Illustration 25 : Evaluate
Lt

Sol.

Lt

1
1
1
1
1 + n + 2 + n + 3 + n + ......... + 2n

1
1
1
1
1 + n + 2 + n + 3 + n + ......... + 2n

= n Lt

r +n

r =1
n

= n Lt

1
1
=
n r
+1
n

r =1

dx
1
x + 1 = [log e ( x + 1)]0 = loge2.

n+2
n+3
3
n +1
+ 2
+ 2
+ ......... +
Illustration 26 : Evaluate n Lt
2
2
n 2 + 12
5
n
n
2
n
+
+
3

2n

Lt

Sol.

n
r =1

Lt
+ r2 = n

r =1

1
n

r
n
2
r
1+
n
1+

Lt

r
= 0, when r = 1, lower limit = 0
n

Lt

r
=
n

and

n
2

2n

n+r

1+ x

1+ x

Lt

2n
= 2, when r = 2n, upper limit = 2
n

dx =

1+ x
0

1
dx +
2

2x

1+ x

dx

= tan1x]20 +
= tan1 2 +

log e (1 + x 2 )
2
0

1
loge5
2

Illustration 27 : Evaluate
1

Lt

n! n
n
n
1

Sol.

n! n
Let y = n Lt
n
n
1
loge
loge y = n Lt

n!
n
n

40

Indefinite & Definite Itegration 40 of 89

pn

For example n Lt

Indefinite & Definite Itegration 41 of 89

1 . 2 . 3 ........n
1

= n Lt
loge

nn

1 log 1 + log 2 + log 3 + ..... + log n


e
e
e
e

= n Lt

n
n
n
n
n
1
= n Lt

log

r =1

log e x dx = x log e x x

= (0 1) Lt + x logex + 0
x 0

= 1 0 = 1
1
e
Self Practice Problems :

y=

Evaluate the following limits


Lt

1
1
1
1
+
+
+ .... +

2
2
2
2
n
n +n
n + 2n
n + n2

Lt

Lt

1.

2.

3.

4.

5.

1
1
1
1
1 + n + 2 + n + 3 + n + ....... + 5n

Ans.

loge5

n
2
3
1 3
+ 2 sin 3
+ 3 sin 3
+ ........ + n sin 3
sin
2
4n
4n
4n
4n
n

Ans.

n r2
2

r=0

3
n

Lt

n
n
n
n
+
+
+ ...... +
1 +

n+3
n+6
n+9
n + 3(n 1)

PART J
Reduction Formulae in Definite Integrals

1.

If n =

sin
0

n 1

x dx , then show that n =
n n2

Proof : n =

( 2 1)

Ans.

n 1

Lt

sin

x dx

41

2
9 2

Ans.

Ans.

(52 15)

] + (n 1) sin

n 2

Indefinite & Definite Itegration 42 of 89

n = sinn1 x cos x

2
0

x . cos 2 x dx

= (n 1)

sin

n2

x . (1 sin2 x ) dx

= (n 1)

sin

n 2

x dx (n 1)

sin

x dx

n + (n 1) n = (n 1) n2
n 1

n =
n n2

sin

Note : 1.

x dx =

cos

x dx

n 1 n 3 n 5


..... or
n =
0
1
n n2 n4

2.

, =1
2 1

according as n is even or odd. 0 =

n 1 n 3 n 5
1
if

........ .

n
n

2
n

2 2

Hence n =
n 1 n 3 n 5
2

........ . 1 if

n
n

2
n

2.

If n =

tan

x dx , then show that n + n2 =

Sol.

n =

(tan x)

n 2

n is even

n is odd

1
n 1

. tan2x dx

(tan x)

n 2

(sec2x 1) dx

(tan x)

n 2

sec2x dx

(tan x)

n 2

dx

(tan x )n 1 4
n2
=
n 1 0

42

1
n2
n 1

n + n2 =

1
n 1

3.

If m,n =

sin

x . cos n x dx , then show that m,n =

Sol.

m,n =

sin

m 1

m 1
,n
m + n m2

x (sin x cos n x ) dx

sinm 1 x . cos n+1 x 2


+
=
n +1

m 1

=
n +1

m 1

=
n +1

sin

m2

cos n +1 x
(m 1) sinm2 x cos x dx
n +1

x . cos n x . cos 2 x dx

(sin

m2

x . cos n x sinm x . cos n x dx

m 1
m 1

=
m2,n
n +1
n + 1 m,n
m 1
m 1
=

1 +
m,n
n +1

n + 1 m2,n
m 1

m,n =
m + n m2,n

Note : 1.

m 1 m 3 m 5


........ or according as m is even or odd.
m,n =
0,n
1,n
m+n m+n2 m+n 4

0,n =

cos

x dx and 1,n =

2.

sin x . cos

x dx =

Wallis Formula

43

1
n +1

Indefinite & Definite Itegration 43 of 89

n =

sin

Illustration 28 : Evaluate

x cos 2 x(sin x + cos x ) dx

Sol.

sin

Given integral =

x cos 2 x dx +

sin

x cos 3 x dx

=0+2

sin

x cos 3 x dx

( sin3x cos2x is odd and sin2x cos3x is even)

1. 2
4
= 2. 5 . 3 . 1 =
15

Illustration 29 : Evaluate

x sin

x cos 6 x dx

Sol.

Let =

x sin

x cos 6 x dx

( x) sin ( x) cos
5

( x ) dx

x sin

sin 5 . cos 6 x dx

x . cos 6 x dx

sin

2 = . 2

x . cos 6 x dx

4 . 2 . 5 . 3 .1
= 11 . 9 . 7 . 5 . 3 . 1

8
693
1

Illustration 30 : Evaluate

(1 x )5 dx

Sol.

Put x = sin2

L.L
:x=0
U.L.

:x=1

dx = 2 sin cos d
=0
=

44

Indefinite & Definite Itegration 44 of 89

(n 1) (n 3) (m 5) .........(n 1) (n 3) (n 5).......
when both m, n are even

(m + n) (m + n 2) (m + n 4)........
2

m,n =

(m 1) (m 3) (m 5) .........( n 1) (n 3) (n 5).......
otherwise

(m + n) (m + n 2) (m + n 4)........

(1 x )5 dx =

sin

(cos 2 )5 2 . sin . cos d

=2.

sin

cos11 d

6 . 4 . 2 . 10 . 8 . 6 . 4 . 2
= 2 . 18 . 16 . 14 . 12 . 10 . 8 . 6 . 4 . 2

1
504

Self Practice Problems:

Evaluate the following

1.

sin

x dx

Ans.

18
15

x cos 4 x dx

Ans.

8
315

Ans.

16

245
14

dx

Ans.

a9
9

2 x dx

Ans.

2.

sin
0
1

3.

sin 1 x dx

(
a

4.

x a2 x 2

7
2

0
2

5.

x
0

3/2

45

Indefinite & Definite Itegration 45 of 89

KEY CONCEPTS
1.

DEFINITION :
If f & g are functions of x such that g(x) = f(x) then the function g is called a PRIMITIVE OR
ANTIDERIVATIVE OR INTEGRAL of f(x) w.r.t. x and is written symbolically as

2.
(i)

d
{g(x) + c} = f(x), where c is called the constant of integration.
dx

f(x) dx = g(x) + c

STANDARD RESULTS :

(iii)
(v)

(ax +

dx =

(ax + b) n +1
a ( n + 1)

(ii)

dx
1
= ln (ax + b) + c
ax + b
a

(iv)

apx+q dx =

1
cos (ax + b) + c
a

(vi)

cos (ax + b) dx =

1
sin (ax + b) + c
a

1
ln sec (ax + b) + c
a

(viii)

cot(ax + b) dx =

1
ln sin(ax + b)+ c
a

+ c n 1

1 ax+b
e
+c
a

eax+b dx =

sin (ax + b) dx =

(vii)

b)n

tan(ax + b) dx =

1 a px + q
(a > 0) + c
p n a

1
tan(ax + b) + c
(x) cosec(ax + b) dx = 1 cot(ax + b)+ c
a
a
1
(xi) sec (ax + b) . tan (ax + b) dx = sec (ax + b) + c
a
(xii) cosec (ax + b) . cot (ax + b) dx = 1 cosec (ax + b) + c
a

(ix)

sec (ax + b) dx =

ln tan + + c

(xiii)

secx dx = ln (secx + tanx) + c

OR

(xiv)

cosec x dx = ln (cosecx cotx) + c

OR ln tan

(xv)

sinh x dx = cosh x + c (xvi)

(xviii)
(xx)

(xxiv)

(xix)

(xxv)

(xxvi)

(xxviii)

dx
1
x
= tan1 + c
a
a 2 +x 2
a
x +a
2

dx
x a
2

a x
dx
2

(xxiii)

x x a

x
+c
a

= ln x + x 2 a 2

OR

cosh1

x
+c
a

(xxvii)

dx
a+x
1
ln
+c
2 =
ax
a x
2a
a 2 x 2 dx =

x
2

a 2 x 2 +

x
a2
sin1 + c
2
a

46

sinh1

sechx dx = tanh x + c

dx

OR

sech x . tanh x dx = sech x + c

= ln x + x 2 + a 2

x
+ c OR ln (cosecx + cotx)
2

(xxi)

cosechx dx = coth x + c

dx

cosh x dx = sinh x + c (xvii)

cosech x . coth x dx = cosech x + c

(xxii)

= sin1

x
+c
a

1
x
sec1 + c
a
a

x a
dx
1
=
ln
+c
2
x
+a
2a
x a
2

x
a2
sinh1 + c
2
a

(xxix)

x 2 + a 2 dx =

x
2

x 2 +a 2 +

(xxx)

x 2 a 2 dx =

x
2

x 2 a 2

(xxxi)

eax. sin bx dx =

(xxxii)

eax . cos bx dx =

3.
(i)

TECHNIQUES OF INTEGRATION :
Substitution or change of independent variable .

e ax
a 2 +b 2

x
a2
cosh1 + c
2
a

(a sin bx b cos bx) + c

e ax
(a cos bx + b sin bx) + c
a 2 +b 2

Integral I = f(x) dx is changed to f( (t)) f (t) dt , by a suitable substitution


x = (t) provided the later integral is easier to integrate .
(ii)

du

. v d x dx where u & v are differentiable

dx

function . Note : While using integration by parts, choose u & v such that

u.v dx = u v dx

du

. v d x dx is simple to integrate.

dx

This is generally obtained, by keeping the order of u & v as per the order of the letters in ILATE,
where ; I Inverse function, L Logarithmic function ,
A Algebraic function, T Trigonometric function & E Exponential function
Partial fraction , spiliting a bigger fraction into smaller fraction by known methods .

(a)

(iii)
4.

Integration by part :

(b)

v dx is simple &

INTEGRALS OF THE TYPE :

(i)

(ii)

ax2 + bx + c

[ f(x)]n f (x) dx
dx

dx

ax + bx + c
2

f ( x )

OR
,

[f ( x )]n

dx

put f(x) = t & proceed .

ax 2 + bx + c dx

Express ax2 + bx + c in the form of perfect square & then apply the standard results .
(iii)

px + q
dx ,
ax + bx + c
2

px + q

dx .

ax 2 + bx + c

Express px + q = A (differential co-efficient of denominator) + B .

(vi)

(vii)

(iv)

ex [f(x) + f (x)] dx = ex . f(x) + c


dx
dx

x x +1
2

(viii)

(ix)

n N , take xn common & put 1+xn = tn

( n 1)
n

dx

xn 1 + xn
dx
a + b sin

[f(x) + xf (x)] dx = x f(x) + c

Take xn common & put 1 + xn = t .

nN

x ( x n +1)

(v)

1/ n

x
r

take xn common as x and put 1 + x n = t .


OR

dx
a + b cos

OR

Multiply N. . & D. . by sec x & put tan x = t .

47

a sin

dx
x + b sin x cos x + c cos

(x)

dx
a + b sin x

dx
a + b cos x

OR

dx
a + b sin x + c cos x

OR

Hint :Convert sines & cosines into their respective tangents of half the angles , put tan
(xi)

(xii)

a .cos x + b .sin x + c
d
dx . Express Nr A(Dr) + B
(Dr) + c & proceed .
 .cos x + m .sin x + n
dx
x 2 +1
x 4 + K x 2 +1

Hint :
(xiii)

(xiv)

(xv)

dx OR

x 2 1
x 4 + K x 2 +1

dx

where K is any constant .

Divide Nr & Dr by x & proceed .

dx
&
(ax + b) px + q

dx

( ax + b)

px 2 + qx + r

(ax

dx
+ bx + c

, put ax + b =

px + q

1
;
t

; put px + q = t2 .

(ax

dx

+ bx + c

px + qx + r
2

x
dx or
x

(x ) ( x) ;

put x = cos2 + sin2

x
dx or
x

( x ) ( x ) ;

put x = sec2 tan2

x
=t
2

dx

( x ) ( x )

, put x =

1
t

; put x = t2 or x = t2 .

DEFINITE

INTEGRAL

f(x) dx = F(b) F(a) where

1.

f(x) dx = F(x) + c

a
b

VERY IMPORTANT NOTE :

If

f(x) dx = 0

then the equation f(x) = 0 has atleast one

root lying in (a , b) provided f is a continuous function in (a , b) .


2.

PROPERTIES OF DEFINITE INTEGRAL :


b

f(x) dx = f(t) dt

P
1
b

P
3

P2

provided f is same

f(x) dx = f(x) dx

f(x) dx = f(x) dx + f(x) dx , where c may lie inside or outside the interval [a, b] . This property
a

to be used when f is piecewise continuous in (a, b) .


a

P
4

f(x) dx = 0

if f(x) is an odd function i.e. f(x) = f(x) .

= 2 f(x) dx if f(x) is an even function i.e. f(x) = f(x) .


0
b

P
5

f(x) dx = f(a + b x) dx ,
a

In particular

f(x) dx = f(a x)dx

48

2a

P
6

f(x) dx

f(x) dx + f(2a x) dx = 2 f(x) dx

=0
P
7

na

f(x) dx = n f(x) dx

b + nT

P
8

; whereais the period of the function i.e. f(a + x) = f(x)

f(x) dx =

a + nT

9
P

if f(2a x) = f(x)

f(x) dx where f(x) is periodic with period T & n I .


a

na

ma

f(x) dx = (n m) f(x) dx if f(x) is periodic with period 'a' .

P
10 If f(x) (x) for a x b then
P
11

if f(2a x) = f(x)

f(x) dx (x) dx

f ( x )d x

f(x)dx .
a
b

P
12 If f(x) 0 on the interval [a, b] , then

f(x) dx 0.
a

3.

WALLIS FORMULA :

sinnx . cosmx dx = [( n 1) ( n 3 ) ( n 5 ).... 1or 2 ][( m 1) ( m 3 ).... 1or 2 ] K

/ 2

( m + n ) ( m + n 2 ) ( m + n 4 ).... 1or 2

if both m and n are even (m, n N)


2
= 1 otherwise
DERIVATIVE OF ANTIDERIVATIVE FUNCTION :
If h(x) & g(x) are differentiable functions of x then ,

Where K =

4.

d
dx

5.

h(x)

f(t) dt = f [h (x)] . h(x) f [g (x)] . g(x)


g(x)

DEFINITE INTEGRAL AS LIMIT OF A SUM :


b

f(x) dx = Limit
n

h [f (a) + f (a + h) + f (a + 2h) + ..... + f (a + n 1 h ) ]

n 1

= Limit
h 0 h f (a + rh) where b a = nh
r=0

n 1

f(x) dx

If a = 0 & b = 1 then , Limit


n h f (rh) =
r=0

Limit 1 n 1 f r =

n
n r =1 n

6.

; where nh = 1

OR

f(x) dx

ESTIMATION OF DEFINITE INTEGRAL :


b

(i)

For a monotonic decreasing function in (a , b) ; f(b).(b a) <

f(x) dx < f(a).(b a) &

a
b

(ii)

For a monotonic increasing function in (a , b) ; f(a).(b a) <

f(x) dx < f(b).(b a)


a

49

7.

SOME IMPORTANT EXPANSIONS :

(i)

1 1 1 1
+ + + ..... = ln 2
2 3 4 5

(ii)

2
1
1
1
1
.....
=
+
+
+
+

6
12 2 2 3 2 4 2

(iii)

1
1
1
1
2
.....

=
12
12 2 2 3 2 4 2

(iv)

1
1
1
1
2
.....
+
+
+
+

=
8
12 3 2 5 2 7 2

(v)

1
1
1
1
2
.....
+
+
+
+

=
24
22 42 62 82

EXERCISE1
Q.1

Q.4

tan 2

cos 6 + sin 6

(x

dx
4

(x

Q.5 Integrate

x x e x
Q.6 e + x nx dx

Q.9

Q.7

a 2 sin 2 x + b 2 cos 2 x
dx
a 4 sin 2 x + b 4 cos 2 x

sin(x a ) dx
sin(x + a )

Q.12

Q.15

Q.17

Q.16

x 2 +1 ln x 2 +1 2ln x
dx
x4

Q.20 Integrate

Q.21

1
2

cosec x cot x
secx
.
cosec x +cot x 1+ 2secx

Q.27

sinx +secx

dx

dx

Q.30

Q.33

Q.28
x

by the substitution z = x + x 2 + 2x 1

x + 2x 1
2

dx

x + x (1 + x )

x
a+x

sin1

Q.18

Q.8

sin

Q.11

Q.14

dx
x + sin 2 x

x + x 2 + 2 dx

cot x dx
(1 sin x ) (sec x +1)

dx

ln (ln x ) + (ln x )

dx

x +1

x (1+ x e x )2

Q.19

dx

Q.22

Q.25

sin x
sin x

sin x2

cos 3 x2

cosx sin x
dx
79sin 2x

tan x.tan 2x.tan 3x dx


Q.31

Q.26
Q.29

dx
secx +cos ecx

dx
sin x sin (2 x + )

3+ 4sin x + 2cosx
dx
3+ 2sin x +cosx

) dx

dx
cos x

x2 + x
dx
Q.23 x
(e + x + 1) 2

dx

n cos x + cos 2x

sin x +

dx

cos + sin

( x cos x sin x)(x sin x + cos x) dx

Q.32

f (x) w.r.t. x4 , where f (x) = tan 1x + ln 1+ x ln 1 x

( x + 1)dx
x ( 3 x + 1)

Q.24

+ x+1

cos 2 x
1 + tan x dx

Q.3

(sin x)11/3 (cos x)1/3dx

1 x dx
1+ x

dx

cos 2 . ln cos sin d

Q.10

Q.13

[(

5 x 4 + 4 x5

Q.2

Q.34

3x 2 + 1
dx
Q.35 2
( x 1) 3

dx
sin x + tan x

50

dx

e cos x ( x sin 3 x + cos x )


dx
Q.36
sin 2 x
(7x 10 x )
2

2 3x
2 + 3x

3/ 2

dx

1+ x
dx
1 x

Q.42

Q.45

(x 2 + 3 x + 3 )

x+2

Q.40

(x 2 1)3 / 2

x c x (ax + b)
2 2

1 + 3 sin 2 x

dx
x +1

Q.46
Q.49

x 4 + 4x 3 6x 2 + 4x + 1

x ln x

cot x tan x

Q.43

x dx

Q.48

Q.37

Q.39

(ax 2 b) dx

dx

dx

2 x x2

cos 2 x
sin x

Q.44

Q.41

(1 x ) 1 x 2
3

dx

1 x dx
1+ x x

4 x 5 7 x 4 + 8x 3 2 x 2 + 4 x 7
dx

x 2 ( x 2 + 1) 2

dx

Q.38

e x 2 x 2

Q.47

(x )

dx
( x )( x )

(1 + x 2 )dx
(0, )
Q.50
1 2x 2 cos + x 4

dx

EXERCISE2

Q.1

x dx
2
9 cos x + sin 2 x

Q.2

Q.3 Evaluate In =

/ 2

/ 2

Q.4

1 sin 2 x
dx
1 + sin 2 x

sin2x arc tan(sinx) dx

/ 4

cos4 3x . sin2 6x dx

Q.5

Q.6

(lnn x) dx hence find I3.


1

x dx
cos x (cos x + sin x)

d
x
(h ( x ) ) = sin
Let h (x) = (fog) (x) + K where K is any constant. If
then compute the
2
dx
cos (cos x )

Q.7

f (x)

value of j (0) where j (x) =

f (t)
dt , where f and g are trigonometric functions.
g( t )
g( x)

Q.8

Find the value of the definite integral

2 sin x + 2 cos x dx .

0
5

Q.9

Evaluate the integral :

x + 2 2 x 4 +

x 2 2 x 4 dx

x dx
dx
x
dx
;
Q
=
and
R
=
1 + x4
1+ x4
1 + x 4 then prove that
0
0
0

(a)
Q = , (b) P = R, (c) P 2 Q + R =
2 2
4
b x n 1 n 2 x 2 + ( n 1)( a + b) x + nab
b n 1 a n 1
Prove that
dx
=
2 ( a + b)
( x + a ) 2 ( x + b) 2
a

Q.10 If P =

Q.11

x 4 (1 x) 4
0 1 + x 2 dx

eb

Q.12
3

Q.15

sin
0

2x
dx
1+ x2

Q.13

x 2 .ln x

1 x 2

0
/ 2

Q.16

Q.19

dx

Q.14 Evaluate:

2
2

a sin x + b cos x
dx
sin ( 4 + x )

x
x
Q.18 e cos 4 + 2 dx

Q.17

2x 7 +3x 6 10x 5 7x 3 12x 2 +x+1


dx
x 2 +2

51

dx
2 + sin 2x

x2 x
x2 + 4

dx

Q.20 Let , be the distinct positive roots of the equation tan x = 2x then evaluate (sin x sin x ) dx ,
0

independent of and .
/ 4

Q.21

cos x sin x
10 + sin 2 x

dx

Q.22

(ax + b)sec x tan x


dx (a,b>0) Q.23
4 + tan 2 x

Evaluate:

(2 x + 3) sin x
dx
(1 + cos 2 x )

Q.24 If a1, a2 and a3 are the three values of a which satisfy the equation
1

3
(sin x + a cos x ) dx
0

4a
x cos x dx = 2
2 0

then find the value of 1000( a12 + a 22 + a 32 ).


p + q

Q.25 Show that

<p<
2
2

| cos x| dx = 2q + sinp where q N &

Q.26 Show that the sum of the two integrals

2/ 3

e(x+5) dx + 3

sin 1 x
x 2 x + 1 dx
0
1

Q.27

/ 2

Q.30

/ 2

Q.28

1/ 3

sin 2 x
2
2
a sin x + b 2 cos 2 x

dx (a>0, b>0)

1+sin x + 1sin x
dx
1+sin x 1sin x

Q.31

Q.32 Comment upon the nature of roots of the quadratic equation


value of k + R.
1

dx
2

0
1

Q.37 (a)

1 x
dx
.
, (b)
1+ x x + x 2 + x 3

1+ 5
2

dz

Q.36

| t + k | dt depending on the
0

x 2 sin 2 x .sin

(2 .cos x )

2x

x2 +1
1

ln 1 + x dx
4
2
x
x x +1

sin

dx

(1 x )

n 1/ n

(n > 1)

dx

if ( 0 , )

8 + sin 2 x

dx

2
sin

if ( , 2 )

Q.40

x 2 (sin2xcos2x )
(1+sin 2x )cos 2 x

dx

dx

x2 + 2x = k +

1 x2

( x a 2 ) (b 2 x 2 )

dx
=
1 + xn

LM
dx
dx
Show that
=2
=M
x + 2x cos + 1 M
x + 2x cos + 1
MN
x sin x
0

Q.39

z
e
0

Q.38

Q.34 Prove that

z x z
x
Q.35 Show that e e dz = e

x3

x.dx

2a x

x sin1 2

1+ x
1 x

3a + b
2

Q.33

2a

Q.29

n

a 2 +b2
2

tan1
0

e9(x 2/3) dx is zero.

Q.41 Prove that

52

f (t ) dt

du =

f (u).(x u) du.

Q.42

dx
(5 + 4 cos x)2

Q.43 Evaluate

ln (

1n

Q.45 Lim n

16

1 x + 1 + x dx

(2006 sin x + 2007 cos x) | x | dx . Q.46 Show that

1 n

Q.44

tan 1

x 1 dx

x ln x
a x dx
dx = ln a . f ( + ).
x
x a x
0

f ( x + a ).
0

Q.47
Q.48

(2 x 332 + x 998 + 4 x1668 sin x 691 )


dx
Evaluate the definite integral,
666
+
x
1
1
Prove that

(a)

ln tan 1 x

(d)

Q.49 If f(x) = (cos x 1) 2


(cos x + 1) 2
Q.50 Evaluate :

where , > 0

4 cos 2 x

dx
=
x ( x ) ( x )

(b)

(c)

( )2

( x )( x ) dx =

(cos x 1) 2 , find

(cos x + 1) 2

dx = ( )

x .d x

= ( + ) where <
( x ) ( x )
2

(cos x + 1) 2

x
x

cos 2 x

f (x)

dx

sin 1 (cos x ) dx .

EXERCISE3
cos x
then show that f(x) is a periodic function .
f (x )

Q.1

If the derivative of f(x) wrt x is

Q.2

Find the range of the function, f(x) =

Q.3

sin x dt
.
1 2t cos x + t 2

A function f is defined in [1 , 1] as f(x) = 2 x sin

1
1
cos ; x 0 ; f(0) = 0;
x
x

f (1/) = 0. Discuss the continuity and derivability of f at x = 0.


Q.4
Q.5

Let f(x) =

[ x1 1

if 2 x 0
and g(x) =
if 0 < x 2

f(t) dt. Define g (x) as a function of x and test the

continuity and differentiability of g(x) in (2, 2).


Prove the inequalities:

(a)
<
6

dx
4x x
2

<

2
8

(b) 2 e1/4 <

x 2 x

dx < 2e.

1
(d)
2

dx
(c) a <
< b then find a & b.
10 + 3 cos x
0

dx

2 + x2

5
6

Q.6

Determine a positive integer n 5, such that

ex (x 1)n

dx = 16 6e.

Q.7

Using calculus

(a)

If x < 1 then find the sum of the series

(b)

1
2x
4x 3
8x 7
+
+
+
+ ...... .
1 + x 1 + x 2 1 + x 4 1 + x8
1 2x
2x 4x 3
4 x 3 8x 7
1 + 2x
If x < 1 prove that
.
+
+
+ ...... =
2
2
4
4
8
1 x + x 1 x + x 1 x + x
1+ x + x2

53

x
x
x
x
1
1
1
1
tan + 2 tan 2 + .... + n 1 tan n1 = n1 cot n 1 2cot 2x
2
2 2
2
2
2
2
2

(c)

Prove the identity f (x)= tanx +

Q.8

If (x) = cos x (x t) (t) dt. Then find the value of (x) + (x).

x
0

Q.9

If y =

1
d2y
f
(
t
)

sin
a
(
x

t
)
dt
then
prove
that
+ a 2 y = f (x).
a 0
dx 2
x

ln t dt
dy
, find
at x = e.
Q.10 If y = x 1
dx

Q.11

If f(x) = x + [xy + xy] f(y) dy where x and y are independent variable. Find f(x).
0

dy
= ex cos x for x [0, 2] and passes through the origin. Prove that the
dx
3

roots of the function (other than zero) occurs in the ranges < x < and
< x < 2.
2
2

Q.12 A curve C1 is defined by:

Q.13(a) Let g(x) = xc . e2x & let f(x) =

e2t . (3 t2 + 1)1/2 dt . For a certain value of 'c', the limit of


0

as x is finite and non zero. Determine the value of 'c' and the limit.
x

(b) Find the constants 'a' (a > 0) and 'b' such that, Lim
Q.14 Evaluate: Lim

x +

d
dx

t2 d t
a+t

f (x)
g (x)

= 1.

x 0

bx sin x

d2U n

= n (n 1) Un2 2 n(2n 1)Un1,

3 x

3t 4 + 1
(t 3)(t 2 + 3) dt
1

2 sin

Q.15 Given that Un = {x(1 x)}n & n 2 prove that

dx 2

further if Vn = ex . Un dx, prove that when n 2, Vn + 2n (2n 1).Vn1 n (n 1) Vn2 = 0

Q.16 If

n t

0 x +t
equation.

Q.17

dt =

n 2
(x > 0) then show that there can be two integral values of x satisfying this
4

1 x
Let f(x) = 0

2
(2 x)

0 x 1

if

if 1 < x 2 . Define the function F(x) =


if 2 < x 3

f(t) dt

and show that F is

continuous in [0, 3] and differentiable in (0, 3).


Q.18 Let f be an injective function such that f(x) f(y) + 2 = f(x) + f(y) + f(xy) for all non negative real x &
y with f (0) = 0 & f (1) = 2 f(0) . Find f(x) & show that, 3 f(x) dx x (f(x) + 2) is a constant.
Q.19

Evaluate: (a) Lim


n

1
22
1 + 2 1 + 2
n
n

1+ 3

n2

1 1
2
3n
+
+ ..... +
(b) Lim
;
n n n + 1
n+2
4n

.....

1+ n

2
2

(c) Lim
n

1/n

n!
nn

;
1/ n

1n

3n C n

(d) Given Lim 2 n

n
C

a
where a and b are relatively prime, find the value of (a + b).
b

54

Q.20 Prove that sin x + sin 3x + sin 5x + .... + sin (2k 1) x =


/ 2

prove that ,

0
/ 2

Q.21 If Un=

sin 2 n x
sin 2 x

sin 2 k x
sin x

sin 2 k x
sin x

, k N and hence

1
1 1 1
.
dx = 1 + + + + ...... +
2k1
3 5 7

dx , then show that U1 , U2 , U3 , ..... , Un constitute an AP .

Hence or otherwise find the value of Un.


Q.22 Solve the equation for y as a function of x, satisfying
x

x y( t ) dt = ( x + 1) t y( t ) dt , where x > 0, given y (1) = 1.


1

Q.23 Prove that :

(a)
(b)

Im , n =
Im , n =

xm . (1 x)n dx =
0
1

m !n !
( m + n +1 )!

m , n N.

n!

xm . (ln x)n dx = (1)n

( m +1) n +1

m , n N.

Q.24 Find a positive real valued continuously differentiable functions f on the real line such that for all x
x

f 2(x) =

(( f (t)) + ( f ' (t)) )dt + e


2

0
x

Q.25 Let f(x) be a continuously differentiable function then prove that,

where [. ] denotes the greatest integer function and x > 1.

[x]

[t] f (t) dt = [x]. f(x) f (k )


k=1

Q.26 Let f be a function such that f(u) f(v) u v for all real u & v in an interval [a, b] . Then:
(i)
Prove that f is continuous at each point of [a, b] .
b

(ii)

Assume that f is integrable on [a, b]. Prove that,

f (x) dx (b a ) f (c)

Q.27 Let F (x) =

(b a )2
, where a c b
2

4 + t 2 dt and G (x) =

4 + t 2 dt then compute the value of (FG)' (0) where dash

denotes the derivative.


Q.28 Show that for a continuously thrice differentiable function f(x)
1x
f (0). x 2
2
+ f ( t )(x t ) dt
20
2
m
1
1
= ( 1)k m
k k+ n+1
k + m + 1 k=0

f(x) f(0) = xf (0) +


n

Q.29 Prove that

( 1)k

k=0

( nk )

( )

Q.30 Let f and g be function that are differentiable for all real numbers x and that have the following
properties:
(i)
f ' (x) = f (x) g (x) (ii)
g ' (x) = g (x) f (x)
(iii)
f (0) = 5
(iv)
g (0) = 1
(a)
Prove that f (x) + g (x) = 6 for all x. (b)
Find f (x) and g (x).

EXERCISE4
Q.1

Find Limit
n Sn , if :

Sn =

1
+
2n

1
4n2 1

+ .......... +

4n2 4

55

1
3n 2 + 2n 1

[REE 97, 6]

Q.2

(a)

If g (x) =

cos

t dt , then g (x + ) equals :

(A) g (x) + g ()
(b)

2n

Limit 1
n
n

(B) g (x) g ()

n2 + r2

r =1

(B) 1 + 5
e

(c)

(d)

d
e sin x
F (x) =
, x > 0 . If
Let
dx
x

values of k is ______.

(e)

Determine the value of


x

(a)

(D) 1 + 2

2 e sin x
dx = F (k) F (1) then one of the possible
x

2 x (1 + sin x)

1 + cos 2 x

Q.3

(C) 1 + 2

sin ( ln x)
dx is _______ .
x

The value of

(D) [ g (x)/g () ]

equals :

(A) 1 + 5
37

(C) g (x) g ()

dx .

[JEE 97, 2 + 2 + 2 + 2 + 5]

f (t ) dt = x + t f (t ) dt , then the value of f (1) is

If

(A) 1/ 2

(B)

(C) 1

1/ 2

(D)

1
1
Prove that tan 1
2
dx
=

tan x dx . Hence or otherwise, evaluate the integral


2
1
x

+
x

0
0
1

(b)

tan

(1 x + x ) dx
2

[JEE98, 2 + 8]

0
1

Q.4

(5 + 2x 2x 2 )(1 + e (2 4 x ) ) dx

Evaluate

[REE 98, 6 ]

Q.5

(a)

If for al real number y, [y] is the greatest integer less than or equal to y, then the value of the
3 / 2

integral

[2 sin x] dx is :

/2

(A)
3 / 4

(b)

/ 4

(B) 2

Integrate :

x3 + 3x + 2

Integrate:

x 2 + 1 (x + 1)

(d)

(D)

dx
is equal to :
1 + cos x

(A) 2
(c)

(C)

(B) 0

(C)

1
2

(D)

1
2

dx

cos x

ecos x + e cos x dx

[JEE '99, 2 + 2 + 7 + 3 (out of 200)]

/6

Q.6

Evaluate the integral

3 cos 2x 1
dx.
cos x
e2

Q.7

(a)

The value of the integral

(A) 3/2

[ REE '99, 6]

loge x
d x is :
x

(B) 5/2

(C) 3

56

(D) 5

(b)

1
1
f (t) 1 for t (0, 1] and 0 f (t)
2
2

f (t) d t , where f is such that

Let g (x) =

for t (1, 2]. Then g (2) satisfies the inequality :


(A)
(c)

3
1
g (2) <
(B) 0 g (2) < 2
2
2
e cos x . sin x for | x | 2

{2

If f (x) =

otherwise

(A) 0

For x > 0, let f (x) =

5
3
< g (2)
2
2

f (x)d x :

(C) 2

(D) 3

n t
dt. Find the function f (x) + f (1/x) and show that,
1+ t

f (e) + f (1/e) = 1/2 .


Q.8

(a)

Sn =

1
1

(b)

1+ n

Given

[JEE 2000, 1 + 1 + 1 + 5]
1

2 + 2n

+ ........ +

1
n+

sin t
d t = , find the value of
1+ t

(D) 2 < g (2) < 4

. Then

(B) 1
x

(d)

(C)

. Find Limit
Sn .
n

sin 2t

4 2

4+2t

d t in terms of .

[ REE 2000, Mains, 3 + 3 out of 100]


Q.9

2x + 2
Evaluate sin 1
dx .
4x 2 + 8x + 13
/ 2

Q.10 (a)

Evaluate

cos9 x
dx .
cos 3 x + sin 3 x

(b)

Evaluate

xdx

1+ cos sin x
0

[ REE 2001, 3 + 5]
x

Q.11

(a)

Let f(x) =

2 t 2 dt . Then the real roots of the equation x2 f (x) = 0 are

(b)

1
(D) 0 and 1
2
2
Let T > 0 be a fixed real number. Suppose f is a continuous function such that for all x R

(A) +1

(B) +

f (x + T) = f (x). If I =
3
(A) I
2

(C) +

3+ 3 T

f(x) dx then the value of f(2x) dx is


(B) 2 I

(C) 3 I

(D) 6 I

1
2

(c)

1 + x
The integral [x] + ln
dx equals
1 x
1

1
(A)
2
(d)

(B) 0

1
(D) 2ln
2
[JEE 2002(Scr.), 3+3+3]

(C) 1

For any natural number m, evaluate

z cx

3m

+ x2 m + x m

h c2 x

2m

Q.12 If f is an even function then prove that

+ 3x m + 6

1
m

dx , where x > 0

[JEE 2002 (Mains),4]

f (cos2x) cosx dx =

f (sin2x) cosx dx

[JEE 2003,(Mains) 2 out of 60]


1

Q.13 (a)

1 x
dx =
1+ x

57


+1
2

(A)

(B)

t2

(b)

If

x f ( x ) dx =
0

(A)

1
2

(C)

(D) 1

4
2 5
t , t > 0, then f =
25
5

2
5

(B)

5
2

(C)

2
5

(D) 1
[JEE 2004, (Scr.)]

(c)

If y(x ) =

(d)

Evaluate

dy
cos x. cos
.d then find
at x = .
2
dx
1
+
sin

2
/ 16

[JEE 2004 (Mains), 2]

/3

+ 4x 3
dx .

/ 3 2 cos | x | +
3

Q.14 (a)

2
t (f (t )) dt

If

sin x

1
is
= (1 sin x), then f
3

(A) 1/3

(x

(B) 1

[JEE 2005 (Scr.)]


(C) 3

(D) 3

(b)

[JEE 2004 (Mains), 4]

+ 3x 2 + 3x + 3 + ( x + 1) cos( x + 1) dx is equal to

[JEE 2005 (Scr.)]

(A) 4

(B) 0

(C) 4

(D) 6

Evaluate: e|cos x| 2 sin cos x + 3 cos cos x sin x dx .


2

(c)

Q.15

x3
(A)

x 2 1
2x 4 2x 2 + 1

[JEE 2005, Mains,2]

dx is equal to

2x 4 2x 2 + 1
+C
x2

2x 4 2x 2 + 1
+C
x
Comprehension

(C)

(B)

2x 4 2x 2 + 1
+C
x3

(D)

2x 4 2x 2 + 1
+C
2x 2

[JEE 2006, 3]

ba
(f (a ) + f (b) ) , for
2
a
a+b
ca
(f (a ) + f (c) ) + b c (f (b) + f (c) ) . When c =
more accurate result for c (a, b) F(c) =
,
2
2
2
b
ba
f (x )dx = 4 (f (a ) + f (b) + 2f (c))
a

Q.16 Suppose we define the definite integral using the following formula f (x ) dx =

/2

(a)

sin x dx is equal to
0

(A)

1+ 2
8

(B)

1+ 2
4

(C)

8 2

58

(D)

4 2

f ( x ) dx
If f (x) is a polynomial and if Lim a

(b)

t a
(f ( t ) + f (a ) )
2

(t a )3

t a

= 0 for all a then the degree of f (x) can

atmost be
(A) 1
(B) 2
(C) 3
(D) 4
If f ''(x) < 0, x (a, b) and c is a point such that a < c < b, and (c, f (c) ) is the point lying on the curve
for which F(c) is maximum, then f '(c) is equal to
f (b ) f (a )
2(f (b ) f (a ))
2f (b ) f (a )
(B)
(C)
(D) 0
(A)
ba
ba
2b a
[JEE 2006, 5 marks each]

(c)

5050 1 x 50
0

Q.17 Find the value of

(1 x
1

100

dx
[JEE 2006, 6]

50 101

dx

ANSWER
EXERCISE1
1 + 1 + 3 cos 2 2
+C
Q.1 ln

cos 2

Q.3

1
4

ln(cos x + sin x) +

Q.2

x
x
3
x 1
1
3
+ (sin 2x + cos 2x) + c Q.4 tan1 x

ln
+c
4
8
2
16
x + 1

4 x 1
8

1
cos + sin 1
(sin 2 ) ln
ln (sec 2 ) + c

2
cos sin 2

1
a 2 tan x
1

x
+
tan
Q.9 2
b2 + c
a + b 2

Q.11

Q.10 2ln

1
tan x
ln
+c
2
tan x + 2

Q.8

t
1
+
+ C when t = x +
2t + 1 2t + 1

3/2
1
2
2
x + x + 2

1/ 2 + c

3
x + x 2 + 2

Q.12 cos a . arc cos cos x sin a . ln

2
2
sin x + sin x sin a

cosa
1
1
x
x
x
Q.14
ln tan + sec + tan + c
2
4
2
2
2

Q.16 (a + x) arc tan


Q.18 xln (lnx)

x
ln x

x
a

+c

ax + c

Q.6 x e + c
e x

Q.5 2 tan1 x + x 2 + 2x 1 + c
Q.7 (c)

x+1
+c
x5 + x + 1

Q.19

59

(x

+1

3 1+ 4 tan 2 x
+c
8(tan x ) 8 / 3

x 1 x 2 1 x + arc cos

Q.15
Q.17

Q.13

+c

x +c

x 2 +1
1

. 2 3ln 1 + 2
9x
x

xe x
ln
x
1+ xe

+ 1
1+ xe x + c

x2 + x

t4 t2
1
2
1
Q.21 6 + t + ln (1 + t ) tan t + C where t = x1/6
2
4 2

Q.20 ln (1 x4) + c

Q.22

+ 2 tan1 cos x2 ln

cos x2

1+

cos x2

cos x2

Q.23 C ln(1 + (x + 1)ex)

+c

1
1 + ( x + 1)e x

1
x
Q24. sin1 sec 2 + c
2

Q.25
Q.27

1
( 4 + 3sin x + 3cos x )
ln
+c
24 ( 4 3sin x 3cos x )
1
2 3

ln

1
1
x
sin x cos x
l n tan + + c
2
2
2 8

Q.26

3 + sin x cos x
+ arc tan (sin x + cos x ) + c
3 sin x + cos x

1
2
Q.29
ln cot x + cot + cot x + 2 cot cot x 1 + c
sin

x
Q.31 2x 3arctan tan +1+c
2

Q.32

1
1

 n (sec 2x) + n (sec 3x) + c


2
3

Q.28 n (sec x)
Q.30 ln

x sin x + cos x
x cos x sin x

((

cos 2x
x cot x . ln e cos x + cos 2x
sin x

t2 2 t + 1
1
1
1
4
ln 2
tan1 t2 + c where t =
Q.33 ln (1 + t) ln (1 + t ) +
t + 2t +1
2
4
2 2
x
1
x 1
x
Q.35 c 2
Q.34 ln tan tan 2 +c
( x 1) 2
2
2 4
2

Q.36 c ecos x (x + cosec x) Q.37


Q.40 arcsecx

Q.42

ln x

x 1
2

+c

ax 2 + b
sin 1
+k
cx

Q.41 n

2 sin 2 x
+c

sin x + cos x

Q.47

Q.49

Q.50

| u 2 1|
u +u +1
4

1+x
+ c Q.39
1x

+ 3 tan 1

1 + 2u 2
3

5 t 1
8 1
1
sin 1 x 1 x 2 + c where t =
n
tan t +
3
2 5
5 t + 1

Q.43 tan1
Q.45

Q.38 ex

2
x
arctan
+c
3
3( x+1)

2 x
.
+c
x

Q.44 4 ln x +

Q.46

)) + c

cot x

2(7x 20)
9 7x 10 x 2

+ c where u = 3

+c

1 x
1+ x

1+ x
1 x

6x
7
+ 6 tan1(x) +
+C
x
1+ x2

4 x + 2 2 2 x x2
2 x x2
2
sin 1 2x + 1 + c
+
n

3
x
4
x

Q.48

1
1
1

ln x + + 2 + x + + 2 12 + C
2
x
x

2 + t 1 1 t
1
ln
ln
where t = cos and = cosec1(cotx)
2 2 t 2 1 + t

x 2 1

cos ec
cos
ec

tan

2x
2
2
2

60

EXERCISE2
2
Q.1
6

Q.3 6 2e

Q.2 ln 2

Q.19

Q.22
Q.28

2 2

16 2
5

2a (a +b)

Q.29
Q.36

Q.43

3
3

1
2

(3

Q.5

32 2

5
64

Q.6

ln 2
8

22

Q.12
7

Q.13

(1 ln 4)
8

2
3 2 2
(a +b)
(e + 1)
Q.17
Q.18
3
5
2 2

1
2
1
Q.21 arc tan
arc tan

3
3
3

5
3

2
Q.27
6 3

Q.24 5250
2
Q.30 3
16

Q.31

12

Q.32 real & distinct k R

2 2

2
; (b) ln 2 Q.39
ln2
ln 2 Q.40
4
8
16
3
3
16
+4
2 3
Q.45 2007
Q.47
Q.44
3
666

Q.37 (a)

ln 2 + 2 1

32
15

1
2

Q.16

( + 3)
Q.23
2

3 3

Q.49 2

Q.15

Q.20 0

(a + 2b)

2
Q.33 a
4
5
Q.42
27

4
3

Q.9 2 2 +

Q.7 1 sec(1) Q.8 2 6


Q.14 4 2 4 ln ( 2 + 1)

Q.4

Q.50

2
1
(1 + ln 2) +
8 4
2

EXERCISE3
Q.2
Q.4


,
2 2

Q.3 cont. & der. at x = 0

g(x) is cont. in (2 , 2); g(x) is der. at x = 1 & not der. at x = 0 . Note that ;
(x + 2)

2
g(x) = 2 + x x2
x2
2 x1

Q.7 (a)

1
1x

for 2 x 0
for

0 <x <1

for

1 x 2

Q.8 cos x

Q.13 (a) c = 1 and Limit


will be
x

Q.10 1 + e

Q.18 f (x) = 1 + x2

1
2
(x2 )3 1
+2
3

if
if

0x1
1<x2

if

2<x3

Q.27 0

61

2
2
& b=
13
7

f(x) = x +

Q.19 (a) 2 e(1/2) ( 4); (b) 3 ln 4; (c)

e 1 x
e
Q.24 f (x) = ex + 1
x3
f (x) = 3 + 2e2x; g (x) = 3 2e2x

Q.22 y =
Q.30

Q.17 F(x) =

Q.11

3
(b) a = 4 and b =1
2

x x2

Q.16 x = 2 or 4

Q.5 (c) a =

Q.6 n = 3

61
80
x+
x
119
119

Q.14 13.5

1
; (d) 43 Q.21
e

Un =

n
2

EXERCISE4
Q.1 /6

Q.4
Q.5
Q.6

Q.2 (a) A (b) B (c) 2 (d) 16

1
2 11
( a

n

(e)

Q.3 (a) A (b) ln2

11 + 1
11 1

C, (b) A ; (c)

2
2 tan1
3

x
1
1
3
tan-1 x ln (1 + x) + ln (1 + x2) +
2 + c, (d)
1+ x
2
4
2
2

Q.8 (a) 2 ln 2, (b)

Q.7 (a) B, (b) B, (c) C, (d)


Q.9 (x + 1) tan1

1 2
ln x
2

2( x + 1) 3
n (4 x 2 + 8 x + 13) + C
3
4


if (0, )
1 5 1

Q.10 (a) , (b) I = sin

8 4 3

( 2) if (,2)
sin

Q.11

1
2x3m + 3x2 m + 6x m
(a) A, (b) C, (c) B, (d)
6 ( m + 1)

Q.13 (a) B, (b) A, (c) 2, (d)

4
1
tan 1
3
2

24
1 e 1
e cos + sin 1
5
2 2 2
Q.16 (a) A, (b) A, (c) A
Q.17 5051

Q.14 (a) C, (b) C, (c)


Q.15 D

62

m+1
m

+C

ELEMENTARY DEFINITE INTEGRAL

(SELF PRACTICE)
Evaluate the following definite integrals.
sin 1 x
dx
x (1 x)

Q.1

Q 5.

/4

sin 2 x
dx
4
sin x + cos4 x

( x 1) (2 x) dx

Q 6.

cos x dx
(1 + sin x) ( 2 + sin x)

Q 9.

/2

Q 8.

Q 11.

1
/2

sin cos

Q 13.

(a

/4

Q14.

/2

dx
(0, )
1 + cos . cos x

Q 19.

sin x dx

Q 30.

dx

1+ x 2

a
1

Q 34.

1 x 2

Q 25.

x5

where a =

1+ x 2
dx
1 x 2

sin2 cos d
0
/ 2

Q 40.

1 sin 2 x dx

Q23.

dx

1+ x 2

4 x 2 dx

Q 28.

)
1

& b=

/2

1
2

x) dx

(1 x )

2 3/ 2

Q 31.
e e
2

/4

dx
3 + 2 sin x + cos x

Q 36.

1 + 2 cos x
dx
( 2 + cos x) 2

Q 39.

0
/2

2 x2
(1+ x) 1 x

1 x 2
dx
1+ x2 + x4

Q33.

dx

Q 42.

sin + cos
d
9 + 16 sin 2
x + sin x
dx
1 + cos x

dx 1 + e

1/ x

63

dx

x2
dx
1+ x 4

a 2 x2
dx
a 2 + x2

sin 1 x

Q 29.

cos3x sin 3x dx Q 41.


0

x
dx
3 x

x (tan

Q 38.

2 3/ 2

Q 35.

(1+ x )

Q 26.

ee
2

dx

dx

x x4 +1

Q 37.

Q 20.

sin x cos x
dx
2
cos x + 2 cos x + 2

Q 32.

x dx
x + 1 + 5x + 1

dx
where < <
x 2 + 2 x cos + 1

( x 1) x 2 2 x

cos 2x

Q 22.

dx

(1 2x )

dx

1/ 2

2
x .

Q17.

/4

Q27.

/2

/2

x + cos3 x

( x + 1)

dx
5 + 4 sin x

Q16.

dx

sin 2 + b 2 cos2 d a b

x cos x cos 3x dx

/2

(sin

3/ 4

Q 12.

Q.24

Q 21.

sin 2 x . cos2 x

Q 18.

1
1
2 dx

n x n x

dx
( x 1) (5 x)

/ 4

Q 15.

sin x dx
1 + cos2 x

x dx
5 4x

Q 10.

Q.3

e2x . cos x dx

Q 7.

x ex dx

/2

Q 4.

3 / 4

n 2

Q.2

dx

dx

ln(x x e x )

Q 43.

Q 44.

Q 45. If f( ) = 2 &

x2 d (ln x)

(f(x) + f (x)) sin x dx = 5, then find f(0)


0

Q.46

/2

Q.48

|x|
dx
x

Q.47

sec x tan x
sec x + tan x

cos ec x
1 + 2 cos ec x

2 3 x
11 x
cos 2
+ dx
cos

8
8
4
4

dx

x f ' ' ( x) dx , where f (x) =cos(tan

Q.49

x)

n 3

f (x)dx, where f(x) = e x + 2 e 2x + 3 e 3x + ......

Q.50

n 2

ANSWER KEY
2
4
1
Q 5.
6

Q 9.

Q 11.

Q 12.

9 + 4 2
1

n
7
2

Q 14.

Q 15.

3
16

Q 16.

1
2
tan1
3
3

Q 22. 1

14
3 2
Q 20.
15
2
1
3
n 2 + 3
Q 23.
Q 24.
2
2

5
1
Q 27.
tan1 2 + ln
2
4
2

Q 10.

1 a 3 b3
3 a 2 b2

Q 17.
3

Q 13.

3
16
32
1
Q 25.
ln
17
4

Q 21.

Q 28.

Q 4.

Q 2.

1
6

3
4

Q 18 sin
3
a2
Q 26.
( 2)
4


1
1 + n 2
2
4 4

Q 31.

2 2

Q 35.
4

Q 39.
2

Q 7.
4

Q 19.

Q 32. 1
1
ln 3
20
5
Q 40.
12

Q 36.

e 2 e 2
2

Q 44.

Q.47

Q.48 /3

1
ln 2
4 2
1
Q 33.
ln 3
2
4
Q 37.
9

Q 41.
2

1
if 0 ;
if = 0
2 sin
2
3 + 8
Q 34.
24
1
Q 38.
2

Q 45. 3

Q.46 | b | | a |

Q 29.

Q 43. ln 2
2

e 2
5
4
Q 8. ln
3

Q3. + tan 1 1

1
e
n
2
2
2
Q6. e
n 2

Q 1.

Q.49 1

3
2 2

64

Q 30.

Q 42.

Q.50

2
1+ e

1
2

EXERCISE5
Part : (A) Only one correct option
1.

[f (x)g( x) f (x)g(x)] dx is equal to


f (x)
(A) g( x )
(C) f(x) g(x) f(x) g(x)

2.

(D) f(x) g(x) + f(x) g(x)

sin x cos x
2

(A)
3.

(B) f(x) g(x) f(x) g(x)

tan x

dx is equal to

n |x|

(B) 2 tan x + c

+c

x 1 + n |x|

2
1 + n x (nx 2) + c
3
1
1 + n x (nx 2) + c
(C)
3

5.

If

6.

sin 8 x cos 8 x

1 2 sin

x cos 2 x

8.

1
sin 2x + c
2

(B)

1
sin 2x + c
2

(B)

2
1 + n x (nx + 2) + c
3

(D) 2 1 + n x (3 nx 2) + c
x 2 + 1 ) + C, then

(B) f(x) = tan1 x, A = 1


(D) f(x) = 2 tan1 x, A = 1

1
sin x + c
2

(C)

(D) sin2x + c

a+x
ax

dx is equal to
ax
a+x

(B)

a2 x 2 + C

x 2 a2 + C

(C)

(D) none of these

tan(x ) tan(x + ) tan 2x dx is equal to


(A) n

sec 2x . sec( x + )
+C
sec( x )

sec 2x
(B) n sec( x ) sec( x + ) + C

(C) n

sec 2x . sec( x + )
+C
sec( x + )

(D) none of these

sec x 1 dx is equal to

x
1
2 x

(A) 2 n cos 2 + cos 2 2 + C

9.

(D) 2 tan x c

dx =

(A) 2 a 2 x 2 + C
7.

+c

x tan 1 x

dx = 1+ x 2 f(x) + A n (x +
1+ x2
(A) f(x) = tan1 x, A = 1
(C) f(x) = 2 tan1 x , A = 1

(A)

tan x

dx equals :

(A)

4.

(C)

x
1
2 x

(C) 2 n cos 2 + cos 2 2 + C

dx
is equal to
3
cos x sin 2x

x
1
2 x

(B) n cos 2 + cos 2 2 + C

(D) none of these

(A)

5/2
x + C
2 cos x + 5 tan

(B)

5/2
x + C
2 tan x + 5 tan

(C)

5/2
x + C
2 tan x 5 tan

(D) none of these

65

10.

11.

Primitive of

(x

3 x4 1
4

+x+1

w.r.t. x is:

(A)

x
+c
x4 + x + 1

(B)

x
+c
x4 + x + 1

(C)

x+1
+c
x4 + x + 1

(D)

x+1
+c
x4 + x + 1

x4 + 1

If

x x +1
2

dx = A n x +

B
+ c, where c is the constant of integration then:
1 + x2
(B) A = 1; B = 1
(D) A = 1; B = 1

(A) A = 1; B = 1
(C) A = 1; B = 1
12.

13.

1 x

1+ x

(A) x

1 x 2 1 x + cos 1

(C) x

1 x 2 1 x cos 1

( x) + c
( x) + c

sin 16 x
+c
1024
1
cos 6 x + sin 6 x

(B)

cos 32 x
+c
1024

dx
3

cos x . sin 2x

( x) + c
( x) + c

1 x + 2 1 x cos 1

cos 32 x
+c
1096

(D)

cos 32 x
+c
1096

(B) tan 1 (tan x + cot x) + c


(D) tan 1 (tan x cot x) + c

(B) n (1 + sin x) + c

(C) x n (1 + sin x) + c (D) n (1 sin) + c

equals:

2
(tan x)5/2 + 2 tan x + c
5

(A)

(B)

2
(tan2 x + 5) tan x + c
5

2
(tan2 x + 5) 2 tan x + c
(D) none
5
dx
= a cot x + b tan 3 x + c where c is an arbitrary constant of integration then the
3
5
sin x cos x

(C)
17.

(C)

ln(1 + sin x) + x tan 4 2 dx is equal to:

(A) x n (1 + sinx) + c
16.

(D) x

1 x + 2 1 x + cos 1

d x equals :

(A) tan 1 (tan x + cot x) + c


(C) tan 1 (tan x cot x) + c
15.

(B) x

sin x. cos x. cos 2x. cos 4x. cos 8x. cos 16 x dx equals:

(A)
14.

dx equals :

If

values of a and b are respectively:


(A) 2 &
18.

2
3

(B) 2 &

2
3

(C) 2 &

2x 4 2x 2 + 1

2x 4 2x 2 + 1
x2

dx is equal to
+c

[IIT - 2006, (3, 1)]

(B)

2x 4 2x 2 + 1
+c
(D)
x
Part : (B) May have more than one options correct

(C)

If

(D) none

x2 1

(A)

19.

2
3

(x 1) dx

2 x 2x + 1
2

(A) f(x) = 2x 2 2x + 1

is equal to

2 x 4 2x 2 + 1
x3

2x 4 2x 2 + 1
2x 2

f (x )
+ c then
g (x )
(B) g(x) = x + 1
66

+c
+c

(C) g(x) = x
20.

(D) f(x) =

x
dx

= tan1 m tan + C then:

2
5 + 4 cos x

(A) l = 2/3
21.

22.

(B) m = 1/3

(C) l = 1/3

(A)

p = 1; q =

(B)

p = 1; q =

(C)

p = 1; q =

(D)

p = 1; q =

; f(x) = x; g(x) = n

1
3

2
3
1
3

; f(x) = x; g(x) = n

; f(x) = x; g(x) = n

; f(x) = x; g(x) = n

3 + tan x
3 tan x
3 + tan x
3 + tan x
3 tan x
3 + tan x
3 tan x

sin 2 x
dx is equal to:
sin x + cos4 x

x 1
n x + 1

(B) cot 1 (tan2 x) + c


(D) tan 1 (cos 2 x) + c

dx equal:

x2 1

(A)

24.

3 tan x

(A) cot 1 (cot 2 x) + c


(C) tan 1 (tan2 x) + c

23.

(D) m = 2/3

3 cot 3x cot x
dx = p f(x) + q g(x) + c where 'c' is a constant of integration, then
tan x 3 tan 3x

If

2x 2 2x

1 2 x 1
n
+c
x +1
2

(B)

1 2 x 1
n
+c
x +1
4

(C)

1 2 x+1
n
+c
x 1
2

(D)

1 2 x +1
n
+c
x 1
4

n (tan x)
dx equal:
sin x cos x

1 2
n (sec x) + c
2
1
(D) n2 (cos x cosec x) + c
2

1 2
n (cot x) + c
2
1
(C) n2 (sin x sec x) + c
2

(A)

(B)

EXERCISE6

cos ec 2 x. sin x

1.

Integrate with respect

2.

Integrate with respect to x

3.

Integrate with respect to x

4.

6.

(x 1)2
x +x +1
4

dx

tan + tan 3
1 + tan
3

(sin x cos x )

. dx

1 x 2
1 x2 + x 4
1

( x + 1) x 2 + 2

5.

7.

67

2 sin 2 cos
6 cos 2 4 sin
cos 5x + cos 4x
dx
1 2 cos 3x

3 + 4 sin x + 2 cos x
dx
3 + 2 sin x + cos x

8.

10.

(x )

12.

ex

14.

x 2 +1 ln x 2 +1 2ln x

x4

16.

cos ec x cot x
.
cos ec x + cot x

18.

2 x x2

20.

22.

23.

24.

Evaluate

25.

Integrate,

26.

dx
( x ) ( x )

x3 x + 2

(x + 1)
2

( b + a sin x)

x 1 x 2 e 2 sin x

(x

x cos + 1

dx

sec x
1 + 2 sec x

3/ 2

n (1 + sin2 x )

cos2 x

dx =

13.

cos

15.

17.

(7 x 10 x )

19.

21.

(x

dx
3

dx

(x + 1)

+ 3 x + 3x + 1
2

x 4 cot 2 x

dx

(a + b cos x)

dx

, (a > b)

x 2 + 2x 3

3/ 2

dx

tan 1 x. n (1 + x 2) dx.

dx

x4 x3 +1

f ( x)
g( x)

+ c then find f(x) and g(x)

dx.

x3 + 3x + 2
2

11.

dx

+ 2 x cos + 1

dx

1 + x cos x

dx

a + b sin x

(cos 2x 3)

dx

{ (

x2

1 + cos cos x
dx
cos + cos x

9.

(x + 1)

dx.

[IIT - 1999, 7]

For any natural number m, evaluate,

(x

3m

+ x2 m + x m

) (2 x

2m

+ 3x m + 6

1/ m

d x, x > 0.

68

[IIT - 2002, 5]

ANSWER
EXERCISE5
1. C

2. A

3. A

4. A

5. B

6. A

7. B

8. C

9. B

10. B

11. C

12. A

13. B

14. C

15. A

16. B

17. A

18. D

x +1

+c
12. ex 2

x + 1
1
tanx.(2 + tan2x). 4 cot 2 x
3

13. c

14.

19. AC 20. AB 21. AD 22. ABCD 23. BD

(x

15.

+1

x2 +1
1
. 2 3 ln 1 + 2

9 x3
x

(a

bsinx
2

b (a + bcosx )
2

2a

(a b )

2 3/2

24. ACD

arctan

EXERCISE6

x
ab
.tan + c
2
a+b

x
1
16. sin1 sec 2 + c
2
2
x
1. n 1+ 2 tan
2

1
1
2

n t 3 + t 3 + 9
3

where t=

17.

+c

tan

+c

2 x x2
2 4x+2 2 2xx
+
ln
x
x
4

2x+1
+ c
sin1
3

19. x tan 1 x. n (1 + x 2) + (tan 1 x)2 2x tan 1 x

2x 2 +1
x 2 1

2 tan 1
3 +c
x 3
3

2
5. 2 n sin 4sin + 5 + 7 tan 1(sin 2) + c

6.

9 7 x 10 x 2

+c

1
x +1

2 (7 x 20)

18.

3.

4.

1
3
x
+ c 2.
n
1
2 3
x+ + x
x
x+

1
1
1
n1+tan+ ntan2 tan + 1+
6
3
3

2 tan 1
tan
+c

2
+ n (1 + x 2) n 1 + x + c

20.

21.

cos x
+c
b + a sin x

2
1
1
x 3 +1
n
3
+c
3
3
x
3x
3 x3 +1

sin 2x
)+ c
2

7.

(sin x +

8.

2 x 3 arc tan tan + 1 + c

9.

10.

11.

22. ln (x esinx) -

cos

x cos + sin n
cos

23. x; x 2 + 2x cos + 1
24. tan x ln (1 + sin2x) 2x +

1 ( x)

2
+c
1 ( + x)

2
x
.
+c
x

x 2 + 2x 3
8 (x + 1)

1
ln (1 - x2 e2 sinx) + c
2

25.

2 tan1 ( 2 .tan x) + c.

x
3
1
1
tan-1 x - n (1 + x) +
n (1 + x2) +
+c
1 + x2
2
2
4
m+1

z m
+ c, where z = 2 x3m + 3 x2m + 6 xm
26.
6 ( m + 1)

2
1
+c
. cos1
x + 1
16
69

EXERCISE7
Part : (A) Only one correct option
1.

1
If f(x) is a function satisfying f + x2 f(x) = 0 for all non-zero x, then
x
(A) sin + cosec
(B) sin2
(C) cosec2
1

2.

The value of the integral

dx
2

(A) sin

4.

, where 0 < <

(B) sin

(D) none of these

, is equal to
2

2 sin

(C)

f (r 1 + x ) dx
f ( x ) dx = a, then
If

=
r =1 0

0
(A) 100 a
(B) a

f ( x) dx equals

sin

(D)

sin
2

100

100

3.

+ 2x cos + 1

cos ec

(C) 0

(D) 10 a

T T
If f(x) is an odd function defined on , and has period T, then (x) =
2 2

(A) a periodic function with period

T
2

f (t ) dt is
a

(B) a periodic function with period T

(C) not a periodic function

(D) a periodic function with period

T
4

5.

If f( ) = 2 and

(f ( x) + f (x)) sin x dx = 5 then f(0) is equal to, (it is given that f(x) is continuous in [0, ])
0

(A) 7

(B) 3

(C) 5

(D) 1

6.

x . f (2x) dx is equal to

If f(0) = 1, f(2) = 3, f (2) = 5 and f (0) is finite, then

(A) zero
7.

2
3
(n 1)
lim sin
. sin
. sin ....... sin

n
2n
2n
2n

(B) e4/

If A =

( x + 2)

dx,

then

(A)

/ 2

(A)

11.

(B)

(D) none of these

/3

f (x) dx is equal to
0

(C) n ( 2 )

(D) n ( 3 )

sin 2x
dx is equal to
x +1

1
A
+2

(C) 1 +

1
A
+2

(D) A

1
1

+2
2

| x | dx
8 cos 2 2x + 1

2
6

has the value


(B)

3n

Lt

1
1
+
A
+2
2

/2

10.

is equal to
(C) e2/

(B) n 2

cos x

(D) none of these

1/ n


f(x) = Minimum {tanx, cot x} x 0, . Then
2

(A) n 2

9.

(C) 2

(A)
8.

(B) 1

r = 2n + 1

2
12

(C)

n
r 2 n 2 is equal to
70

2
24

(D) none of these

2
3

(A) log
y

12.

If

x2

cos t dt =
2

13.

2
3

(D) log

2 sin x

2 sin x 2

(B)

x cos2 y

(C)

x cos y 2

2 sin x 2

y 2
x 1 2 sin
2

0 , where x =
, n = 1, 2, 3.....
, then the value of
If f(x) =
n +1
1 , else where
(A) 1
(B) 0
(C) 2
1

14.

(C) log

x dx

(1 x)

3
2

dy
sin t
is
dt , then the value of
dx
t

(A)

3
2

(B) log

(D) none of these

f ( x) dx
0

(D)

3/ 4

(A)

15
16
2

15.

16
5

(B)

Let 1 =

dx
1+ x

(A) 1 > 2

and 2 =

(C)

3
16

(D) none

dx
x , then

(B) 2 > 1

(C) 1 = 2

(D) 1 > 22

1
(C) 2 [ x]

(D) none of these

[ x]

16.

( x [x]) dx is

The value of

(A)

1
[x]
2

(B) 2[x]
3

17.

The value of the integeral

tan

(A)

x +1
2

(B) 2

+ tan 1

x 2 + 1
dx is equal to
x

(C) 4

(D) none of these

/2

18.

log | tan x + cot x | dx is

The value of

(A) log 2
1

19.

If

x2

(B) log 2

(A) 1 < < 2

(B) < 0

Suppose for every integer n,


(A) 16
Let A =

e dt
dt then
1+ t

(A) Aea

x4

5/2

(A)

(D)

log 2
2

a 1

(C) 19

(D) = 0

f (x)dx is :
(D) 21

et
dt has the value :
t a 1
(C) aea

(D) Aea

(D) none

2 3

22.

2
f (x)dx = n . The value of

(B) Aea

(25 x )

(C) 0 < < 1

n +1

(B) 14
1

21.

log 2
2

(x ) dx = 0, then

20.

(C)

dx equals to :
(B)

2
3

(C)

71

23.

dt

The function f(x) =

satisfies

[IIT - 1996]

(A) f(x + y) = f(x) + f(y)

x
(B) f y = f(x) + f(y)

(C) f(xy) = f(x) + f(y)

(D) none of these

(C) /2

(D) 2

(C) 1

(D) 2 ln (1/2)

24.

The value of

cos 2 x
dx , a > 0 is
x
1+ a

(A)

(B) a
1/ 2

25.

The integral
(A) 1/2

1 + x
[x] +  n
d x equals:
1 x
1/ 2

(B) 0
1

26.

If (m, n) =

(1 + t )n dt, then the expression of (m, n) in terms of (m + 1, n 1) is

[IIT - 2003]
n

(A)

n
2

(m + 1, n 1)
m +1 m +1

(B)

n
(m + 1, n 1)
m +1

(C)

n
2n

(m + 1, n 1)
m
+1
m 1

(D)

n
(m +1, n 1)
m +1

27.

If

1
is
t 2 (f(t)) dt = (1 sinx), then f
3

sin x

(A) 1/3

(B) 1/ 3

[IIT - 2005]

(C) 3

(D)

28.

{x

+ 3x 2 + 3 x + 3 + ( x + 1) cos( x + 1)} dx is equal to

[IIT - 2005]

(A) 4

(B) 0

(C) 4

(D) 6

(C) 0

(D) none of these

Part : (B) May have more than one options correct

29.

The value of integral

xf (sin x) dx is
0

(A)

/2

f (sin x) dx

(B)

f (sin x ) dx

0
2

30.

If f(x) is integrable over [1, 2], then

f ( x) dx is equal to
1

1
(A) nlim
n
1
(C) nlim
n

r =1
n

r =1

r
f
n

1
(B) nlim
n

r +n
f

1
(D) nlim
n

2n

r =n+1
2n

r
f
n

f n

r =1

31.

If f(x) =

(cos

t + sin 4 t ) dt, f (x + ) will be equal to

(A) f(x) + f()


1

32.

The value of
0

(A)

(B) f(x) + 2 f()

2x 2 + 3x + 3

(x + 1) x 2 + 2x + 2

+ 2 ln2 tan1 2
4

(B)

(C) f(x) + f
2

(D) f(x) + 2f
2

dx is:

+ 2 ln2 tan1 (C) 2 ln2 cot1 3


4
3
72

(D)

+ ln4 + cot1 2
4

33.

Given f is an odd function defined everywhere, periodic with period 2 and integrable on every interval. Let
x

f(t) dt. Then:

g(x) =

(A) g(2n) = 0 for every integer n


(C) g(x) and f(x) have the same period
/2

34.

dx

If =

1 + sin 3 x

(A) 0 < < 1


1

35.

If In =

(B) g(x) is an even function


(D) none

, then
(B) >

dx

(1 + x )
2

(C) <

; n N, then which of the following statements hold good?

1
+
8 4
5

(D) I3 =
16 48

(A) 2n In + 1 = 2 n + (2n 1) In
(C) I2 =

(D) > 2

(B) I2 =

8 4

EXERCISE8

1.

e cos

2x

cos3 (2n + 1) x d x, n I

2.

If f, g, h be continuous functions on [0, a] such that f (a x) = f (x), g (a x) = g (x)


a

and 3 h (x) 4 h (a x) = 5, then prove that,

f (x) g (x) h (x) = 0.

3.

Assuming

log 2, show that,

log sin x d x =

3 2
log
2

3 log sin d =

Show that

0
x

5.

Prove that

7.

f (t ) dt du =

1
n

f (u).(x u) du.

Prove that

6.

x dx

ex + 1

Evaluate

9.

dx

(1 x )

n 1/ n

(n > 1)

Evaluate, I =

x t. cos t dt where x is any real number


0

dx

(i)

p
p+r
2p
2 p 2
2 p 3
2p
cos 2n + cos 2n + cos 2n + ...... + cos 2 = 4 r

r =1

a 2 cos 2 x + b 2 sin 2 x
1 2 x
1 2 x
1 3 cos
2
+ tan

x + 1
1 x2

11.

dx
=
1 + xn

10.

a x ln x
a x dx
f ( + ).
dx = ln a . f ( + ).
x a x
x a x
0

Prove that Limit


n

8.

2 sin d .

4.

2 sin (p t) sin (q t) d t, if:

p & q are different roots of the equation, tan x = x.


73

p & q are equal and either is root of the equation tan x = x.

(ii)

sin x

x +1

Prove that

12.

dx 0 for x 0.

Let f(x) be a continuous functions x R, except at x = 0 such that

13.

f ( x)dx , a R

exists. If

0
a

g(x) =

f (t)
t dt, prove that

g( x ) dx =

sin x
x (0, ], prove that,
If f(x) =
x
2

14.

2e
d
e
F (x) =
, x > 0. If
x
dx
x
1

Let

15.

16.

/2

f ( x ) f x dx =
2

f (x) dx
0

dx = F (k) F (1) then one of the possible values of k is ______.

x4
2x
cos1
dx.
1 x4
1+ x2

Evaluate

17.

sin x 2

sin x

f (x) dx

|cos x|

[IIT - 1995, 5 + 2 + 2 ]

2 sin cos x + 3 cos cos x sinx dx.


2
2

[IIT - 2005, 2]

(1 x
The value of 5050

18.

50 100

dx

0
1

is

(1 x

50 101

[IIT - 2006, (6, 0)]

dx

ANSWER
EXERCISE7
EXERCISE8
1. D

2. C

3. B

4. B

5. B

6. C
1. 0

7. B

8. D

9. A

10. B

11. B

12. B

9.

2
2

2
13. C

14. D

15. B

16. A

17. B

18. A

20. C

21. B

22. A

23. C

15. 16
25. A

26. A

27. C

28. C

29. AB 30. BC
17.

31. AD 32. AD 33. ABC

34. BC

4a b

cos x for 0 < x < 1 ;

for x 1 &

2 3

24. C

3 3

10.
19. C

2 a 2 +b 2

8.

16.

2
2

for x 0

11. (i) 0

(ii)

74

1 + p2

n 2 + 3 +

4
12
3

1 1
1
24
e cos + e sin 1
5
2 2
2

35. AB

p2

18. 5051

ASSERTION AND REASON


Some questions (AssertionReason type) are given below. Each question contains Statement 1 (Assertion) and Statement 2
(Reason). Each question has 4 choices (A), (B), (C) and (D) out of which ONLY ONE is correct. So select the correct choice :
Choices are :
(A) Statement 1 is True, Statement 2 is True; Statement 2 is a correct explanation for Statement 1.
(B) Statement 1 is True, Statement 2 is True; Statement 2 is NOT a correct explanation for Statement 1.
(C) Statement 1 is True, Statement 2 is False.
(D) Statement 1 is False, Statement 2 is True.

INDEFINITE & DEFINITE INGEGRATION


129.

Let F(x) be an indefinite integral of cos2x.


Statement-1: The function F(x) satisfies F(x + ) = F(x) real x
Statement-2: cos2(x + ) = cos2x.
1

130.

Statement-1: |x| dx can not be found while

| x |dx can be found.

Statement-2: |x| is not differentiable at x = 0.

1 + x

1
2
dx = tan (x ) + C

Statement-2:

131.

Statement-1:

132.

Statement-1: If y is a function of x such that y(x y)2 = x then


Statement-2:

dx

x 3y

1+ x
dx

dx = tan1x + C

x 3y = 2 log(x y)

= log (x 3y) + c

x2
Statement2 : f(x) is periodic
2
x9/ 2
2
dx = ln x11/ 2 + 1 + x11 + c
11
11
1+ x
dx
= ln | x + 1 + x 2 | + c
2
1+ x

133.

Statement1 : f(x) = logsecx

134.

Statement1 :

Statement2 :

10

135.

Statement1 : tan 1 x dx = 10 tan1 ; where [x] = G.I.F.

Statement2 : [tan1 x] = 0 for 0 < x < tan 1 and [tan1 x] = 1 for tan 1 x < 10.
/ 2

136.

Statement1 :

dx
1 + tan 3 x

1 + tan x
3

0
/2

Statement2 :
/2

dx

f (x) dx = f ( a + x ) dx

=
3
1 + cot x 4

dx

f (x) dx = f (a x) dx .

137.

Statement1 :

1 sin 2 xdx = 0

Statement2 :

cos x dx = 0 .
0

75 of 89
75

138.

Statement1 : e x tan x + sec 2 x dx = e x tan x + c


x
x
e ( f (x) + f (x) ) dx = e f (x) + c .

Statement2 :

139.

Statement1 : If f(x) satisfies the conditions of Rolle's theorem in [, ], then

f (x) dx =

Statement2 : If f(x) satisfies the conditions of Rolle's theorem in [, ], then

f (x) dx = 0

140.

Statement1 :

[| sin x | + | cos x |]dx , where [] denotes G.I.F. equals 8.


0

Statement2 : If f(x) = |sinx| + |cosx|, then 1 f(x)

2.

n +1

141.

Let f(x) be a continuous function such that

f (x) dx = n , nI
3

n
2

Statement1 :

f (x) dx = 27

Statement2 :

142.

Let In =

( nx )

f (x) dx = 27

dx, n N

StatementI : I1. I2, I3 . . . is an increasing sequence.


: n x is an increasing function.

StatementII

f (t) dt and h(x) = g(x + 2) g(x).

143.

Let f be a periodic function of period 2. Let g(x) =

144.

Statement2 : g(x + 2) g(x) = g(2).


Statement1 : h is a periodic function.
x
e
Statement1 : (1 + x log x ) dx = e x log x + c
x
Statement2 : e x ( f (x) + f (x) ) dx = e x f (x) + c .

145.

Statement1 : If I1 =

dt
x 1 + t 2 and I2

1/ x

dt

1+ t

, x > 0 then I1 = I2.

Statement2 :

min .{x [x], x [ x]} dx = 0

146.

Statement1 : 8 < 2x dx < 12 .


4

Statement2 : If m is the smallest and M is the greatest vlaue of a function f(x) in an interval (a, b),
b

then the vlaue of the integral f (x)dx is such that for a < b, we have M(b a)
a

147.

ax

sin bxdx =

f (x) dx M(b a) .
a

ax

e
(asinbx bcosbx)+c
A
x 1 + sin x cos x
x
Statement2 : e
dx = e tanx + c
2
cos
x

Statement1 :

Then A is

76 of 89
76

a 2 + b2

148.

Statement1 :

Statement2 :

d(x 2 + 1)
+2
xa / 2

is equal to

2 x2 + 2 + c

dx is 2/11 ln |x + 1 + x11 | + c

1+ x
1
Statement1 :
is /12
1 + tan 3 x
/6
11

/3

149.

Statement2 :

f (x) dx = f (a + b x) dx

150.

Statement1 : If f satisfies f(x + y) = f(x) + f(y) x , y R then

f (x) dx = 0

Statement2 : If f is an odd function then

f (x) dx = 0

f (t) dt is an even function of (n)

151.

Statement1 : If f(x) is an odd function of x then

152.

Statement2 : If graph of y = f(x) is symmetric about yaxis then f(x) is always an even function.
Statement1 : Area bounded by y = {x}, {x} is fractional part of x = 0, x = 2 and xaxis is 1.
Statement2 : Area bounded by y = |sinx|, x = 0, x = 2 is 2 sq. unit.

153.

Statement-1: lim

154.

+ .... +

1
=
3n 3

Statement-1: If In = tann x dx, then 5 (I4 + I6) = tan5x .


Statement-2: If In = tan4x dx, then

155.

4n 2
4n 1
1
n
1 r
Statement-2: lim f = f (x) dx , symbols have their usual meaning.
n
r =1 n n
0
n

Statement-1:
1

tan n 1 x
- In-2 = In, nN.
n

If a > 0 and b2 4ac < 0, then the value of the integral

x+A

+ c , where A, B, C, are constants.


B

ax

dx
will be of the type tan+ bx + c

Statement-2: If a > 0, b2 4ac < 0 then ax2 + bx + c can be written as sum of two squares.
156.

157.

158.

x2 x +1 x
ex
x
Statements-2: e (f (x) + f (x) dx = ex f(x) + c
e
dx
=
+c
2
3/
2
(x + 1)
2
x +1
2
x 2
Statements-1:
dx = log |tan-1 (x + 2/x)| + c
2

x
+
2
(x 4 + 5x 2 + 4) tan 1

x
dx
1
x
Statements-2: 2
= tan 1 + c
2
a +x
a
a
x
ln
e = x +c
Statements-1:
Statements-2: ex (f(x) + f(x)) dx = ex f(x) + c.
(ln x)2 ln x
Statements-1:

dx =

1
1
1 + 4 + c Statements-2: For integration by parts we have to follow ILATE rule.
2
x

159.

Statements-1:

160.

Statements-1: A function F(x) is an antiderivative of a function f(x) if F (x) = f(x)

1+ x

Statements-2: The functions x 2 + 1, x2 , x 2 +

2 are all antiderivatives of the function 2x.

77 of 89
77

161.

Statements-1:

dx = b a , a < b

Statements-2: If f(x) is a function continuous every where in the interval (a, b) except x = c then
b

f (x)dx = f (x)dx + f (x)dx


c
3

162.

Statements-1: 4

3 + x 3 dx 2 30

Statements-2: m and M be the least and the maximum value of a continuous function
b

y = f(x) in [a, b] then m(b a) f (x)dx M(b a)


a
1

163.

Statements-1: 1 < e dx < e


x2

Statements-2: if f(x) g(x) h(x) in (a, b) then

f (x)dx g(x)dx h(x)dx

164.

Statements-1:

1 + x 4 dx < 1.2

Statements-2: For any functions f(x) and g(x), integrable on the interval (a,b), then
b

f (x)g(x)dx f
a

(x)dx g 2 (x)dx

165.

1
Statements-1: 2 dx = 2
x
1
b

Statements-2: If F(x) is antiderivative of a continuous function (a, b) then

f (x)dx =

F(b) F(a)

166.

Statements-1:

cos x
can be integrated by substitution it sinx = t.
(1 + sin x)2

Statements-2: All integrands are integrated by the method of substitution only.


167.

Statement-1 :

1 + sin x cos
x

dx = e tan x + c
2
cos x

Statement-2 : ex (f(x) + f (x)dx = ex f(x) + c


168.

169.

Statements-2:

170.

171.

e (x + 1) cos (x.e )dx = 2 x.e + 4 sin 2(x.e


Statements-2: f ( (x) ) '(x)dx, {(x) = t} equals f (t)dt .
Statements-1: log xdx = x log x x + c
Statements-1:

du

)+C

uvdx = u vdx + dx vdx dx

2
ex
x
x
x x + 4x + 2
e
dx
=
x 2 + 4x + 4 (x + 2)2 + C Statements-2: e ( f (x) + f '(x) ) dx = e f (x) + C
1
1
a
a
a
sin x x 2
x2
Statements-2: f (x) = dx = f (x)dx + f ( x)dx
Statements-1:
= 2
3+ | x |
3+ | x |
1
0
a
0
0

Statements-1:

78 of 89
78

172.

Statements-1: The value of

(1 + x)(1 + x 3 )dx can not exceed

15
8

Statements-2: If m f(x) M x [a, b] then m(b a) f (x)dx (b a)M


a
/2

173.

Statements-1:

174.

175.

176.

Statements-1:

9
(sin x)

Statements-2: Area bounded by y = 3x and y = x2 is = sq. units


dx =
5/ 2
5/ 2
(sin x) + (cos x)
4
2
5/ 2

10x 9 + 10 x log e 10
dx = log|10 x + x10| + c Statements-2:
10x + x10

e x (1 + x)
2
x
cos2 (xe x ) dx = tan (xe ) + c Statements-2: sec xdx = tan x + c
x
ln t dt
1
Statement-1 : f(x) =
(x > 0), then f(x) = - f
2
1+ t + t
x
1
Statements-1:

Statements-2: f(x) =

Statement-1 :

1 1
f = (ln x)2
x 2

ln t dt
, then f(x) +
t +1
1

sin x x 2
2x 2
=
dx
3 | x |
0 3 | x | dx .
1
1

177.

f (x)

f (x) dx = log| f (x) | + c

sin x
is an odd function. So, that
3 | x |

Statements-2: Since

sin x

3 | x | = 0 .

n+ t

178.

Statements-1 :

| sin x |dx = (2n + 1) COSt (0 t )

Statements-2:

179.

c
1

f (x) dx = f (x) dx + f (x) dx

Statements-1: The value of the integral

na

f (x) dx = n f (x) dx if f(a + x) = f(x)

and

dx belongs to [0, 1]

Statements-2: If m & M are the lower bound and the upper bounds of f(x) over [a, b] and f is integrable, then m (b a)
b

f (x) dx M(b a).


a

180.

Statements-1:

[cot

x]dx = cot1, where [] denotes greatest integer function.

0
b

Statements-2:

f (x) dx is defined only if f(x) is continuous in (a, b) [] function is discontinuous at all integers
a

(
4

181.

Statements-1:

1 + x + x 2 1 x + x 2 dx = 0 Statements-2:

f (x)dx = 0 if f(x) is an odd function.

182.

Statements-1: All continuous functions are integrable


Statements-2: If a function y = f(x) is continuous on an interval [a,b] then its definite integral over [a, b] exists.

183.

Statements-1: If f(x) is continuous on [a, b], a b and if

f (x) dx = 0 , then f(x) = 0 at least once in [a, b]


a

79 of 89
79

Statements-2: If f is continuous on [a, b], then at some point c in [a, b] f(c) =


4

184.

| x + 2 |dx = 50

Statements-1:

Statements-2:

1+ x
Statements-1: log
dx = 0
1 x
2

1
f (x) dx
b a a

f (x) dx = f (x) dx + f (x) dx where C (A, B)

185.

186.

ax
e dx =

Statement-1 If

0
10

187.

1
then
a

Statements-2: If f is an odd function

d n kx
d n 1 (1)n n!
m!
n kx
:
=
k
e
and
Statement-2
(e
)
= n +1
dx n x
x
a m +1
dx n

m ax
x e dx =
0

na

{x [x]dx = 5

Statement-1 :

Statements-1:

f (x)dx = n f (x) dx

Statements-2:

188.

| cos x | dx = 2

Statements-2:

ecos x
Statements-1: cos x
dx =
e + e cos x
0
1000

190.

Statements-1:

x [ x ]

Statements-2:

dx =1000 (e 1)

Statements-2:

dx

Statements-1:
=
tan x
1+ 2
2
0

191.

f (x)dx = f (x)dx + f (x)dx where a < c < b.


a

189.

f (x) dx = 0

Statements-2:

a
n

f (x)dx = f (a + b x)dx
e

x [x ]

dx = n e x [ x ]dx

0
b

f (x) dx = f (a + b x) dx

ANSWER
129.
137.
145.
153.
161.
169.
177.
185.

D
D
C
D
A
C
A
A

130.
138.
146.
154.
162.
170.
178.
186.

B
A
A
C
A
A
A
A

131.
139.
147.
155.
163.
171.
179.
187.

D
A
D
A
A
A
D
C

132.
140.
148.
156.
164.
172.
180.
188.

C
D
C
C
A
A
A
A

133.
141.
149.
157.
165.
173.
181.
189.

A
D
A
A
D
B
A
D

134.
142.
150.
158.
166.
174.
182.
190.

A
D
A
A
C
A
B
A

Que. from Compt. Exams


(Indefinite Integral)
1.

2.

dx
=
cos( x a) cos( x b)
sin( x a)
(a) cosec (a b ) log
+c
sin( x b )
sin( x b)
(c) cosec (a b) log
+c
sin( x a)
dx
=
x +a + x +b
2
(a)
[(x + a) 3 / 2 (x + b )3 / 2 ] + c
3(b a)
2
(c)
[(x + a) 3 / 2 + ( x + b) 3 / 2 ] + c
3(a b )

(b)
(d)

cos( x a)
+c
cos( x b)
cos( x b)
cosec (a b) log
+c
cos( x a)
cosec (a b) log

[AISSE 1989]

(b)
(d)

2
[(x + a)3 / 2 (x + b )3 / 2 ] + c
3(a b)
None of these

80 of 89
80

135. A
143. A
151. C
159. B
167. C
175. A
183. A
191. A

136.
144.
152.
160.
168.
176.
184.

C
A
C
B
A
D
A

3.

4.

5.

3 cos x + 3 sin x
[EAMCET 1991]
dx =
4 sin x + 5 cos x
27
3
27
3
(a)
(b)
x
log( 4 sin x + 5 cos x )
x+
log( 4 sin x + 5 cos x )
41
41
41
41
27
3
(d)
None of these
(c)
x
log(4 sin x 5 cos x )
41
41
1
If (sin 2 x + cos 2 x ) dx =
[Roorkee 1978]
sin(2 x c) + a , then the value of a and c is
2
(b)
(a) c = / 4 and a = k (an arbitrary constant)
c = / 4 and a = / 2
(c) c = / 2 and a is an arbitrary constant
(d)
None of these

(a)

6.

7.

(c)

11.

12.

13.

1 2 sin 2 x cos 2 x

x 2 dx

(a)

10.

sin 8 x cos 8 x

dx =

[AI CBSE 1985]

x +1 x2
log
+c

2
x 1

sin 2 x + c

(c)

9.

(1 x 2 )

(a)

(a)

8.

x3 x 2

(a + bx ) 2

x 1 x2
(b) log
+c
+
2
x +1

dx =

x +1 x2
(c) log
+c
+
2
x 1

x 1 x 2
(d) log
+c

2
x +1

[IIT 1986]

(b)

1
sin 2 x + c
2

(c)

1
sin 2 x + c
2

(d)

sin 2 x + c

[IIT 1979]

1
2a
a2
1
x+
log( a + bx )

2
b
b a + bx
b

1
2a
a2
1
x+
log(a + bx ) +

2
b
b a + bx
b
dx
=
2
2
(1 + x ) p + q 2 (tan 1 x ) 2
1
log[ q tan 1 x + p 2 + q 2 (tan 1 x ) 2 ] + c
q
2 2
( p + q 2 tan 1 x )3 / 2 + c
3q

(b)

1
2a
a2
1
x
log(a + bx ) +

2
b
b a + bx
b

(d)

1
b2

(b)

log[q tan 1 x + p 2 + q 2 (tan 1 x ) 2 ] + c

(d)

None of these

a 2a
a2 1
log(a + bx )
x +

b b
b a + bx

x5

[IIT 1985]
dx =
1+ x3
2
(a)
(b)
(1 + x 3 )3 / 2 + c
9
2
2
(c)
(d)
(1 + x 3 )3 / 2 (1 + x 3 )1 / 2 + c
3
9
dx
equals
[MP PET 2002]
sin x cos x + 2
1
1
x
x
(a)
(b)
(c)
tan + + c
tan + + c
2
8
2
2

2 8
a dx
[MP PET 1988; BIT Ranchi 1979]
=
b + ce x
ex
b + ce x
a
a
+c
+c
(a)
(b)
(c)
log
log
x
x

b
b
b + ce
e

2
2
(1 + x 3 )3 / 2 + (1 + x 3 )1 / 2 + c
3
9

None of these

x
cot + + c
2
2 8

x
cot + + c
2
2 8

(d)

(d)

b + ce x
b
log
x
a
e

sin

x dx =

[Roorkee 1977]

(a)

2[sin x cos x ] + c

(b) 2[sin x x cos x ] + c

(c)

2[sin x + cos x ] + c

(d) 2[sin x + x cos x ] + c

x2
(9 x 2 ) 3 / 2

ex
b
log
x
a
b + ce

dx =

81 of 89
81

+c

+c

(a)

14.

(a)

If

17.

18.

19.

20.

dx =
1+ x2
1
[sin 1 x 2 + 1 x 4 ] + c
2

f ( x ) sin x cos x dx =

x
+c
3

(c) sin 1

x
9 x2

+c

(d) sin 1 x 2 + 1 x 2 + c

(b)

a 2 sin 2 x b 2 cos 2 x

(log x )

(c)

1
a 2 cos 2 x + b 2 sin 2 x

(d)

1
a 2 cos 2 x b 2 sin 2 x

[AISSE 1986]

(b)

x2 +1

dx =
x x2 +1
1 + x2
+c
(a) tan 1

tan x

+ c (c) 1 tan 1 2 tan x + c


tan 1

5
2
5
5

(d) None of these

[MP PET 1991]

1 + x 2
(b) cot 1
x

dx =

+c

(c)

x 2 1
+ c (d)
tan 1

x (log x ) 2 2 x log x 2 x + c

(b)

x (log x ) 2 2 x log x x + c

(c)

x (log x ) 2 x log x + 2 x + c

(d)

x (log x ) 2 2 x log x + x + c

The value of

(x 2 a 2 )
dx will be
x

[UPSEAT 1999]

(a)

(x 2 a 2 )

( x 2 a 2 ) a tan 1

(b)

(c)

( x 2 a 2 ) + a 2 tan 1 [ x 2 a 2 ]

(d)

tan

x 2 1
+c
cot 1

[IIT 1971, 77]

(a)

(d) None of these

1
[sin 1 x 2 + 1 x 2 ] + c
2

2 x sec 2 x dx =

(x 2 a 2 )

( x 2 a 2 ) + a tan 1

tan 1 x / a + c

[IIT 1977]

1
1
sec 3 2 x sec 2 x + c
6
2
1
1
2
sec 2 x sec 2 x + c
9
3

(b)

1
1
sec 3 2 x + sec 2 x + c
6
2

(d) None of these

x sin 1 x dx =

[MP PET 1991]

x 2 1 1
x
(a)
sin x +
1 x2 +c
2
4
4

(b)

x 2 1 1
x
2

2 + 4 sin x + 4 1 x + c

x 2 1 1
x
2

(c)
2 4 sin x 4 1 x + c

(d)

x 2 1 1
x
2

2 + 4 sin x 4 1 x + c

(b)

sin 1

x x
+
a2 x 2 + c
a a

(d)

sin 1

x x

a2 x 2 + c
a a

ax
dx =
x

(a) a sin 1

(c)
23.

(b)

(c)

22.

+ sin 1

1
log( f ( x )) + c , then f (x ) =
2(b 2 a 2 )

a 2 sin 2 x + b 2 cos 2 x
dx
=
4 sin 2 x + 5 cos 2 x
2 tan x
1
+c
(a)
tan 1
5
5

(a)

21.

9 x2

1 x2

(a)

16.

(b)

sin 1 x 2 + 1 x 4 + c

(c)
15.

x
+c
3

sin 1

9 x2

x
+
a

x
a

ax
+c
a

x x

a sin 1
a2 x 2 + c
a
a

3
If x , , then
4 4

sin x cos x
1 sin 2 x

e sin x cos x dx =

82 of 89
82

24.

25.

(a) e sin x + c
(b) e sin x cos x + c
sin x + cos x
(c) e
+c
(d) e cos x sin x + c
x
x
4 e + 6e
If
dx = Ax + B log(9 e 2 x 4 ) + C , then A, B and C are [IIT 1990]
9e x 4e x
3
36
3
(a) A = , B =
, C = log 3 + constant
2
35
2
3
35
3
(b) A = , B =
, C = log 3 + constant
2
36
2
3
35
3
(c) A = , B =
, C = log 3 + constant
2
36
2
(d) None of these

The value of

sec

x dx will be

1
[ sec x tan x + log(sec
2
1
(b)
[ sec x tan x + log(sec
3
1
(c)
[ sec x tan x + log(sec
4
1
(d)
[ sec x tan x + log(sec
8
x 1 x
e dx =
( x + 1)3

(a)

26.

(c)

ex
( x + 1)
ex

( x + 1)

If I =

+c

29.

x + tan x )]
x + tan x )]
x + tan x )]
[IIT 1983; MP PET 1990]

ex

(b)

+c

(d)

( x + 1) 2
ex
( x + 1) 3

+c

+c

sin 2 x dx , then for what value of K, KI = e x (sin 2 x 2 cos 2 x ) + constant

(a) 1
(c) 5
28.

x + tan x )]

(a)

27.

[UPSEAT 1999]

The value of

(b) 3
(d) 7
dx

3 2x x

will be

[UPSEAT 1999]

(a)

1
3+ x
log

4
1 x

(b)

(c)

1
3 + x
log

2
1 x

1 x
(d) log

3+ x

1
3 + x
log

3
1 x

x 2 x + 3 dx =

x
1
(2 x + 3 ) 3 / 2
(2 x + 3)5 / 2 + c
3
15
x
1
(b)
(2 x + 3 ) 3 / 2 +
(2 x + 3)5 / 2 + c
3
15
x
1
(c)
(2 x + 3)3 / 2 + (2 x + 3)5 / 2 + c
2
6
(d) None of these
cos + sin
cos 2 log
d =
cos sin

[AISSE 1985]

(a)

30.

[IIT 1994]

cos + sin
(a) (cos sin ) 2 log

cos sin
cos + sin
(b) (cos + sin ) 2 log

cos sin

(c)

(cos sin )2
cos sin
log

2
cos + sin

83 of 89
83

[MP PET 1992]

(d)
31.

x2

(x sin x + cos x )
(c)

If u =

dx =

ax

[MNR 1989; RPET 2000]

(b)

x sin x cos x
x sin x + cos x

(d) None of these

cos bx dx and v =

sin bx dx , then (a 2 + b 2 )(u 2 + v 2 ) =

ax

2e ax

(b) (a 2 + b 2 )e 2 ax

(c) e 2 ax

(d) (a 2 b 2 )e 2 ax

(a)

33.

sin x + cos x
x sin x + cos x
sin x x cos x
x sin x + cos x

(a)

32.

1
sin 2 log tan + log sec 2
2
4
2

If In = (log x )n dx , then In + nIn 1 =


[Karnataka CET 2003]

x (log x )n

(a)

(c) (log x )n 1

34.

35.

36.

x
sin + dx =
2 4
x
x /2
(a) e
cos + c
2
x
x /2
(c) e
sin + c
2
2x + 3
If
dx = 9 ln(x
x 2 5x + 6
(a) 5 ln( x 2) + constant
(c) Constant
dx
=
2 + cos x

(b) (x log x )n
(d) n(log x )n

x/2

[Roorkee 1982]

x
+c
2
x
sin + c
2

(b)

2 e x / 2 cos

(d)

2e x / 2

3) 7 ln(x 2) + A , then A =

[MP PET 1992]

(b) 4 ln( x 3 ) + constant


(d) None of these

1
x
2 tan 1
tan + c
2
3

(a)

(b)

1
x
tan 1
tan + c
2
3
3

1
x
tan 1
tan + c (d) None of these
2
3
x
[MP PET 2004]
dx equal to
x4 + x2 +1
2x 2 + 1
2x 2 + 1
1
1

(a)
(b)
tan 1
tan 1

3
3
3
3

1
(c)
tan 1 (2 x 2 + 1)
(d) None of these
3
dx
[IIT 1984]
=
(sin x + sin 2 x )
1
1
2
(a)
log(1 cos x ) + log(1 + cos x ) log(1 + 2 cos x )
6
2
3
2
(b) 6 log(1 cos x ) + 2 log(1 + cos x ) log(1 + 2 cos x )
3
1
2
(c) 6 log(1 cos x ) + log(1 + cos x ) + log(1 + 2 cos x )
2
3
(d) None of these

(c)

37.

38.

39.

If

1
2x + 3
2 ( x 2 + 1)a
dx
=
log
(
x

1
)
tan 1 x + A ,

e
(x 1)( x 2 + 1)

where A is any arbitrary constant, then the value of a is


[MP PET 1998]

(a) 5/4

(b) 5/3

84 of 89
84

(c) 5/6

40.

(d) 5/4
a
b

(2 x + 1) dx
x +1 x 2
If
= log

+ C, then the values of a and b are respectively
2
2
(x 4 ) (x 1)
x 1 x + 2
(a) 1/2, 3/4
(b) 1, 3/2
(c) 1, 3/2
(d) 1/2,
2

[Roorkee 2000]

(Definite Integral)
1.

If I is the greatest of the definite integrals


1

I1 =

I3 =

x 2

cos 2 x dx , I 2 =

dx , I 4 =

cos 2 x dx

x 2 / 2

dx , then

(a) I = I1
(c) I = I 3
2.

x 2

(b) I = I 2
(d) I = I 4

Let f (x ) be a function satisfying f (x ) = f (x ) with f (0) = 1 and g(x ) be the function satisfying f (x ) + g(x ) = x 2 . The value of
integral

f (x ) g(x ) dx

is equal to

[AIEEE 2003; DCE 2005]

1
(e 7)
4
1
(e 3)
2

(a)
(c)
3.

1
(b)
(e 2)
4

(d) None of these

If Im = (log x )m dx satisfies the relation Im = k lIm 1 , then


1

(a) k = e

4.

(b) l = m
1
(c) k =
(d) None of these
e
Let f be a positive function. Let
I1 =

1k

x f {x (1 x )}dx , I2 =

1 k

f {x (1 x )} dx

when 2k 1 > 0 . Then I1 / I 2 is


(a) 2
(b) k
(c) 1 / 2
(d) 1
5.

If

[IIT 1997 Cancelled]

f (t) dt = x + t f (t) dt , then the value of

f (1) is
[IIT 1998; AMU 2005]

(a) 1/2
(c) 1
6.

x7

(b) 0
(d) 1/2

1 x4

dx is equal to

[AMU 2000]

(a) 1
(c)
7.

(b)

2
3

(d)

If n is any integer, then

cos 2 x

1
3

cos 3 (2n + 1)x dx =

[IIT 1985; RPET 1995; UPSEAT 2001]

(a) x
(c) 0
8.

(b) 1
(d) None of these

The value of the definite integral

(a) 0,
17
(c) 0,

27

x dx
lies in the interval [a, b]. The smallest such interval is
x 3 + 16

(b) [0, 1]
(d) None of these

85 of 89
85

9.

Let a,b,c be non-zero real numbers such that

(1 + cos

x )(ax 2 + bx + c) dx =

(1 + cos 8 x )(ax 2 + bx + c) dx

Then the quadratic equation ax 2 + bx + c = 0 has


[IIT 1981; CEE 1993]

(a)
(b)
(c)
(d)
10.

No root in (0, 2)
At least one root in (0, 2)
A double root in (0, 2)
None of these
x

If f (x ) = | t | dt , x 1, then
(a)
(b)
(c)
(d)

11.

and f are continous for x + 1 > 0


is continous but f is not continous for x + 1 > 0
and f are not continous at x = 0
is continous at x = 0 but f is not so

f
f
f
f

1
1
for t (1, 2] , then
f (t) 1, t [0, 1] and 0 f (t)
2
2

Let g(x ) = f (t) dt where


0

12.

[MNR 1994]

3
1
g(2) <
2
2
3
5
< g(2)
2
2

(a)

(b) 0 g(2) < 2

(c)

(d) 2 < g(2) < 4

The value of

cos 2 x

1 + ax

dx , a > 0, is

(a)

[IIT Screening 2001; AIEEE 2005]

(b) a

(c)

[IIT Screening 2000]

(d) 2

2
x

e
, I1 =
1+ ex

f (a)

xg { x (1 x )}dx , and I 2 =

f (a)

13.

If f (x ) =

14.

(a) 1
(b) 3
(c) 1
(d) 2
Let f : R R and g : R R be continuous functions, then the value of the integral

f ( a)

g{ x (1 x ))}dx , then the value of

f ( a )

I2
is
I1
[AIEEE 2004]

/2

/ 2

[ f ( x ) + f ( x )] [g( x ) g( x )] dx =
[IIT 1990; DCE 2000; MP PET 2001]

15.

(a)
(b) 1
(d) 0
(c) 1
The numbers P, Q and R for which the function f (x ) = Pe 2 x + Qe x + Rx satisfies the conditions f (0 ) = 1, f (log 2) = 31 and
log 4

39
are given by
2
(a) P = 2, Q = 3, R = 4 (b) P = 5, Q = 2, R = 3
(c) P = 5, Q = 2, R = 3 (d) P = 5, Q = 6, R = 3

[ f ( x ) Rx ] dx =

16.

10

n =1

10
sin 27 x dx +
2 n 1
n =1

2n

2 n +1

2n

sin 27 x dx equals

[MP PET 2002]

(b) 54
(d) 0

(a) 27 2
(c) 36
17.

Let

f (x )dx = 1, x f (x )dx = a and

Given that

(x a)

f (x ) dx =

[IIT 1990]

(b) a 2
(d) a 2 2 a + 2

(a) 0
(c) a 2 1
18.

x 2 f (x ) dx = a 2 , then the value of

x 2 dx

( x + a )(x + b )(x + c )
2

2(a + b)(b + c)(c + a)

then the value of

x 2 dx
is
(x + 4 )(x 2 + 9 )
2

[Karnataka CET 1993]

(a)

60

(b)

20

86 of 89
86

(c)
19.

(d)

40

80

If l(m , n) = t m (1 + t)n dt , then the expression for l(m , n) in terms of l(m + 1, n 1) is

[IIT Screening 2003]

2
n

l(m + 1, n 1)
m +1 m +1
n
l(m + 1, n 1)
(b)
m +1

(a)

2n
n
+
l(m + 1, n 1)
m +1 m +1
m
(d)
l(m + 1, n 1)
n +1

(c)

20.

lim

1 + 2 4 + 3 4 + .... + n 4
n5

lim

1 + 2 3 + 3 3 + .... + n 3
n5

[AIEEE 2003]

1
30
1
4

(a)
(c)
21.

If

(b) Zero
(d)

t2

xf (x )dx = 5 t

1
5

4
, t > 0, then f
=
25
[IIT Screening 2004]

(a)

2
5

(c)

(b)
2
5

5
2

(d) None of these

22.

For which of the following values of m, the area of the region bounded by the curve y = x x 2 and the line y = mx equals

23.

(b) 2
(a) 4
(c) 2
(d) 4
Area enclosed between the curve y 2 (2a x ) = x 3 and line x = 2a above x-axis is

9
2

[IIT 1999]

(a) a

(c) 2 a 2
24.

25.

26.

(d) 3 a 2

What is the area bounded by the curves x 2 + y 2 = 9 and y 2 = 8 x is


(a) 0

[MP PET 2001]

3 a 2
(b)
2

(b)

[DCE 1999]

2 2 9
1
+
9 sin 1
3
2
3

(c) 16
(d) None of these
The area bounded by the curves y =| x | 1 and y = | x | +1 is
[IIT Screening 2002]
(a) 1
(b) 2
(c) 2 2
(d) 4
The volume of spherical cap of height h cut off from a sphere of radius a is equal to
(a)
(c)

h (3 a h)
3
4 3
h
3
2

[UPSEAT 2004]

(b) (a h)(2a h ah)


2

(d) None of these


3 / 2

27.

If for a real number y, [y ] is the greatest integer less than or equal to y, then the value of the integral

[2 sin x ] dx

is

/2

(a)
(c)

[IIT 1999]

(b) 0
(d)

87 of 89
87

28.

1
2A
x
1
f ( x ) dx =
, then the constants A and B are respectively
+ B, f = 2 and
0

2
2

2
3
(a)
and
(b)
and
2
2

4
4
and 0
(d) 0 and
(c)

If f (x ) = A sin

29.

If In =

e x

n 1

dx , then

30.

In

n 1

(b)

dx =

In

(d) n In

In =

(a) In
(c)

[IIT 1995]

/4

tan n x dx , then lim n [In + In 2 ] equals


n

[AIEEE 2002]

31.

(a) 1/2
(b)
(c)
(d)
The area bounded by the curves
(b)
(a) 4 sq. unit
(c) 10 sq. unit
(d)

33.

y = ln x , y = ln | x | , y =| ln x | and y =| ln | x || is

[Kurukshetra CEE 1998]

(a) n

(b) (2n + 1)

(c)

(d) 0

If

[AIEEE 2002]

6 sq. unit
None of these

sin n + x
2

dx , (n N ) equals
sin x

32.

1
0

e x (x ) dx = 0, then

(a) 1 < < 2


(c) 0 < < 1
34.

10

| sin x | dx is

[AIEEE 2002]

(a) 20
(c) 10
35.

(b) 8
(d) 18

2 x (1 + sin x )

1 + cos 2 x
(a) 2 /4

dx is

[AIEEE 2002]

(b) 2
(d) /2

(c) 0
36.

5 7

On the interval ,
, the greatest value of the function f ( x ) =
(6 cos t 2 sin t) dt =
5 / 3
4
3
(a) 3 3 + 2 2 + 1
(c) Does not exist

37.

(b) 3 3 2 2 1
(d) None of these
1

If I1 = 2 x dx , I2 = 2 x dx , I3 = 2 x dx , I4 = 2 x dx , then
2

(a) I3 = I4
(c) I2 > I1
38.

[MNR 1994; Pb. CET 2001; UPSEAT 2000]

(b) < 0
(d) None of these

[AIEEE 2005]

(b) I3 > I4
(d) I1 > I2
1
x

If 2 f (x ) 3 f = x , then

f (x ) dx

is equal to

[J & K 2005]

(a)
(c)

3
ln 2
5
3
ln 2
5

3
(b)
(1 + ln 2)
5

(d) None of these

88 of 89
88

39.

If

x 3 dx = 0 and

x 2 dx =

2
, then the value of a and b will be respectively
3
(b) 1,1
(d) 1,1

[AMU 2005]

(a) 1, 1
(c) 1,1
40. The sine and cosine curves intersects infinitely many times giving bounded regions of equal areas. The area of one of such
region is
[DCE 2005]
(a) 2
(b) 2 2
(c) 3 2

(d) 4 2

(Indefinite Integral)
1
6
11
16
21
26
31
36

b
b
a
c
a
b
c
b

2
7
12
17
22
27
32
37

1
6
11
16
21
26
31
36

d
b
b
d
a
a
a
b

2
7
12
17
22
27
32
37

b
d
b
c
a
c
c
b

3
8
13
18
23
28
33
38

a
a
a
c
a
a
a
a

4
9
14
19
24
29
34
39

a
c
a
a
d
a
d
d

5
10
15
20
25
30
35
40

d
d
a
a
a
d
c
a

c
b
d
a
b
c
d
d

5
10
15
20
25
30
35
40

a
a
d
d
b
b
b
b

(Definite Integral)
d
c
c
a
b
c
c
d

3
8
13
18
23
28
33
38

b
a
d
a
b
c
c
b

4
9
14
19
24
29
34
39

89 of 89
89

STUDY PACKAGE
Target: IIT-JEE (Advanced)
SUBJECT: MATHEMATICS
TOPIC: 23 XII M 6. Area Under Curves
Index:
1. Key Concepts
2. Exercise I to V
3. Answer Key
4. Assertion and Reasons
5. 34 Yrs. Que. from IIT-JEE
6. 10 Yrs. Que. from AIEEE

Area Under Curve


1.

Curve Tracing :
To find the approximate shape of a curve, the following procedure is adopted in order:

(a)

Symmetry:
(i)
Symmetry about x axis:
If all the powers of ' y ' in the equation are even then the curve is symmetrical about the x axis.

E.g.: y2 = 4 a x.
(ii)

Symmetry about y axis:


If all the powers of ' x ' in the equation are even then the curve is symmetrical about the y axis.

E.g.: x2 = 4 a y.
(iii)

Symmetry about both axis;


If all the powers of ' x ' and ' y ' in the equation are even, the curve is symmetrical about the axis of ' x
' as well as ' y '.

E.g.: x2 + y2 = a2.
(iv)

Symmetry about the line y = x:


If the equation of the curve remains unchanged on interchanging ' x ' and ' y ', then the curve is
symmetrical about the line y = x.

E.g.: x3 + y3 = 3 a x y.
(v)

Symmetry in opposite quadrants:


If the equation of the curve remains unaltered when ' x ' and ' y ' are replaced by x and y respectively,
then there is symmetry in opposite quadrants.

E.g.: x y = c2.
(b)

Find the points where the curve crosses the xaxis and also the yaxis.
2

(c)

dy
Find dx and equate it to zero to find the points on the curve where you have horizontal tangents.

(d)

Examine if possible the intervals when f (x) is increasing or decreasing.

(e)

Examine what happens to y when x or x .

(f)

Asymptotes :
Asymptoto(s) is (are) line (s) whose distance from the curve tends to zero as point on curve moves towards
infinity along branch of curve.
(i)
If Lt f(x) = or Lt f(x) = , then x = a is asymptote of y = f(x)
xa

xa

Lt

If

(iii)

f (x)
If x Lt
= m1, x Lt

(f(x) m 1x) = c, then y = m1x + c1 is an asymptote. (inclined to right)


x

(iv)
Example :
Solution.

x +

f(x) = k or

Lt

(ii)

f(x) = k, then y = k is asymptote of y = f(x)

f ( x)
= m2, Lt (f(x) m2x) = c2, then y = m2x + c2 is an asymptote (inclined to left)
x
x
Find asymptote of y = ex
Lim y = Lim ex = 0

If

Lt

y = 0 is asymptote.

Example :

Find asymptotes of xy = 1 and draw graph.

Solution

y=

1
x

Lim y = Lim 1 = x = 0 is asymptote.


x 0
x 0 x
Lim y = Lim 1 = 0 y = 0 is asymptote.
x
x x
1
and sketch the curve.
x

Example :

Find asymptotes of y = x +

Solution

Lim y = Lim x + = + or
x 0
x 0
x

x = 0 is asymptote.
1

Lim y = Lim x + =
x 0
x 0
x

there is no asymptote of the type y = k.


1

Lim y = Lim 1 +
=1
x x
x
x2
1

Lim (y x) = Lim x + x = Lim 1 = 0


x
x
x x
x

y = x + 0 y = x is asymptote.
A rough sketch is as follows

2.

Quadrature :

(a)

If f(x) 0 for x [a, b], then area bounded by curve y = f(x), x-axis, x-axis, x = a and x = b is

f (x) dx
a

Example :

Find area bounded by the curve y = n x + tan1x and x-axis between ordinates x = 1 and x = 2.

Solution

y = n x + tan1x
dy
1
1
=
+
>0
dx
x
1+ x2
It is increasing function

Domain x > 0

Lt y = Lt (n x + tan1x) =
x

Lt

x 0 +

Lt + (n x + tan1x) =
y = x
0

A rought sketch is as follows


2

Required area =

(n x + tan

x ) dx

1
2
= x n x x + x tan x n (1 + x )
2

= 2 n 2 2 + 2 tan12
=

1
1
n 5 0 + 1 tan1 1 +
n 2
2
2

5
1

n 2
n 5 + 2 tan12
1
2
2
4
b

(b)

If f(x) 0 for x [a, b], then area bounded by curve y = f(x), x-axis, x = a and x = b is

f ( x ) dx
a

Example :

Find area bounded by y = log 1 x and x-axis between x = 1 and x = 2.


2

Solution.

A rought sketch of y = log 1 x is as follows


2

Area

log 1 x dx =
2

log

x . log 1 e dx
2

= log 1 e . [ x loge x

x]12

= log 1 e . (2 loge2 2 0 + 1)
2

= log 1 e . (2 loge 2 1)
2

Note : If y = f(x) does not change sign an [a, b], then area bounded by y = f(x), x-axis between
b

ordinates x = a, x = b is

f (x ) dx

(c)

If f(x) > 0 for x [a,c] and f(x) < 0 for x [c,b] (a < c < b) then area bounded by curve y = f(x) and xaxis
c

between x = a and x = b is
Example :

f ( x ) dx f ( x ) dx .

Find the area bounded by y = x3 and xaxis between ordinates x = 1 and x = 1.


4

Solution

Required area

x 3 dx + x 3 dx

0
0

x4
x3
= 4 + 4
1
0
1
1
1
= 0 +
0=
4
4
2
b

Note : Area bounded by curve y = f(x) and xaxis between ordinates x = a and x = b is | f ( x ) | dx .
a

(d)

If f(x) > g(x) for x[a,b] then area bounded by curves y = f(x) and

y = g(x) between ordinates x = a and

x = b is

(f ( x ) g( x ))dx .
a

Example :
Solution.

Find the area enclosed by curve y = x2 + x + 1 and its tangent at (1,3) between ordinates x = 1 and
x = 1.
dy
= 2x + 1
dx
dy
= 3 at x = 1
dx
Equation of tangent is
y 3 = 3 (x 1)
y = 3x
1

Required area

(x

+ x + 1 3 x ) dx

( x 2 2x + 1) dx =

x3
x 2 + x
3
1

= 1 + 1 1 1
3

2
8
+2=
3
3
Note : Area bounded by curves y = f(x) and y = g (x) between ordinates x = a and x = b is

| f ( x) g( x ) | dx .
a

(e)

If g (y) 0 for y [c,d] then area bounded by curve x = g(y) and yaxis between abscissa y = c and
d

y = d is

g( y)dy

y =c

Example :
Solution

Find area bounded between y = sin1x and yaxis between y = 0 and y =

.
2

y = sin1 x
x = sin y

Required area

sin y dy
0

cos y

2
0

= (0 1) = 1

Note : The area in above example can also evaluated by integration with respect to x.

Area = (area of rectangle formed by x = 0, y = 0 , x = 1, y =

) (area bounded by y = sin1x,


2

xaxis between x = 0 and x = 1)


1

=
1
2

sin

x dx =

(x sin1x +
2

1
1 x2 )


= + 0 0 1 = 1

2 2

Some more solved examples


Example :

Find the area contained between the two arms of curves (y x)2 = x3 between x = 0 and x = 1.
(y x)2 = x3 y = x x

Solution

3/2

For arm
dy
3 1/2
x >0
=1+
dx
2
y is increasing function.

y = x + x3/2

x > 0.

For arm
dy
3 1/2
x
=1
dx
2

y = x x3/2

dy
4 d2 y
4
3
= x 2 < 0 at x =
=0 x= ,
2
dx
9 dx
9
4

at x =

4
y = x x3/2 has maxima.
9

Required are a =

(x + x

3/2

x + x 3 / 2 ) dx

0
1

=2

x 3 / 2 dx =

Example :
Solution.

2 x5 / 2

5 / 2

=
0

4
5

Find area contained by ellipse 2x2 + 6xy + 5y2 = 1


5y2 + 6xy + 2x2 1 = 0
y=

6 x 36 x 2 20(2x 2 1)
10

3x 5 x 2
5
y is real R.H.S. is also real.

If

x=

If

x=

If

x = 0,

y=

5 <x<
5 ,
5,

5
y=3 5
y = 3 5
y=+

1
5
6

If

y = 0,

x=+

3x + 5 x 2 3x 5 x 2

5
5
5

Required area

dx

2
=
5

5 x 2 dx

5
5

4
=
5

5 x 2 dx

Put x = 5 sin : dx =
L.L : x = 0 = 0
5 =

U.L : x =

4
5

5 cos d

5 5 sin2

5 cosd

=0

cos

=4

d = 4

Example :
Solution.

1
=
2 2

Let A (m) be area bounded by parabola y = x2 + 2x 3 and the line y = mx + 1. Find the least area
A(m).
Solving we obtain
x2 + (2 m) x 4 = 0
Let , be roots + = m 2, = 4

A (m)

(mx + 1 x

2x + 3) dx

( x

+ (m 2) x + 4) dx

x3

x2

+
(
m

2
)
+ 4x
= 3

3 3 m 2 2
+
( 2 ) + 4 ( )
3
2

= | |.

A(m)

1 2
(m 2)
( + + 2 ) +
( + ) + 4
3
2

1
(m 2)
2
(m 2)2 + 16 3 (m 2) + 4 + 2 (m 2) + 4

(m 2)2 + 16

1
8
(m 2 ) 2 +
6
3

1
3/2
2
6 (m 2) + 16

Leas A(m) =

1
32
(16)3/2 =
.
6
3

Self Practice Problems


1.

2.

Find the area between curve y = x2 3x + 2 and xaxis


(i) bounded between x = 1 and x = 2.

Ans.

(ii) bound between x = 0 and x = 2.

Ans.

1
6
1

Find the area included between curves y = 2x x2 and y + 3 = 0.


Ans.

32
3

3.

Find area between curves y = x2 and y = 3x 2 from x = 0 to x = 2.


Ans.
1

4.

Curves y = sinx and y = cosx intersect at infinite number of points forming regions of equal area between
them calculate area of one such region.
Ans.
2 2

5.

Find the area of the region bounded by the parabola (y 2)2 = (x 1) and the tangent to it at ordinate y = 3
and xaxis.
Ans.
9

6.

Find the area included between y = tan1x, y = cot1x and yaxis.


Ans.
n2

7.

Find area common to circle x2 + y2 = 2 and the parabola y2 = x.


Ans.

8.

Find the area included between curves y =


Ans.

9.

4 x2
4 + x2

and 5y = 3|x| 6.

8
5

Find the area bounded by the curve |y| +


Ans.

10.

3 2

3
2
3

1
= e|x|.
2

2 (1n2)

Find the area of loop y2 = x (x 1)2.


Ans.

8
15

11.

Find the area enclosed by |x| + |y| < 3 and xy > 2.


Ans.
34n2

12.

Find are bounded by x2 + y2 < 2ax and y2 > ax, x > 0.


Ans.

3 8

a2.
6

KEY CONCEPTS
THINGS TO REMEMBER :
1.

The area bounded by the curve y = f(x) , the x-axis and the ordinates
at x = a & x = b is given by,
b

A=

f (x) dx =

2.

y dx.

If the area is below the xaxis then A is negative. The convention is


to consider the magnitude only i.e.
b

A=

y dx in this case.

3.

Area between the curves y = f (x) & y = g (x) between the


ordinates at x = a & x = b is given by,
b

A=

f (x) dx

g (x) dx =

4.

[ f (x) g (x) ] dx.

Average value of a function y = f (x) w.r.t. x over an


interval a x b is defined as :
y (av) =

1
ba

f (x) dx.

The area function Axa satisfies the differential equation

5.

d A xa
= f (x) with initial condition A aa = 0.
dx

Note : If F (x) is any integral of f (x) then ,


A xa = f (x) dx = F (x) + c
A aa = 0 = F (a) + c c = F (a)
hence A xa = F (x) F (a). Finally by taking x = b we get , A ab = F (b) F (a).
6.
(a)
(i)
(ii)
(iii)
(iv)
(v)
(b)
(c)
(d)

CURVE TRACING :
The following outline procedure is to be applied in Sketching the graph of a function y = f (x) which in
turn will be extremely useful to quickly and correctly evaluate the area under the curves.
Symmetry : The symmetry of the curve is judged as follows :
If all the powers of y in the equation are even then the curve is symmetrical about the axis of x.
If all the powers of x are even , the curve is symmetrical about the axis of y.
If powers of x & y both are even, the curve is symmetrical about the axis of x as well as y.
If the equation of the curve remains unchanged on interchanging x and y, then the curve is symmetrical
about y = x.
If on interchanging the signs of x & y both the equation of the curve is unaltered then there is symmetry
in opposite quadrants.
Find dy/dx & equate it to zero to find the points on the curve where you have horizontal tangents.
Find the points where the curve crosses the xaxis & also the yaxis.
Examine if possible the intervals when f (x) is increasing or decreasing. Examine what happens to y
when x or .

7.
(i)
(ii)
(iii)

USEFUL RESULTS :
Whole area of the ellipse, x2/a2 + y2/b2 = 1 is ab.
Area enclosed between the parabolas y2 = 4 ax & x2 = 4 by is 16ab/3.
Area included between the parabola y2 = 4 ax & the line y = mx is 8 a2/3 m3.

Q.1

Find the area bounded on the right by the line x + y = 2, on the left by the parabola y = x2 and below by
the x axis.

Q.2

Find the area of the region bounded by the curves, y = x + 2 ; y = x ; x = 0 & x = 3.

EXERCISEI

Q.3
Q.4
Q.5

Find the area of the region {(x , y) : 0 y x + 1 , 0 y x + 1 , 0 x 2}.


Find the value of c for which the area of the figure bounded by the curves y = sin 2x, the straight lines
x = /6, x = c & the abscissa axis is equal to 1/2.
The tangent to the parabola y = x2 has been drawn so that the abscissa x0 of the point of tangency
belongs to the interval [1 , 2]. Find x0 for which the triangle bounded by the tangent, the axis of ordinates
& the straight line y = x 20 has the greatest area.

Q.6

Compute the area of the region bounded by the curves y = e. x. ln x & y = ln x/(e. x) where ln e=1.

Q.7

A figure is bounded by the curves y = 2 sin

Q.8

xaxis straight lines must be drawn through (4 , 0) so that these lines partition the figure into three parts
of the same size.
Find the area of the region bounded by the curves , y = loge x , y = sin4 x & x = 0.

Q.9

x
, y = 0 , x = 2 & x = 4. At what angles to the positive
4

Find the area bounded by the curves y = 1 x 2 and y = x3 x. Also find the ratio in which the y-axis
divided this area.

Q.10 If the area enclosed by the parabolas y = a x2 and y = x2 is 18 2 sq. units. Find the value of 'a'.
Q.11
Q.12
Q.13
Q.14
Q.15
Q.16
Q.17
Q.18
Q.19

The line 3x + 2y = 13 divides the area enclosed by the curve,


9x2 + 4y2 18x 16y 11 = 0 into two parts. Find the ratio of the larger area to the smaller area.
Find the area of the region enclosed between the two circles x + y = 1 & (x 1) + y = 1
Find the values of m (m > 0) for which the area bounded by the line y = mx + 2 and
x = 2y y2 is , (i) 9/2 square units & (ii) minimum. Also find the minimum area.
Find the ratio in which the area enclosed by the curve y = cos x (0 x /2) in the first quadrant is
divided by the curve y = sin x.
Find the area enclosed between the curves : y = loge (x + e) , x = loge (1/y) & the xaxis.
Find the area of the figure enclosed by the curve (y arc sin x)2 = x x2.
For what value of 'a' is the area bounded by the curve y = a2x2 + ax + 1 and the straight line y = 0,
x = 0 & x = 1 the least ?
Find the positive value of 'a' for which the parabola y = x2 + 1 bisects the area of the rectangle with
vertices (0, 0), (a, 0), (0, a2 + 1) and (a, a2 + 1).
Compute the area of the curvilinear triangle bounded by the yaxis & the curve,
y = tan x & y = (2/3) cos x.

Q.20 Consider the curve C : y = sin 2x 3 sin x, C cuts the x axis at (a, 0) , a ( , ).
A1 : The area bounded by the curve C & the positive x axis between the origin & the ordinate at x = a.
A2 : The area bounded by the curve C & the negative x axis between the ordinate x = a & the origin.
Prove that A1 + A2 + 8 A1 A2 = 4.
Q.21 Find the area bounded by the curve y = x ex ; xy = 0 and x = c where c is the x-coordinate of the curve's
inflection point.
Q.22 Find the value of 'c' for which the area of the figure bounded by the curve, y = 8x2 x5, the straight lines
x = 1 & x = c & the abscissa axis is equal to 16/3.
Q.23 Find the area bounded by the curve y = x & x = y.
x
Q.24 Find the area bounded by the curve y = x e , the x-axis, and the line x = c where y (c) is maximum.
Q.25 Find the area of the region bounded by the xaxis & the curves defined by,
2

y = tan x , / 3 x / 3
y = cot x , / 6 x 3 / 2

10

EXERCISEII
Q.1
Q.2

In what ratio does the x-axis divide the area of the region bounded by the parabolas y = 4x x
& y = x x ?
Find the area bounded by the curves y = x4 2 x2 & y = 2 x2.

Sketch the region bounded by the curves y = 5 x 2 & y = x 1 & find its area.
Find the equation of the line passing through the origin and dividing the curvilinear triangle with vertex at
the origin , bounded by the curves y = 2 x x2 , y = 0 and x = 1 into two parts of equal area.
Q.5 Consider the curve y = xn where n > 1 in the 1st quadrant. If the area bounded by the curve, the x-axis
and the tangent line to the graph of y = xn at the point (1, 1) is maximum then find the value of n.
Q.6 Consider the collection of all curve of the form y = a bx2 that pass through the the point (2, 1), where
a and b are positive constants. Determine the value of a and b that will minimise the area of the region
bounded by y = a bx2 and x-axis. Also find the minimum area.
Q.7 In the adjacent graphs of two functions y = f(x) and y = sinx
are given. y = sinx intersects, y = f(x) at A (a, f(a)); B(, 0)
and C(2, 0). Ai (i = 1, 2, 3,) is the area bounded by the
curves y = f (x) and y = sinx between x = 0 and x = a; i = 1,
between x = a and x = ; i = 2, between x = and x = 2;
i = 3. If A1 = 1 sina + (a 1)cosa, determine the function f(x).
Hence determine a and A1. Also calculate A2 and A3.
Q.8 Consider the two curves y = 1/x & y = 1/[4 (x 1)].
(i)
At what value of a (a > 2) is the reciprocal of the area of the fig. bounded by the curves, the lines x = 2
& x = a equal to a itself ?
(ii)
At what value of b (1 < b < 2) the area of the figure bounded by these curves, the lines x = b &
x = 2 equal to 1 1/b.
ln x c
Q.9 Show that the area bounded by the curve y =
, the x-axis and the vertical line through the
x
maximum point of the curve is independent of the constant c.
Q.10 For what value of 'a' is the area of the figure bounded by the lines,
Q.3
Q.4

y=

1
4
1
,y=
, x = 2 & x = a equal to ln
?
x
2x 1
5

Q.11 Compute the area of the loop of the curve y = x [(1 + x)/(1 x)].
Q.12 Find the value of K for which the area bounded by the parabola y = x2 + 2x 3 and the line
y = Kx + 1 is least. Also find the least area.
Q.13 Let An be the area bounded by the curve y = (tan x)n & the lines x = 0, y = 0 & x = /4. Prove that for
n > 2 , An + An2 = 1/(n 1) & deduce that 1/(2n + 2) < An < 1/(2n 2).
Q.14 If f (x) is monotonic in (a, b) then prove that the area bounded by the ordinates at x = a ; x = b ; y = f (x)
a+b
and y = f (c), c (a, b) is minimum when c =
.
2
x3
x 2 + a , the straight lines x = 0, x = 2 and the
Hence if the area bounded by the graph of f (x) =
3
x-axis is minimum then find the value of 'a'.

Q.15 Consider the two curves C1 : y = 1 + cos x & C2 : y = 1 + cos (x ) for 0 , ; x [0, ]. Find

the value of , for which the area of the figure bounded by the curves C1, C2 & x = 0 is same as that of
the figure bounded by C2 , y = 1 & x = . For this value of , find the ratio in which the line y = 1 divides
the area of the figure by the curves C1, C2 & x = .
Q.16 Find the area bounded by y = 4 (x + 1), y = 4 (x 1) & y = x above axis of x.
Q.17 Compute the area of the figure which lies in the first quadrant inside the curve

11

x + y = 3 a & is bounded by the parabola x = 2 ay & y = 2 ax (a > 0).


Q.18 Consider a square with vertices at (1 , 1), ( 1 , 1), (1 , 1) & (1 , 1). Let S be the region consisting
of all points inside the square which are nearer to the origin than to any edge. Sketch the region S & find
its area.
Q.19 Find the whole area included between the curve x y = a (y x) & its asymptotes (asymptotes are the
lines which meet the curve at infinity).
Q.20 For what values of a [0 , 1] does the area of the figure bounded by the graph of the function y = f (x)
and the straight lines x = 0, x = 1 & y = f(a) is at a minimum & for what values it is at a maximum if
f (x) = 1 x 2 . Find also the maximum & the minimum areas.

Q.21 Find the area enclosed between the smaller arc of the circle x + y 2 x + 4 y 11 = 0 & the parabola
y = x + 2 x + 1 2 3 .
Q

. 2

(x) = cos 1 (4x3 3x) , x [1 , 1] and find the area


enclosed between the graph of the function and the xaxis as x varies from 0 to 1.

r a w

n e a t

a n d

c l e a n

g r a p h

t h e

f u n c t i o

Q.23 Let C1 & C2 be two curves passing through the origin as shown in the figure.
A curve C is said to "bisect the area" the region between C1 & C2, if for each
point P of C, the two shaded regions A & B shown in the figure have equal
areas. Determine the upper curve C2, given that the bisecting curve C has
the equation y = x2 & that the lower curve C1 has the equation y = x2/2.
Q.24 For what values of a [0 , 1] does the area of the figure bounded by the graph of the function y = f (x)
& the straight lines x = 0, x = 1, y = f(a) have the greatest value and for what values does it have the least
value, if, f(x) = x + 3x , , R with > 1, > 1.
x

Q.25 Given f (x) = e t (log sec t sec 2 t ) dt ; g (x) = 2ex tan x. Find the area bounded by the curves
0

y = f (x) and y = g (x) between the ordinates x = 0 and x = .


3

EXERCISEIII
Q.1
Q.2
Q.3

Q.4
Q.5
(a)

(b)

Let f (x) = Maximum {x2 , (1 x)2 , 2 x (1 x)} , where 0 x 1. Determine the area of the region
bounded by the curves y = f (x) , x axis , x = 0 & x = 1.
[ JEE '97, 5 ]
2
Indicate the region bounded by the curves x = y , y = x + 2 and xaxis and obtain the area enclosed by
them.
[ REE '97, 6 ]
2
Let C1 & C2 be the graphs of the functions y = x & y = 2x,
0 x 1 respectively. Let C3 be the graph of a function y = f (x),
0 x 1, f(0) = 0. For a point P on C1, let the lines through P,
parallel to the axes, meet C2 & C3 at Q & R respectively (see
figure). If for every position of P (on C1), the areas of the shaded
regions OPQ & ORP are equal, determine the function f(x).
[JEE '98, 8]
Indicate the region bounded by the curves y = x ln x & y = 2x 2x2 and obtain the area enclosed by
them.
[ REE '98, 6 ]
For which of the following values of m, is the area of the region bounded by the curve y = x x2 and the
line y = mx equals 9/2 ?
(A) 4
(B) 2
(C) 2
(D) 4
for x 1
x + ax + b for x > 1
2x

Let f(x) be a continuous function given by f(x) =

Find the area of the region in the third quadrant bounded by the curves, x = 2y2 and

12

y = f(x) lying on the left of the line 8x + 1 = 0.

[ JEE '99, 3 + 10 (out of 200) ]

Q.6

Find the area of the region lying inside x2 + (y 1)2 = 1 and outside c2x2 + y2 = c2 where c = 2 1 .
[REE '99, 6]

Q.7

Find the area enclosed by the parabola (y 2)2 = x 1 , the tangent to the parabola at (2, 3) and the
x-axis.
[REE 2000,3]
Let b 0 and for j = 0, 1, 2,.........n, let Sj be the area of the region bounded by the y axis and the curve
j
( j + 1)
xeay = sinby, y
. Show that S0, S1, S2,............Sn are in geometric progression. Also,
b
b
find their sum for a = 1 and b = .
[JEE'2001, 5]

Q.8

Q.9

The area bounded by the curves y = | x | 1 and y = | x | + 1 is


(A) 1
(B) 2
(C) 2 2

(D) 4

[JEE'2002, (Scr)]
Q.10 Find the area of the region bounded by the curves y = x2 , y = | 2 x2 | and y =2 , which lies to the right
of the line x = 1.
[JEE '2002, (Mains)]
Q.11

If the area bounded by y = ax2 and x = ay2 , a > 0, is 1, then a =


1
1
(B)
(A) 1
(C)
3
3

1
3
[JEE '2004, (Scr)]
Q.12(a) The area bounded by the parabolas y = (x + 1)2 and y = (x 1)2 and the line y = 1/4 is
(A) 4 sq. units
(B) 1/6 sq. units
(C) 4/3 sq. units
(D) 1/3 sq. units
[JEE '2005 (Screening)]
(b) Find the area bounded by the curves x2 = y, x2 = y and y2 = 4x 3.
(D)

4a 2 4a 1 f (1) 3a 2 + 3a
2
(c) If 4b 4b 1 f (1) = 3b 2 + 3b , f (x) is a quadratic function and its maximum value occurs at
2
4c
4c 1 f (2) 3c 2 + 3c

a point V. A is a point of intersection of y = f (x) with x-axis and point B is such that chord AB subtends
a right angle at V. Find the area enclosed by f (x) and chord AB.
[JEE '2005 (Mains), 4 + 6]

Q.13 Match the following


2

(i)

(sin x)

cos x

(cos x cot x log(sin x )sin x ) dx

(A) 1

(ii) Area bounded by 4y2 = x and x 1 = 5y2


(iii) Cosine of the angle of intersection of curves
y = 3x 1 log x and y = xx 1 is

(B) 0
(C) 6 ln 2
(D) 4/3

[JEE 2006, 6]

ANSWER
EXERCISEI
Q.1

5/6 sq. units

Q 4. c =

or
3
6

Q 7. tan 1
Q 9.

1
;
2 +1

Q 2. 21/2 sq. units

Q 3. 23/6 sq. units

Q 5. x0 = 2, A(x0) = 8

Q 6. (e2 5)/4 e sq. units

2 2
; tan 1 4 2
3
3

Q 8.
Q 10. a = 9

11
sq. units
8

Q 11.

13

3 + 2
2

Q 12.

2
3

sq. units
3
2

Q 13. (i) m = 1, (ii) m = ; Amin= 4/3

Q 14. 2

Q 15. 2 sq. units

Q 16. /4

Q 17. a= 3/4

Q 18.

Q 19.

Q 21. 1 3e2

Q 22. C = 1 or 8 17

Q 23. 1/3

Q 24.

3
1
+  n sq. units
3
2

1/ 3

1
(1 e1/2 )
2

Q 25. ln 2

EXERCISEII
Q 1. 4 : 121

Q 3. (5 2)/4 sq. units

Q 2. 128/15 sq. units

Q 4. y = 2 x/3
Q 5. 2 + 1
Q 6. b = 1/8, Aminimum = 4 3 sq. units
Q 7. f(x) = x sinx, a = 1; A1 = 1 sin1; A2 = 1 sin1; A3 = (3 2) sq. units
Q 8. a = 1 + e2 , b = 1 + e 2

Q.9

Q 11. 2 (/2) sq. units

Q 12. K = 2 , A = 32/3

Q.10 a = 8 or

1/2

Q.14 a =

Q 15. = /3 , ratio = 2 : 3

Q 16.

8 8

3 2 2
3 3

2 3
+ . arc sin
2
3

Q 18.

1
16 2 20
3

Q 17.

1 2
a sq. units
3

1
2

Q 20. a = 1/2 gives minima, A =

3/ 2

(6

21

2
3

2 22

2
5

Q 19. 4a2

3 3
; a = 0 gives local maxima A(0) = 1 ;
4
12

a = 1 gives maximum value , A(1) = /4


Q 21.

8
32
4 3+
3
3

Q 22. 3

3 1 sq. units

Q 24. for a = 1, area is greatest, for a = 1/2, area is least

Q 23. (16/9) x2
Q25.

e 3 log 2 sq. units

EXERCISEIII
Q.1

17/27

Q.5

(a) B, D (b) 257/192 ; a = 2 ; b = 1

Q.7

Q.9
Q.12

9 sq. units

Q.2 5/6 sq. units

Q.8

Q.3

f(x) = x3 x2

Q.6

2

sq. units
2 2

7/12

a
e b + 1
b
a

Sj
(e + 1)
for a = 1, b = , S = 2
and r =
= e b ;S =
0
0
+1
S j+1
a 2 + b2

20

4 2 sq. units

1
125
sq. units
(a) D ; (b) sq. units ; (c)
3
3

Q.4

Q.10

Q.11

Q.13 (i) A, (ii) D, (iii) A

14

EXERCISEIV
1.

The area bounded by the curve x 2 = 4y, x-axis and the line x = 2 is
(A) 1

2.

2
3

(C)

3
2

(D) 2

The area bounded by the x-axis and the curve y = 4x x 2 3 is


(A)

3.

(B)

1
3

(B)

2
3

(C)

4
3

(D)

8
3

The area bounded by the curve y = sin ax with x-axis in one arc of the curve is
4
a

(A)

(B)

2
a

(C)

1
a

(D) 2a

4.

The area contained between the curve xy = a2, the vertical line x = a, x = 4a (a > 0) and x-axis is
(A) a2 log 2
(C) a log 2
(D) 2a log 2
(B) 2a2 log 2

5.

The area of the closed figure bounded by the curves y = x , y = 4 3x & y = 0 is:
(A)

6.

8
9

(C) 16

(D) none

+4
4

(B)

3
4

(C)

3 + 4
4

x & x = is

2
(D)

3 4
4

The area included between the curve xy 2 = a2 (a x) & its asymptote is:
(A)

8.

(B)

The area of the closed figure bounded by the curves y = cos x; y = 1 +

(A)

7.

4
9

a2
2

(B) 2 a2

The area bounded by x + y 2 x = 0 & y = sin

(A)

(B)

(C) a2

(D) none

x
in the upper half of the circle is:
2
(C)

(D) none

9.

The area of the region enclosed between the curves 7x 2 + 9y + 9 = 0 and 5 x 2 + 9 y + 27 = 0 is:
(A) 2
(B) 4
(C) 8
(D) 16

10.

The area bounded by the curves y = x (1 ln x); x = e


x = e 1 and x = e is :

e 2 4 e 2

(A)

11.

e 2 5 e 2

(B)

4 e 2 e 2

(C)

and a positive Xaxis between

5 e 2 e 2

(D)

1
The area enclosed between the curves y = loge(x + e), x = loge y and the x-axis is

(A) 2

(B) 1

(C) 4

15

(D) none of these

12.

The area bounded by the curves


(A)

13.

1
3

(B)

x + y = 1 and x + y = 1 is

1
6

(C)

1
2

(D) none of these

The area bounded by x-axis, curve y = f(x), and lines x = 1, x = b is equal to

(b 2 + 1)

2 for all

b > 1, then f(x) is


( x 1)

(A)
14.

2
(A) (3 2x x ) dx

2
(B) (3 2x x ) dx

16.

( x 2 + 1)

(C)

(D) x / (1 + x 2 )

The area of the region for which 0 < y < 3 2x x 2 and x > 0 is
3

15.

( x + 1)

(B)

2
(C) (3 2x x ) dx

The area bounded by y = x 2 , y = [x + 1], x 1 and the y-axis is


(A) 1/3
(B) 2/3
(C) 1

(D)

(3 2 x x

) dx

(D) 7/3

The area bounded by the curve x = acos3t, y = a sin3 t is


(A)

3a 2
8

(B)

3a 2
16

(C)

3a 2
32

(D) 3a2

17.

If A1 is the area enclosed by the curve xy = 1, x -axis and the ordinates x = 1, x = 2; and A2 is the area
enclosed by the curve xy = 1, x -axis and the ordinates x = 2, x = 4, then
(A) A1 = 2 A2
(B) A2 = 2 A1
(C) A2 = 2 A1
(D) A1 = A2

18.

The area bounded by the curv e y = f (x), x -axis and the ordinates x = 1 and x = b is
(b 1) sin (3b + 4), b R, then f(x) =
(A) (x 1) cos (3x + 4)
(B) sin (3x + 4)
(C) sin (3x + 4) + 3(x 1) cos (3x + 4)
(D) none of these

19.

Find the area of the region bounded by the curv es y = x 2 + 2, y = x, x = 0 and x = 3.


(A)

20.

(B) 22 sq. unit

(C) 21 sq. unit

(D) none of these

The areas of the figure into which curve y2 = 6x divides the circle x 2 + y2 = 16 are in the ratio
(A)

21.

21
sq. unit
2

2
3

(B)

4 3
8 + 3

(C)

4 + 3
8 3

(D) none of these

The triangle formed by the tangent to the curve f(x) = x2 + bx b at the point (1, 1) and the coordinate axes,
lies in the first quadrant . If its area is 2, then the value of b is
[IIT - 2001]
(A) 1
(B) 3
(C) 3
(D) 1

EXERCISEV
1.

Find the area of the region bounded by the curve y2 = 2y x and the y-axis.

2.

Find the value of c for which the area of the figure bounded by the curves y = sin 2x, the straight lines
x = /6, x = c & the abscissa axis is equal to 1/2.

3.

For what value of 'a' is the area bounded by the curve y = a2x 2 + ax + 1 and the straight line
y = 0, x = 0 & x = 1 the least?
Find the area of the region bounded in the first quadrant by the curve C: y = tan x, tangent drawn to

4.

16

C at x =

and the x axis.


4

5.

Find the values of m (m > 0) for which the area bounded by the line y = mx + 2 and
x = 2y y2 is, (i) 9/2 square units & (ii) minimum. Also find the minimum area.

6.

Consider the two curves y = 1/x & y = 1/[4 (x 1)].


At what value of a (a > 2) is the reciprocal of the area of the figure bounded by the curves, the
(i)
lines x = 2 & x = a equal to a itself?
(ii)
At what value of b (1 < b < 2) the area of the figure bounded by these curves, the lines
x = b & x = 2 equal to 1 1/b.

7.

A normal to the curve, x 2 + x y + 2 = 0 at the point whose abscissa is 1, is parallel to the line
y = x. Find the area in the first quadrant bounded by the curve, this normal and the axis of ' x '.

8.

Find the area between the curve y2 (2 a x) = x 3 & its asymptotes.

9.

Draw a neat & clean graph of the function f (x) = cos 1 (4x 3 3x), x [1, 1] & find the area enclosed
between the graph of the function & the xaxis as x varies from 0 to 1.

10.

Find the area of the loop of the curve, a y2 = x 2 (a x).

11.

Let b 0 and for j = 0, 1, 2,......, n, let Sj be the area of the region bounded by the yaxis and the curve
xeay = sin by,

12.

their sum for a = 1 and b = .

[IIT - 2001, 5]

Find the area of the region bounded by the curves, y = x 2, y = 2 x 2 & y = 2


which lies to the right of the line x = 1.

[IIT - 2002, 5]

4a 2
2
4b
If 2
4c

13.

( j + 1)
j
y
. Show that S0, S1, S2,....., Sn are in geometric progression. Also, find
b
b

4a 1

4b 1
4c 1

3a 2 + 3a
f ( 1)
2

3b + 3b
f (1) =
, f(x) is a quadratic function and its maximum value occurs at a
3c 2 + 3c
f (2)

point V. A is a point of intersection of y = f(x) with x-axis and point B is such that chord AB subtends a
right angle at V. Find the area enclosed by f(x) and cheord AB.
[IIT - 2005, 6]

ANSWER
EXERCISEIV
1. B

2. C

3. B

4. B

5. B

6. D

6. a = 1 + e2, b = 1 + e 2

7. C

8. A

9. C

10. B

11. A

12. A

13. D

14. C

15. B

16. A

17. D

18. C

19. A

20. C

21. C

EXERCISEV
1. 4/3 sq. units

4.

2. c =

1
1
ln 2
5. (i) m = 1,
2
4

or
3
6

3. a =

7.

9.

3 1 sq. units

10.

11.

20
4 2 sq. units
3

13.

3
4

(ii) m = ; Amin = 4/3

17

7
6

8. 3 a2

8 a2
15
125
square units.
3

AREA UNDER THE CURVES


Some questions (AssertionReason type) are given below. Each question contains Statement 1 (Assertion) and
Statement 2 (Reason). Each question has 4 choices (A), (B), (C) and (D) out of which ONLY ONE is correct. So select
the correct choice :
Choices are :
(A)
Statement 1 is True, Statement 2 is True; Statement 2 is a correct explanation for Statement 1.
(B)
Statement 1 is True, Statement 2 is True; Statement 2 is NOT a correct explanation for Statement 1.
(C)
Statement 1 is True, Statement 2 is False.
(D)
Statement 1 is False, Statement 2 is True.
209.

Let |A1| be the area bounded between the curves y = |x| and y = 1 |x| ; |A2| be the area bounded between the
curves y = |x| and y = |x| 1.
Statement-1: |A1| = |A2|
Statement-2: Area of two similar parallelograms are equal.

210.

Statement-1: Area bounded between the curves y = |x 3| and y = cos1 (cosx) is 2/2
Statement-2: |x 3| = 3 x for 5/2 x 3
cos1 (cosx) = x 2, 2 x 3

211.

Statement-1: Area of the ellipse

212.

Statement-1: Area enclosed by the curve | x | + | y | = 2 is 8 units


Statement-2: | x | + | y |= 2 represents an square of side length 8 unit.

213.

Statement-1: The area bounded by y = x(x 1)2, the yaxis and the line y = 2 is

x 2 y2
+
= 1 in the first quadrant is equal to
4
1
x 2 y2
Statement-2: Area of the ellipse 2 + 2 = a 2 is ab.
a
b

2
(x (x 2) 2) dx

is equal to

10
.
3

Statement-2: The curve y = x(x 1)2 is intersected by y = 2 at x = 2 only and for 0 < x < 2, the curve y = x(x
1)2 lies below the line y = 2.
214.

Let f be a nonzero odd function and a > 0.


a

Statement-1:

f (x) = 0 . Because

Statement-2: Area bounded by y = f(x), x = a, x = a and xaxis is zero.

215.

Statement-1: The area of the curve y = sin2 x from 0 to will be more than that of the curve
y = sin x from 0 to .
Statement-2: x2 > x if x > 1.

216.

Statement-1: The area bounded by the curves y =x2 3 and y = kx + 2 is least if k = 0.


Statement-2: The area bounded by the curves y = x2 3 and y = kx + 2 is

217.

k 2 + 20 .

Statement-1: The area of the ellipse 2x2 + 3y2 = 6 will be more than the area of the circle
x2 + y2 2x + 4y + 4 = 0.
Statement-2: The length of the semi-major axis of ellipse 2x2 + 3y2 = 6 is more than the radius of the circle x2 +
y2 2x + 4y + 4 = 0.
18

18

218.

Statement-1: Area included between the parabolas y = x2/4a and the curve
y=

a2
8ab
is
(6 4) sq. units.
x 2 + 4a 2
3
x2

Statement-2: Both the curves are symmetrical about y-axis and required area is

(y

y1 ) dx

x1

219.

Statement-1: The area of the region bounded by y2 = 4x , y = 2x is 1/3 sq. units.


Statement-2: The area of the region bounded by y2 = 4ax, y = mx is

220.

8a 2
sq. units.
3m3

Statement-1: Area under the curve y = sinx, above x axis between two ordinates x = 0 & x = 2 is 4 units.
2

Statement-2:

sin x dx = 4
0

221.

Statement-1: Area under the curve y = [|sinx| + |cosx|], where [] denotes the greatest integer function. above x
axis and between the ordinates = 0 & x = is units.
Statement-2: f(x) = |sinx| + |cosx| is periodic with fundamental period /2.

222.

Statement-1: Area between y = 2 x2 & y = x is equal to

(2 + x x

)dx

Statement-2: When a region is determined by curves that intersect, the intersection points give the units of
integration.
223.

Statement-1: Area of the region bounded by the lines 2y = -x + 8, x-axis and the lines x = 3 and x = 5 is 4 sq.
units.
b

Statement-2: Area of the region bounded by the lines x = a, x = b, x-axis and the curve y = f(x) is

f (x) dx .
a

224.

Statement-1: The area of the region

included

between the parabola

y=

3x 2y + 12 = 0 is 27 sq. units.

3x
4

and the line

Statement-2: The area bounded by the curve y = f(x) the x-axis and x = a, x = b is

f (x)dx, where f is a
a

continuous function defined on [a, b].


225.

(x, y) : 0 y x 2 + 1,
23
sq. units.
=
0 y x + 1, 0 x 2 3

Statement-1: The area of the region

Statement-2: The area bounded by the curves y = f(x), x-axis ordinates x = a, x = b is

f (x)dx
a

226.

Statement-1: Area bounded by y2 = 4x and its latus rectum = 8/3


Statement-2: Area of the region bounded by y2 = 4ax and it is latus rectum 8a2/3

Answer Key
209. A 210. A
217. B 218. A
224. A 225. D

211. D
219. A
226. A

212. A
220. C

213. A
221. B

19

19

214. C
222. B

215. D 216. C
223. A

Details Solution
209.

Clearly |A1| = |A2|

y = |x| - 1
y = |x|

y = -|x|

y = 1 - |x|
A2
A1
3

210.

=2

( x 2 ) ( 3 x ) dx = 2

5 / 2

212.
213.

(2x 5)dx = 2/2.

5 / 2

x
y
1

+
= 1 in the first quadrant = 2 1 = .
4
1
4
2
(A) Clearly | x | + | y | = 2 represents a square of 8 units and area of square is equal to square
of the side length.
2

211.

(d) Area of ellipse

Solving y = x(x 1)2 and y = 2, we get x = 2. Hence y = x(x 1)2 intersects the line y = 2 at
x = 2 only.
Statement II is true because of above and the graphs of y = 2 and y = x(x 1)2.
Statement I is obviously true and it is because of statement II.
Hence (a) is the correct answer.
y

O
1

214.

Statement I is true, as this is a property of definite integral.


As f is nonzero function, area bounded by given boundaries can not be zero.
Hence statement II is false.
Hence (c) is the correct answer.

215.

sin2x sin x : x (0, )


Therefore area of y = sin2 x will be lesser from area of y = sin x.
Statement II is obviously true.
Hence (d) is the correct answer.

216.

Let the line y = kx + 2 cuts y = x2 3 at x = and = , area bounded by the curves =

(y

y 2 ) = ( kx + 2 ) ( x 2 3) dx

(k
f(k) =

+ 20 )

3/ 2

6
which clearly shows that statement II is false but f(k) is least when k = 0.
Hence (c) is the correct answer.
217.

Option (b) is correct.

20

20

x 2 y2
+
= 1 & the circles is (x 1)2 + (y + 2)2 = 1.
3 2
Area of ellipse = 3 2 = 6 and area of circle = . (1)2 =
The ellipse

The Statement-2 is true in this particular example. In general, this may not be true.

2a

218.

2a 2

8a 3
x

dx
dx
2
2

4a
0
0 x + 4a

Required area = 2

a2
(6 - 4)
3
4a / m2

219.

Req. area =

4ax mx dx

8a 2
sq. units
3m 3

220.

sin x dx = [ cos x ]

2
0

= [-cos2 - (-cos(0))]

= [1 (1)] = 0
So, c is correct.
221.

1 |sinx| + |cosx| 2
So [|sinx| + |cosx|] = 1

So 1.dx =
0

b is correct.
5

8x
1
x2
= 4 sq. units.
dx
=
8x

3 2
2
2 3
5

223.

Area =

224.

(A)
Required area

3x + 12 3 2
x dx = 27 sq. units.
2
4
2
4

225.

(D)
Required area is
1

2
(x + 1)dx + (x + 1)dx =

226.

23
sq. units.
6

area = ar (OAS)
1

x dx

0
1

4
2 3/ 2 4
=2
3 .x = 3 = 3
0
Whose area = 2

8a 2
4 8
= that is latus rectum by reason have latus rectum =
3
3 3

21

21

Ans. (A)

STUDY PACKAGE
Target: IIT-JEE (Advanced)
SUBJECT: MATHEMATICS
TOPIC: 24 XII M 7. Differential
Equations
Index:
1. Key Concepts
2. Exercise I to VIII
3. Answer Key
4. Assertion and Reasons
5. 34 Yrs. Que. from IIT-JEE
6. 10 Yrs. Que. from AIEEE

Differential Equation
1.

Introduction :
An equation involving independent and dependent variables and the derivatives of the dependent variables is
called a differential equation. There are two kinds of differential equation:
1.1

Ordinary Differential Equation : If the dependent variables depend on one independent variable
x, then the differential equation is said to be ordinary.
dy
dz
+
= y + z,
for example
dx
dx
d3 y

dy
+ xy = sin x ,
dx

for example

2.

+2

dy
+ y = ex ,
dx

3/2

dy 2
dy
dy
1
+

, y=x
k 2 =
+ k 1 + dx
dx

dx
dx
Partial differential equation : If the dependent variables depend on two or more independent
variables, then it is known as partial differential equation
2

d2 y

1.2

dx

y2

2
2
z
2z
+y
= ax, z + z = 0
x
y
2
x
y2

Order and Degree of a Differential Equation:


2.1
2.2

Order : Order is the highest differential appearing in a differential equation.


Degree :
It is determined by the degree of the highest order derivative present in it after the differential
equation is cleared of radicals and fractions so far as the derivatives are concerned.
n

n
dm y 1
dm1y 2
dy k
f 1 (x, y) m + f 2 (x, y) m1 + ........ fk(x, y) = 0
dx
dx
dx

The above differential equation has the order m and degree n1.

Example :
Find the order & degree of following differential equations.

(i)

(iii)

d2 y
dx 2

dy

y
+

=
dx

1/ 4

dy d2 y

sin dx + 2 = y
dx

dy d2 y
+

dx dx 2

(ii)

y= e

(iv)

ey xy + y = 0

Solution.
4

(i)
(ii)
(iii)

6
d2 y

= y + dy
dx 2
dx

d2 y
dx

d2 y
dx

d3 y

order = 2, degree = 4

dy
= ny
dx

order = 2, degree = 1

dy
= sin1 y
dx

order = 2, degree = 1

3
e dx

d2 y

+y=0

equation can not be expressed as a polynomial in differdx 2


ential coefficients, so degree is not applicable but order is 3.
Self Practice Problems :
Find order and degree of the following differential equations.
1.
dy
1
(i)
+y=
Ans.
order = 1, degree = 2
dy
dx
dx
3
d5 t

dy d y

+ 1
dx dx 3 = n
(ii)
Ans.
order = 5, degree = not applicable.
5

e
dx

(iv)

(iii)

3.

dy 1 / 2

+ y
dx

d2 y

Ans.

dx 2

order = 2, degree = 2

Formation of Differential Equation:

Differential equation corresponding to a family of curve will have :


(a) Order exactly same as number of essential arbitrary constants in the equation of curve.
(b) no arbitrary constant present in it.
The differential equation corresponding to a family of curve can be obtained by using the following
steps:
(a)
Identify the number of essential arbitrary constants in equation of curve.
NOTE : If arbitrary constants appear in addition, subtraction, multiplication or division, then we can club
them to reduce into one new arbitrary constant.
(b)
Differentiate the equation of curve till the required order.
(c)
Eliminate the arbitrary constant from the equation of curve and additional equation obtained in step
(b) above.
Example :
Form a differential equation of family of straight lines passing through origin.
Sol.
Family of straight lines passing through origin is y = mx wherem is parameter.
Differentiating w.r.t. x
dy
=m
dx
Eliminating m from both equations
dy
y
=
dx
x
which is the required differential equation.
Example :
Form a differential equation of family of circles touching x-axis at the origin ?
Sol.
Equation of family of circles touching x-axis at the origin is
x2 + y2 + y = 0
..........(i)
where is parameter
dy
dy
2x + 2y
+
=0
.........(ii)
dx
dx
Eliminating from (i) and (ii)
2xy
dy
= 2
x y2
dx
which is required differential equation.
Self Practice Problems :
Obtain a differential equation of the family of curves y = a sin (bx + c) where a and c being arbitrary constant.
1.

2.

d2 y

+ b2y = 0
dx 2
Show the differential equation of the system of parabolas y2 = 4a(x b) is given by
Ans.

d2 y

3.

dy
y
+ =0
dx 2
dx
Form a differential equation of family of parabolas with focus origin and axis of symmetry along the
x-axis.
2

Ans.

4.

dy
dy
y2 = y2 + 2xy
dx
dx

Solution of a Differential Equation:


Finding the dependent variable from the differential equation is called solving or integrating it. The solution or
the integral of a differential equation is, therefore, a relation between dependent and independent variables
(free from derivatives) such that it satisfies the given differential equation
NOTE : The solution of the differential equation is also called its primitive, because the differential equation
can be regarded as a relation derived from it.
There can be three types of solution of a differential equation:
(i) General solution (or complete integral or complete primitive) : A relation in x and y satisfying a
given differential equation and involving exactly same number of arbitrary constants as order of
differential equation.
(ii) Particular Solution : A solution obtained by assigning values to one or more than one arbitrary constant of general solution.
(iii) Singular Solution : It is not obtainable from general solution. Geomatrically, General solution acts
as an envelope to singular solution.

5.

Differential Equation of First Order and First Degree :


A differential equation of first order and first degree is of the type
dy
+ f(x, y) = 0, which can also be written as :
dx
Mdx + Ndy = 0, where M and N are functions of x and y.
3

6.

Elementary Types of First Order and First Degree Differential


Equations :
6.1

Variables separable : If the differential equation can be put in the form, f(x) dx = (y) dy w e
say that variables are separable and solution can be obtained by integrating each side separately.

Example :
Solution.

A general solution of this will be f ( x ) dx = ( y ) dy + c, where c is an arbitrary constant


Solve the differential equation
(1 + x) y dx = (y 1) x dy
The equation can be written as y 1
1+ x
dy

dx =
x
y
1

x + 1 dx = 1 y dy

Solution.

n x + x = y ny + c
ny + nx = y x + c
xy = ceyx
dy
Solve :
= (ex + 1) (1 + y2)
dx
The equation can be written asdy
= (e x + 1)dx
1+ y2
Integrating both sides,
tan1 y = ex + x + c.

Example :

Solve : y x

Example :

Solution.

2 dy
dy

= a y +
dx
dx

The equation can be written as dy


y ay2 = (x + a)
dx
dx
dy
=
x + a y ay 2

dx
1
=
x + a y(1 ay ) dy
1
dx
a

= +
x + a y 1 ay dy
Integrating both sides,
n (x + a) = n y n (1 ay) + n c
cy
n (x + a) = n 1 ay

cy = (x + a) (1 ay)
where 'c' is an arbitrary constant.
6.1.1 Sometimes transformation to the polar co-ordinates facilitates separation of variables. In this
connection it is convenient to remember the following differentials:
If x = r cos ; y = r sin then,
(i)
x dx + y dy = r dr (ii) dx2 + dy2 = dr2 + r2d2
(iii) x dy y dx = r2d
If x = r sec & y = r tan then
(i)
x dx y dy = r dr (ii) x dy y dx = r2 sec d.
Example :
Solve the differential equation xdx + ydy = x (xdy ydx)
Solution.
Taking x = r cos, y = r sin
x2 + y2 = r2
2x dx + 2ydy = 2rdr
xdx + ydy = rdr
.........(i)
y
= tan
x
dy
d
y
d
dx
= sec2 .
dx
x2
xdy y dx = x2 sec2 . d
xdy ydx = r2 d
........(ii)
Using (i) & (ii) in the given differential equation then it becomes
r dr = r cos. r2 d
4

dr

= cos d
r2
1
= sin +
r
1

=
2
x + y2

y
x + y2
2

y +1

6.1.2

Example :
Solution.

= c where = c
x2 + y2
2
(y + 1) = c(x2 + y2)
Equations Reducible to the Variables Separable form : If a differential equation can be reduced into a variables separable form by a proper substitution, then it is said to be
dy
Reducible to the variables separable type. Its general form is
= f(ax + by + c) a, b 0. To
o
dx
solve this, put ax + by + c = t.
dy
Solve
= (4x + y + 1)2
dx
Putting 4x + y + 1 = t
dy
dt
4+
=
dx
dx
dy
dt
=
4
dx
dx
Given equation becomes
dt
4 = t2
dx
dt
= dx
(Variables are separated)
t2 + x
Integrating both sides,
dt
= dx
4 + t2
4x + y + 1
1
t
1
=x+c

tan1
=x+c

tan1
2

2
2
2

Example :
Solution.

dy
sin1 = x + y
dx
dy
= sin (x + y)
dx
putting x + y = t
dy
dt
=
1
dx
dx
dt

1 = sin t
dx
Integrating both sides,
dt
dx
1 + sin t =

Solve

dt
= 1 + sin t
dx

1 sin t

cos
(sec

dt = x + c

t sec t tan t ) dt = x + c
tan t sec t = x + c
1 sin t
=x+c

cos t
t
t
cos sin
2
2

t
t =x+c
cos + sin
2
2
t
tan = x + c
4 2
2

dt
1 + sin t = dx

x+y
+x+c=0
tan
2
4
Self Practice Problems :

1.

2.

Solve the differential equation


dy
x2 y
= (x + 1) (y + 1)
dx

Ans.

Solve the differential equation


xdx + ydy
ydx xdy
=
x2 + y2
x2
Solve :

4.

Solve : xy

5.

Solve

6.
7.

x2 + y2 +

Ans.

dy
= ex + y + x2ey
dx

3.

y n (y + 1) = nx

= ex +

1
+c
x

y
=c
x
x3
+c
3

Ans.

Ans.

y = x + n |x (1 + y)| + c

Ans.

ey x = x + c

dy
= sin(x + y) + cos (x + y)
dx

Ans.

log tan

dy
= x tan (y x) + 1
dx
6.2
Homogeneous Differential Equations :

Ans.

sin (y x) = ex + c

dy
= 1 + x + y + xy
dx

dy
= 1 + ex y
dx

Solution.

x+y
+1 = x + c
2

f ( x, y )
dy
=
where f and g are homogeneous function of
g( x, y )
dx
x and y, and of the same degree, is called homogeneous differential equaiton and can be solved
easily by putting y = vx.
y 2
y

dy
Solve 2 + x 1
x

dx
Putting y = vx
dy
dv
=v+x
dx
dx
dv

=0
2v + (v 2 1) v + x
dx

A differential equation of the from

Example :

ey

v+x
x

dv
2v
= 2
dx
v 1

dv
v(1 + v 2 )
=
dx
v2 1

v2 1

v(1 + v
2v

dv =

dx

1
dv = n x + c
v
2
n (1 + v ) n v = n x + c

1 + v

1+ v2
.x
n
v
x2 + y2
n
y
x2 + y2 = yc'

=c

=c
where

c = ec

Example :

Solve : (x2 y2) dx + 2xydy = 0 given that y = 1 when x = 1

Solution.

x2 y2
dy
=
2xy
dx
6

y = vx
dy
dv
=v +
dx
dx
2v

1+ v

at

6.2.1

dv =

v+x

dv
1 v2
=
dx
2v

dx

n (1 + v 2) = nx + c
x = 1, y = 1

v=1
n 2 = c

y 2

n 1 + 2 . x = n2
x

2
2
x + y = 2x

Equations Reducible to the Homogeneous form


dy
ax + by + c
=
Equations of the form
.........(1)
dx Ax + By + C
can be made homogeneous (in new variables X and Y) by substituting x = X + h and y = Y + k,
aX + bY + (ah + bk + c )
dY
where h and k are constants, we get
=
.
.........(2)
AX
+ BY + ( Ah + Bk + C)
dX
Now, h and k are chosen such that ah + bk + c = 0, and Ah + Bk + C = 0; the differential equation
can now be solved by putting Y = vX.
x + 2y 5
dy
= 2x + xy 4
dx
x = Y + h,
y=Y+k
dy
dy dY dX
=
.
.
dx
dY dX dx
dY
= 1.
. 1.
dX
dY
dY
X + h + 2( Y + k ) 5
=

=
dX
dX
2 X + 2h + Y + k 4
X + 2Y + (h + 2k 5)
=
2 X + Y + (2h + k 4)
h & k are such that
h + 2k 5 = 0 & 2h + k 4 = 0
h = 1, k = 2
dY
X + 2Y
=
which is homogeneous differential equation.
dX
2x + Y

Example :

Solve the differential equation

Solution.

Let

Now, substituting Y = vX
dY
dv
=v+X
dX
dX
dv
1 + 2v

X
=
v
dX
2+v
2+v
dx
2 dv =
X
1 v

3
1

dv = n X + c
+
2
(
v
+
1
)
2
(
1
v )

1
3
n (v + 1)
n (1 v) = n X + c
2
2
v +1
n
= nX2 + 2c
(1 v )3
( Y + Y)

X2

= e2c
( X Y) X2
X + Y = c(X Y)3
where e2c = c1
x 1 + y 2 = c ( 1 y + 2)3
x + y 3 = c (x y + 1)3
3

Special case :
(A)
In equation (1) if aB = Ab, then the substitution ax + by = v will reduce it to the form in which
variables are separable.
7

2x + 3 y 1
dy
= 4 x + 6y 5
dx

Example :

Solve

Solution.

Putting u = 2x + 3y
du
dy
= 2+3.
dx
dx

1 du
u 1
2 =

3 dx
2u 5

du
3u 3 + 4u 10
=
dx
2u 5
2u 5
dx = dx
7u 13
1
2
9
1.du

. du = x + c
7
7 7u 13
2
9 1

u .
n (7u 13) = x + c
7
7 7
9

4x + 6y
n (14x + 21y 13) = 7x + 7c
7
9

3x + 6y
n (14x + 21y 13) = c
7
In equation (1), if b + A = 0, then by a simple cross multiplication equation (1) becomes an
exact differential equation.

(B)

Example :
Solution.

x 2y + 5
dy
= 2x + y 1
dx
Cross multiplying,
2xdy + y dy dy = xdx 2ydx + 5dx
2 (xdy + y dx) + ydy dy = xdx + 5 dx
2 d(xy) + y dy dy = xdx + 5dx
On integrating,

Solve

y2
x2
y=
+ 5x + c
2
2
2
2
x 4xy y + 10x + 2y = c
2xy +

(C)

where

c = 2c

If the homogeneous equation is of the form :


yf(xy) dx + xg(xy)dy = 0, the variables can be separated by the substitution xy = v.

Self Practice Problems :


Solve the following differential equations
1.
2.

dy

y tan1 y = x given that y = 0 at x = 1


x
dx

x
dy
y
x
= y x tan
dx
x

Ans.

x 2 + y 2 = e x tan

Ans.

x sin

1 y

y
=C
x

3.

x + 2y 3
dy
= 2x + y + 3
dx

Ans.

x + y = c (x y + 6)3

4.

x + y +1
dy
= 2 x + 2y + 3
dx

Ans.

3(2y x) + log (3x + 7y + 4) = C

5.

3 x + 2y 5
dy
= 3 y 2x + 5
dx

Ans.

3x2 + 4xy 3y2 10x 10y = C

6.3

Exact Differential Equation :

dy
=0
...........(1)
dx
Where M and N are functions of x and y is said to be exact if it can be derived by direct differentiation
(without any subsequent multiplication, elimination etc.) of an equation of the form f(x, y) = c
dx
e.g.
y2 dy + x dx +
= 0 is an exact differential equation.
x

The differential equation M + N

M N
The necessary condition for (1) to be exact is y = x .
(ii)
For finding the solution of Exact differential equation, following exact differentials must be remembered :
y
xdy ydx
= d
(c) 2(x dx + y dy) = d (x2 + y2)
(a) xdy + y dx = d(xy)
(b)
2
x
x

NOTE : (i)

(d)

xdy + ydx

(g)

xdy ydx

xdy ydx
y
ln
=
d
xy
x
x2 y2

(e)

x +y
2

1 y

= d tan
x

(f)

xdy + ydx
= d(ln xy)
xy

1
= d xy

xdy ydx

Example :

Solve : y dx + x dy =

Solution.

ydx + xdy =

x2 + y2

xdy ydx
x2 + y2
d (xy) = d (tan y/x)
Integrating both sides xy = tan1 y/x + c
1

Example :
Solution.

x2

Solve : (2x ny) dx + y + 3 y dy = 0

The given equation can be written as dy


ny (2x) dx + x2 y + 3y2 dy = 0

ny d (x2) + x2 d (ny) + d (y3) = 0

d (x2 ny) + d (y3) = 0


Now integrating each term, we get
x2 ny + y3 = c

Self Practice Problems :


1.

Solve : xdy + ydx + xy ey dy = 0

2.

Solve : yex/y dx (xex/y + y3) dy = 0

6.4

Linear Differential Equation :

Ans.
Ans.

n (xy ) + ey = c

2ex/y + y2 = c

When the dependent variable and its derivative occur in the first degree only and are not multiplied together,
the differential equation is called linear
The mth order linear differential equation is of the form.
dm y

dm1y

dy
m1 + .................... + Pm1 (x) dx + Pm (x) y = (x),
dx
dx
where P0(x), P1(x) ..................Pm(x) are called the coefficients of the differential equation.
dy
NOTE :
+ y2 sinx = lnx is not a Linear differential equation.
dx
Linear differential equations of first order :
dy
The differential equation
+ Py = Q , is linear in y..
dx
where P and Q are functions of x.
Integrating Factor (I.F.) : - It is an expression which when multiplied to a differential equation converts it
into an exact form.
Pdx
I.F for linear differential equation = e
(constant of integration will not be considered)
after multiplying above equation by .F it becomes;
Pdx
Pdx
dy Pdx
.e
+ Py . e
= Q. e
dx
d
Pdx

( y. e Pdx ) = Q. e
dx

P0(x)

+ P1(x)

Pdx
+C.

y. e Pdx = Q. e
NOTE : Some times differential equation becomes linear if x is taken as the dependent variable and y as
independent variable. The differential equation has then the following form :

dx
dy + P1 x = Q1.
where P1 and Q1 are functions of y.
P1 dy
The .F. now is e
Example :
Solve

Solution.

dy
3x 2
sin2 x
y=
+
3
dx
1+ x
1+ x3
dy
+ Py = Q
dx

P=

3x 2
1+ x3
3x2

F = e P.dx = 1+ x 3 dx = e n(1+ x
e
General solution is

y(F) =

= 1 + x3

Q(IF).dx + c
sin2 x

1+ x

y (1 + x3) =

(1 + x3) dx + c

1 cos 2x
dx + c
2
1
sin 2x
y(1 + x3) =
x
+c
2
4

y(1 + x3) =

Example :
Solution.

dy
+ y = 2 n x
dx
dy
1
2
+
y=
dx
xnx
x
1
2
P=
,Q=
xnx
x

Solve : x nx

IF = e P.dx = e xnx dx = e n( nx ) = n x

General solution is
2
.nx.dx + c
y. (n x) =
x
y (n x) = (n x)2 + c

Example :
Solution.

Solve the differential equation


t (1 + t2) dx = (x + xt2 t2) dt and it given that x = /4 at t = 1
t (1 + t2) dx = [x (1 + t2) t2] dt
x
t
dx
= t
(1 + t 2 )
dt
dx x
t
=
dt t
1+ t2
which is linear in
Here, P =
IF = e

dx
dt

1
t
, Q=
t
1+ t 2

1
t dt = ent = 1

t
General solution is 1
t
1
.
dt + c
x- =
t 1+ t2
t
x
= tan1 t + c
t
putting x = /4, t = 1
/4 = /4 + c
c=0

x = t tan1 t

10

Equations reducible to linear form


6.4.1 By change of variable.
dy
Example :
Solve : y sinx
= cos x (sinx y2)
dx
Solution.
The given differential equation can be reduced to linear form by change of variable by a suitable
subtitution.
Substituting y2 = z
dy
dz
=
2y
dx
dx
differential equation becomes
sin x dz
+ cos x.z = sin x cos x
2 dx
dz
dz
+ 2 cot x . z = 2 cos x which is linear in
dx
dx

IF = e

2 cot x dx

z. sin2 x =

= e 2n sin x = sin2 x

2 cos x. sin

General solution is -

x. dx + c

2
sin3 x + c
3
Bernoullis equation :
dy
+ Py = Q.yn, n 0 and n 1
Equations of the form
dx
where P and Q are functions of x, is called Bernoullis equation and can be made linear in v by
dividing by yn and putting y n+1 = v. Now its solution can be obtained as in (v).
dy
e.g. : 2 sin x
y cos x = xy3 ex .
dx

y2 sin2x =

6.4.2

Example :
Solution.

dy y y 2
=
(Bernoulli's equation)
dx x x 2
2
Dividing both sides by y
1 dy 1
1

= 2
..... (1)
2 dx
xy x
y

Solve :

1
Putting y = t
1 dy dt
=
y 2 dx dx
differential equation (1) becomes,
dt t
1
=

dx x x 2
dt t
1
dt
+ = 2 which is linear differential equation in
dx x
dx
x

IF = e x

dx

= enx = x

General solution is 1
t. x = 2 . x dx + c
x
tx = nx + c
x
y = nx + c
Self Practice Problems :

1.

Solve : x (x2 + 1)

2.

Solve : (x + 2y3)

3.

Solve : x

4.

dy
= y (1 x2) + x2 nx
dx
dy
=y
dx

dy
+ y = y2 log x
dx
Solve the differential equation

Ans.

x 2 + 1

x y = x n x x + c

Ans.

x = y (c + y2)

Ans.

y (1 + cx + log x) = 1

dy
xy2 2y3 = 2x3 given y = 1 at x = 1
dx

Ans.
11

y3 + 2x3 = 3x6

7.

Clairauts Equation :
The differential equation
y = px + f(p),

..............(10), where p =

dy
dx

is known as Clairouts Equation.


To solve (10), differentiate it w.r.t. x, which gives
dp
=0p=c
...........(11)
either
dx
or
x + f(p) = 0
............(12)
NOTE : (i)
If p is eliminated between (10) and (11), the solution obtained is a general solution of (10)
(ii)
If p is eliminated between (10) and (12), then solution obtained does not contain any arbitrary
constant and is not particular solution of (10). This solution is called singular solution of (10).
dy
Example :
Solve : y = mx + m m3
where, m =
dx
Solution.
y = mx + m m3
..... (i)
The given equation is in clairaut's form.
Now, differentiating wrt. x dy
dm dm
dm
=m+x
+
3m 2
dx
dx
dx
dx
dm dm
dm
+
3m 2
m=m+x
dx
dx
dx
dm
(x + 1 3m2) = 0
dx
dm
=0

m=c
..... (ii)
dx
x +1
or
x + 1 3m2 = 0

m2 =
..... (iii)
3
Eliminating 'm' between (i) & (ii) is called the general solution of the given equation.
y = cx + c c3 where, 'c' is an arbitrary constant.
Again, eliminating 'm' between (i) & (iii) is called singular solution of the given equation.
y = m (x + 1 m2)
1/ 2

x + 1

y=
3

1/ 2

x + 1

y=
3

x + 1

x + 1

2
(x + 1)
3

3/2

x + 1

y=2
3
4
(x + 1)3
y2 =
27
27y2 = 4 (x + 1)3
Self Practice Problems :
1.
Solve the differential equation
dy
dx
Ans.
General solution : y = cx + 2/c where c is an arbitrary constant
Singular solution : y2 = 8x
dy
Solve : sin px cos y = cos px sin y + p where p =
dx
Ans.
General solution : y = cx sin1 (c) where c is an arbitrary constant.

Y = mx + 2/m

2.

where, m =

Singular solution :

y=

Orthogonal Trajectory :

x 2 1 sin1

x2 1
x2

An orthogonal trajectory of a given system of curves is defined to be a curve which cuts every member of
a given family of curve at right angle.
Steps to find orthogonal trajectory :
(i)
Let f (x, y, c) = 0 be the equation of the given family of curves, where 'c' is an arbitrary constant.
(ii)
Differentate the given equation w.r.t. x and then eliminate c.
12

(iii)
(iv)

dy
dx
by
in the equation obtained in (ii).
dx
dy
Solve the differential equation obtained in (iii).
Hence solution obtained in (iv) is the required orthogonal trajectory.

Replace

Example :
Solution.

Find the orthogonal trajectory of family of straight lines passing through the origin.
Family of straight lines passing through the origin is y = mx ..... (i)
where 'm' is an arbitrary constant.
Differentiating wrt x
dy
=m
..... (ii)
dx
Eliminate 'm' from (i) & (ii)
dy
x
y=
dx
dx
dy
Replacing
by dy ,
we get
dx
dx
y = dy x
x dy + y dy = 0
Integrating each term,

x2 y2
+
=c
2
2

x2 + y2 = 2c
which is the required orthogonal trajectory.
Example :
Solution.

Find the orthogonal trajectory of y2 = 4ax (a being the parameter).


y2 = 4ax
..... (i)
dy
2y
= 4a
..... (ii)
dx
Eliminating 'a' from (i) & (ii)
dy
y2 = 2y
x
dx
dx
dy
by dy , we get
Replacing
dx
dx
y = 2 dy x

2 x dx + y dy = 0
Integrating each term,

y2
=c
2
2
2
2x + y = 2c
which is the required orthogonal trajectories.
x2 +

Self Practice Problems :


1.
Find the orthogonal trajectory of family of circles concentric at (a, 0)
Ans.
y = c (x a)
where c is an arbitrary constant.
2.

Find the orthogonal trajectory of family of circles touching x axis at the origin.
Ans.
x2 + y2 = cx
where c is an arbitrary constant.

3.

Find the orthogonal trajectory of the family of rectangular hyperbola xy = c2


Ans.
x2 y2 = k
where k is an arbitrary constant.

Geometrical application of differential equation :


Example :
Solution.

Find the curves for which the portion of the tangent included between the co-ordinate axes is
bisected at the point of contact.
Let P (x, y) be any point on the curve.
Equation of tangent at P (x, y) is -

13

B
P(
x,
y)

Y y = m (X x) where m =

dy
is slope of the tangent at P (x, y).
dx

mx y
, 0 & B (0, ymx)
Co-ordinates of A
m

P is the middle point of A & B


mx y

= 2x
m

mx y = 2mx

mx = y
dy

x=y
dx
dy
dx

+ y =0
x

nx + ny = nc

xy = c

Example :
Solution.

Example :
of
which
Solution.

Show that (4x + 3y + 1) dx + (3x + 2y + 1) dy = 0 represents a hyperbola having as asymptotes


the lines x + y = 0 and 2x + y + 1 = 0
(4x + 3y + 1) dx + (3x + 2y + 1) dy = 0
4xdx + 3 (y dx + x dy) + dx + 2y dy + dy = 0
Integrating each term,
2x2 + 3 xy + x + y2 + y + c = 0
2x2 + 3xy + y2 + x + y + c = 0
which is the equation of hyperbola when x2 > ab & 0.
Now, combined equation of its asymptotes is 2x2 + 3xy + y2 + x + y + = 0
which is pair of straight lines

=0
1 1 3
1
1
9
1.

=0

2.1 + 2 . . . 2 .
2 2 2
4
4
4

= 0

2x2 + 3xy + y2 + x + y = 0
(x + y) (2x + y) + (x + y) = 0
(x + y) (2x + y + 1) = 0
x+y=0
or
2x + y + 1 = 0
The perpendicular from the origin to the tangent at any point on a curve is equal to the abscissa
the point of contact. Find the equation of the curve satisfying the above condition and
passes through (1, 1)
Let P (x, y) be any point on the curve
Equation of tangent at 'P' is Y y = m (X x)
mX Y + y mx = 0
Now,
y mx

2 = x
1
+
m

y2 + m2x2 2mxy = x2 (1 + m2)


y 2 x 2 dy
=
which is homogeneous equation
2xy
dx
Putting y = vx
dy
dv
=v+x
dx
dx
dv
v2 1

v+x
=
dx
2v
x

dv v 2 1 2v 2
=
dx
2v

14

2v

dx
x
+1
n (v 2 + 1) = n x + n c

dv =

y2

x 2 + 1 = c
x

Curve is passing through (1, 1)

c=2
x2 + y2 2x = 0
Example :
Solution.

Find the nature of the curve for which the length of the normal at a point 'P' is equal to the radius
vector of the point 'P'.
Let the equation of the curve be y = f(x). P(x, y) be any point on the curve.
dy
=m
Slope of the tanget at P(x, y) is
dx

Slope of the normal at P is

P(x,y)

m =

1
m

Equation of the normal at 'P'


1
(X x)
Yy=
m
Co-ordinates of G (x + my, 0)
Now, OP2 = PG2
x2 + y2 = m2y2 + y2
x
m= y
x
dy
= y
dx
Taking as the sign
x
dy
= y
dx
y . dy = x . dx

y2
x2
=
+
2
2
x2 y2 = 2
x 2 y2 = c
(Rectangular hyperbola)
Again taking as ve sign
x
dy
= y
dx
y dy = x dx
y2
x2
=
+
2
2
2
2
x + y = 2
x2 + y2 = c
(circle)

15

G(x+my,0)

SHORT REVISION
DIFFERENTIAL EQUATIONS OF FIRST ORDER AND FIRST DEGREE DEFINITIONS:
1.
2.

An equation that involves independent and dependent variables and the derivatives of the
dependent variables is called a DIFFERENTIAL EQUATION.
A differential equation is said to be ordinary , if the differential coefficients have reference to a
single independent variable only and it is said to be PARTIAL if there are two or more
independent variables . We are concerned with ordinary differential equations only.
u u u
= 0 is a partial differential equation.
eg.
+
+
x

3.

Finding the unknown function is called SOLVING OR INTEGRATING the differential equation . The
solution of the differential equation is also called its PRIMITIVE, because the differential equation
can be regarded as a relation derived from it.
4.
The order of a differential equation is the order of the highest differential coefficient occuring in it.
5.
The degree of a differential equation which can be written as a polynomial in the derivatives is the
degree of the derivative of the highest order occuring in it , after it has been expressed in a form
free from radicals & fractions so far as derivatives are concerned, thus the differential equation :
p
q
dmy
d m 1 ( y )
f(x , y) m + (x , y)
m 1 + ....... = 0 is order m & degree p.
d x
dx

Note that in the differential equation ey xy + y = 0 order is three but degree doesn't apply.
6.
FORMATION OF A DIFFERENTIAL EQUATION :
If an equation in independent and dependent variables having some arbitrary constant is given ,
then a differential equation is obtained as follows :

Differentiate the given equation w.r.t. the independent variable (say x) as many times
as the number of arbitrary constants in it .

Eliminate the arbitrary constants .
The eliminant is the required differential equation . Consider forming a differential
equation for y = 4a(x + b) where a and b are arbitary constant .
Note : A differential equation represents a family of curves all satisfying some common properties.
This can be considered as the geometrical interpretation of the differential equation.
7.
GENERAL AND PARTICULAR SOLUTIONS :
The solution of a differential equation which contains a number of independent arbitrary constants equal
to the order of the differential equation is called the GENERAL SOLUTION (OR COMPLETE INTEGRAL OR
COMPLETE PRIMITIVE) . A solution obtainable from the general solution by giving particular values to the
constants is called a PARTICULAR SOLUTION.
Note that the general solution of a differential equation of the nth order contains n & only n independent
arbitrary constants. The arbitrary constants in the solution of a differential equation are said to be
independent, when it is impossible to deduce from the solution an equivalent relation containing fewer
arbitrary constants. Thus the two arbitrary constants A, B in the equation y = A ex + B are not independent
since the equation can be written as y = A eB. ex = C ex. Similarly the solution y = A sin x + B cos (x + C)
appears to contain three arbitrary constants, but they are really equivalent to two only.
8.
Elementary Types Of First Order & First Degree Differential Equations .
TYPE
1. VARIABLES SEPARABLE : If the differential equation can be expressed as ;
f (x)dx + g(y)dy = 0 then this is said to be variable separable type.
A general solution of this is given by f(x) dx + g(y) dy = c ;
where c is the arbitrary constant . consider the example (dy/dx) = exy + x2. ey.
Note : Sometimes transformation to the polar coordinates facilitates separation of variables.
In this connection it is convenient to remember the following differentials.
If x = r cos ; y = r sin then,
(i) x dx + y dy = r dr
(ii) dx2 + dy2 = dr2 + r2 d2 (iii) x dy y dx = r2 d
If x = r sec & y = r tan then x dx y dy = r dr and x dy y dx = r2 sec d .
TYPE
2 :

dy
= f (ax + by + c) , b 0.
dx

To solve this , substitute t = ax + by + c. Then the equation reduces to separable type in the
variable t and x which can be solved.

16

Consider the example (x + y)2 d y = a2 .


dx

TYPE
3. HOMOGENEOUS EQUATIONS :
A differential equation of the form

f (x , y )
dy
=
(x , y )
dx

where f (x , y) & (x , y) are homogeneous functions of x & y , and of the same degree , is called
HOMOGENEOUS . This equation may also be reduced to the form

dy
= g x & is solved by
dx
y

putting y = vx so that the dependent variable y is changed to another variable v, where v is some
unknown function, the differential equation is transformed to an equation with variables separable.
Consider

y (x + y )
dy
+
= 0.
x2
dx

TYPE
4. EQUATIONS REDUCIBLE TO THE HOMOGENEOUS FORM :
a x + b1y + c1
If d y = 1
dx

(i)
(ii)

a 2 x + b 2 y + c2

where a1 b2 a2 b1 0,

i.e.

a1
a
2
b1
b2

then the substitution x = u + h, y = v + k transform this equation to a homogeneous type in the new
variables u and v where h and k are arbitrary constants to be chosen so as to make the given
equation homogeneous which can be solved by the method as given in Type 3. If
a1 b2 a2 b1 = 0 , then a substitution u = a1 x + b1 y transforms the differential equation to an equation
with variables separable. and
b1 + a2 = 0 , then a simple cross multiplication and substituting d (xy) for x dy + y dx & integrating
term by term yields the result easily.
x 2y + 5
2 x + 3y 1
Consider dy =
; dy =

dx

2x + y 1

dx

4x + 6y 5

&

dy
2x y + 1
=
dx
6 x 5y + 4

(iii)

In an equation of the form : yf (xy) dx + xg (xy)dy = 0 the variables can be separated by the substitution
xy = v.
IMPORTANT NOTE :
(a)
The function f (x , y) is said to be a homogeneous function of degree n if for any real number
t ( 0) , we have f (tx , ty) = tn f(x , y) .
For e.g. f(x , y) = ax2/3 + hx1/3 . y1/3 + by2/3 is a homogeneous function of degree 2/3 .
(b)

A differential equation of the form

dy
= f(x , y) is homogeneous if f(x , y) is a homogeneous
dx

function of degree zero i.e. f(tx , ty) = t f(x , y) = f(x , y). The function f does not depend on
y
x
x & y separately but only on their ratio
or .
x
y
LINEAR DIFERENTIAL EQUATIONS :
A differential equation is said to be linear if the dependent variable & its differential coefficients occur
in the first degree only and are not multiplied together .
The nth order linear differential equation is of the form ;
a0 (x)

dn y
d n 1 y
+
a
(x)
+ ...... + an (x) . y = (x) . Where a0(x) , a1(x) ..... an(x) are called the
1
d xn
d x n 1

coefficients of the differential equation.


Note that a linear differential equation is always of the first degree but every differental equation of the
3

d2 y d y
first degree need not be linear. e.g. the differential equation 2 + + y2 = 0 is not linear, though
dx
dx

its degree is 1.
TYPE 5. LINEAR DIFFERENTIAL EQUATIONS OF FIRST ORDER :
The most general form of a linear differential equations of first order is
functions of x .

dy
+ Py = Q , where P & Q are
dx

Pdx
To solve such an equation multiply both sides by e
.

17

Pdx
NOTE : (1) The factor e
on multiplying by which the left hand side of the differential equation
becomes the differential coefficient of some function of x & y , is called integrating factor of the
differential equation popularly abbreviated as I. F.
(2)
It is very important to remember that on multiplying by the integrating factor , the left hand side becomes
the derivative of the product of y and the I. F.
(3)
Some times a given differential equation becomes linear if we take y as the independent variable
and x as the dependent variable. e.g. the equation ;

(x + y + 1)

dy
dx
= y2 + 3 can be written as (y2 + 3)
= x + y + 1 which is a linear differential
dx
dy

equation.
TYPE
6. EQUATIONS REDUCIBLE TO LINEAR FORM :
The equation

dy
+ py = Q . yn where P & Q functions of x , is reducible to the linear form by
dx

dividing it by yn & then substituting yn+1 = Z . Its solution can be obtained as in Type
5. Consider
the example (x3 y2 + xy) dx = dy.
The equation
9.

dy
+ Py = Q . yn is called BERNOULIS EQUATION.
dx

TRAJECTORIES :
Suppose we are given the family of plane curves.
(x, y, a) = 0
depending on a single parameter a.
A curve making at each of its points a fixed angle with the curve of the family passing through that point
is called an isogonal trajectory of that family ; if in particular = /2, then it is called an orthogonal
trajectory.
Orthogonal trajectories : We set up the differential equation of the given family of curves. Let it be of
the form
F (x, y, y') = 0
The differential equation of the orthogonal trajectories is of the form

1
F x, y, = 0
y

The general integral of this equation


1 (x, y, C) = 0
gives the family of orthogonal trajectories.
Note : Following exact differentials must be remembered :

x
ydx xdy
x dy yd x
y
= d
= d (iii)
2
2
x
y
y
x
xdy + ydx
x dy ydx
dx + dy
y
= d (ln xy) (v)
= d ln
(iv)
= d (ln (x + y))
(vi)
x
xy
xy
x+y
xdy ydx
x

y
ydx xdy
ydx xdy
x

= d tan 1 (ix)
= d ln (viii)
= d tan 1
(vii)
2
2
2
2

x
xy
y
x +y
y

x +y

(i)

xdy + y dx = d(xy)

(ii)

(x)

xdx + ydy
= d ln x 2 + y 2 (xi)

x2 + y2

(xiii)

e y x e y dy e y dx
d =
x2
x

1 x dy + y dx
(xii)
d =
xy
x2 y2

e x y e x dx e x dy
d =
y2
y

EXERCISEI

[ FORMATION & TYPE 1 & TYPE 2 ]


Q.1

State the order and degree of the following differential equations :


3

(i)
Q.2

2
d 2y d y
= 1 +
d x2 d x

d 2x
dx
2 + xt = 0
dt
d t

(ii)

3/ 2

Form a differential equation for the family of curves represented by ax + by = 1 , where a & b

18

Q.3
Q.4

are arbitary constants.


Obtain the differential equation of the family of circles x2 + y2 + 2gx + 2fy + c = 0 ; where
g , f & c are arbitary constants.
Form the differential equation of the family of curves represented by,
c (y + c)2 = x3 ; where c is any arbitrary constant.

Q.5

n (sec x + tan x)
n (sec y + tan y)
dx =
dy
cos x
cos y

Q.7

dy
+
dx

Q.9

x dx y dy
=
x dy y dx

(x

)(

Q.6

(1 x) (1 y) dx = xy (1 + y) dy

Q.8

yx

1 y2 1
xy

=0

1 + x2 y2
x2 y2

Q.10

dy
dy
= a y2 +

dx
dx

dy
= sin (x + y) + cos (x + y)Q.111
dx

dy
x (2 ln x + 1)
=
dx
sin y + y cos y

Q.12 It is known that the decay rate of radium is directly proportional to its quantity at each given instant. Find
the law of variation of a mass of radium as a function of time if at t = 0 , the mass of the radius was m0
and during time t0 % of the original mass of radium decay.
Q.13

x+y
xy
dy
= sin
+ sin
2
2
dx

Q.14 Sin x .

dy
= y . lny if y = e , when x =
2
dx

Q.15 e(dy/dx) = x + 1 given that when x = 0 , y = 3


Q.16 A normal is drawn at a point P(x , y) of a curve. It meets the x axis at Q. If PQ is of constant length
dy

Q.17
Q.18
Q.19
Q.20

k, then show that the differential equation describing such curves is, y
= k 2 y 2 . Find the
dx
equation of such a curve passing through (0, k).
Find the curve for which the sum of the lengths of the tangent and subtangent at any of its point
is proportional to the product of the coordinates of the point of tangency, the proportionality factor is
equal to k.
Obtain the differential equation associated with the primitive ,
y = c1 e3x + c2 e2x + c3 ex , where c1 , c2 , c3 are arbitrary constants .
A curve is such that the length of the polar radius of any point on the curve is equal to the length of the
tangent drawn at this point . Form the differential equation and solve it to find the equation of the curve.
Find the curve y = f (x) where f (x) 0 , f (0) = 0 , bounding a curvilinear trapezoid with the base
[0, x] whose area is proportional to (n + 1)th power of f (x). It is known that f (1) = 1.

EXERCISEII
[ TYPE
3 & TYPE
4]

Q.1
Q.2
Q.3
Q.4
Q.5
Q.6
Q.7
Q.8
Q.9

Q.10

dy
x 2 + xy
= 2
dx
x + y2

Find the equation of a curve such that the projection of its ordinate upon the normal is equal to its abscissa.
The light rays emanating from a point source situated at origin when reflected from the mirror of a search
light are reflected as beam parallel to the x axis. Show that the surface is parabolic, by first forming the
differential equation and then solving it.
The perpendicular from the origin to the tangent at any point on a curve is equal to the abscissa of the point
of contact. Find the equation of the curve satisfying the above condition and which passes through (1, 1).
Find the equation of the curve intersecting with the x- axis at the point x = 1 and for which the length of
the subnormal at any point of the curve is equal to the arthemetic mean of the co-ordinates of this point
(y x)2(x + 2y) = 1.
dy
Use the substitution y2 = a x to reduce the equation y3 .
+ x + y2 = 0 to homogeneous form and
d
x
hence solve it.
Find the isogonal trajectories for the family of rectangular hyperbolas x2 y2 = a2 which makes with it
an angle of 45.
(x3 3xy2) dx = (y3 3x2y) dy
dy f ( x , y)
Show that every homogeneous differential equation of the form
=
where f and g are
dx g ( x , y)
homogeneous function of the same degree can be converted into variable separable by the substitution
x = r cos and y = r sin.
y
y

x cos x + y sin x y

y
y dy

y sin x x cos x x d x = 0

19

Q.11

Find the curve for which any tangent intersects the yaxis at the point equidistant from the point of
tangency and the origin.

Q.12 (x y) dy = (x + y + 1) dx
Q.15

x+y+1
dy
Q.16
=
2
x
+ 2y + 3
dx

dy
dy
yx+1
= x + 2y 3
Q.14
=
y
+x+5
dx
d
x
2x + y 3
2
2 (y + 2)
cos x (3 cos y 7 sin x 3)
dy
dy
= 0
=
+
2 Q.17
(x + y 1)
sin y (3 sin x 7 cos y + 7)
dx
dx

Q.13

Q.18 Show that (4x + 3y + 1) dx + (3x + 2y + 1) dy = 0 represents a hyperbola having an asymptotes,


x + y = 0 & 2x + y + 1 = 0.
Q.19 If the normal drawn to a curve at any point P intersects the x-axis at G and the perpendicular from P on
the x-axis meets at N , such that the sum of the lengths of PG and NG is proportional to the abscissa of
the point P, the constant of proportionality being k. Form the differential equation and solve it to show
1

k2 x2

k2 x2

or y2 =
cx k , where c is any arbitrary
that the equation of the curve is, y2 = cx k
2k + 1
2k 1
constant.
Q.20 Show that the curve such that the distance between the origin and the tangent at an arbitrary point is
equal to the distance between the origin and the normal at the same point ,
x2 + y2 = c e

tan 1 x
y

EXERCISEIII
[ TYPE
5 & TYPE
6 ]

Q.1
Q.2
Q.3

(x + tan y) dy = sin 2y dx
Show that the equation of the curve whose slope at any point is equal to y + 2x and which pass through
the origin is y = 2 (ex x 1).
dy
dy
x
1
+
y=
Q.4 (1 x)
+ 2xy = x (1 x)1/2
1 + x2

Q.5

dx
2x(1 + x 2 )
Find the curve such that the area of the trapezium formed by the coordinate axes, ordinate of an
arbitrary point & the tangent at this point equals half the square of its abscissa .

Q.6
Q.8

x (x 1)
(x 2) y = x3 (2x 1)
Q.7 (1 + y + xy) dx + (x + x3) dy = 0
dx
Find the curve possessing the property that the intercept , the tangent at any point of a curve cuts off on
the yaxis is equal to the square of the abscissa of the point of tangency.

Q.9

sin x

dx

dy

dy

dy
+ 3y = cos x
dx

Q.10 x(x + 1)

dy
= y (1 x) + x3 . lnx
dx

y = 2 x cosec 2 x
Q.12 (1 + y) dx = (tan1 y x)dy
Q.11 x
dx
Q.13 Find the curve such that the area of the rectangle constructed on the abscissa of any point and the initial
ordinate of the tangent at this point is equal to a2. (Initial ordinate means y intercept of the tangent).
Q.14 Let the function ln f(x) is defined where f(x) exists for x 2 & k is fixed positive real number, prove
that if

d
(x . f (x)) k f (x) then f(x) A x 1 k where A is independent of x.
dx
x
x

Q.15 Find the differentiable function which satisfies the equation f (x) = f ( t ) tan t dt + tan( t x ) dt

where x ( 2 , 2)
Q.16 y x Dy = b(1 + xDy)

dy

Q.17 Integrate (1 + x)
+ 2yx 4x = 0 and obtain the cubic curve satisfying this equation and
dx
passing through the origin.
dy

Q.18 If y1 & y2 be solutions of the differential equation


+ Py = Q, where P & Q are functions of x
dx
alone, and y2 = y1 z, then prove that
Q

dx
z = 1 + a e y 1 , 'a' being an arbitrary constant .
dy y
dy
y
+ ln y = 2 (ln y)2 Q.20
+ xy = yex/2 . sin x
Q.19
dx x
dx
x
dy
dy
Q.21 2
y sec x = y3 tan x
Q.22 x2 y x3
= y4 cos x
dx
dx
Q.23 y (2 xy + ex) dx ex dy = 0

20

Q.24 Find the curve for which the area of the triangle formed by the xaxis, the tangent line and radius vector
of the point of tangency is equal to a2.
Q.25 A tank contains 100 litres of fresh water. A solution containing 1 gm/litre of soluble lawn fertilizer runs
into the tank at the rate of 1 lit/min, and the mixture is pumped out of the tank at the rate of 3 litres/min.
Find the time when the amount of fertilizer in the tank is maximum.

EXERCISEIV

Q 1.

(GENERAL CHANGE OF VARIABLE BY A SUITABLE SUBSTITUTION)


(x y) dx + 2xy dy = 0
Q 2. (x3 + y2 + 2) dx + 2y dy = 0

Q 3.

Q 6.

dy
dy
+ x y = 0 Q 7.
(x + y)
dx
dx

Q 9.

dy
= exy (ex ey)
dx

dy
dy
tan y
= (1 + x) ex sec yQ 5.
+ y lny = xyex Q 4.

dx
dx
1+ x
2

dy
y2 x
=
dx
2 y (x + 1)

Q 8.

dy
ey 1
= 2
dx
x
x

(1 xy + x2 y2) dx = x2 dy

Q 10. y y sin x = cos x (sin x y2)

EXERCISEV(MISCELLANEOUS)
Q.1
Q.2

dy
y ln 2 = 2sin x . (cos x 1) ln 2 , y being bounded when x + .
dx
1
dy
= y + y dx given y = 1 , where x = 0
dx
0
x

Q.3

Given two curves y = f(x) passing through the points (0, 1) & y =

f(t) dt passing through the points

Q.4
(i)
(ii)
(iii)

(0, 1/2). The tangents drawn to both curves at the points with equal abscissas intersect on the x axis.
Find the curve f(x).
dy
Consider the differential equation
+ P(x)y = Q(x)
dx
If two particular solutions of given equation u(x) and v(x) are known, find the general solution of the
same equation in terms of u(x) and v(x).
If and are constants such that the linear combinations u(x) + v(x) is a solution of the given
equation, find the relation between and .
v(x) u(x)
.
If w(x) is the third particular solution different from u(x) and v(x) then find the ratio
w(x) u(x)
dy
= y3 + y2
dx

y2 x2

Q.5

x3

Q.6

Find the curve which passes through the point (2, 0) such that the segment of the tangent between the
point of tangency & the y axis has a constant length equal to 2 .

Q.7

x dy + y dx +

Q.9

Find the equation of the curve passing through the orgin if the middle point of the segment of its normal
from any point of the curve to the x-axis lies on the parabola 2y2 = x.

Q.10

xdy ydx
= 0 Q.8
x2 + y2

y dx x dy

(x y)

dx

2 1 x2

, given that y = 2 when x = 1

Find the continuous function which satisfies the relation, t f ( x t ) dt = f ( t ) dt + sin x + cosx x 1,
for all real number x.

dy
2
2
+ x (x2 + y2 a2) = 0 Q.12(1 x2)2 d y + y 1 x x 1 x d x = 0
dx
dy
2
2
Q.13 3 x y + cos (xy) xy sin (xy) +
{2x3y x2 sin (xy)} = 0.
dx
dy
1
Q.14 Find the integral curve of the differential equation, x (1 x l n y). + y = 0 which passes through 1, .
dx
e

Q.11

(x2 + y2 + a2) y

Q.15 Find all the curves possessing the following property ; the segment of the tangent between the
point of tangency & the x-axis is bisected at the point of intersection with the y-axis.
Q.16 y2(y dx + 2x dy) x2(2y dx + x dy) = 0
21

Q.17 A perpendicular drawn from any point P of the curve on the xaxis meets the xaxis at A . Length of
the perpendicular from A on the tangent line at P is equal to 'a' . If this curve cuts the y-axis orthogonally,
find the equation to all possible curves , expressing the answer explicitly .
Q.18 Find the orthogonal trajectories for the given family of curves when 'a' is the parameter.
(i) y = ax2
(ii) cos y = a e x
(iii) xk + yk = ak
Q.19 A curve passing through (1 , 0) such that the ratio of the square of the intercept cut by any tangent off the y-axis
to the subnormal is equal to the ratio of the product of the co-ordinates of the point of tangency to the product
of square of the slope of the tangent and the subtangent at the same point. Determine all such possible curves.
Q.20 A & B are two separate reservoirs of water. Capacity of reservoir A is double the capacity of reservoir
B. Both the reservoirs are filled completely with water, their inlets are closed and then the water is
released simultaneously from both the reservoirs. The rate of flow of water out of each reservoir at any
instant of time is proportional to the quantity of water in the reservoir at that time. One hour after the
water is released , the quantity of water in reservoir A is 1.5 times the quantity of water in reservoir B.
After how many hours do both the reservoirs have the same quantity of water ?
Q.21 (2x + 3y 7)x dx = (3x + 2y2 8)y dy
Q.22 Find the curve such that the segment of the tangent at any point contained between the x-axis and the
straight line y = ax + b is bisected by the point of tangency.
Q.23 Find the curve such that the ratio of the distance between the normal at any of its point and the origin to the
distance between the same normal and the point (a , b) is equal to the constant k. Interpret the curve. (k > 0)
Q.24 Let f (x, y, c1) = 0 and f (x, y, c2) = 0 define two integral curves of a homogeneous first order differential
equation. If P1 and P2 are respectively the points of intersection of these curves with an arbitrary line,
y = mx then prove that the slopes of these two curves at P1 and P2 are equal.
Q.25 Find the curve for which the portion of y-axis cut-off between the origin and the tangent varies
as cube of the absissa of the point of contact.

EXERCISEVI
(PROBLEMS ASKED IN JEE & REE)
Q.1

Determine the equation of the curve passing through the origin in the form y = f (x) , which satisfies the
dy
= sin (10 x + 6 y) .
[ JEE '96 , 5 ]
dx
dy

Solve the differential equation ; cos2 x


(tan 2x) y = cos4x , x< , when y (/ 6)=3 3 /8.
dx
4
y
y

Solve the diff. equation ; y cos (x dy y dx) + x sin ( x dy + y dx) = 0 , when y (1) =

differential equation

Q.2
Q.3
Q.4

du
dv
Let u (x) & v (x) satisfy the differential equations
+ p (x) u = f (x) &
+ p (x) v = g (x) where p
dx
dx

(x), f (x) & g (x) are continuous functions. If u (x1) > v (x1) for some x1 and f (x) > g (x) for all
x > x1 , prove that any point (x , y) where x > x1 does not satisfy the equations y = u (x) &
y = v (x).
[ JEE '97 , 5 ]
Q.5(i) The order of the differential equation whose general solution is given by
x+C

5
y = (C1 + C2) cos(x + C3) C4 e
where C1, C2, C3, C4, C5 are arbitrary constants, is
(A) 5
(B) 4
(C) 3
(D) 2
(ii) A curve C has the property that if the tangent drawn at any point P on C meets the coordinate axes at
A and B, then P is the mid-point of AB. The curve passes through the point (1, 1). Determine the
equation of the curve.
Q.6 Solve the differential equation (1 + tany) (dx dy) + 2x dy = 0
[ REE '98 , 6 ]

dy
dy
Q.7(a) A soluton of the differential equation, x
+ y = 0 is :
dx
dx

(A) y = 2

(B) y = 2x

(C) y = 2x 4

(D) y = 2x2 4

(b) The differential equation representing the family of curves, y2 = 2 c x + c , where c is a positive
parameter, is of :
(A) order 1
(B) order 2
(C) degree 3
(D) degree 4
(c) A curve passing through the point (1, 1) has the property that the perpendicular distance of the origin
from the normal at any point P of the curve is equal to the distance of P from the x axis . Determine the
[ JEE '99, 2 + 3 + 10, out of 200 ]
equation of the curve .

22

Q.8
Q.9

Solve the differential equation, (x2 + 4y2 + 4xy) dy = (2x + 4y + 1) dx .


[ REE '99, 6 ]
A country has a food deficit of 10 % . Its population grows continuously at a rate of 3 %. Its annual food
production every year is 4 % more than that of the last year . Assuming that the average food requirement
per person remains constant, prove that the country will become self-sufficient in food after 'n ' years,
where ' n' is the smallest integer bigger than or equal to ,
 n 10  n 9
.
 n (104
. ) 0.03

[ JEE '2000 (Mains) 10 ]

Q.10 A hemispherical tank of radius 2 metres is initially full of water and has an outlet of 12 cm2 cross sectional
area at the bottom. The outlet is opened at some instant. The flow through the outlet is according to the
law V(t) = 0.6 2gh(t) , where V(t) and h(t) are respectively the velocity of the flow through the outlet
and the height of water level above the outlet at time t, and g is the acceleration due to gravity. Find the
time it takes to empty the tank.
[ JEE '2001 (Mains) 10 ]
Q.11 Find the equation of the curve which passes through the origin and the tangent to which at every point
x 4 + 2xy 1
.
[ REE '2001 (Mains) 3 ]
(x, y) has slope equal to
1 + x2
x
Q.12 Let f(x), x > 0, be a nonnegative continuous function, and let F(x) = f (t )dt , x > 0. If for some c > 0,
0

f(x) < cF(x) for all x > 0, then show that f(x) = 0 for all x > 0.
[ JEE 2001 (Mains) 5 out of 100 ]
Q.13(a) A right circular cone with radius R and height H contains a liquid which evaporates at a rate proportional
to its surface area in contact with air (proportionality constant = k > 0). Find the time after which the
cone is empty.
dP ( x )
(b) If P(1) = 0 and
> P(x) for all x > 1 then prove that P(x) > 0 for all x > 1.
dx
[JEE 2003, (Mains) 4 + 4]
2 + sin x dy

Q.14(a) If
= cos x, y (0) = 1, then y =
2
1 + y dx
1
1
1
(A) 1
(C)
(D) [JEE 2004 (Scr.) ]
(B)
2
3
4
( x + 1) 2 + y 3
. Find the
(b) A curve passes through (2, 0) and the slope of tangent at point P (x, y) equals
( x + 1)
equation of the curve and area enclosed by the curve and the x-axis in the fourth quadrant.
Q.15(a) The solution of primitive integral equation (x2 + y2)dy = xy dx, is y = y(x). If y(1) = 1 and y(x0) = e, then
x0 is
e2 + 1
(A) 2(e 1)
(B) 2(e + 1)
(C) 3 e
(D)
2
(b) For the primitive integral equation ydx + y2dy = xdy; xR, y > 0, y = y(x), y(1) = 1, then y(3) is
(A) 3
(B) 2
(C) 1
(D) 5 [JEE 2005 (Scr.)]
(c) If length of tangent at any point on the curve y = f (x) intercepted between the point and the xaxis is of
length 1. Find the equation of the curve.
[JEE 2005 (Mains)]
Q.16 A tangent drawn to the curve, y = f (x) at P(x, y) cuts the x-axis at A and B respectively such that
BP : AP = 3 : 1, given that f (1) = 1, then
dy
dy
(A) equation of the curve is x
3y = 0
(B) equation of curve is x
+ 3y = 0
dx
dx
(C) curve passes through (2, 1/8)
(C) normal at (1, 1) is x + 3y = 4
[JEE 2006, 5]
2

ANSWER KEY

EXERCISEI
d 2y
Q 2. xy 2 + x
dx

Q 1. (i) order 2 & degree 3 (ii) order 2 & degree 2

dy
dy
=0
y
d
x

dx

Q 3. [1 + (y)] .y 3y(y) = 0

Q 4. 12y (y) = x[8(y)3 27]

Q 5. ln2 (sec x + tan x) ln2 (sec y + tan y) = c

Q 6. ln x (1 y) = c

23

1
1
y 2y + x
2
2

Q 7.

x 2 1 sec1 x +

Q 9.

x2 y2 +

Q 8. y = c (1 ay) (x + a)
x + y

Q 10. ln 1 + tan
=x+c
2

y2 1 = c

1 + x2 y2 =

c (x + y )
x2 y2

Q 11. y sin y = x ln x + c

Q 12. m = m0 e k t where k =

y
x
= c 2 sin
4
2
Q 15. y = (x + 1) . ln (x + 1) x + 3

Q.14 y = etan(x/2)

Q 13. ln tan

Q 16. x2 + y2 = k2

Q 17. y =  n c k x 1
k
Q 19. y = k x or x y = c
2 2

ln 1

t0

100

d3 y

d2 y

dy

6
+ 11
6y=0
Q 18.
dx
d x 3 1/n d x 2
Q 20. y = x

EXERCISEII

1
x + 2y
x
Q.1 c (x y)2/3 (x + xy + y)1/6 = exp
tan 1
where exp x e
3
x
3

2
y y 2 x 2 c

n
=
Q.2

x 3 , where same sign has to be taken.


x2
Q 4. x + y 2x = 0
Q 5. (x y)2 (x + 2y) = 1
1
a
Q 6.
lnx2 + a2 tan1 = c , where a = x + y2
Q 7. x2 y2 + 2xy = c ; x2 y2 2xy = c
2
x
y
Q 8. y x = c (y + x)
Q 10. xy cos = c
x
2y + 1
2
2
= ln c x 2 + y 2 + x + y + 1
Q 11. x + y = cx
Q 12. arctan

y2 y y2 x 2

Q 13. (x + y 2) = c (y

x)3

Q 14.

Q 15. x + y + = ce3(x2y)
3 x 1)2 (cos y + sin x 1)5 = c
Q 17. (cos y sin

tan 1

Q 16. e

2x + 1
y+3
+ ln c
x+2

2 tan 1

y + 2
x 3

(y + 3)

+ (x + 2) = 0
2

= c . (y + 2)

EXERCISEIII
Q 1. x cot y = c + tan y

Q 2. y = 2 (ex x 1)

1 1
1

1 + x2 1
Q 3. y 1 + x 2 = c + ln tan arc tan x Another form is y 1 + x 2 = c + ln
2 2
2

x
2
2
2
2
Q 4. y = c (1 x) + 1 x Q 5. y = cx x Q 6. y (x 1) = x (x x + c)Q 7. xy = c arc tan x
1
3

Q 10. 4 (x + 1) y + x3 (1 2 lnx) = cx

x
x
= c + 2 tan x
2
2
Q 11. y = cx + x ln tan x

Q 12. x = cearctany + arc tan y 1

Q 13. y = cx

Q 8. y = cx x2

Q 9. + y tan3

a2
2x

1
2

Q.15 cos x 1 Q 16. y(1 + bx) = b + cx Q 17. 3y (1 + x) = 4x3 Q 19. x = lny cx 2 +

1
x
= 1 + (c + x) cot +
2
2
y
2
7
Q 22. x3 y3 = 3 Sinx + c Q 23. y1 ex = c x Q 24. x = cy a Q 25. 27 minutes
9
y
Q 20. ex/2 = y (c + cosx)

Q 21.

24

EXERCISEIV

Q 1. y + x ln ax = 0

Q 2. y = 3x 6x x3 + cex + 4

Q 3. x ln y = ex (x 1) + c

Q 4. sin y = (ex + c) (1 + x)

Q 5. cx + 2xey = 1

Q 6. y = cex ; y = c +

Q 7. y2 = 1 + (x + 1)  n

x2
2

1
tan  n cx
x
2
c
Q 10. y2 = sin x + 2
3
sin x

c
c
or x + (x + 1) ln
x +1
x +1

Q 8. y =

Q 9. ey = c . exp (ex) + ex 1

EXERCISEV
Q.1 y = 2sin x

Q.2 y =

1
(2 ex e + 1)
3 e

Q.3 f (x) = e2x

Q.5 (i) y = u(x) + K(u(x) v(x)) where K is any constant ; (ii) + = 1 ; (iii) constant

Q.5 xy = c y + y 2 x 2
Q.8

Q.6 y = 4 x 2 + 2 n

sin 1x y
+
= 2
2
x y 4

Q.9 y2 = 2x + 1 e2x

Q.11 (x + y) + 2a (y x) = c

Q.7 xy + tan1

y
=c
x

Q.10 f (x) = ex cos x

Q.12 y =

Q.13 x (x y + cos xy) = c

+c e

1 x2

x
1 x2

Q.14 x (e y + l n y + 1) = 1

Q.16 x2 y2 (y2 x2) = c

Q.15 y = cx

2 4 x2

Q.17 y = a .

ex/a + e x/a
& y=a
2

Q.18 (i) x2 + 2y2 = c, (ii) siny = ce x , (iii) y = cx if k = 2 and

1
x

k 2

k 2

=
c

k 2

if k 2

2 y/ x

Q.19 x = e 2 y / x ; x = e
Q.20 T = log4/3 2 hrs from the start
Q.21 (x2 y2 1)5 = c (x2 + y2 3)
Q.22 y2 a xy by = c
Q.23 (k + 1) x2 + (k + 1) y2 2kax 2kby = c
or (k 1) x2 + (k 1) y2 2kax 2kby = c both represents a circle. Q.25 2y + Kx3 = cx

EXERCISEVI
Q.1 y =

5 tan 4x 5x
1
tan 1

3
3
4 3 tan 4x

Q.5 (i) C

(ii) xy = 1 (x > 0 , y > 0)

Q.7 (a) C

(b) A, C

Q.2 y =

1
y
tan 2x . cos2x Q.3 xy sin =
2
x 2
Q.6 x ey (cosy + siny) = ey siny + C

(c) x2 + y2 2x = 0

Q.8 y = ln ((x + 2y)2 + 4(x + 2y) + 2)

3
2 2

x + 2y + 2 2
+ c
x + 2y + 2 + 2

ln

7 x 10 5
Q.10

135 g

Q.13 (a) T =

sec.
H
k

Q.15 (a) C; (b) A; (c) 1 y 2 + ln

Q.11

y = (x 2tan1x) (1 + x2)

Q.14 (a) C ; (b) y = x2 2x , area =


1 1 y2
=x+c
y

25

Q.16 B, C

4
sq. units
3

EXERCISEVII
Only one correct option
1.

The degree of differential equation satisfying the relation


1+ x 2 +

(A) 1
2.
3.

1+ y 2 = (x
(B) 2

5.

=a

9.

dx 2

d2 y
= a 2
dx

(C)

e6 + 9
2

(D) loge 6

(C) 3 e2

(D) 2 e3

If integrating factor of x(1 x 2) dy + (2x 2 y y ax 3) dx = 0 is e

2x ax

p . dx

2x 1

(B) (2x 2 1)

Integral curve satisfying y =


5
3

(C)

x +y

k
t
m

(D)

(2x 2 1)
x(1 x 2 )

(D) 1 + x

x2 y2

, y(1) = 2, has the slope at the point (1, 2) of the curve, equal to

(B) 1

The solution of
(A) v = ce

, then P is equal to

x(1 x 2 )
ax 3
dy
If
= 1 + x + y + xy and y ( 1) = 0, then function y is
dx
2
2
(A) e(1 x ) / 2
(B) e(1+ x ) / 2 1
(C) loge (1 + x) 1

(A)

13.

12.

dy 2
(B) 1 +
dx

d2 y

If (x) = (x) and (1) = 2, then (3) equals


(A) e2
(B) 2 e2

(A)

11.

dy

dy
(B) y + x = 1 +
dx

dx

(B) e6 + 1

10.

dy
+ x3
dx

d2 y
dy
(C) 1 + = a2 2
(D) none of these
dx
dx
The differential equation of all circles which pass through the origin and whose centres lie on y-axis is
dy
dy
(A) (x 2 y 2)
2xy = 0
(B) (x 2 y2)
+ 2xy = 0
dx
dx
dy
dy
(C) (x 2 y2)
xy = 0
(D) (x 2 y 2)
+ xy = 0
dx
dx
dy
= e2y and y = 0 when x = 5, the value of x for y = 3 is
If
dx

(A) e5
8.

dy
dy

dy

dy
(C) y x = 1 +
(D) y + x = 1
dx
dx

dx

dx
The differential equation obtained on eliminating A and B from = y = A cos (t) + B sin (t) is
(A) y + y = 0
(B) y 2y = 0
(C) y = 2y
(D) y + y = 0
The differential equation whose solution is (x h)2 + (y k)2 = a2 is (a is a constant)

7.

(D) none of these

d y

dx 2 + xy = cos x, then

(A) p < q
(B) p = q
(C) p > q
(D) none of these
The differential equation for all the straight lines which are at a unit distance from the origin is

dy 2
(A) 1 +
dx

6.

1+ x 2 ) is :
(C) 3

If p and q are order and degree of differential equation y

dy

dy
=1
(A) y x
dx

dx

4.

1+ y 2 y

(C) 1

(D)

5
3

dv
k
+
v = g is
dt
m

mg
k

(B) v = c

mg m t
e
k

The solution of the differential equation

(C) v e

k
t
m

=c

dy
a + x dx + xy = 0 is

26

mg
k

(D) v e m = c

mg
k

x+a

(A) y = Ae2/3 (2a x)

(B) y = Ae2/3 (a x)

(C) y = Ae (2a + x) x + a
Where A is an arbitrary constant.
2/3

14.

15.

17.

(2a x) x + a

dy
+ y(x) y = r(x) then y1(x) + y2(x) is solution of :
dx
dy
dy
(A)
+ f(x) y = 0
(B)
+ 2f(x) y = r(x)
dx
dx
dy
dy
+ f(x) y = 2 r(x)
(D)
+ 2f (x) y = 2r(x)
(C)
dx
dx
2
The differential equation of all 'Simple Harmonic Motions' of given period
is
n
d2 x
dt

+ nx = 0

(B)

If

( x 2 + y 2 ) = ae tan

(A)

a /2
e
2

The function f() =

( y / x)

d2 x
dt

+ n2 x = 0

d2 x
dt

n2 x = 0

(D)

d2 x
dt

1
n2

x = 0.

, a > 0. Then y(0), equals

(B) ae/2
d
d

(C)

(C)

dx

1 cos cos x

2 /2
e
a

(D)

a /2
e
2

satisfies the differential equation

df
df
df
+ 2f() cot = 0 (B)
2f() cot = 0 (C)
+ 2f() = 0
d
d
d
2
The solution of the differential equation y1 y3 = 3y2 is
(A) x = A1y2 + A2 y + A3
(B) x = A1 y + A2
(C) x = A1 y2 + A2 y
(D) none of these

(A)

18.

x+a

If y1(x) and y2(x) are two solutions of

(A)
16.

(D) y = Ae

2/3

(D)

df
2f () = 0
d

The solution of y dx x dy + 3x 2 y2 e x dx = 0 is
x
x
x
3
3
3
(A) y + e x = C
(B) y e x = 0
(C) y + e x = C
(D) none of these
20.
The solution of the differential equation
(x 2 sin3 y y2 cos x) dx + (x 3 cos y sin2 y 2y sin x) dy = 0 is
3
(B) x 3 sin3 y + 3y2 sin x = C
(A) x sin3 y = 3y2 sin x + C
(C) x 2 sin3 y + y3 sin x = C
(D) 2x 2 sin y + y2 sin x = C
One or more than one options correct
21.
The differential equation of the curve for which the initial ordinate of any tangent is equal to the corresponding subnormal
(A) is linear
(B) is homogeneous
(C) has separable variables
(D) is none of these
22.
The solution of x 2 y12 + xy y1 6y2 = 0 are
1
(A) y = Cx 2
(B) x 2 y = C
(C)
log y = C+ log x (D) x 3 y = C
2
19.

23.

dy
The orthogonal trajectories of the system of curves = a/x are
dx

(A) 9 a(y + c) = 4x 3

(B) y + C =

2
3 a

24.

dy
The solution of (x 2 y3 + xy ) = 1 is
dx

(A)

1/x = 2 y2 + C e y / 2 (B)

(C)

2/x = 1 y2 + ey /2

(D)

x 3/2

(C) y + C =

2
3 a

x 3/2

the solution of an equation which is reducible to linear equation.


1 2x
2
= y2 + Ce y /2
x

EXERCISEVIII
1.
2.

(D) none of these

x dx y dy
1+ x2 y2
=
Solve : x dy y dx
x2 y2
Solve :

27

(a)

x2 dy + y(x + y) dx = 0, given that y = 1, when x = 1

(b)

y
y
x cos + y sin y
x
x

y
y
dy
y sin x cos x
=0
x
x
dx

3.

dy y 2 2xy x 2
Find the equation of the curve satisfying dx = 2
and passing through (1, 1).
y + 2xy x 2

4.

Find the solution of the differential equation

5.

Solve :(i)

Solve :(i)
(iii)

dx

=8

d2 y
dx 2

dy
= y, y > 0
dx

dy
= y tanx 2sinx
dx

(iii)
6.

(x + 3y2)

d3 y

(iv)

y(x2y + ex) dx = ex dy

satisfying y(0) =

1
, y (0) = 0 and y2(0) = 1.
8 1

(ii)

(1 + y + x2y) dx + (x + x3)dy = 0

(1 + x2)

dy
+ 2xy = cosx
dx

(ii)

dy
+ y = x2y4
dx

2y sinx dy + (y2 cosx + 2x) dx = 0


3

7.
8.
9.
10.

11.

12.
13.

14.

x
dy
2y
+
=
dx
x +1
y2
Find the curve y = f(x) where f(x) 0, f(0) = 0, bounding a curvilinear trapezoid with the base [0, x] whose area
is propostinal to (n + 1)th power of f(x). It is known that f(1) = 1
Find the nature of the curve for which the length of the normal at the point P is equal to the radius vector of the
point P.
A particle, P, starts from origin and moves along positive direction of y-axis. Another particle, Q, follows P i.e.
its velocity is always directed towards P, in such a way that the distance between P and Q remains
constant. If Q starts from (2, 0), find the equation of the path traced by Q. Assume that they start moving at
the same instant.

Solve the following differential equations.

dy x 2 y 2
Let c1 and c2 be two integral curves of the differential equation dx = 2
. A line passing through origin
x + y2
meets c1 at P(x1, y1) and c2 at Q(x2, y2). If c1 : y = f(x) and c2 : y = g(x) prove that f (x1) = g(x2).
dy
+ y = 0 which passes through (1, 1/e).
Find the integral curve of the differential equation x(1 xy)
dx
dy
Show that the integral curves of the equation (1 x2)
+ xy = ax are ellipses and hyperbolas, with the
dx
centres at the point (0, a) and the axes parallel to the co-ordinate axes, each curve having one constant axis
whose length is equal to 2.

If y1 & y2 be solutions of the differential equation

dy
+ Py = Q, where P & Q are functions of x alone,
dx
yQ dx

15.
16.
17.

18.

19.

and y2 = y1 z, then prove that


z = 1 + ae 1 ,'a' being an arbitrary constant.
Find the curve for which the sum of the lengths of the tangent and subtangent at any of its point is
proportional to the product of the coordinates of the point of tangency, the proportionality factor is
equal to k.
Find all the curves possessing the following property; the segment of the tangent between the point of
tangency & the xaxis is bisected at the point of intersection with the yaxis .
A curve passing through (1 , 0) such that the ratio of the square of the intercept cut by any tangent off
the yaxis to the subnormal is equal to the ratio of the product of the coordinates of the point of
tangency to the product of square of the slope of the tangent and the subtangent at the same point.
Determine all such possible curves.
A & B are two separate reservoirs of water. Capacity of reservoir A is double the capacity of reservoir B.
Both the reservoirs are filled completely with water , their inlets are closed and then the water is
released simultaneously from both the reservoirs. The rate of flow of water out of each reservoir at any
instant of time is proportional to the quantity of water in the reservoir at that time. One hour after the
water is released , the quantity of water in reservoir A is 1.5 times the quantity of water in reservoir B.
After how many hours do both the reservoirs have the same quantity of water ?
A curv e y = f(x ) passes through the point P (1 ,1). The normal to the curv e at P is ;
a (y 1) + (x 1) = 0. If the slope of the tangent at any point on the curve is proportional to the ordinate
of the point, determine the equation of the curve. Also obtain the area bounded by the yaxis, the curve
& the normal to the curve at P.
[IIT - 1996, 5 ]

28

Let u (x) & v (x) satisfy the differential equations

20.

du
dv
+ p (x) v = g (x) where p (x) ,
+ p (x) u = f (x) &
dx
dx

f (x) & g (x) are continuous functions. If u (x1) > v (x 1) for some x 1 and f (x) > g (x) for all x > x 1 , prove that
any point (x , y) where x > x 1 does not satisfy the equations y = u (x) & y = v (x).
A curve passing through the point (1, 1) has the property that the perpendicular distance of the origin
from the normal at any point P of the curve is equal to the distance of P from the x axis. Determine the
equation of the curve.
[IIT - 1999, 10 ]
A country has a food deficit of 10 %. Its population grows continuously at a rate of 3 % per year. Its
annual food production every year is 4 % more than that of the last year. Assuming that the average
food requirement per person remains constant, prove that the country will become selfsufficient in

21.
22.

food after ' n ' years , where ' n ' is the smallest integer bigger than or equal to,

 n 10  n 9
.
. ) 0.03
 n (104

[IIT - 2000 (Mains) 10 ]

An inverted cone of height H and radius R is pointed at bottom. It is filled with a volatile liquid completely.
If the rate of evaporation is directly proportional to the surface area of the liquid in contact with air
(constant of proportionality k > 0) , find the time in which whole liquid evaporates.
[IIT - 2003 (Mains) 4]

23.

EXERCISEVII
1. A

2. C

3. C

4. C

5. B

6. A

7. C

8. B

9. D

10. B

11. A

12. A

13. A

14. C

15. B

16. C

17. A

18. A

19. A

20. A

21. AB

22. ACD

23. ABC

24. ABD

EXERCISEVIII
1.

x2 y2 + 1+ x2 y2 =

2. (a) 3x2y = 2x + y

c( x + y )
x2 y2

y
(b) xy cos = c
x

3. x + y = 0
4. 64y = (e8x 8x) + 7
x
5. (i) y = 3y + c
(ii) xy = c arc tanx
(iii) y = cosx + c secx (iv) y (1 + x2) = c + sinx.

1
x3
x
e
=

+c
y
3
2
2
(iii) y sinx = x + c

6. (i)

7. y3 (x + 1) 2 =
8. y = x1/n

(ii)

= 3x2 + cx3

2
1
x6
+ x5 + x4 + c
5
4
6
9. Rectangular hyperbola or circle.

10. y = 2n x 2n (2

4 x2 )

12. x(ey + ny + 1) = 1 15. y =


16. y = cx

y3

17. x = e

1
a

2 y/ x

; x =e

a 1 + e a

, sq. unit
2

21. (c) x 2 + y2 2x = 0
23. t = H/k

29

1
 n c k 2x 2 1
k

18. T = log4/3 2 hrs from the start


19. ea(x1)

4 x2

2 y/ x

DIFFERENTIAL EQUATION
Some questions (AssertionReason type) are given below. Each question contains Statement 1 (Assertion) and
Statement 2 (Reason). Each question has 4 choices (A), (B), (C) and (D) out of which ONLY ONE is correct. So select
the correct choice :
Choices are :
(A)
Statement 1 is True, Statement 2 is True; Statement 2 is a correct explanation for Statement 1.
(B)
Statement 1 is True, Statement 2 is True; Statement 2 is NOT a correct explanation for Statement 1.
(C)
Statement 1 is True, Statement 2 is False.
(D)
Statement 1 is False, Statement 2 is True.
227.
Statement-1: The order of the differential equation whose general solution is y = c1cos2x + cos2sin2x + c3cos2x +
2x + c

228.

229.

6
is 3
c4e2x + c5 e
Statement-2: Total number of arbitrary parameters in the given general solution in the statement (1) is 6.
Statement-1: Degree of differential equation of parabolas having their axis along xaxis and vertex at (2, 0) is 2.
Statement-2: Degree of differential equation of parabola having their axis along xaxis and vertex at (1, 0) is 1.

x3
dy y
+c.
+ = x is xy =
dx x
3
dy
Statement2 : Solution of the differential equation
+ PY = Q is
dx
pdx
pdx
Ye
= Q.e
dx + c where P and Q are function of x alone.
Statement1 : Solution of the differential equation

230.

231.

Let the general solution of a differential equation be y = aebx + c .


Statement1 : Order of the differential equation is 3.
Statement2 : Order of the differential equation is equal to the number of actual constant of
the solution
Let F be the family of ellipses on the Cartesian plane, whose directrices are x = 2.

Statement1 : The order of the differential equation of the family F is 2.


Statement2
232.

: F is a two parameter family.

d2 y

dy
.
dx
dx
Statement1 : For any member of this family y as x .
Consider the differential equation (x 2 + 1).

= 2x.

Statement2
: Any solution of this differential equation is a polynomial of odd degree with positive
coefficient of maximum power.

233.

Statement1 : The solution of the differential equation x

y = xecx.

dy
= y ( log y log x + i )
dx

is

dy
dy
Statement2 : A solution of the differential equation x + y = 0 is y = 2.
dx
dx
234.

235.

Statement-1: Order of the differential equation of family of parabola whose axis is perpendicular to yaxis and
ratus rectum is fix is 2.
Statement-2: Order of first equation is same as actual no. of abitrary constant present in diff. equation.
Statement-1: Solution of y dy = x x as is family of rectangular hyperbola
Statement-2: Solution of y

236.

dy
= 1 is family of parabola
dx

Statement-1: Solution of differential equation dy (x 2y 1) + dx (y2x 1) = 0 is

x 2 y2
= x + y+c
2

Statement-2: Order of differential equation of family of circle touching the coordinate axis is 1.
237.

Statement-1: Integrating factor of

dy
+ y = x 2 is ex
dx
30 of 35

30

Statement-2: Integrating factor of


238.

239.

p(x )dx
dy
+ p(x)y = Q(x) is e
dx

Statement-1: The differential equation of all circles in a plane must be of order 3.


Statement-2: There is only one circle passing through three non-collinear points.

d3 y
Statement-1: The degree of the differential equation 3
dx

2/3

+62

dy
d2 y
+ 15
= 0 is 3.
2
dx
dx

Statement-2: The degree of the highest order derivative occuring in the D.E. when the D.E. has been expressed as
a polynomial of derivatives.

dy
2
2
2
2
dx = x cos (x + y ) is x - tan (x2 + y2) = c
Statement-1: Solution of
dy
y2
y3
yx
dx
x+y

240.

Statement-2: Since the given differential equation is homogenous can be solved by putting y = vx
241.

Statement-1: The order of the differential equation formed by the family of curve
x +c

y = c1ex + (c2 + c3) e 4 is 1. Here c1, c2, c3, c4 are arbitrary constant.
Statement-2: The order of the differential equation formed by any family of curve is equal to the number of
arbitrary constants present in it.
2

242.

d2 y
dy
Statement-1: The degree of differential equation 3 1 +
=
log
2 is not defined.

dx
dx
Statement-2: The degree of differential equation is the power of highest order derivative when differential
equation has been expressed as polynomial of derivatives.

243.

Statement-1: The order of differential equation of family of circles passing then origin is 2.
Statement-2: The order of differential equation of a family of curve is the number of independent parameters
present in the equation of family of curves

244.

Statement-1: Integrating factor of

245.

Statement-1: The differentiable equation y3dy + (x + y2) dx = 0 becomes homogeneous if we put y2 = t.


Statement-2: All differential equation of first order and first degree becomes homogeneous if we put y = tx.

246.

247.

xdy
+ 3y = x is x3
dx
dy
Statement-2: Integrating factor of
+ p(x)y = Q(x) is epdx
dx

p(x)dx
dy
+ P(x) y = Q(x) is e
+c
dx
p(x )dx
dy
+ P(x) y = Q(x) is e
Statement-2: Integrating factor of
dx

Statement-1: The general solution of

Statement-1: The general solution of

dy
+ y = 1 is yex = ex + c
dx

Statement-2: The number of arbitrary constants in the general solution of the differential equation is equal to the
order of differential equation.
2

248.

dy
dy
Statement-1: Degree of the differential equation y = x
+ 1 + is 2.
dx
dx
Statement-2: In the given equation the power of highest order derivative when expressed as a polynomials in
derivatives is 2.
31 of 35

31

Statement-1: The differential equation of the family of curves represented by y = A.ex is given by

249.

dy
= y is valid for every member of the given family.
dx

Statement-2:

250.

dy
=y.
dx

Statement-1: The differential equation

dy
2xy
= 2
can be solved by putting y = vx
dx x + y 2

Statement-2: Since the given differentiable equation is homogenous


Statement-1: A differential equation

251.

Pdx
dy y
+ = x 2 can be solved by finding. If = e
dx x

= e
= e = x then solution y.x = x3dx + c
Statement-2: Since the given differential equation in of the form dy/dx + py = wherep, are function of x
1/ xdx

252.

log x

Statement-1: The differential equation of all circles in a plane must be of order 3.


Statement-2: There is only on circle passing through three non collinear points.

ANSWER
227.
232.
239.
246.

A
228.
A 233. C 234.
D 240. C 241.
D 247. B 248.

D
229. A 230. D
231. A
A
235. D 236. B
237. A 238. A
C 242. A 243. A
244. A
245. C
A 249. A
250. A 251. A 252. A

DETAILS SOLUTION
227.

y = c1 cos2x + c2 sin2x + c3 cos2x + c4e2x + c5 e

2x + c6

1 cos 2x
cos 2x 1
+ c3
+ c 4e 2x + c5e 2x .ec6

2
2

c 2 c3

c 2 c3
= c1
+ cos 2x + + (c4 + c5 )e2x = 1 cos2x + 2 e2x + 3
2 2
2 2

= c1cos2x + c2

Total number of independent parameters in the given general solution is 3.


228.

Equation of parabola will be y2 = ap (x 1)


2y

229.

dy
dy
= p D.E. is y = 2
(x 1) degree of this D.E. is 1.
dx
dx

(a)
e

Pdx

=e

dx
x

=x

Sol. is xy =
xy =

230.

Ans. : A

dx + c

x3
+c.
3

(D)
y = aebx + c = aec. ebx = Aebx
order is two.

32 of 35

32

Ans. : D

231.

Statement II is true as any member of the family will have equation

x2
a2

( y )2

a 2 1 e2

= 1 , where 0 < e < 1, a >

0, b R and ae = 2.
Hence F is a two parameter family.
Statement I is true, because of statement II, because order of a differential equation of a n parameter family is
n.
Hence (a) is the correct answer.

232.

dy
d
dx = 2x dx
The given differential equation is
dy
x2 +1
dx
x3

dy
dy
2
n
= c x 2 + 1 y = c + x + c , c R.
= n x + 1 + nc, c > 0
3

dx
dx

Obviously y ,as x ; as c > 0


Hence (a) is the correct answer.

233.

The given equation can be rearranged as,


dy y
ye
= log
dx x
x
dv v log v
dv
dx
dy
dv
=

=
put y = vx

y = xecx
= v+x
dx
x
v log v
x
dx
dx
dy
for II, put
= p p 2 xp + y = 0
dx
dp
dp
dp
y= px p2 p = p + x

2p
= 0 or x 2p = 0 y = 2x + c
dx
dx
dx
Hence (c) is the correct answer.

234.

(x h)2 = 4b (y k)
here b is constant and h, k are parameters
Hence order is 2.
(D)
ydy = dx dx

235.

y2 x 2
+
= x + c is family of circle
2 2
236.

Ans. : A
ydy = dx

y2
= x + c which is family of parabola
2

xy d (xy) = d(x + y)

x 2 y2
=x+ y+c
2

let circle is (x h)2 + (y h)2 = h2


Hence order of differential equation will be 1.
237.
238.

Option (a) is correct.


Option (a) is correct
The equation of circle contains. Three independent constants if it passes through three non-collinear points,
therefore a is true and follows from R.
3

239.

Ans. : B

f .dx
I.F. = e
= ex

d3 y d 2 y

dy
3 = 2 2 15 6
dx
dx dx

33 of 35

33

Hence degree is 2.
240.

Ans. (D)

2x dx + 2y dy 2x ydx xdy
=

cos 2 (x 2 + y 2 ) y
y2

sec2 (x2 + y2) (2x dx + 2y dy) = 2

tan (x2 + y2) =

2. ( x 2 / y 2 )
2

y .d y

+c

x2
Ans. (C)
tan (x 2 + y 2 ) = c
2
y
c
c
y = c1ex + (c2 + c3) ex e 4 = ex (c1 + (c2 + c3) e 4 )
y = cex (1)
here c = c1 + (c 2 + c3 )ec4

241.

dy
= ce x
dx
dy
y = dx
ex
ex

dy
c = dxx Put in (1)
e
dy
So
= y and order is 1.
dx

c is correct.

242.

d2y
dy
1 + = log 2
dx
dx

2
d2 y
dy
1 + = log 2
dx
dx

degree is not defined as it is not a polynomial of derivatives.


a is correct.
244.

I.F. epdx = e

x dx

dy 3y
+
= 1 = x3.
dx x
245.

(C)

dy x + y 2
=
cannot be made homogenous by putting y = tx.
dx y + x 2
dy dt
But if we put y2 = t in the differential equation in assertion A then 2y
=
dx dx
1
And differential equation becomes t . dt + (x + t) dx = 0
2
t
or dx/dt
which is homogeneous.
2(x + t)
R is false since

246.

(D)
Statement-1 is false
Statement-2 is true.

34 of 35

34

247.

dy
dy
= dx
+ y= 1
1 y
dx
dy
1 y = dx log (1 y) = x

(b)

1 y = ex, yex = ex + c
order of differential equation is the number of arbitrary constants.
Both one true, but Statement-2 is not the correct explanation.
248.

(A)
2

y=x

dy
dy
+ 1 + becomes
dx
dx
2

dy
dy
(x 2 1) 2xy + (y 2 1) = 0 ,
dx
dx
249.

when expressed as a polynomial in derivatives.

(A)
y = A.ex
on differentiation we get

250.

dy
= A.e x
dx

dy
2xy
=
... (1)
dx x 2 + y 2
This is homogenous differential equation put y = vx
from (1)

dy
dv
=v+x
dx
dx

xdv
2x 2 v
= 2
dx x (1 + v 2 )
dv
2v
2v v v3 v(1 v 2 )

=
=
x =
v
dx 1 + v 2
1 + v2
1 + v2
2
(1 + v )
dx
v(1 v2 ) dv = x

v+

251.

dy/dx + y/x = x2 ... (1)


This is term of linear differential equation dy/dx + py = ... (2)
from (1) and (2) p = -1/x , = x2
I.f. e
= e
y.I.f = xI.fd + c
yx = x 3 dx + c.
Ans. (A)
Pdx

252.

1/ xdx = x

(A)
The equation of circle contains three independent constants if it passes through three non-collinear points therefore
A is true and follows from statement-2

35 of 35

35

También podría gustarte